Você está na página 1de 228

Remedial Law 2- Brondial Notes AY 2017- 2018

Remedial Law 2 Replevin- It shall serve as the provisional


remedy while the principal action shall be the
Provisional Remedy (Rule 57-61)
recovery of possession of personal property.
AIRRS JORDAN
- It also pertains to immediate recovery
A-Preliminary Attachment possession of personal property
I-Preliminary Injunction Support Pende Lite- The Principal Action is
R- Receivership support

R- Replevin *When a party avails of Provisional Remedy,


there is no need for Barangay Conciliation
S- Support Pendete Lite
Q: Which court has jurisdiction over Provisional
*Provisional Remedy is temporary in nature and Remedies?
it cannot stand alone.
A: The court which has jurisdiction over the
- Dependent on a principal action principal action shall also have jurisdiction over
- If there is no principal action, provisional the provisional remedy. (RTC or MTC)
remedy shall cease Exception: Support Pende lite- Action Incapable
of Pecuniary Estimation- RTC

Example: The Doctrine in Bristol Meyers Exception to the Exception: When a party
joined support towards another action and it is
- The Bond was filed for the preliminary
cognizable by the MTC hence there is no need
injuction but the principal action was
to file two separate action
already dismissed therefore the
provisional remedy shall also cease. Example: A party filed a criminal case of
seduction (Cognizable by MTC) and
Principal Action:
simultaneously filed support
Preliminary Attachment- Recovery of either
Q: What are the requirements in provisional
real or personal property
remedies?
Preliminary Injunction- The Principal action is
A: (A) Affidavit and (B) Bond
inherent under this rule.
Exception: Support Pendente Lite- Only Verified
Receivership- It shall serve as the principal
Complaint
action, the provisional remedy is the
appointment of a receiver. Q: Can there be a claim for damages for a
provisional remedy?

Rolan Jeff A. Lancion


Arellano University School of Law Page 1
Remedial Law 2- Brondial Notes AY 2017- 2018

A: YES, it is provided under Rule 57 Sec 20- or fraudulently taken, detained


Claim for damages on account of improper or converted, when the
irregular or excessive attachment property, or any part thereof,
has been concealed, removed,
Exception: Support Pendete Lite- Restitution or
or disposed of to prevent its
Reimbursement
being found or taken by the
Rule 57- Preliminary Attachment applicant or an authorized
Section 1- Ground upon which attachment may person;
issue - (ii) Kinds of Adverse Party
- (i)Kinds of Action o (d) In an action against a party
o (a) In an action for the recovery who has been guilty of a fraud
of a specified amount of money in contracting the debt or
or damages, other than moral incurring the obligation upon
and exemplary, on a cause of which the action is brought, or
action arising from law, in the performance thereof;
contract, quasi-contract, delict o (e) In an action against a party
or quasi-delict against a party who has removed or disposed
who is about to depart from the of his property, or is about to
Philippines with intent to do so, with intent to defraud his
defraud his creditors; creditors; or
o (b) In an action for money or o (f) In an action against a party
property embezzled or who does not reside and is not
fraudulently misapplied or found in the Philippines, or on
converted to his own use by a whom summons may be served
public officer, or an officer of a by publication.
corporation, or an attorney,
Period: At the commencement of the action or
factor, broker, agent, or clerk,
at any time before entry of judgment
in the course of his
employment as such, or by any Jurisdiction: It will depend on the principal
other person in a fiduciary action but generally RTC
capacity, or for a willful
Brondial: An unwritten purpose is to force the
violation of duty;
adverse party to come in to the negotiating
o (c) In an action to recover the table.
possession of property unjustly

Rolan Jeff A. Lancion


Arellano University School of Law Page 2
Remedial Law 2- Brondial Notes AY 2017- 2018

-When a property is attached, it shall become required by the next succeeding section, must
custodia legis or custody of the law. be duly filed with the court before the order
issues.

Requirements:
Principal Purpose: The written Purpose is to
“Secure the Satisfaction of favorable judgment” - Valid Cause of Action or Principal Action

-If a party that attaches a property obtain a - Valid Ground Sec 1 (a) to (f)
favorable judgment, he can proceed
- No Security
immediately to the sale of it or sale on
attachment. - The amount due or value of the
property sought to be recovered must
-Under the jurisdiction over the res, if the court
be equal to the sum prayed for in the
cannot acquire jurisdiction over the defendant,
order less other counter-claim
a party should acquire jurisdiction over the
thing or property. The limitation provided by *There is no limitation in the attachment of the
law is that a party cannot go beyond the res. property as long as the order prayed for must
be equal to the claim.

Bond (Do not confuse with counter-bond)


Two basic requirement required:
-The applicant shall put up this toward the court
- Affidavit - Bond
that has jurisdiction.

-It must be duly accredited otherwise the


Section 3- Affidavit & Bond preliminary attachment shall be null & void.

An order of attachment shall be granted only Case: Torres vs Satsatin


when it appears by the affidavit of the
Doctrine: The bond must be duly accredited by
applicant, or of some other person who
the Supreme Court otherwise the preliminary
personally knows the facts, that a sufficient
attachment shall be null & void.
cause of action exists, that the case is one of
those mentioned in section 1 hereof, that there Facts: The Torres siblings wanted to sold a
is no other sufficient security for the claim property through an agent which is Mr.
sought to be enforced by the action, and that Satsatin. In return, Mr. Satsatin sold the real
the amount due to the applicant, or the value of property that was given to him for Php 35
the property the possession of which he is million but he refused to give the entire sold
entitled to recover, is as much as the sum for property to Torres siblings. The latter wanted to
which the order is granted above all legal recover the amount and the applied for
counterclaims. The affidavit, and the bond preliminary attachment. Torres siblings posted

Rolan Jeff A. Lancion


Arellano University School of Law Page 3
Remedial Law 2- Brondial Notes AY 2017- 2018

a bond, but Mr. Satsatin questioned it since *In the first two-the court is not required to
Preliminary Attachment is issued before the have jurisdiction but in the last- court should
action. The court denied the motion but have issued a summons.
granted post of the counter-bond.
Q: What are the ways to lift a preliminary
Issue: WON the posting of the bond in this case attachment?
is proper?
A: Posting of counter bond/cash deposit or
Held: NO – The posting of the bond is incorrect Filing of a motion assailing the propriety and
since it is not duly accredited by the Supreme regularity of the issuance.
Court. From the facts given, it was shown that
Section 5- The means to lift is through cash
the bonding company that issued the bond to
deposit
secure the judgment of Preliminary Attachment
was only accredited in Makati, Pasay and Pasig Section `12- The means to lift is through cash
but it was not accredited in Dasmarinas, Cavite deposit/ counterbond and the writ has already
where the principal action is tried. In this case issued and implemented
the issuance of the Preliminary Attachment is Section 13- The mode to discharge is through a
null and void. motion assailing or questioning the propriety/
Case Doctrine: Mangila vs Court of Appeals regularity of its issuance

Supreme Court ruled that the issuance of the Case: Chuidian vs Sandiganbayan
writ of preliminary attachment was wrong on Doctrine: When the ground of the principal
the ground that there was no earnest effort of action and writ are the same, a party cannot
serving the summons. The Supreme Court used discharge the writ of PA pursuant to Section 13.
“SEC 5- The requirement of prior service of
summons shall not apply where the summons Facts: Vicente Chuidian was one of the cronies
could not be served personally or by substituted during the Marcos Regime and he was able to
diligent efforts”. The sheriff should exert effort secure a loan from a government bank in
to serve the summons to Mangila before she millions of dollar, to be used in the Philippines.
left for Guam. Chuidian instead invested such money in Silicon
Valley, Californian since there is the growth of
Q: What are the stages of preliminary Information technology in the early 1980’s. The
attachment? bank ran after him for the purpose of payment
A: (1) Order of granting the issuance, (2) and filed a case against before the court of San
Issuance of writ and (3) Order of Mateo, California which ended up in a
implementation compromise agreement. After the EDSA
Revolution, Cory Aquino created Ill-gotten
wealth act for the purpose of recovery of such

Rolan Jeff A. Lancion


Arellano University School of Law Page 4
Remedial Law 2- Brondial Notes AY 2017- 2018

ill-gotten wealth coming from the Marcos and sum not less than the value of the property
its cronies. The government sought to attach levied upon. In case of disagreement as to such
the Letters of Credit of Chuidian in California value, the same shall be decided by the court
USA. The writ of preliminary attachment was issuing the writ of attachment. No claim for
implemented when he arrived in the country. damages for the taking or keeping of the
property may be enforced against the bond
Issue: WON the Sandiganbayan abused its
unless the action therefor is filed within one
discretion for the denial of Chuidian’s motion?
hundred twenty (120) days from the date of the
Held: NO – The Supreme Court ruled that When filing of the bond.
the ground for the cause of action in the same
The sheriff shall not be liable for damages for
as for the application of preliminary
the taking or keeping of such property to any
attachment, a party cannot discharge the writ
such third-party claimant, if such bond shall be
through a mere motion otherwise you will also
filed. Nothing herein contained shall prevent
resolve the principal action through motion. In
such claimant or any third person from
this case, the principal action is for the recovery
vindicating his claim to the property, or prevent
of letters of credit and the ground for the
the attaching party from claiming damages
discharge of the writ is under fraud. If the court
against a third-party claimant who filed a
will release the preliminary attachment through
frivolous or plainly spurious claim, in the same
a motion, the court is also resolving the
or a separate action.
principal action. The proper remedy is Sec 12 or
posting a counter bond. When the writ of attachment is issued in favor
of the Republic of the Philippines, or any officer
Section 14. Proceedings where property
duly representing it, the filing of such bond shall
claimed by third person. — If the property
not be required, and in case the sheriff is sued
attached is claimed by any person other than
for damages as a result of the attachment, he
the party against whom attachment had been
shall be represented by the Solicitor General,
issued or his agent, and such person makes an
and if held liable therefor, the actual damages
affidavit of his title thereto, or right to the
adjudged by the court shall be paid by the
possession thereof, stating the grounds of such
National Treasurer out of the funds to be
right or title, and serves such affidavit upon the
appropriated for the purpose.
sheriff while the latter has possession of the
attached property, and a copy thereof upon the Q: Can a party attach a property that belongs to
attaching party, the sheriff shall not be bound the estate?
to keep the property under attachment, unless
A: NO, a party cannot attach the property
the attaching party or his agent, on demand of
belonging to the estate but he can attach his
the sheriff, shall file a bond approved by the
court to indemnify the third-party claimant in a

Rolan Jeff A. Lancion


Arellano University School of Law Page 5
Remedial Law 2- Brondial Notes AY 2017- 2018

interest over the estate since the property is in Nothing herein contained shall prevent the
custodia legis. party against whom the attachment was issued
from recovering in the same action the
damages awarded to him from any property of
Q: When a party discharge the writ under Sec the attaching party not exempt from execution
12, can he still apply under Sec 13? should the bond or deposit given by the latter
A: Yes, a party is now seeking to recover the be insufficient or fail to fully satisfy the award.
bond he put up (i) Bond- coming from a -When a party is granted of preliminary
Bonding Company and the party is only paying attachment does not automatically mean that
the premium (ii) Cash Deposit- Payment of cash he will get a favorable judgment.
directly to the court.
Cases:
Section 20. Claim for damages on account of
ALFREDO C. LIM, JR., Petitioner, v. SPOUSES
improper, irregular or excessive attachment. —
TITO S. LAZARO AND CARMEN T.
An application for damages on account of
LAZARO,Respondents.G.R. No. 185734, July 03,
improper, irregular or excessive attachment
2013
must be filed before the trial or before appeal is
perfected or before the judgment becomes Facts:
executory, with due notice to the attaching
On August 22, 2005, Lim, Jr. filed a
party and his surety or sureties setting forth the
complaint for sum of money with prayer for the
facts showing his right to damages and the
issuance of a writ of preliminary attachment
amount thereof. Such damages may be
before the RTC, seeking to recover from
awarded only after proper hearing and shall be
respondents-spouses Tito S. Lazaro and Carmen
included in the judgment on the main case.
T. Lazaro (Sps. Lazaro) the sum of
If the judgment of the appellate court be P2,160,000.00, which represented the amounts
favorable to the party against whom the stated in several dishonored checks issued by
attachment was issued he must claim damages the latter to the former, as well as interests,
sustained during the pendency of the appeal by attorney’s fees, and costs. The RTC granted the
filing an application in the appellate court, with writ of preliminary attachment application and
notice to the party in whose favor the upon the posting of the required P2,160,000.00
attachment was issued or his surety or sureties, bond, issued the corresponding writ on October
before the judgment of the appellate court 14, 2005.
becomes executory. The appellate court may
On September 22, 2006, the parties entered
allow the application to be heard and decided
into a Compromise Agreement whereby Sps.
by the trial court.
Lazaro agreed to pay Lim, Jr. the amount of

Rolan Jeff A. Lancion


Arellano University School of Law Page 6
Remedial Law 2- Brondial Notes AY 2017- 2018

P2,351,064.80 on an installment basis, following upon the relief sought and expected to be
a schedule of payments covering the period granted in the main or principal action; it is a
from September 2006 until October 2013, measure auxiliary or incidental to the main
under the following terms, among others: (a) action. As such, it is available during its
that should the financial condition of Sps. pendency which may be resorted to by a litigant
Lazaro improve, the monthly installments shall to preserve and protect certain rights and
be increased in order to hasten the full payment interests during the interim, awaiting the
of the entire obligation; and (b) that Sps. ultimate effects of a final judgment in the case.
Lazaro’s failure to pay any installment due or
In this relation, while the provisions of Rule 57
the dishonor of any of the postdated checks
are silent on the length of time within which an
delivered in payment thereof shall make the
attachment lien shall continue to subsist after
whole obligation immediately due and
the rendition of a final judgment, jurisprudence
demandable.
dictates that the said lien continues until the
Sps. Lazaro filed an Omnibus Motion, seeking to debt is paid, or the sale is had under execution
lift the writ of preliminary attachment issued on the judgment or until the judgment
annotated on the subject TCTs, which the RTC is satisfied, or the attachment discharged or
granted on March 29, 2007. It ruled that a writ vacated in the same manner provided by law.
of preliminary attachment is a mere provisional Records indicate that while the parties have
or ancillary remedy, resorted to by a litigant to entered into a compromise agreement which
protect and preserve certain rights and had already been approved by the RTC in its
interests pending final judgment. Considering January 5, 2007 Amended Decision, the
that the case had already been considered obligations thereunder have yet to be fully
closed and terminated by the rendition of the complied with – particularly, the payment of
January 5, 2007 Amended Decision on the basis the total compromise amount of P2,351,064.80.
of the September 22, 2006 compromise Hence, given that the foregoing debt remains
agreement, the writ of preliminary attachment unpaid, the attachment of Sps. Lazaro’s
should be lifted and quashed. properties should have continued to subsist.

Issue: WON the preliminary attachment was The Court holds that the writ of preliminary
properly lifted? attachment subject of this case should be
restored and its annotation revived in the
Held:
subject TCTs, re-vesting unto Lim, Jr. his
By its nature, preliminary attachment, under preferential lien over the properties covered by
Rule 57 of the Rules of Court (Rule 57), is an the same as it were before the cancellation of
ancillary remedy applied for not for its own sake the said writ. Lest it be misunderstood, the lien
but to enable the attaching party to realize or security obtained by an attachment even

Rolan Jeff A. Lancion


Arellano University School of Law Page 7
Remedial Law 2- Brondial Notes AY 2017- 2018

before judgment, is in the nature of a vested On February 18, 2003, a similar complaint for
interest which affords specific security for the collection of sum of money, damages, and
satisfaction of the debt put in suit. cancellation of title with prayer for issuance of a
writ of preliminary attachment was lodged
LETICIA P. LIGON, Petitioner, v. THE REGIONAL
before the Makati City RTC, docketed as Civil
TRIAL COURT, BRANCH 56 AT MAKATI CITY
Case No. 03–186 (Makati City Case), by Spouses
AND ITS PRESIDING JUDGE, JUDGE REYNALDO
Cecilia and Gil Vicente (Sps. Vicente) against
M. LAIGO, SHERIFF IV LUCITO V. ALEJO, ATTY.
Sps. Baladjay, Polished Arrow, and other
SILVERIO GARING, MR. LEONARDO J. TING,
corporations. In that case, it was established
AND MR. BENITO G. TECHICO, Respondents.
that Sps. Baladjay solicited millions of pesos in
G.R. No. 190028, February 26, 2014 investments from Sps. Vicente using conduit
Facts: companies that were controlled by Rosario, as
President and Chairperson. During the
On November 20, 2002, petitioner Leticia P. proceedings therein, a writ of preliminary
Ligon (Ligon) filed an amended complaint attachment also against the subject property
before the Regional Trial Court of Quezon City, was issued and annotated on the dorsal
Branch 101 (Quezon City RTC) for collection of portion of TCT No. 9273 on March 12, 2003.
sum of money and damages, rescission of
contract, and nullification of title with prayer On September 25, 2008, the March 26, 2008
for the issuance of a writ of preliminary Decision of the Quezon City RTC became final
attachment, docketed as Civil Case No. Q–10– and executory. However, when Ligon sought its
48145 (Quezon City Case), against Sps. Baladjay, execution, she discovered that the December 3,
a certain Olivia Marasigan (Marasigan), Polished 2002 attachment annotation had been deleted
Arrow Holdings, Inc. (Polished Arrow), and its from TCT No. 9273 when the subject property
incorporators, namely, Spouses Julius Gonzalo was sold by way of public auction on September
and Charaine Doreece Anne Fuentebella (Sps. 9, 2005 to the highest bidder, respondent Ting,
Fuentebella), Ma. Linda Mendoza (Mendoza), for the amount of P9,000,000.00 during the
Barbara C. Clavo (Clavo), Bayani E. Arit, Jr. (Arit, execution proceedings in the Makati City Case,
Jr.), and Peter M. Kairuz (Kairuz), as well as the as evidenced by the Officer’s Final Deed of
latter’s spouses (individual defendants). Sale dated October 27, 2006 (Officer’s Final
Deed of Sale) issued by Sheriff Alejo. Atty.
In her complaint, Ligon alleged, inter alia, that Garing manifested before the Makati City RTC
Rosario Baladjay (Rosario) enticed her to extend that it submitted the matter en consulta to the
a short–term loan in the amount of Land Registration Authority (LRA) as he was
P3,000,000.00, payable in a month’s time and uncertain whether the annotations on TCT No.
secured by an Allied Bank post–dated check for 9273 should be carried over to TCT No. 8502.
the same amount.

Rolan Jeff A. Lancion


Arellano University School of Law Page 8
Remedial Law 2- Brondial Notes AY 2017- 2018

In a Decision dated October 30, 2009, the CA been duly levied upon a property, a purchaser
dismissed Ligon’s certiorari petition, finding that thereof subsequent to the attachment takes the
the Makati City RTC did not gravely abuse its property subject to the said attachment.
discretion in issuing the Assailed Orders, adding
in order to secure the satisfaction of a favorable
further that the same was tantamount to a
judgment in the Quezon City Case, applied for
collateral attack against the titles of both Ting
and was eventually able to secure a writ of
and Techico, which is prohibited under Section
preliminary attachment over the subject
48 of Presidential Decree No. (PD) 1529.
property on November 25, 2002, which was
Issue: WON Makati City Judge erred in its later annotated on the dorsal portion of TCT No.
issuance of Preliminary Attachment 9273 in the name of Polished Arrow on
December 3, 2002. Notwithstanding the
HELD:
subsequent cancellation of TCT No. 9273 due to
YES- Attachment is defined as a provisional the Makati City RTC’s December 9, 2004
remedy by which the property of an adverse Decision rescinding the transfer of the subject
party is taken into legal custody, either at the property from Sps. Baladjay to Polished Arrow
commencement of an action or at any time upon a finding that the same was made in fraud
thereafter, as a security for the satisfaction of of creditors, Ligon’s attachment lien over the
any judgment that may be recovered by the subject property continued to subsist since the
plaintiff or any proper party. Case law instructs attachment she had earlier secured binds the
that an attachment is a proceeding in rem, and, property itself, and, hence, continues until the
hence, is against the particular property, judgment debt of Sps. Baladjay to Ligon as
enforceable against the whole world. adjudged in the Quezon City Case is satisfied, or
Accordingly, the attaching creditor acquires a the attachment discharged or vacated in some
specific lien on the attached property which manner provided by law.
nothing can subsequently destroy except the
very dissolution of the attachment or levy itself.
Such a proceeding, in effect, means that the
property attached is an indebted thing and a
Alejandro Ng Wee vs Tankiansee, G.R. No.
virtual condemnation of it to pay the owner’s
171124, Feb 13, 2008
debt. The lien continues until the debt is paid,
or sale is had under execution issued on the
judgment, or until the judgment is satisfied, or Facts:
the attachment discharged or vacated in some
manner provided by law. Thus, a prior Sometime in February 2000, petitioner received
registration of an attachment lien creates a disturbing news on Wincorps financial condition
preference, such that when an attachment has prompting him to inquire about and investigate

Rolan Jeff A. Lancion


Arellano University School of Law Page 9
Remedial Law 2- Brondial Notes AY 2017- 2018

the companys operations and transactions with 2001, the RTC, in an Omnibus Order, denied all
its borrowers. He then discovered that the the motions for the discharge of the
company extended a loan equal to his total attachment. The defendants, including
money placement to a corporation [Power respondent herein, filed their respective
Merge] with a subscribed capital of motions for reconsideration but the trial court
only P37.5M. This credit facility originated from denied the same on October 14, 2002.
another loan of about P1.5B extended by
On September 30, 2004, respondent filed
Wincorp to another corporation [Hottick
before the trial court another Motion to
Holdings]. When the latter defaulted in its
Discharge Attachment, re-pleading the grounds
obligation, Wincorp instituted a case against it
he raised in his first motion but raising the
and its surety. Settlement was, however,
following additional grounds: (1) that he was
reached in which Hotticks president, Luis Juan L.
not present in Wincorps board meetings
Virata (Virata), assumed the obligation of the
approving the questionable transactions; and
surety.
(2) that he could not have connived with
Wincorp and the other defendants because he
and Pearlbank Securities, Inc., in which he is a
Under the scheme agreed upon by Wincorp and
major stockholder, filed cases against the
Hotticks president, petitioners money
company as they were also victimized by its
placements were transferred without his
fraudulent schemes.
knowledge and consent to the loan account of
Power Merge through an agreement that Issue: WON there was irregularity in the
virtually freed the latter of any liability. issuance of writ of preliminary attachment?
Allegedly, through the false representations of
Held:
Wincorp and its officers and directors,
petitioner was enticed to roll over his A writ of attachment can only be granted on
placements so that Wincorp could loan the concrete and specific grounds and not on
same to Virata/Power Merge. general averments quoting perfunctorily the
words of the Rules. Connivance cannot also be
On October 26, 2000, on the basis of the
based on mere association but must be
allegations in the complaint and the October 12,
particularly alleged and established as a fact.
2000 Affidavit of petitioner, the trial court
Respondent further contends that the trial
ordered the issuance of a writ of preliminary
court, in resolving the Motion to Discharge
attachment against the properties not exempt
Attachment, need not actually delve into the
from execution of all the defendants in the civil
merits of the case. All that the court has to
case subject, among others, to petitioners filing
examine are the allegations in the complaint
of a P50M-bond. The writ was, consequently,
and the supporting affidavit. Petitioner cannot
issued on November 6, 2000. On October 23,

Rolan Jeff A. Lancion


Arellano University School of Law Page 10
Remedial Law 2- Brondial Notes AY 2017- 2018

also rely on the decisions of the appellate court not to pay, or without any showing of how
in CA-G.R. SP No. 74610 and this Court in G.R. respondent committed the alleged fraud, the
No. 162928 to support his claim because general averment in the affidavit that
respondent is not a party to the said cases. respondent is an officer and director of Wincorp
who allegedly connived with the other
For a writ of attachment to issue under this
defendants to commit a fraud, is insufficient to
rule, the applicant must sufficiently show the
support the issuance of a writ of preliminary
factual circumstances of the alleged fraud
attachment.
because fraudulent intent cannot be inferred
from the debtors mere non-payment of the Phil-Air Conditioning Center vs RCJ Lines and
debt or failure to comply with his obligation. Rolando Abadilla, Jr., G.R. No. 193821,
November 23, 2015
“To sustain an attachment on this ground, it
must be shown that the debtor in contracting
the debt or incurring the obligation intended to
Facts:
defraud the creditor. The fraud must relate to
the execution of the agreement and must have On various dates between March 5, 1990, and
been the reason which induced the other party August 29, 1990, petitioner Phil-Air sold to
into giving consent which he would not have respondent RCJ Lines four Carrier Paris 240 air
otherwise given. To constitute a ground for conditioning units for buses (units). The units
attachment in Section 1 (d), Rule 57 of the Rules included compressors, condensers,
of Court, fraud should be committed upon evaporators, switches, wiring, circuit boards,
contracting the obligation sued upon. A debt is brackets, and fittings.
fraudulently contracted if at the time of
contracting it the debtor has a preconceived
plan or intention not to pay, as it is in this case. Phil-Air allegedly performed regular
Fraud is a state of mind and need not be proved maintenance checks on the units pursuant to
by direct evidence but may be inferred from the the one-year warranty on parts and labor. RCJ
circumstances attendant in each case.” Lines issued three post-dated checks in favor of
Phil-Air to partly cover the unpaid balance.
The affidavit, being the foundation of the
writ, must contain such particulars as to how
the fraud imputed to respondent was All the post-dated checks were dishonored
committed for the court to decide whether or when Phil-Air subsequently presented them for
not to issue the writ. Absent any statement of payment. Check No. 479759 was returned
other factual circumstances to show that because it was drawn against insufficient funds,
respondent, at the time of contracting the while Check Nos. 479760 and 479761 were
obligation, had a preconceived plan or intention returned because payments were stopped.

Rolan Jeff A. Lancion


Arellano University School of Law Page 11
Remedial Law 2- Brondial Notes AY 2017- 2018

Issue: WON Phil-Air should reimburse RCJ Lines


for the counter-bond premium and its alleged
Before presenting the third check for payment,
unrealized profits
Phil-Air sent a demand letter to Rolando
Abadilla, Sr. asking him to fund the post-dated Held:
checks. In view of the failure of RCJ Lines to pay
A writ of preliminary attachment is a provisional
the balance despite demand, Phil-Air filed on
remedy issued by the court where an action is
April 1, 1998 the complaint for sum of money
pending to be levied upon the property or
with prayer for the issuance of a writ of
properties of the defendant. The property is
preliminary attachment.
held by the sheriff as security for the
satisfaction of whatever judgment that might
be secured by the attaching party against the
In its answer with compulsory counterclaim, RCJ
defendant.
Lines admitted that it purchased the units in the
total amount of P1,240,000.00 and that it had
only paid P400,000.00. It refused to pay the
The grant of the writ is conditioned not only on
balance because Phil-Air allegedly breached its
the finding of the court that there exists a valid
warranty.
ground for its issuance. The Rules also require
the applicant to post a bond.

RCJ Lines averred that the units did not


sufficiently cool the buses despite repeated
Section 4 of Rule 57 of the Rules of Civil
repairs. Phil-Air purportedly represented that
Procedure (Rules) provides that “the party
the units were in accord with RCJ Lines’ cooling
applying for the order must…give a bond
requirements as shown in Phil-Air’s price
executed to the adverse party in the amount
quotation. The price quotation provided that
fixed by the court in its order granting the
full payment should be made upon the units’
issuance of the writ, conditioned that the latter
complete installation. Complete installation,
will pay all the costs that may be adjudged to
according to RCJ Lines, is equivalent to being in
the adverse party and all damages that he may
operational condition.
sustain by reason of the attachment, if the
court shall finally adjudge that the applicant
was not entitled thereto.”
RCJ Lines claimed that it was also entitled to be
reimbursed for costs and damages occasioned
by the enforcement of the writ of attachment.
The enforcement of the writ notwithstanding,
the party whose property is attached is
afforded relief to have the attachment lifted.

Rolan Jeff A. Lancion


Arellano University School of Law Page 12
Remedial Law 2- Brondial Notes AY 2017- 2018

There are various modes of discharging an cash, and after hearing to determine the
attachment under Rule 57, viz.: sufficiency of the cash deposit or counter-bond.
On the other hand, the discharge under Section
(1) by depositing cash or posting a counter-
13 takes effect only upon showing that the
bond under Section 12;
plaintiff’s attachment bond was improperly or
(2) by proving that the attachment bond was irregularly issued, or that the bond is
improperly or irregularly issued or enforced, or insufficient. The discharge of the attachment
that the bond is insufficient under Section 13; under Section 13 must be made only after
(3) by showing that the attachment is excessive hearing.
under Section 13; and (4) by claiming that the
property is exempt from execution under
As discussed above, it is patent that under the
Section 2.
Rules, the attachment bond answers for all
damages incurred by the party against whom
RCJ Lines availed of the first mode by posting a the attachment was issued. Thus, Phil-Air
counter-bond. cannot be held directly liable for the costs
adjudged to and the damages sustained by RCJ
Lines because of the attachment. Section 4 of
Under the first mode, the court will order the Rule 57 positively lays down the rule that the
discharge of the attachment after (1) the attachment bond will pay “all the costs which
movant makes a cash deposit or posts a may be adjudged to the adverse party and all
counterbond and (2) the court hears the motion damages which he may sustain by reason of
to discharge the attachment with due notice to the attachment, if the court shall finally
the adverse party. adjudge that the applicant was not entitled
thereto.”

The amount of the cash deposit or counter-


bond must be equal to that fixed by the court in The RTC, instead of declaring Phil-Air liable for
the order of attachment, exclusive of costs. The the alleged unrealized profits and counter-bond
cash deposit or counter-bond shall secure the premium, should have ordered the execution of
payment of any judgment that the attaching the judgment award on the attachment bond.
party may recover in the action. To impose direct liability to Phil-Air would
defeat the purpose of the attachment bond,
which was not dissolved despite the lifting of
The discharge under Section 12 takes effect the writ of preliminary attachment.
upon posting of a counter-bond or depositing

Rolan Jeff A. Lancion


Arellano University School of Law Page 13
Remedial Law 2- Brondial Notes AY 2017- 2018

The order to refund the counter-bond premium Exception: Joinder of Action- If a party cause of
is likewise erroneous. The premium payment action with injunction it will depend to which
may be deemed a cost incurred by RCJ Lines to court has jurisdiction
lift the attachment. Such cost may be charged
-If injunction is filed with the Supreme Court, it
against the attachment bond.
is not solely filed in that court but with another
cause of action

Rule 58- Preliminary Injunction Kinds:

Principal Action- Injunction Mandatory- It seeks to return to the status quo


and is applicable to continuing acts. (Ibabalik sa
Provisional Remedy- Injunction
dati)
Sec 1- Definition
Prohibitory- It seeks to maintain the status quo
A preliminary injunction is an order granted at or no disturbance. (Walang gagalawin)
any stage of an action or proceeding prior to
GR: Preliminary Injunction cannot be granted
the judgment or final order, requiring a party or
without notice and hearing
a court, agency or a person to refrain from a
particular act or acts. It may also require the Temporary Restraining Order (TRO)
performance of a particular act or acts, in which
-The court can grant such TRO ex-parte
case it shall be known as a preliminary
mandatory injunction. Kinds:

Sec 2- Who may grant the Preliminary 72 hrs TRO- It takes effect upon issuance of an
Injunction order

A preliminary injunction may be granted by the 20 day TRO- It takes effect upon receipt of the
court where the action or proceeding is party
pending. If the action or proceeding is pending -The court is not precluded in setting for a
in the Court of Appeals or in the Supreme summary hearing in the issuance of TRO
Court, it may be issued by said court or any
member thereof. - The grant of TRO is discretionary upon the
judge who will issue it.
Q: Which court has jurisdiction to grant
Preliminary Injunction? -If a court grants a 72 hour TRO and the party
would like to extend it, the court shall only
A: Only Cognizable by the Regional Trial Court grant an extension of 17 more days and it shall
(Original & Exclusive) begin to run from the receipt of the order
granting the TRO.

Rolan Jeff A. Lancion


Arellano University School of Law Page 14
Remedial Law 2- Brondial Notes AY 2017- 2018

Period for TRO: amount of damages to be awarded to either


party, upon the bond of the adverse party, shall
Sandiganbayan/Court of Appeals- 60 days
be claimed, ascertained, and awarded under
Supreme Court—Upon further notice the same procedure prescribed in section 20 of
Not Covered of TRO Rule 57.

-Infrastructure project of the government Cases:

-Prosecution of criminal offense TERESITA V. IDOLOR, petitioner, vs. HON.


COURT OF APPEALS, SPS. GUMERSINDO DE
-Labor Cases GUZMAN and ILUMINADA DE GUZMAN and
-Execution of judgment as a matter of right HON. PRUDENCIO CASTILLO, JR., Presiding
Judge, Regional Trial Court, National Capital
Judicial Region, Branch 220, Quezon
Section 3. Grounds for issuance of preliminary City, respondents., [G.R. No. 141853. February
injunction. 7, 2001]

(a) That the applicant is entitled to the relief Facts:


demanded, and the whole or part of such relief
On March 21, 1994, to secure a loan of
consists in restraining the commission or
P520,000.00, petitioner Teresita Idolor
continuance of the act or acts complained of, or
executed in favor of private respondent
in requiring the performance of an act or acts
Gumersindo De Guzman a Deed of Real
either for a limited period or perpetually;
Estate Mortgage with right of extra-judicial
(b) That the commission, continuance or non- foreclosure upon failure to redeem the
performance of the act or acts complained of mortgage on or before September 20,
during the litigation would probably work 1994. The object of said mortgage is a 200-
injustice to the applicant; or square meter property with improvements
located at 66 Ilocos Sur Street, Barangay Ramon
(c) That a party, court, agency or a person is
Magsaysay, Quezon City covered by TCT No.
doing, threatening, or is attempting to do, or is
25659. On September 21, 1996, private
procuring or suffering to be done some act or
respondent Iluminada de Guzman, wife of
acts probably in violation of the rights of the
Gumersindo de Guzman, filed a complaint
applicant respecting the subject of the action or
against petitioner Idolor before the Office of the
proceeding, and tending to render the
Barangay Captain of Barangay Ramon
judgment ineffectual.
Magsaysay, Quezon City, which resulted in a
Section 8. Judgment to include damages Kasunduang Pag-aayos which agreement is
against party and sureties. — At the trial, the quoted in full.

Rolan Jeff A. Lancion


Arellano University School of Law Page 15
Remedial Law 2- Brondial Notes AY 2017- 2018

On May 23, 1997, the mortgaged property was Issue: WON the Injunction filed by Idolor is
sold in a public auction to respondent proper on the ground that he has still a right
Gumersindo, as the highest bidder and over the property
consequently, the Sheriffs Certificate of Sale
Held:
was registered with the Registry of Deeds of
Quezon City on June 23, 1997. Injunction is a preservative remedy aimed at
protecting substantive rights and interests.
On June 25, 1998, petitioner filed with the
Before an injunction can be issued, it is
Regional Trial Court of Quezon City, Branch 220,
essential that the following requisites be
a complaint for annulment of Sheriffs
present: 1) there must be a right in esse or the
Certificate of Sale with prayer for the issuance
existence of a right to be protected; 2) the act
of a temporary restraining order (TRO) and a
against which the injunction is to be directed is
writ of preliminary injunction against private
a violation of such right. The existence of a right
respondents, Deputy Sheriffs Marino Cachero
violated, is a prerequisite to the granting of an
and Rodolfo Lescano and the Registry of Deeds
injunction. Injunction is not designed to protect
of Quezon City alleging among others alleged
contingent or future rights. Failure to establish
irregularity and lack of notice in the extra-
either the existence of a clear and positive right
judicial foreclosure proceedings subject of
which should be judicially protected through
the real estate mortgage. In the meantime, a
the writ of injunction or that the defendant has
temporary restraining order was issued by the
committed or has attempted to commit any act
trial court.
which has endangered or tends to endanger the
On July 28, 1998, the trial court issued a writ of existence of said right, is a sufficient ground for
preliminary injunction enjoining private denying the injunction. It is to be resorted to
respondents, the Deputy Sheriffs and the only when there is a pressing necessity to avoid
Registry of Deeds of Quezon City from causing injurious consequences which cannot be
the issuance of a final deed of sale and remedied under any standard of compensation.
consolidation of ownership of the subject
In the instant case, we agree with the
property in favor of the De Guzman spouses.
respondent Court that petitioner has no more
The trial court denied the motion for
proprietary right to speak of over the foreclosed
reconsideration filed by the de Guzman
property to entitle her to the issuance of a writ
spouses.
of injunction. It appears that the mortgaged
On September 28, 1999, the respondent court property was sold in a public auction to private
granted the petition and annulled the assailed respondent Gumersindo on May 23, 1997 and
writ of preliminary injunction. Teresita Idolor the sheriffs certificate of sale was registered
filed her motion for reconsideration which was with the Registry of Deeds of Quezon City on
denied in a resolution dated February 4, 2000. June 23, 1997. Petitioner had one year from the

Rolan Jeff A. Lancion


Arellano University School of Law Page 16
Remedial Law 2- Brondial Notes AY 2017- 2018

registration of the sheriffs sale to redeem the On May 20, 1997, his opponent filed an election
property but she failed to exercise her right on protest case, docketed as Civil Case No. 703-M,
or before June 23, 1998, thus spouses de before the MCTC of Victorias-Manapla, Negros
Guzman are now entitled to a conveyance and Occidental. Libo-on sought the recounting of
possession of the foreclosed property. ballots in two precincts, preliminary prohibitory
injunction, and damages. On May 29, 1997,
RIMEO S. GUSTILO, complainant, vs. HON.
respondent judge issued a temporary
RICARDO S. REAL, SR., Presiding Judge, 2nd
restraining order (TRO) and annulled the
Municipal Circuit Trial Court of Victorias-
proclamation of complainant as the duly
Manapla, Negros Occidental, respondent.,
elected punong barangay of Punta Mesa,
A.M. No. MTJ-00-1250. February 28, 2001
Manapla.
Facts:
On May 30, 1997, complainant took his oath of
In a verified complaintdated June 15, 1997, office as punong barangay. That same day, he
Rimeo S. Gustilo charged respondent Judge also filed a petition for certiorari before the
Ricardo S. Real, Sr., of the Municipal Circuit Trial Regional Trial Court of Silay City, Negros
Court of Victorias-Manapla, Negros Occidental Occidental, Branch 69. On June 5, 1997, the RTC
with gross misconduct, gross incompetence, lifted the TRO issued by respondent and
gross ignorance of the law, and violation of the declared as null and void the order nullifying
Anti-Graft and Corrupt Practices Act relative to complainants proclamation as duly
Civil Case No. 703-M entitled Weddy C. Libo-on elected punong barangay.
v. Rimeo S. Gustilo, et al. for recounting of
Issue: WON Judge Real erred in his issuance of
ballots of Precinct Nos. 27 and 27-A, Barangay
Temporary Restraining Order
Punta Mesa, Manapla, Negros Occidental.
Complainant avers that he was a candidate Held:
for punong barangay of Barangay Punta Mesa,
Supreme Court Administrative Circular No. 20-
Manapla, Negros Occidental in the May 12,
95 provides: 2. The application for a TRO shall
1997 elections. His lone opponent was Weddy
be acted upon only after all parties are heard in
C. Libo-on, then the incumbent punong
a summary hearing conducted within twenty-
barangay and the representative of the
four (24) hours after the records are
Association of Barangay Captains (ABC) to
transmitted to the branch selected by raffle.
the Sangguniang Bayan of Manapla and
The records shall be transmitted immediately
the Sangguniang Panlalawigan of Negros
after raffle.
Occidental. Both complainant and Libo-on
garnered eight hundred nineteen (819) votes The foregoing clearly show that whenever an
during the elections, resulting in a tie. application for a TRO is filed, the court may act
on the application only after all parties have

Rolan Jeff A. Lancion


Arellano University School of Law Page 17
Remedial Law 2- Brondial Notes AY 2017- 2018

been notified and heard in a summary hearing. Facts:


In other words, a summary hearing may not be
Michael J. Lagrosas was employed by Bristol-
dispensed with. In the instant case, respondent
Myers Squibb (Phil.), Inc./Mead Johnson Phil.
admits that he issued the injunctive writ sought
from January 6, 1997 until March 23, 2000 as
on May 29, 1997 after receiving the applicants
Territory Manager in its Medical Sales Force
evidence ex parte. His failure to abide by
Division. On February 4, 2000, Ma. Dulcinea S.
Administrative Circular No. 20-95 in issuing the
Lim, also a Territory Manager and Lagrosas
first TRO is grave abuse of authority,
former girlfriend, attended a district meeting of
misconduct, and conduct prejudicial to the
territory managers at
proper administration of justice.
McDonalds Alabang Town Center. After the
A judge is expected to know the jurisdictional meeting, she dined out with her friends. She left
boundaries of courts and quasi-judicial bodies her car at McDonalds and rode with Cesar R.
like the COMELEC as mapped out by the Menquito, Jr. When they returned to
Constitution and statutes and to act only within McDonalds, Lim saw Lagrosas car parked beside
said limits. A judge who wantonly arrogates her car. Lim told Menquito not to stop his car
unto himself the authority and power vested in but Lagrosas followed them and slammed
other agencies not only acts in oppressive Menquitos car thrice. Menquito and Lim
disregard of the basic requirements of due alighted from the car. Lagrosas approached
process, but also creates chaos and contributes them and hit Menquito with a metal steering
to confusion in the administration of justice. wheel lock. When Lim tried to intervene,
Respondent, in transgressing the jurisdictional Lagrosas accidentally hit her head.
demarcation lines between his court and the
Upon learning of the incident, Bristol-Myers
COMELEC, clearly failed to realize the position
required Lagrosas to explain in writing why he
that his court occupies in the interrelation and
should not be dismissed for assaulting a co-
operation of the countrys justice system. He
employee outside of business hours. While the
displayed a marked ignorance of basic laws and
offense is not covered by the Code of Discipline
principles.
for Territory Managers, the Code states that
other infractions not provided for herein shall
be penalized in the most appropriate manner at
Lagrosas vs Bristol-Myers Squib (Phil), G.R. No.
the discretion of management.
168637 and G.R. No. 170684, September 12,
2008 On March 23, 2000, Bristol-Myers dismissed
Lagrosas effective immediately. Lagrosas then
Doctrine: The bond which was posted has no
filed a complaint for illegal dismissal, non-
more legal basis considering that the principal
payment of vacation and sick leave benefits,
action was already dismissed
13th month pay, attorney’s fees, damages and

Rolan Jeff A. Lancion


Arellano University School of Law Page 18
Remedial Law 2- Brondial Notes AY 2017- 2018

fair market value of his Team Share Stock Issue: WON the Court of Appeals erred in
Option Grant. disallowing the discharge and release of the
injunction cash bond
On February 28, 2002, Labor Arbiter Renaldo O.
Hernandez rendered a Decision declaring the Held:
dismissal illegal. He noted that while Lagrosas
A preliminary injunction may be granted only
committed a misconduct, it was not connected
when, among other things, the applicant, not
with his work. On appeal, the National Labor
explicitly exempted, files with the court where
Relations Commission (NLRC) set aside the
the action or proceeding is pending,
Decision of Labor Arbiter Hernandez in its
a bond executed to the party or person
Decision dated September 24, 2002. It held that
enjoined, in an amount to be fixed by the court,
Lagrosas was validly dismissed for serious
to the effect that the applicant will pay such
misconduct in hitting his co-employee and
party or person all damages which he may
another person with a metal steering wheel
sustain by reason of the injunction or
lock. The gravity and seriousness of his
temporary restraining order if the court should
misconduct is clear from the fact that he
finally decide that the applicant was not
deliberately waited for Lim and Menquito to
entitled thereto. Upon approval of the requisite
return to McDonalds.
bond, a writ of preliminary injunction shall be
Bristol-Myers moved to quash the writ of issued.
execution contending that it timely filed a
The injunction bond is intended as a security for
petition for certiorari with the Court of
damages in case it is finally decided that the
Appeals. The appellate court gave due course to
injunction ought not to have been granted. Its
Bristol-Myers petition and issued a temporary
principal purpose is to protect the enjoined
restraining order (TRO) enjoining the
party against loss or damage by reason of the
enforcement of the writ of execution and
injunction, and the bond is usually conditioned
notices of garnishment. Upon the expiration of
accordingly.
the TRO, the appellate court issued a writ of
preliminary injunction dated September 17, By its Decision dated January 28, 2005, the
2004. appellate court disposed of the case by granting
Bristol-Myers petition and reinstating the
Bristol-Myers moved to release the TRO cash
Decision dated September 24, 2002of the NLRC
bond and injunction cash bond in view of the
which dismissed the complaint for dismissal. It
Decision dated January 28, 2005. On August 12,
also ordered the discharge of the TRO cash
2005, the appellate court denied the motion as
bond and injunction cash bond. Thus, both
premature since the decision is not yet final and
conditions of the writ of preliminary injunction
executory due to Lagrosas appeal to this Court.
were satisfied.

Rolan Jeff A. Lancion


Arellano University School of Law Page 19
Remedial Law 2- Brondial Notes AY 2017- 2018

Jenosa vs Delariarte, 630 SCRA On 3 January 2003, petitioners filed a complaint


for injunction and damages with the Regional
Doctrine: A party should come to court with
Trial Court, Branch 29, Iloilo City (trial court)
clean hands including the application for a writ
docketed as Civil Case No. 03-27460. Petitioners
of preliminary injunction
assailed the Principals decision to order the
Facts: immediate transfer of petitioner students as a
On 22 November 2002, some students of the violation of their right to due process because
University, among them petitioners Nio Carlo the COSD was not convened. On 5 February
Jenosa, Patrick Canto, Cyndy Apalisok, Clint 2003, the trial court issued a writ of preliminary
Eduard Vargas, and Nonell Gregory Duro injunction and directed respondents to admit
(petitioner students), were caught engaging in petitioner students during the pendency of the
hazing outside the school premises. The hazing case.
incident was entered into the blotter of the The Court of Appeals granted respondents
Iloilo City Police. petition and ordered the trial court to dismiss
During the 28 November 2002 meeting, the Civil Case Nos. 03-27460 and 03-27646 for lack
parties agreed that, instead of the possibility of of jurisdiction over the subject matter because
being charged and found guilty of hazing, the of petitioners failure to exhaust administrative
students who participated in the hazing incident remedies or for being premature. According to
as initiators, including petitioner students, the Court of Appeals, petitioners should have
would just transfer to another school, while waited for the action of the DepEd or of the
those who participated as neophytes would be University President before resorting to judicial
suspended for one month. The parents of the action.
apprehended students, including petitioners, Issue: WON the writ of preliminary injunction is
affixed their signatures to the minutes of the improperly issued
meeting to signify their conformity.
Held:
On 10 December 2002, petitioner parents also
Since injunction is the strong arm of equity, he
wrote a letter to Mrs. Ida B. Endonila, School
who must apply for it must come with equity or
Division Superintendent, Department of
with clean hands. This is so because among the
Education (DepEd), Iloilo City, seeking her
maxims of equity are (1) he who seeks equity
intervention and prayed that petitioner
must do equity, and (2) he who comes into
students be allowed to take the home study
equity must come with clean hands. The latter
program instead of transferring to another
is a frequently stated maxim which is also
school. The DepEd asked the University to
expressed in the principle that he who has done
comment on the letter.
inequity shall not have equity. It signifies that a

Rolan Jeff A. Lancion


Arellano University School of Law Page 20
Remedial Law 2- Brondial Notes AY 2017- 2018

litigant may be denied relief by a court of equity On October 5, 2000, claiming that the interests,
on the ground that his conduct has been penalties and charges imposed by CBC were
inequitable, unfair and dishonest, or fraudulent, iniquitous and unconscionable and to enjoin
or deceitful as to the controversy in issue. CBC from initiating foreclosure proceedings, SBI
and MFII filed a Complaint "To Compel
Petitioners, having reneged on their agreement
Execution of Contract and for Performance and
without any justifiable reason, come to court
Damages, With Prayer for Writ of Preliminary
with unclean hands. This Court may deny a
Injunction and Ex-Parte Temporary Restraining
litigant relief if his conduct has been
Order" in the Regional Trial Court (RTC) of Pasig
inequitable, unfair and dishonest as to the
City. The case was docketed as Civil Case No.
controversy in issue.
68105 and assigned to Branch 264. After
SOLID BUILDERS, INC. and MEDINA FOODS hearing the parties, the trial court issued an
INDUSTRIES, INC., Petitioners, Order dated December 14, 2000 granting the
vs. CHINA BANKING application of SBI and MFII for the issuance of a
CORPORATION, Respondent. G.R. No. 179665 writ of preliminary injunction. The trial court
April 3, 2013 held that SBI and MFII were able to sufficiently
Facts: comply with the requisites for the issuance of
an injunctive writ:
During the period from September 4, 1992 to
March 27, 1996, China Banking Corporation It is well-settled that to be entitled to an
(CBC) granted several loans to Solid Builders, injunctive writ, a party must show that: (1) the
Inc. (SBI), which amounted to ₱139,999,234.34, invasion of right sought to be protected is
exclusive of interests and other charges. To material and substantial; (2) the right of
secure the loans, Medina Foods Industries, Inc. complainant is clear and unmistakable; and, (3)
(MFII) executed in CBC’s favor several surety there is an urgent and paramount necessity for
agreements and contracts of real estate the writ to prevent serious damage.
mortgage over parcels of land in the Loyola The Court of Appeals found that, on its face, the
Grand Villas in Quezon City and New Cubao trial court’s Order dated December 14, 2000
Central in Cainta, Rizal. granting the application of SBI and MFII for the
In a letter dated March 20, 2000 addressed to issuance of a writ of preliminary injunction had
CBC, SBI requested the restructuring of its no basis as there were no findings of fact or law
loans, a reduction of interests and penalties and which would indicate the existence of any of
the implementation of a dacion en pago of the the requisites for the grant of an injunctive writ.
New Cubao Central property. Issue: WON the issuance of writ of preliminary
injunction is proper?

Rolan Jeff A. Lancion


Arellano University School of Law Page 21
Remedial Law 2- Brondial Notes AY 2017- 2018

Held: petitioners and render the whole case (sans


trial on the merits) moot and academic. Second,
A preliminary injunction is an order granted at
even the Order dated December 14, 2000 of the
any stage of an action prior to judgment of final
trial court, which granted the application for the
order, requiring a party, court, agency, or
issuance of a writ of preliminary injunction,
person to refrain from a particular act or acts. It
recognizes that the parties still have to be heard
is a preservative remedy to ensure the
on the alleged lack of "fairness of the increase
protection of a party’s substantive rights or
in interests and penalties" during the trial on
interests pending the final judgment in the
the merits.
principal action. A plea for an injunctive writ lies
upon the existence of a claimed emergency or It was likewise shown that plaintiffs SBI and
extraordinary situation which should be MFII had the clear right and urgency to ask for
avoided for otherwise, the outcome of a injunction because of the issue of validity of the
litigation would be useless as far as the party increase in the amount of the loan obligation.
applying for the writ is concerned.
SPOUSES SILVESTRE O. PLAZA AND ELENA Y.
A writ of preliminary injunction is an PLAZA, Petitioners,
extraordinary event which must be granted only vs. GUILLERMO LUSTIVA, ELEODORA VDA. DE
in the face of actual and existing substantial MARTINEZ AND VICKY SAYSON
rights. The duty of the court taking cognizance GOLOSENO, Respondents. G.R. No. 172909,
of a prayer for a writ of preliminary injunction is March 5, 2014
to determine whether the requisites necessary
Facts:
for the grant of an injunction are present in the
case before it. In this connection, a writ of On September 14, 1999, Vidal’s son and
preliminary injunction is issued to preserve the daughter-in-law, the petitioners, filed a
status quo ante, upon the applicant’s showing Complaint for Injunction, Damages, Attorney’s
of two important requisite conditions, namely: Fees with Prayer for the Issuance of the Writ of
(1) the right to be protected exists prima facie, Preliminary Injunction and/or Temporary
and (2) the acts sought to be enjoined are Restraining Order against the respondents and
violative of that right. It must be proven that the City Government of Butuan. They prayed
the violation sought to be prevented would that the respondents be enjoined from
cause an irreparable injury. unlawfully and illegally threatening to take
possession of the subject property. According to
No legal justification for the Honorable Court of
the petitioners, they acquired the land from
Appeals to lift/dissolve the injunction issued by
Virginia Tuazon in 1997; Tuazon was the sole
the trial court, otherwise, respondent bank – on
bidder and winner in a tax delinquency sale
the basis of this illegal imposition of interest –
can already foreclose the properties of

Rolan Jeff A. Lancion


Arellano University School of Law Page 22
Remedial Law 2- Brondial Notes AY 2017- 2018

conducted by the City of Butuan on December preliminary injunction may be issued only upon
27, 1996. clear showing of an actual existing right to be
protected during the pendency of the principal
In its December 14, 1999 order, the Regional
action. When the complainant’s right or title is
Trial Court (RTC) of Butuan City, Branch 5,
doubtful or disputed, he does not have a clear
reconsidered its earlier order, denied the prayer
legal right and, therefore, the issuance of
for a Writ of Preliminary Injunction, and
injunctive relief is not proper.
ordered that the possession and occupation of
the land be returned to the respondents. The Upon the dismissal of the main case by the RTC
RTC found that the auction sale was tainted on August 8, 2013, the question of issuance of
with irregularity as the bidder was a the writ of preliminary injunction has become
government employee disqualified in moot and academic.
accordance with Section 89 of the Local
Novecio vs Lim, 754 SCRA 111
Government Code of 1991.
Facts:
In its October 24, 2005 decision, the CA
Respondents Maria Carmen J. Tuazon and
affirmed the RTC’s ruling, found the petitioners
Manuel V. Nieto, represented by their attorney-
guilty of forum shopping, dismissed the case,
in-fact, Lope Durotan , filed complaints for
and referred the case to the Court and to the
forcible entry with damages against petitioners
Integrated Bar of the Philippines for
Saturnino Novecio, Gavino Novecio, Anastacio
investigation and institution of the appropriate
Golez.
administrative action.
The petitioners, on the other hand, contended
Issue: WON petitioners failed to show clear and
that they have already been in possession of
unmistakable rights to be protectedby the writ;
the land for more than two years when the
the present action has been rendered moot and
complaints were filed. They maintained that
academic by the dismissal of the main action
they have planted the land with corn, durian,
Held: coconut, mango, jackfruit, rambutan, etc. for
their livelihood. They also alleged that they
As the lower courts correctly found, Tuazon had
were harassed by some men armed with
no ownership to confer to the petitioners
shotguns and pistols on February 12, 2004.
despite the latter’s reimbursement of Tuazon’s
purchase expenses. Because they were never The MTC found that the respondents anchored
owners of the property, the petitioners failed to their alleged prior possession on the fact that
establish entitlement to the writ of preliminary they have applied title for the land as shown by
injunction. "[T]o be entitled to an injunctive a certification authorizing land survey. Other
writ, the right to be protected and the violation than this, the respondents had no evidence of
against that right must be shown. A writ of their actual and physical possession of the land.

Rolan Jeff A. Lancion


Arellano University School of Law Page 23
Remedial Law 2- Brondial Notes AY 2017- 2018

The MTC also found that they were not even intended merely to give the court an evidence
residents of the place and never personally of justification for a preliminary injunction
appeared in court during trial. pending the decision on the merits of the case,
and is not conclusive of the principal action
The RTC held that the MTC ignored some pieces
which has yet to be decided. In a prayer for
of evidence, warranting the reversal of the
preliminary injunction, the plaintiff is not
decision.
required to submit conclusive and complete
The RTC ruled that the MTC should have given
evidence. He is only required to show that he
credence to the certification issued by the
has an ostensible right to the final relief prayed
Department of Environment and Natural
for in his complaint.
Resources - Community Environment and
Natural Resources Office (DENR-CENRO) The petitioners have adequately shown their
showing that the land in litigation is the subject entitlement to a preliminary injunction. First,
of an application for title and claim by the the relief demanded consists in restraining the
respondents. execution of the RTC decision ordering their
ejectment from the disputed land. Second, their
This Court, acting on the petitioners' prayer,
ejectment from the land from which they derive
issued a TRO on October 18, 2010, enjoining the
their source of livelihood would work injustice
RTC from executing its decision. The TRO
to the petitioners. Finally, the execution of the
remains effective until this day. Finally, it
RTC decision is probably in violation of the
appears that the CA has yet to issue a decision
rights of the petitioners, tending to render the
on the Petition for Review.
MTC judgment dismissing the forcible entry
Issue: WON CA abused its discretion when it cases ineffectual.
denied the petition for preliminary injunction

Held:
Rule 59- Receivership
A preliminary injunction is proper when the
Receivership is the principal action, the
plaintiff appears to be clearly entitled to the
provisional remedy is the appointment of a
relief sought and has substantial interest in the
receiver which is cognizable by the MTC or RTC.
right sought to be defended. As this Court has
previously ruled, "while the existence of the Section 1. Appointment of receiver. — Upon a
right need not be conclusively established, it verified application, one or more receivers of
must be clear." the property subject of the action or proceeding
may be appointed by the court where the
A writ of preliminary injunction is generally action is pending or by the Court of Appeals or
based solely on initial or incomplete evidence. by the Supreme Court, or a member thereof, in
Such evidence need only be a sampling the following cases:

Rolan Jeff A. Lancion


Arellano University School of Law Page 24
Remedial Law 2- Brondial Notes AY 2017- 2018

(a) When it appears from the verified appointed to be subject to the control of said
application, and such other proof as the court court.
may require, that the party applying for the
PAD- Preservation, Administration &
appointment of a receiver has an interest in the
Disposition
property or fund which is the subject of the
action or proceeding, and that such property or Q: Why is there a need for the appointment of a
fund is in danger of being lost, removed, or receiver?
materially injured unless a receiver be A: In order to preserve the property, administer
appointed to administer and preserve it; such property or proper disposal of such
(b) When it appears in an action by the property.
mortgagee for the foreclosure of a mortgage *Receivership has the longest time frame to file
that the property is in danger of being wasted an application
or dissipated or materially injured, and that its
value is probably insufficient to discharge the Receivership- At the commencement of the
mortgage debt, or that the parties have so action or even after execution
stipulated in the contract of mortgage; Preliminary Attachment- At the
(c) After judgment, to preserve the property commencement of the action or before entry of
during the pendency of an appeal, or to dispose judgment
of it according to the judgment, or to aid Preliminary Injunction- At the commencement
execution when the execution has been of the action or before final judgment
returned unsatisfied or the judgment obligor
Replevin- Any time before the defendant files
refuses to apply his property in satisfaction of
his answer
the judgment, or otherwise to carry the
judgment into effect; Support Pendte Lite- It vary depending on the
circumstances surrounding it
(d) Whenever in other cases it appears that the
appointment of a receiver is the most Q: Who may be appointed as a receiver?
convenient and feasible means of preserving,
A: Not necessarily the applicant since it is not a
administering, or disposing of the property in
guaranty that he will be appointed by the court.
litigation.
The court is not bound and it has the discretion
During the pendency of an appeal, the appellate to which he shall appoint.
court may allow an application for the
-The applicant for the appointment of a receiver
appointment of a receiver to be filed in and
is required to post a BOND.
decided by the court of origin and the receiver

Rolan Jeff A. Lancion


Arellano University School of Law Page 25
Remedial Law 2- Brondial Notes AY 2017- 2018

Reason: By mere application, damage may same; to make transfers; to pay outstanding
occur over the property. debts; to divide the money and other property
that shall remain among the persons legally
Example: BDO is placed under liquidator
entitled to receive the same; and generally to
ship/receivership, when a depositor applies for
do such acts respecting the property as the
receivership he should post a bond in order to
court may authorize. However, funds in the
protect the interest of BDO
hands of a receiver may be invested only by
-By applying for a receivership, a bond is order of the court upon the written consent of
necessary and once appointed by the court as a all the parties to the action.
receiver, he is required to post another bond.
No action may be filed by or against a receiver
(Double the amount principle)
without leave of the court which appointed
Section 2. Bond on appointment of receiver. — him.
Before issuing the order appointing a receiver
Section 8. Termination of
the court shall require the applicant to file a
receivership; compensation of receiver. —
bond executed to the party against whom the
Whenever the court, motu proprio or on motion
application is presented, in an amount to be
of either party, shall determine that the
fixed by the court, to the effect that the
necessity for a receiver no longer exists, it shall,
applicant will pay such party all damages he
after due notice to all interested parties and
may sustain by reason of the appointment of
hearing, settle the accounts of the receiver,
such receiver in case the applicant shall have
direct the delivery of the funds and other
procured such appointment without sufficient
property in his possession to the person
cause; and the court may, in its discretion, at
adjudged to be entitled to receive them and
any time after the appointment, require an
order the discharge of the receiver from further
additional bond as further security for such
duty as such. The court shall allow the receiver
damages.
such reasonable compensation as the
Section 6. General powers of receiver. — circumstances of the case warrant, to be taxed
Subject to the control of the court in which the as costs against the defeated party, or
action or proceeding is pending a receiver shall apportioned, as justice requires.
have the power to bring and defend, in such
-When the property is in the possession of
capacity, actions in his own name; to take and
another, receivership is not the proper remedy.
keep possession of the property in controversy;
to receive rents; to collect debts due to himself Case Doctrine: Koruga vs Arcenas
as receiver or to the fund, property, estate,
The jurisdiction in action for receivership and
person, or corporation of which he is the
financial institution shall be under the
receiver; to compound for and compromise the
Monetary Board of the Central Bank in

Rolan Jeff A. Lancion


Arellano University School of Law Page 26
Remedial Law 2- Brondial Notes AY 2017- 2018

accordance with Revised Central Bank Law. In his agent, she also filed for the appointment of
other entities, an action for receivership shall a receiver.
be under the regular courts.
Issue: WON the application for receivership is
Case: Sps Larrobis vs Phil Veterans Bank valid?

Facts: Sps Larrobis secured a loan from Phil Held: No- The appointment of a receiver in this
Veterans Bank, at one point during the case is not proper since the property is intact
pendency of the loan. PVB was placed under and it is not materially lost therefore there is no
receivership, the bank was later on ground for such appointment
rehabilitated, the management found that Sps.
Case: Tantano vs Espina-Caboverde 702 SCRA
Larrobis is not yet paid with their loan. PVB
foreclosed the property, Sps Larrobis put up the Facts:
defense of prescription since they filed the case Petitioners files a complaint of annulment of
after 14 years which is beyond the 10 year the Deed of Sale purportedly transferring lots
period. PVB argued that they undergo from their parents Maximo and Dominalda.
receivership therefore the prescriptive period During the pendency of the case the parties
was stalled. executed a Partial Settlement Agreement (PSA)
Issue: WON prescription shall run against PVB where they fixed the sharing of the
that is placed under receivership uncontroverted properties among themselves,
in particular, the adverted additional eight (8)
Held: YES- One of the duties & responsibilities
parcels of land including their respective
of a receiver is to collect debts in order to
products and improvements. Under the PSA,
rehabilitate such bank. The prescriptive period
Dominalda’s daughter, Josephine, shall be
therefore shall not be stalled. When banks are
appointed as Administrator. The PSA provided
placed in a receivership, they are only
that Dominalda shall be entitled to receive a
prohibited to conduct business banking
share of one-half (1/2) of the net income
transaction. When PNB failed to file an action
derived from the uncontroverted properties.
against Sps, Larrobis, they already slept on their
The PSA also provided that Josephine shall have
rights.
special authority, among others, to provide for
Case: Chavez vs Court of Appeals the medicine of her mother.

Facts: One lawyer who is residing in Manila but


has a plenty of properties in Bicol. His agent is
Both Annabelle Saldia and Jesus Tan then took
not remitting the money coming from the
their respective oaths of office and filed a
property. She filed civil/criminal cases against
motion to fix and approve bond which was

Rolan Jeff A. Lancion


Arellano University School of Law Page 27
Remedial Law 2- Brondial Notes AY 2017- 2018

approved by the trial court over petitioners’ requirement of a receiver’s bond rests upon the
opposition. discretion of the court. Sec. 2 of Rule 59 clearly
states that the court may, in its discretion, at
any time after the appointment, require an
Petitioners harp on the fact that the court a quo additional bond as further security for such
failed to require Dominalda to post a bond prior damages.
to the issuance of the order appointing a
Rule 60- Replevin
receiver, in violation of Section 2, Rule 59 of the
Rules of court Section 1. Application. — A party praying for
the recovery of possession of personal property
may, at the commencement of the action or at
Respondents insist that where there is sufficient any time before answer, apply for an order for
cause to appoint a receiver, there is no need for the delivery of such property to him, in the
an applicant’s bond because under Sec. 2 of manner hereinafter provided.
Rule 59, the very purpose of the bond is to
Principal Action is for the recovery of
answer for all damages that may be sustained
possession of personal property and the
by a party by reason of the appointment of a
Provisional Remedy is Replevin
receiver in case the applicant shall have
procured such appointment without sufficient Purpose: Immediate recovery of possession of
cause. personal property and not applicable in real
property.
Issue:
Example: Car (Vios)
WON posting bond is required in receivership?
-When a person bought a car, the seller will ask
Held:
you to sign a document pertaining to Chattel
Mortgage that has a clause ”The Company can
Sec. 2 of Rule 59 is very clear in that before judicially or extra-judicially foreclose/retrieve it
issuing the order appointing a receiver the court upon failure to pay the necessary payment”.
shall require the applicant to file a bond The mortgage in an action for the foreclosure
executed to the party against whom the can file a replevin.
application is presented. The use of the word Time Period: At the commencement of an
"shall" denotes its mandatory nature; thus, the action or before the defendant files his answer
consent of the other party, or as in this case,
-Once the answer is submitted to the court,
the consent of petitioners, is of no moment.
issues shall be joined and the court should hear
Hence, the filing of an applicant’s bond is
both sides. Upon the issuance of the writ, it
required at all times. On the other hand, the
shall be forwarded to the sheriff and he can

Rolan Jeff A. Lancion


Arellano University School of Law Page 28
Remedial Law 2- Brondial Notes AY 2017- 2018

recover it anywhere within the Philippines. (No preliminary attachment, or otherwise placed
jurisdictional limitation) under custodia legis, or if so seized, that it is
exempt from such seizure or custody; and
Requirements: -Bond -Affidavit
(d) The actual market value of the property.
-The bond should be twice the value of the
property.

Example: Juan bought a car for P 1 million Section 4. Duty of the sheriff. — Upon receiving
pesos, the seller seek to recover such property such order, the sheriff must serve a copy
by posting twice the value of the car. thereof on the adverse party, together with a
copy of the application, affidavit and bond, and
-The bond mentioned is not a cash deposit but
must forthwith take the property, if it be in the
rather a SURETY BOND
possession of the adverse party, or his agent,
-The applicant shall only pay the premium of and retain it in his custody. If the property or
the bond (3%-15%) any part thereof be concealed in a building or
-The bond shall be twice the value of the enclosure, the sheriff must demand its delivery,
property and if it be not delivered, he must cause the
building or enclosure to be broken open and
First Value- It answers for the value of the take the property into his possession. After the
property sheriff has take possession of the property as
Second Value- It answers for the damages of herein provided, he must keep it in a secure
the property place and shall be responsible for its delivery to
the party entitled thereto upon receiving his
Section 2. Affidavit and bond. — The applicant
fees and necessary expenses for taking and
must show by his own affidavit or that of some
keeping the same.
other person who personally knows the facts:
-When the sheriff takes possession of the
(a) That the applicant is the owner of the
personal property, he has the obligation to keep
property claimed, particularly describing it, or is
it for 5 days. (Mandatory Provision)
entitled to the possession thereof;
-He cannot give the property to the applicant
(b) That the property is wrongfully detained by
before the expiration of 5 days otherwise he
the adverse party, alleging the cause of
can be made liable for administratively.
detention thereof according to the best of his
knowledge, information, and belief ; Purpose: -To determine the sufficiency of the
bond posted.
(c) That the property has not been distrained or
taken for a tax assessment or a fine pursuant to -To await of the possibility of posting of
law, or seized under a writ of execution or counter-bond/ re-delivery bond.

Rolan Jeff A. Lancion


Arellano University School of Law Page 29
Remedial Law 2- Brondial Notes AY 2017- 2018

Section 7. Proceedings where property claimed When the writ of replevin is issued in favor of
by third person. — If the property taken is the Republic of the Philippines, or any officer
claimed by any person other than the party duly representing it, the filing of such bond shall
against whom the writ of replevin had been not be required, and in case the sheriff is sued
issued or his agent, and such person makes an for damages as a result of the replevin, he shall
affidavit of his title thereto, or right to the be represented by the Solicitor General, and if
possession thereof, stating the grounds held liable therefor, the actual damages
therefor, and serves such affidavit upon the adjudged by the court shall be paid by the
sheriff while the latter has possession of the National Treasurer out of the funds to be
property and a copy thereof upon the applicant, appropriated for the purpose.
the sheriff shall not be bound to keep the
-The bond is for the benefit of the sheriff
property under replevin or deliver it to the
applicant unless the applicant or his agent, on -The sheriff will ask the applicant to post a bond
demand of said sheriff, shall file a bond equivalent to the value of the property
approved by the court to indemnify the third- THRICE THE VALUE DOCTRINE
party claimant in a sum not less than the value
of the property under replevin as provided in Q: What are the remedy for frivolous claim?
section 2 hereof. In case of disagreement as to A: File for damage in the same case or separate
such value, the court shall determine the same. action:
No claim for damages for the taking or keeping,
Same Case- if the court has jurisdiction
of the property may be enforced against the
bond unless the action therefor is filed within Separate Action- If the court has no jurisiction
one hundred twenty (120) days from the date
Section 9. Judgment. — After trial of the issues
of the filing of the bond.
the court shall determine who has the right of
The sheriff shall not be liable for damages, for possession to and the value of the property and
the taking or keeping of such property, to any shall render judgment in the alternative for the
such third-party claimant if such bond shall be delivery thereof to the party entitled to the
filed. Nothing herein contained shall prevent same, or for its value in case delivery cannot be
such claimant or any third person from made, and also for such damages as either
vindicating his claim to the property, or prevent party may prove, with costs.
the applicant from claiming damages against a
Navarro vs Escobido Doctrine
third-party claimant who filed a frivolous or
plainly spurious claim, in the same or a separate “Demand is not necessary in a writ of replevin
action. since it destroy the very essence or purpose of
replevin”

Rolan Jeff A. Lancion


Arellano University School of Law Page 30
Remedial Law 2- Brondial Notes AY 2017- 2018

Hao vs Andres Doctrine July 28, 1984 as well as three (3) consecutive
installments which fell due on August 28,
“Sheriff has the obligation to safe keep the
September 28, and October 28, 1984.
property for 5 days. “ The sheriff immediately
released the property even prior to the 5 day The trial court ruled that private respondent
period. The writ of replevin indicated was 20 FCP had no reason to file the present action
cars but it turned out that only 9 was in his since petitioner already paid the installments
possession and the 3 cars were lost under his for the months of July to November 1984,
custody. which are the sole bases of the complaint. The
lower court declared that private respondent
Case: Orosa vs Court of Appeals
was not entitled to the writ of replevin, and was
Doctrine: A party cannot recover the value of liable to petitioner for actual damages under
the car if he already received the value of the the replevin bond it filed.
car.
Issue: WON Mr Orosa can recover the value of
Facts: the car and of the property simultaneously?
On December 6, 1984, private respondent FCP Held:
Credit Corporation filed a complaint for replevin
NO- Mr. Orosa cannot recover the value of the
and damages in the Regional Trial Court of
car if he already received the car since it will
Manila against petitioner Jose S. Orosa and one
produce unjust enrichment.
John Doe to recover possession of a 1983 Ford
Laser 1.5 Sedan with Motor and Serial No. Case: Smart Communication vs Astorga
SUNKBT-14584. The complaint alleged that on
Facts:
September 28, 1983, petitioner purchased the
subject motor vehicle on installment from Astorga was hired by Smart Communication as a
Fiesta Motor Sales Corporation. He executed manager and she was given a car plan. She
and delivered to Fiesta Motor Sales Corp. a failed the re-evaluation by Smart and as a result
promissory note in the sum of P133,824.00 she was to different location. Astorga did not
payable in monthly installments. To secure follow order and as a result she was dismissed
payment, petitioner executed a chattel from Smart on the ground of insubordination.
mortgage over the subject motor vehicle in She filed a case towards the NLRC and during
favor of Fiesta Motor Sales Corp. On September the pendency of the case, Smart demand to
28, 1983, Fiesta Motor Sales assigned the return the car but Astorga refused. Smart filed a
promissory note and chattel mortgage to writ of replevin for the recovery of the car,
private respondent FCP Credit Corporation. The Astorga filed a Motion to dismiss on the ground
complaint further alleged that petitioner failed of lack of jurisdiction but it was denied the RTC.
to pay part of the installment which fell due on CA reversed the ruling of the RTC since the

Rolan Jeff A. Lancion


Arellano University School of Law Page 31
Remedial Law 2- Brondial Notes AY 2017- 2018

cause of action arises from an employer- interest of 6% per month shall be imposed for
employee relationship therefore RTC has no failure to pay each installment on or before the
jurisdiction. stated due date. For failure to pay four
successive installments from May 15, 2002 to
August 15, 2002, respondent, through counsel,
sent to petitioners a demand letter dated
Issue: WON RTC has jurisdiction over the writ of August 29, 2002, declaring the entire obligation
replevin as due and demandable and requiring to pay
Php576,664.04, or surrender the mortgaged
Held: The car involved in this case under did not vehicle immediately upon receiving the letter.
fall under employer-employee relationship but
rather for creditor-debtor relationship A writ of replevin was issued. Despite this, the
therefore it is cognizable by regular courts. subject vehicle was not seized. Trial on the
merits ensued. On August 11, 2005, the Manila
The RTC rightfully assumed jurisdiction over the RTC Br. 33 ruled for the respondent and
suit and acted well within its discretion in ordered petitioners to jointly and severally pay
denying Astorgas motion to dismiss. SMARTs the amount of Php576,664.04 plus interest at
demand for payment of the market value of the the rate of 72% per annum from August 20,
car or, in the alternative, the surrender of the 2002 until fully paid, and the costs of suit.
car, is not a labor, but a civil, dispute. It involves
the relationship of debtor and creditor rather Petitioners appealed the decision to the Court
than employee-employer relations. As such, the of Appeals (CA), but the CA affirmed the lower
dispute falls within the jurisdiction of the court’s decision and, subsequently, denied the
regular courts. motion for reconsideration;

Sps Agner vs BPI Family Savings Bank G.R. No. Issue: WON the writ of replevin is properly
182963, June 3, 2013 issued

Facts: Held:

On February 15, 2001, petitioners spouses Deo Records bear that both verbal and written
Agner and Maricon Agner executed a demands were in fact made by respondent
Promissory Note with Chattel Mortgage in favor prior to the institution of the case against
of Citimotors, Inc. The contract provides, among petitioners. Even assuming, for argument’s
others, that: for receiving the amount of sake, that no demand letter was sent by
Php834, 768.00, petitioners shall pay Php respondent, there is really no need for it
17,391.00 every 15th day of each succeeding because petitioners legally waived the necessity
month until fully paid; the loan is secured by a of notice or demand in the Promissory Note
2001 Mitsubishi Adventure Super Sport; and an with Chattel Mortgage, which they voluntarily

Rolan Jeff A. Lancion


Arellano University School of Law Page 32
Remedial Law 2- Brondial Notes AY 2017- 2018

and knowingly signed in favor of respondent’s -The only interlocutory order subject for
predecessor-in-interest. execution

Further, the Court even ruled in Navarro v. - A judgment or order for support pendent lite
Escobido that prior demand is not a condition never becomes final
precedent to an action for a writ of replevin,
- Res Judicata is not applicable
since there is nothing in Section 2, Rule 60 of
the Rules of Court that requires the applicant to Section 1. Application. — At the
make a demand on the possessor of the commencement of the proper action or
property before an action for a writ of replevin proceeding, or at any time prior to the
could be filed. judgment or final order, a verified application
for support pendente lite may be filed by any
party stating the grounds for the claim and the
Rule 61- Support PendeLite financial conditions of both parties, and
accompanied by affidavits, depositions or other
Principal Action: Support
authentic documents in support thereof.
Provisional Remedy: Support Pende Lite
Section 2. Comment. — A copy of the
Jurisdiction application and all supporting documents shall
RTC- Incapable of Pecuniary Estimation be served upon the adverse party, who shall
have five (5) days to comment thereon unless a
Exception: different period is fixed by the court upon his
-In Criminal Cases, when the civil aspect is motion. The comment shall be verified and
deemed instituted shall be accompanied by affidavits, depositions
or other authentic documents in support
*In support of Minor- Family Court
thereof.
Legal Age- RTC
Periods:
Q: What is the venue of such action?
5 days- Comment to the application
A: Residence of the plaintiff/ defendant
3 days- Set the application of support for
Requirement: hearing

 Verified application Situation:

*Support does not need an affidavit & bond  “Maria Clara and Jose” Love Story

*When an order granting support pende lite is  Maria Clara is working as an entertainer
issued by the court, it is an interlocutory order in a videoke bar while Jose is a grab
driver. They have a sexual relationship

Rolan Jeff A. Lancion


Arellano University School of Law Page 33
Remedial Law 2- Brondial Notes AY 2017- 2018

for 1 year and as a result a child was Reimbursement- If a third person is giving
born out of wedlock named Mario. support in lieu of the adverse party, the party is
obliged to return the 100k
Situation # 1: Maria Clara filed a case of support
along with a writ of support pendete lite -It shall be in a separate action since the third
towards the RTC of Quezon City person is not a party to the principal action and
the court has no jurisdiction over such party.
- The rule proves that within a period of
5 days, the adverse party is mandated Situation# 3- The court rendered a decision on
to file a comment. (Rule 61, Sec 2). If the principal action that Jose should give 5k
the adverse party failed to file a support to Maria Clara.
comment within the reglementary
-The 5k judgment for support never becomes
period, the court shall render a
final since Maria Clara or Jose can come to court
decision/order on the application for
to change the amount.
support pendete lite.
*If there is an increase or decrease in the
- The court shall issue an order directing
amount of support, a person through motion
Jose to give support pendete lite
can file it the same court and docket number.
Situation #2: After 1 year of trial, 100k was
- In the determination for amending the
already given as support pendete lite by Jose.
amount of support, two criteria must be
RTC of Quezon City rendered a decision on the
established:
principal action dismissing the case.
- Capacity of the person giving support
Q: What is the remedy for Jose?
- Necessity of the person to be supported
A: There should be Restitution, Maria Clara is
obliged to return the 100k plus interest Case: People vs Manahan 315 SCRA

*In support pendete lite, there is no damages Doctrine:”Compulsory Acknowledgment of a


but only restitution child if there is legal impediment is not
necessary in support”
Restitution vs Reimbursement

Restitution- The person liable to provide for


support pendete lite which is not proven
through judgment shall be entitled to return the Facts:
amount given along with the interest.
- On January 5, 1995, Manuel
-It shall be filed within the court Manahan committed his lustful
desire against Terisita a 16 year old

Rolan Jeff A. Lancion


Arellano University School of Law Page 34
Remedial Law 2- Brondial Notes AY 2017- 2018

minor. The crime resulted into Asis. Defandant-De Asis is the father of
pregnancy. When the parents the minor.
learned of such incident, they
- During the proceeding of the trial, both
jreported it to the local police.
the plaintiff & defendant filed a motion
- The Trial court of Dagupan to dismiss provided that the latter shall
convicted Manahan for the crime of not pursue his counter-claim.
rape.
- On September 7, 1995, another
Issue: WON Manahan is liable for support even complaint for support was filed by
if he did not acknowledge the child. Vircel de Andres against Manuel de
Asis. Defendant moved to dismiss the
complaint on the ground of res judicata
Held: but it was denied by the trial court.
YES- In the decision of the trial court, it provides - On certiorari, the Court of Appeals
that persons guilty of rape shall also be affirmed the denial of the motion to
sentenced to acknowledge the offspring unless dismiss filed by defendant de Asis.
the law should prevent him to do so is
Issue: WON Support can be a subject of res
erroneous.
judicata
-Compulsory acknowledgment of a child is not
Held:
proper there being a legal impediment in doing
so. NO- The right to receive support cannot be
renounced nor it can be transmitted to third
-Married men are not forced to recognize a
persons.
child out of wedlock. The purpose is to preserve
the sanctity of the family. -Filiation and/or support must be judicially
established and it is for the court to declare its
existence and absence.
Case: De Asis vs CA 303 SCRA
-The right of support cannot be waived or
Doctrine: “An action for support can never be transferred and future support cannot be a
under the principle of res judicata” subject of future compromise there the present
Facts: action for support can be brought
notwithstanding the fact the previous case was
- Vircel Andres, the legal guardian of Glen dismissed.
de Asis brought an action for
maintenance & support of Camil de Lim vs Lim 604 SCRA

Rolan Jeff A. Lancion


Arellano University School of Law Page 35
Remedial Law 2- Brondial Notes AY 2017- 2018

Doctrine: “Grandparents shall be liable to their support passes on to ascendants not only upon
grandchildren in case their parents cannot default of the parents but also or the latter
provide for sufficient support.” inability to provide for support.

Facts: -This obligation imposed by law is only limited


to relatives by blood. Cheryl’s right to receive
- Cheryl Lim married Edward Lim, they
support is only towards Edward since they are
had three three (3) children and they
legally married.
resided at the house of the petitioner-
grandparents. Case: Lim-Lua vs Lim 697 SCRA

- Cheryl Lim abandoned their residence Doctrine: “Supreme Court laid down the factors
in Forbes Park bringing the children in determining the amount of provisional
after she discovered the illicit affairs of support”
his husband Edward towards the in-
Facts:
house midwife.
- Susan Lim-Lua filed an action for the
- Cheryl sued the petitioners in the RTC
declaration of nullity of her marriage
of Makati City for support.
with Danito Lua. In her prayer, she
- RTC of Makati rendered a decision alleged for support pendete lite in the
ordering Edward and petitioners to amount of Php 500,000 monthly
jointly provide for Php 40,000 monthly support.
support while the husband shall
- The trial court rendered a decision that
shoulder the remaining balance.
Danilo shall only be liable for the
- The Court of Appeals affirmed the amount of Php 250,000.
decision of the trial court on the ground
- On appeal, CA reversed the decision of
the parents and their legitimate
the RTC on the ground that the amount
children are obliged to mutually
is excessive and there is no proof of
support each other and this obligation
such.
extends to the grandchildren.

Issue: WON petitioner-grandparents are liable


for support Issue: WON the expenses already incurred by
the respondent may be deducted from the total
Held:
amount of support?
YES- Obligation to provide support arising from
parental authority ends upon the emancipation
of the child. This obligation to provide for legal Held:

Rolan Jeff A. Lancion


Arellano University School of Law Page 36
Remedial Law 2- Brondial Notes AY 2017- 2018

No- The amount of support which those related -Matusalem prayed for support pendete lite
by marriage and family relations shall be in which was granted by the Trial Court.
proportion to the resources or means of the
-RTC rendered a decision in favor of Matusalem
giver and to the needs of the receipient
and ordering for the monthly support. CA
-The court may motu proprio or upon verified affirmed the decision of the trial court.
application of any of the parties, temporary
grant support pendete lite prior to the
judgment. The court is tasked to determin the Issue: WON Salas is liable to give support to
kind and amount of evidence which may be Matusalem
enable to resolve the application.

-Factors in determining the amount of support: Held:


(a) financial resources of the custodial & non-
custodial parent, (b) physical and emotional No- Illegitimate filiation can be established
health of the child has been accustomed and (4) through record of birth appearing in the civil
non- monetary contribution that the parents registrar or final judgment or admission of
will make. legitimate filiation in a public or private
instrument.

-Matusalem failed to establish the filiation


Case: Salas vs Matusalem 705 SCRA 560 between Salas and Christian Paulo through clear
Doctrine: “Support can only be granted only if and convincing evidence.
paternity or filiation is established by clear and
convincing evidence”
- An order of recognition and support may
create an unwholesome situation to the lives of
Facts: the parties therefore it can only be issued by
clear and convincing evidence.
- Annabelle Matrusalem filed a complaint for
support and damages against Narciso Salas
towards RTC of Cabanatuan City. Case: Gotardo vs Buling 678 SCRA
-Matusalem claimed tha Salas is the father of Doctrine: “The amount of support is variable
Christian Paulo who was born on December 28, therefore res judicata is not applicable”
1994. Salas allegedly rented an apartment for
the benefit of Matusalem and shouldered all Facts:
the necessary expenses for the birth of the
child.

Rolan Jeff A. Lancion


Arellano University School of Law Page 37
Remedial Law 2- Brondial Notes AY 2017- 2018

-Buling filed a complaint with the RTC of Case: Del Socorro vs Van Wilsem 744 SCRA 516
Southern Leyte for compulsory recognition and
Doctrine: “If the foreign law, judgment or
support pendete lite against Gotardo.
contract is contrary to sound and established
-Petitioner Gotardo and Buling started their policy, it shall be applied in this jurisdiction”
intimate sexual relation with Buling in their
Facts:
boarding house. As a result of this action, Buling
got pregnant. - Del Socorro and Van Wilsem contracted
a marriage in The Netherlands on
- During the pendency of the action, RTC
September 25, 1990.
granted the Php 2,000 monthly support. In its
decision, it dismissed the action for insufficiency - Their marriage ended by virtue of a
of evidence to prove filiation. Divorcee decree applied by Van Wilsem
in his country (The Netherlands). He
-CA reversed the decision of the RTC and
however promised that he will give
ordered a grant of Php 2,000 monthly support.
support to their child but later on did
not materialize.

Issue: WON the Court of Appeals erred in its - Del Socorro filed a criminal complaint
decision to recognize and provide for support. for VAWC but it was dismissed by the
Trial Court. She later on filed a civil
Held:
action for support.
NO – Buling established a prima facie evidence
- Van Wilsem argued that under the
that Gotardo is the putative father of Glifze
Dutch law, he is not obliged to give
through the testimony that she had been
support since his marriage with Del
sexually involved with only one man.
Socorro is already divorced.
-Support follow as a matter of right, parents are
Issue: WON Van Wilsem is obliged to give
obliged to support his/her children whether
support even if his national law does not
legitimate or illigitmate.
provide for it.

Held:
- The amount of support is variable and for this
YES- When the foreign law, judgment is
reason no final judgment on the amount of is
contrary to the sound policy, morals, it shall not
made as the amount shall be in proportion to
be applied in this country.
the resources, means of the giver and
necessities of the recipient. -Even if the laws of The Netherlands neither
enforce parental obligation to support his child,
such obligation is still enforceable in the

Rolan Jeff A. Lancion


Arellano University School of Law Page 38
Remedial Law 2- Brondial Notes AY 2017- 2018

Philippines because it would be a great injustice Rule 62- Interpleader


to the child to be denied of such support.
Section 1. When interpleader proper. —
Whenever conflicting claims upon the same
subject matter are or may be made against a
person who claims no interest whatever in the
SPECIAL CIVIL ACTION (Rule 62-71) subject matter, or an interest which in whole
or in part is not disputed by the claimants, he
Rule 62- Interpleader may bring an action against the conflicting
claimants to compel them to interplead and
Rule 63- Declaratory Relief and Similar
litigate their several claims among themselves.
Remedies
Requisites:
Rule 64- Judgment, Final Order Coming from
COA, COMELEC and CSC - There should be more than one
Rule 65- Certiorari, Prohibition and Mandamus defendant
- Conflicting claim over the subject
Rule 66- Quo Warranto matter
- Plaintiff has no legal interest
Rule 67- Expropriation
*There can be no interpleader if there is only
Rule 68- Foreclosure of Real Estate Mortgage
one defendant.
Rule 69- Partition
Interpleader vs Intervention
Rule 70- Unlawful Detainer and Forcible Entry
Rule19- Intervention
Rule 71- Contempt
- Ordinary Civil Action
- The person who intervene must have
Q: Why are they called Special Civil Actions? interest over the subject matter
- It should be filed any time before the
A: In cases filed under SCA, the rules provides rendition of final judgment
for its particular rules to be applied. If there is a
conflict between the regular rules and sca, the Rule 62- Interpleader
latter shall prevail.
- Special Civil Action
- The interpleader must have no interest
over the subject matter
-Only when there is no particular application of - It should be filed during the complaint
the rule, the regular civil action will supplement
the SCA. Subject Matter:

Rolan Jeff A. Lancion


Arellano University School of Law Page 39
Remedial Law 2- Brondial Notes AY 2017- 2018

- Real Property (A,B,C) are claiming interest over the bank


- Personal Property deposit.
- Performance of specific
action/obligation A- The account number belongs to my
father (deceased) and based on his will
it belongs to me.
GR: The jurisdiction of the court shall depend B- The account was already donated to me
on the value of the property over which persons (Deed of Donation)
have. C- I have the authority to withdraw based
on a letter
XPN: Performance of a specific action-
Incapable of Pecuniary Estimation- RTC Remedy: File an action for interpleader in the
RTC since it is incapable of pecuniary
Real Property- In all civil actions which involve estimation.
the title to, or possession of, real property, or
any interest therein, where the assessed value Section 2. Order. — Upon the filing of the
of the property involved exceeds Twenty complaint, the court shall issue an order
thousand pesos (P20,000,00) or, for civil actions requiring the conflicting claimants to
in Metro Manila, where such value exceeds Fifty interplead with one another. If the interests of
thousand pesos (P50,000.00). justice so require, the court may direct in such
order that the subject matter be paid or
Personal Property- In all other cases in which delivered to the court.
the demand, exclusive of interest, damages of
whatever kind, attorney's fees, litigation
expenses, and costs or the value of the property
in controversy exceeds One hundred thousand Section 3. Summons. — Summons shall be
pesos (P100,000.00) or, in such other cases in served upon the conflicting claimants, together
Metro Manila, where the demand exclusive of with a copy of the complaint and order.
the abovementioned items exceeds Two
Hundred thousand pesos (P200,000.00). Section 4. Motion to dismiss. — Within the
time for filing an answer, each claimant may file
Example: Abel found a wallet along the corridor a motion to dismiss on the ground of
of Arellano and there are three claimants impropriety of the interpleader action or on
(X,Y,Z). A can file an action for interpleader other appropriate grounds specified in Rule 16.
since there are 3 defendants. The period to file the answer shall be tolled and
if the motion is denied, the movant may file his
-This kind of action should be filed with the MTC answer within the remaining period, but which
based on the value of the property. shall not be less than five (5) days in any event,
reckoned from notice of denial.
Example 2: If the subject matter of interpleader
is a bank deposit. The saving account no. (Fresh 5 day period shall be applicable)
123456789 and there are three individual
Grounds:

Rolan Jeff A. Lancion


Arellano University School of Law Page 40
Remedial Law 2- Brondial Notes AY 2017- 2018

-Impropriety of the interpleader Held:

-Grounds under Rule 16 of the ROC YES –The essence of interpleader aside from
disavowal of interest in the property in litigation
Section 7. Docket and other lawful fees, costs is also the deposit of property or funds in court.
and litigation expenses as liens. — The docket
and other lawful fees paid by the party who -The need for deposit in question has been
filed a complaint under this Rule, as well as the established not applicable in the lower courts,
costs and litigation expenses, shall constitute a Court of Appeals but also in the Supreme Court
lien or change upon the subject matter of the where such deposit is required.
action, unless the court shall order otherwise.
Case: Wack-Wack Golf and Country Club vs
Case: Eternal Gardens vs IAC 165 SCRA Won & Tan 70 SCRA

Doctrine: “The subject matter of an Doctrine: “Interpleader is considered as


interpleader should be deposited in court” compulsory”

Facts: Facts:

-Eternal Gardens and Mission Realty - Mr Won claims ownership of its


executed a Land Development membership fee certificate 201 by
Agreement whereby the later will virtue of the decision rendered by CFI of
introduce and construct a Memorial lot Manila.
over the property of Mission. - Mr. Tan also claim ownership over the
certificate 201 based on a deed of
-Out of the proceeds from the sale, assignment given by “Swan, Culbertson
Eternal Garden shall be liable for 60% and Fritz”.
and 40% will go to Mission. Maysila - Wack Wack filed an interpleader
Estate asserted claim of ownership over towards the CFI of Manila, Mr. Won
the land. filed a motion to dismiss based on res
judicata since the court already
-Eternal Garden filed a complaint for rendered a decision as to the ownership
interpleader, Mission during the course of such certificate.
of the proceeding filed a motion that
the amount due must be judicially Issue: WON the MTD filed by Mr. WON should
deposit. prosper as to the determination of ownership

-The motion filed by Mission was Held:


denied.
YES- An action for interpleader is a remedy
Issue: WON the amount involved in this case whereby a person who was personal property in
should be placed under judicial deposit his possession belonging to different claimants

Rolan Jeff A. Lancion


Arellano University School of Law Page 41
Remedial Law 2- Brondial Notes AY 2017- 2018

in which he has no interest over the subject -Pasricha is also entitled to judicial consignation
matter. where it shall be made by depositing the thing
due at the disposal of judicial authority.
-Wack Wack is now estopped since it is already
a party to the previous case of WON in which a -Instead of availing of the available remedies,
court order was issued. petitioner refrains from making payments.

-Interpleader is late if a stakeholder defends a Case: Bank of Commerce vs Planters


suit filed by one of the adverse claimants and it Development Bank 681 SCRA
is allowed to proceed to final judgment, he
cannot later on have part on the litigation. Doctrine: “Interpleader is also applicable in
Cross-claim or Third-Party Complaint”
Case: Pasricha vs Don Luis Dizon Realty 548
SCRA Facts:

Facts: - RCBC owned two sets of Central Bank


Bill (1) CB Bill was worth Php 70 million
- Don Luis Realty and Pasricha entered and (2) 2 CB Bill worth Php 20 million.
into a lease contract of apartment units The first set was sold to BOC while the
in Ermita, Manila. second set was sold by RCBC to PDB.
- Pasricha failed to pay the rentals of the - Upon learning of the transfer involving
leased unit despite repeated demands, CB Bills, PDB requested the BSP to
Don Luis filed an ejectment case before record its claim but it was denied.
the MTC of Manila. Pascricha contend - PDB filed with the RTC two separate
that he is clueless as to whom he should petitions for Mandamus with Prayer of
made the payment. Preliminary Injunction.
- MTC dismissed the complaint for - BSP asked that an interpleader suit be
ejectment but it was overturned by the allowed among the claimants to the
RTC. subject CB Bills. BOC and PDB entered
into two separate Escrow Agreement.
Issue: WON Pasricha should file an action for
interpleader to determine who shall be entitled Issue: WON Interpleader may be made an
to the payment alternative defense

Held: Held:

YES- The defense of Pasricha is untenable in YES- Rule 62 does not expressly authorize the
order to escape the payment of rentals. filing of a complaint in interpleader as separate
and independent. This also allows “counter-
-An action for interpleader is proper when the complaint/cross-claim for interpleader” run
lessee does not know to whom payment of counter to general procedure.
rentals should be made due to conflicting
claims.

Rolan Jeff A. Lancion


Arellano University School of Law Page 42
Remedial Law 2- Brondial Notes AY 2017- 2018

-BSP did not initiate the interpleader suit the declaration shall be made parties; and no
through an original complaint but through declaration shall, except as otherwise provided
answer. in these Rules, prejudice the rights of persons
not parties to the action.

Section 6. Conversion into ordinary action. — If


Rule 63- Declaratory Relief and Similar before the final termination of the case, a
Remedies breach or violation of an instrument or a
statute, executive order or regulation,
Section 1. Who may file petition. — ordinance, or any other governmental
regulation should take place, the action may
(a) Any person interested under a deed, thereupon be converted into an ordinary
will, contract or other written action, and the parties shall be allowed to file
instrument, or whose rights are such pleadings as may be necessary or proper.
affected by a statute, executive order or
regulation, ordinance, or any other
governmental regulation may, before
breach or violation thereof bring an Case:
action in the appropriate Regional Trial
Almeda vs Bathala Marketing, GR No. 150806,
Court to determine any question of
construction or validity arising, and for January 28, 2008
a declaration of his rights or duties,
Doctrine: “6 requisites for declaratory relief”
thereunder.
Facts:
Jurisdiction – RTC
- Bathala Marketing renewed its Contract of
(b) An action for the reformation of an
Lease with Potenciano Almeda. He agree to
instrument, to quiet title to real
property or remove clouds therefrom, lease a portion of their property located in
or to consolidate ownership under Makati City.
Article 1607 of the Civil Code, may be
brought under this Rule. -Based on the provision of their contract,
Almeda is informing Bathala Marketing about
Jurisdiction- Depending on the the increase in the monthly rendtal due to
assessed value of the property extra-ordinary inflation rate in the country.

-Bathala opposed the demand and later on filed


for an action for declaratory relief for the
correction of condition 6&7 of the contract. The
trial court denied their claim for increase in
Section 2. Parties. — All persons who have or rental.
claim any interest which would be affected by

Rolan Jeff A. Lancion


Arellano University School of Law Page 43
Remedial Law 2- Brondial Notes AY 2017- 2018

Doctrine: “Rule 63 par (1) is cognizable by RTC


while par (2) shall depend on the assessed value
Issue: WON the action for declaratory relief is of the land”
proper
Facts:
Held:
- Malana, et,al filed before the RTC their
YES- 1) the subject matter of the controversy complaint for Reinvindication, Quieting of Title
must be a deed, will, contract or other written
and Damages.
instrument, statute, executive order or
regulation, or ordinance; 2) the terms of said -RTC issued an order dismissing the complaint
documents and the validity thereof are doubtful on the ground of lack of jurisdiction. It found
and require judicial construction; 3) there must that the subject property has a value less than
have been no breach of the documents in Php 20,000 therefore it is cognizable by MTC.
question; 4) there must be an actual justiciable
controversy or the ripening seeds of one
between persons whose interests are adverse; Issue: WON the judge erred in its decision of
5) the issue must be ripe for judicial dismissing the complaint
determination; and 6) adequate relief is not
available through other means or other forms Held:
of action or proceeding. It is beyond cavil that
NO- RTC correctly made a distinction between
the foregoing requisites are present in the
first & second paragraph.
instant case, except that petitioners insist that
respondent was already in breach of the -The first paragraph shall be brought before the
contract when the petition was filed. RTC while in the second paragraph for action for
the reformation of interest, action to quiet title
- The Court is disposed to entertain the instant
and action to consolidate ownership must be
declaratory relief action instead of dismissing it,
read together with the Judiciary Act wherein
notwithstanding the pendency of the
jurisdiction shall depend on the assessed value
ejectment/rescission case before the trial court.
of the land.
The resolution of the present petition would
write finis to the parties dispute, as it would
settle once and for all the question of the
proper interpretation of the two contractual -In this case, the assessed value of the land is
stipulations subject of this controversy. only Php 410 which clearly cognizable by the
MTC.

Sps Sabitsana vs Muertegui, GR No. 181359,


Malana vs Tappa, GR No. 181303, September August 5, 2013
17, 2009

Rolan Jeff A. Lancion


Arellano University School of Law Page 44
Remedial Law 2- Brondial Notes AY 2017- 2018

Doctrine: “All cases of Declaratory Relief shall Facts:


be cognizable by RTC”
-Orbecido married Lady Myros in Ozamis City.
Facts: The wife went to the United States along with
their son.
- Alberto Garcia execute a Deed of Sale in favor
Muertegui over a 7, 500 sq meter over a -Lady Myros obtained a US Citizenship and later
property in Biliran. The father of Juanito and his on their son Kristoffer told Orbecido that her
brother took possession of it. wife already obtained a divorcee decree in
the United States.

-Orbecido filed a petition for authority to re-


- Garcia sold the lot to Atty. Sabitsana through a marry and for declaratory relief for the
deed of absolute sale. application of Art 26 par 2 of the FC

Issue: WON the petition for declaratory relief


- Juanito filed an action for quieting of title shall prosper
against Sabitsana towards the Regional Trial Held:
Court. A decision was made in favor of Juanito
by the trial court and it was affirmed by the YES- The requisites for declaratory relief are (a)
Court of Appeals. justiciable controversy, (b) controversy
must be between person whose interest
Issue: WON RTC has jurisdiction over the case
are adverse, (c) party has legal interest and (d)
ripe for judicial determination.

-The issue raised by Orbecido is already ripe for


judicial determination in order to answer
the valdity of his second marriage.

Held:

YES- RTC has jurisdiction over the suit for


Anti- Terrorism Council vs Southern
quieting of title. It is clear that an action for
Hemisphere, GR No. 178552
quieting of title may be instituted with the RTC
regardless of the assessed value.
Facts:
-If the action is for declaratory relief, it shall be
cognizable by the RTC. -There are six petitions challenging the
constitutionality of RA 9372 or the Human
Republic vs Orbecido
SecurityAct of 2007.

Rolan Jeff A. Lancion


Arellano University School of Law Page 45
Remedial Law 2- Brondial Notes AY 2017- 2018

Issue: WON RA 9372 is constitutional in nature - Harry Roque et. Al filed a petition for
based on the allegations made by the declaratory relief before the RTC
petitioners challenging the constitutionality of RA
9372 or Anti- Human Security Act for
being void for vagueness violating the
right to privacy of communication.
Held: - On Oct 5, 2010, SC released a decision
in the Southern Hemisphere case
YES- The petition for certiorari does not lie dismissing its petition.
against the respondent who does not exercise - RTC held that the respondent has legal
judicial or quasi-judicial function. standing to assail the petition for
declaratory relief.
-The Supreme Court gave the four requisites for Issue: WON RTC gravely abused it discretion
judicial review: (a) there must be an actual case when it denied the subject motion to dismiss
or controversy, (b) the question of
Held:
constitutionality must be raised at the earliest
possible opportunity, (c) the question of YES- The requisites for an action of declaratory
constitutionality must be raised at the earliest relief: (a) subject matter of deed, will, contract
possible opportunity and (d) the issue of or other written instrument, (b) the terms of
constitutionality must be the lis mota of the such document require judicial construction, (c)
case. no breach of the document in question, (d)
justiciable controversy, (e) ripe for judicial
-In this case, petitioners have not presented any
determination and (f) adequate relief is not
personal stake in the outcome of the
available.
controversy since none of the faces any charge
of RA 9372. -Based on the Southern Hemisphere Case, there
was no imminent danger/threat towards the
-The obscure allegations of sporadic
petitioner. Without any justiciable controversy,
surveillance and being tagged as communist
the petition have become pleas for declaratory
front in no way approximate credible threat of
relief over the Court has no original jurisdiction.
prosecution.
Absent the requisites (d),(e) and (f), the petition
- The allegations and issues were not valid to should be dismissed.
held RA 9372 as unconstitutional.

Dept. of Finance vs De la Cruz Jr., 768 SCRA 73


Republic vs Harry Roque 706 SCRA
Facts:
Facts:
- There was an issuance of E.O 140 on
Sept 2, 2013 which created the Customs

Rolan Jeff A. Lancion


Arellano University School of Law Page 46
Remedial Law 2- Brondial Notes AY 2017- 2018

Policy research in the Department of Rule 64


Finance. In the said Executive Order,
the CPRO shall be responsible for - The petition shall be filed within thirty (30)
reviewing the customs administration days from notice of the judgment or final order
policies, rules & procedure. or resolution sought to be reviewed. The filing
- Respondent filed an action for of a motion for new trial or reconsideration of
declaratory relief with application for said judgment or final order or resolution, if
TRO and/or Preliminary Injunction allowed under the procedural rules of the
before the RTC of Manila and Judge Commission concerned, shall interrupt the
Dela Cruz granted such petition. period herein fixed. If the motion is denied, the
aggrieved party may file the petition within the
- Petitioner filed a petition for certiorari
and prohibition before the Supreme remaining period, but which shall not be less
Court. than five (5) days in any event, reckoned from
notice of denial.
Issue: RTC has jurisdiction over the case
-Filing of a motion for new trial or
Held: reconsideration of said judgment or resolution
shall interrupt the period therein.
YES- Respondents alleged that CPO 189-2013
was issued to beat the deadline of the
Rule 65
COMELEC’s ban on personnel movement due to
the scheduled brgy. Election.
-The petition shall be filed not later than sixty
-The issue took the case beyond the scope of (60) days from notice of the judgment, order
the CSC’s jurisdiction because the matter is no or resolution.
longer limited to personnel action therefore
RTC has jurisdiction over the case. -Motion for Reconsideration is mandatory and
the 60 days period shall be counted from the
notice of denial

RULE 64- REVIEW OF JUDGMENT AND FINAL *the Fresh Period shall not be applicable in Rule
ORDER OR RESOLUTION OF COMELEC AND 12, 16, 62 and 64
COA

-Any Decision, Final Order, Resolution of the


COA and COMELEC, it shall be assailed by Rule
Case: Alliance for Nationalism and Democracy
64 towards the Supreme Court and it is only
vs COMELEC 716 SCRA 340
through certiorari under Rule 65.

Rule 64 vs Rule 65 Facts:

Rolan Jeff A. Lancion


Arellano University School of Law Page 47
Remedial Law 2- Brondial Notes AY 2017- 2018

-On November 7, 2012, Comelec En Banc abuse of discretion amounting to lack or


promulgated a Resolution cancelling the excess of jurisdiction, and there is no appeal,
Certificate of Recognition based on three or any plain, speedy, and adequate remedy in
grounds: (a) ANAD does not represent the the ordinary course of law, a person aggrieved
thereby may file a verified petition in the
marginalized sector, (b) No actual showing of
proper court, alleging the facts with certainty
nominees and (c) it failed to submit its and praying that judgment be rendered
statement of contribution and expenditure annulling or modifying the proceedings of such
tribunal, board or officer, and granting such
Issue: WON COMELEC gravely abused its incidental reliefs as law and justice may require.
discretion in promulgating the assailed decision
Section 2. Petition for prohibition. — When the
Held: proceedings of any tribunal, corporation,
board, officer or person, whether exercising
judicial, quasi-judicial or ministerial functions,
NO- ANAD was already afforded of summary
are without or in excess of its or his
hearing during which Mr. Balang answered jurisdiction, or with grave abuse of discretion
questions from COMELEC. amounting to lack or excess of jurisdiction, and
there is no appeal or any other plain, speedy,
-Compliance with Sec 8 of RA 7941 is essential and adequate remedy in the ordinary course of
as it as a safeguard against arbitrariness. law, a person aggrieved thereby may file a
verified petition in the proper court, alleging
-The factual finding of administrative bodies will the facts with certainty and praying that
judgment be rendered commanding the
not be disturbed by the courts of justice except
respondent to desist from further proceedings
when there is no evidence or substantial in the action or matter specified therein, or
evidence in support of such finding. otherwise granting such incidental reliefs as law
and justice may require.
-The COMELEC may motu proprio cancel, after
due notice and hearing, the registration of any Section 3. Petition for mandamus. — When any
party list if it violates or fails to comply with law, tribunal, corporation, board, officer or person
unlawfully neglects the performance of an act
rules, or regulation relating to election.
which the law specifically enjoins as a duty
resulting from an office, trust, or station, or
unlawfully excludes another from the use and
enjoyment of a right or office to which such
RULE 65- CERTIORARI, PROHIBITION AND other is entitled, and there is no other plain,
MANDAMUS speedy and adequate remedy in the ordinary
course of law, the person aggrieved thereby
Section 1. Petition for certiorari. — When any may file a verified petition in the proper court,
tribunal, board or officer exercising judicial or alleging the facts with certainty and praying
quasi-judicial functions has acted without or in that judgment be rendered commanding the
excess its or his jurisdiction, or with grave respondent, immediately or at some other time

Rolan Jeff A. Lancion


Arellano University School of Law Page 48
Remedial Law 2- Brondial Notes AY 2017- 2018

to be specified by the court, to do the act Officer- Herbosa (SEC) Judges of the trial courts
required to be done to protect the rights of the
petitioner, and to pay the damages sustained by 3. What are the grounds?
the petitioner by reason of the wrongful acts of
the respondent. Certiorari- Lack of jurisdiction, Excess of
jurisdiction and grave abuse of discretion
6 Issues under Rule 65 amounting to lack of jurisdiction

1. Who are the petitioners? Prohibition- Lack of jurisdiction, Excess of


jurisdiction and grave abuse of discretion
2. Who are the respondents? amounting to lack of jurisdiction

3. What are the grounds? Mandamus- Neglect in the performance of duty


enjoined by law and exclusion of a person from
4. What functions did the respondent an office or franchise
performed?
Q: What kind of jurisdiction?
5. What is the common condition?
A: Jurisdiction over the subject matter
6. What is the prayer?
-Jurisdictional estoppel only applies to subject
matter (Boston Equity vs CA)

1. Who are the petitioners? Q: What is excess of jurisdiction?

A: Aggrieved Party- A party was aggrieved by A: Over stepping the authority or going beyond
the actuation of the respondent. the authority

2. Who are the respondents? Example:

A: Certiorari- Tribunal, Board, Officer Judge X of MTC rendered a penalty of reclusion


temporal in a case of qualified seduction. The
Prohibition- Tribunal, Board, Officer, judge exceeded his authority since he can only
Corporation and Natural Person impose penalty below 6 years.

Mandamus- Tribunal, Board, Officer, Q: What is grave abuse of discretion amounting


Corporation and Natural Person to lack of jurisdiction?

Tribunal- Regional Trial Court or Municipal Trial A: It amounts to gross violation of the rule.
Court (Whimsical, Capricious, Arbitrary and Despotic)

Board- Energy Regulatory Board, SEC Whimsical and Capricious- No Basis at all

Rolan Jeff A. Lancion


Arellano University School of Law Page 49
Remedial Law 2- Brondial Notes AY 2017- 2018

Arbitrary and Despotic- Out of passion or Ministerial- When a party or officer has no
emotion option or choice in the performance of duty.

Mandamus Example: Motion for Execution as a matter of


right
-Neglect in performance of duty enjoined by
law -The motion shall only determine if the
judgment is already executory
-Exclusion of a person from an office or
franchise Case: Tuason vs RD of Caloocan City

Example: Mayor Erap Estrada removed the city Facts:


engineer and in his place is his niece
-Mr Tuason is a retired public school teacher
Q: What are the remedies available to the city wherein his retirement pay was used to buy a
engineer? property in Kalookan. During the time of Martial
Law, President Marcos enacted a Presidential
A: He can file a mandamus and quo warranto Decree which invalidate all of the land titles
against Mayor Erap and his niece. The usurper situated in Kalookan and was given to the
in this problem is the niece and the cause of members of PSG. After the Edsa Revolution,
action is quo warranto while for Mayor Erap Tuason filed a petition for certiorari asking for
who cause the usurpation shall be mandamus. the nullification of the Presidential Decree.

-A party can join its cause of action quo Issue: WON the petition for certiorari shall
warranto & mandamus simultaneously. apply to Marcos that exercise executive
function
4. What are the functions of the respondents?
Held: YES- The petition for certiorari is proper to
Certiorari- It must be discretionary question the Presidential Decree since Marcos
is a usurper of judicial function. The cancellation
Prohibition- It must be discretionary or of existing titles belongs to the judiciary and not
ministerial the executive department. Only judicial courts
can nullify such titles.
Mandamus- Ministerial function
Judicial Courts- RTC, MTC, Sandiganbayan, CTA
Discretionary- When a party or officer is given and Court of Appeals
an option or choice in the performance of duty.
Quasi-judicial- it may fall under executive or
Example: Granting a motion for extension of legislative department
appeal
Example: HLURB, SEC, Energy Regulatory Board

Rolan Jeff A. Lancion


Arellano University School of Law Page 50
Remedial Law 2- Brondial Notes AY 2017- 2018

5. What is the common condition in certiorari, Q: Can DeAndre use petition for certiorari as a
prohibition and mandamus? remedy in the decision rendered by the court
that has no jurisdiction over the case?
A: - There should be no appeal, or any other
plain, adequate remedy in the ordinary course A: NO, since DeAndre has the remedy of Appeal
of law. in the final judgment rendered by such court
even if it has no jurisdiction over the case.
-No appeal, No Motion for Reconsideration or
No New Trial 6. What is the prayer?

-Certiorari and Appeal are mutually exclusive Certiorari- To nullify the proceeding, judgment,
order rendered by a judicial or quasi- judicial
Remington Doctrine: Certiorari and Appeal can
be availed of in one case since they are directed Prohibition-To cease/desist from doing
to different acts/ orders. something

Example Mandamus- To do or perform a particular act


enjoined by law
A vs X/Y- Civil Case
*When a criminal information is already filed in
X and Y filed a motion to dismiss but it was court, it shall exercise absolute discretion over
subsequently denied. The denial for an order of it. (Crespo vs Mogul)
MTD is an interlocutory order. X filed a petition
for certiorari towards the appellate court -Every Motion filed in court, the court has the
against the interlocutory order. In the course of discretion whether to grant or deny it and it
the proceeding final judgment was rendered by cannot be subjected to Mandamus.
the court a quo therefore it has a legal remedy
of appeal. Case Doctrine: Social Justice System vs
Atienza- Based on the ordinance enacted by the
If A files a case against X and a Motion to City Council of Manila, it is ministerial to
Dismiss was filed relocate them based on the prevailing
ordinance due to imminent threat.
Granted – It is considered as Final Order and
the remedy is APPEAL. Prohibition vs Preliminary Injunction

Denied – It is considered as Interlocutory Order Prohibition Preliminary Injunction


therefore the remedy is Certiorari. Grounds: Grounds:

Blake filed a case against DeAndre in the Lack of jurisdiction Valid Cause of Action
amount of Php 275,000 towards the RTC and
judgment was rendered in favor Blake. Excess of jurisdiction Violation of Right

Rolan Jeff A. Lancion


Arellano University School of Law Page 51
Remedial Law 2- Brondial Notes AY 2017- 2018

Grave abuse of Cause irreparable *When a party filed a petition for certiorari,
discretion damage prohibition or mandamus, the court will not
Nature: Nature: issue summons to the respondents.

Special Civil Action Ordinary Civil Q: How will the court acquire jurisdiction over
the person of the respondent?
Case: UP Board of Regents vs Judge Ligot-Telan
A: The court shall acquire jurisdiction over the
Facts: Nadal (law student) applied for a respondent which can either be a tribunal,
scholarship in UP Law but it was later on board, officer, corporation or natural person in
discovered that there is misrepresentation. He which the petition should be valid in form and
was expelled by the UP Board but he applied for substance. The court shall issue an order for the
reconsideration in which downgraded the respondent to file comment or opposition to
penalty to Honorable dismissal. Nadal filed an the petition.
injunction, certiorari, prohibition/mandamus
against UP Board for his reinstatement in which -Upon receipt of the order to file
Judge Ligot-Telan granting its petition. comment/opposition, that is the time the court
acquires jurisdiction over the respondent.
Issue: WON Mandamus shall be applicable in
this case? -For Court of Appeals and Supreme Court, no
need to set for hearing of the petition.
Held:
Q: Can the court dismiss the petition through a
NO- Mandamus is not applicable since UP minute resolution?
exercise a valid academic freedom. SC
reiterated the doctrine in Arellano vs Cui that A: Yes it can be dismissed on the grounds of (a)
school has an academic freedom to whom they patently without merit and (b) filed manifestly
will accept in their institution. UP has therefore to incur delay
the discretion in this case that the court should
respect therefore Mandamus shall not prosper. Q: Should the court acquire jurisdiction over the
petition after it was dismissed?
Common Provision (Sec 4-9)
A: YES
-Number of Copies of the petition:
-If the respondent already filed their
Supreme Court- 5 copies comment/opposition to the petition, the court
can already resolve it based on the papers
Court of Appeals- 3 copies submitted to them since issues are already
joined.
-Parties should observe efficient paper rule
-The court still has the discretion if they will
need corresponding additional paper, oral
argument or memorandum.

Rolan Jeff A. Lancion


Arellano University School of Law Page 52
Remedial Law 2- Brondial Notes AY 2017- 2018

Q: What is the remedy of a party in a decision Case: AL Ang Network, Inc vs Mondejar 714
of certiorari of Court of Appeals? SCRA 1/28/14

A: The remedy shall be Rule 45- Petition for


review on certiorari towards the Supreme Court
based on pure questions of law. Facts:

Q: Is it possible to file a certiorari on a decision


for certiorari?
-A.L Ang Network filed a small claims complaint
A: YES, Certiorari is under the concurrent before the MTC against Sps. Mondejar for the
jurisdiction of RTC, CA and SC provided that the amount of Php 23, 111.71 for unpaid waterbill.
three grounds are available.

-The court shall be a nominal party in a petition


for certiorari, prohibition & mandamus -MTC rendered a decision in favor A.L Ang
Network ordering the Sps. Mondejar to pay.
-The court in this case is not a real party in
interest.
-RTC dismissed the petition for certiorari finding
-Even if the court is a principal party in his
petition, the private respondent should also be it was only filed to circumvent the rules of small
included. claims that it is not non-appelable.

Example:
Issue: WON the petition for certiorari is
RTC- A (Plaintiff) vs XYZ (defendant)
erroneously filed
CA- XYZ (Petitioner) vs Judge K of RTC Manila &
A (Respondents)
Held:
SC- XYZ (Petitioner) vs Judge K of RTC Manila &
A (Respondents)

-The judge of court is prohibited to participate NO- In Small Claims Cases, the decision shall be
in the proceeding. final and executory. The court ruled that
certiorari may only be invoked when there is no
-The private respondent shall be the one to file
appeal nor plain, speedy and adequate remedy
their comment/opposition.
in the course of law.
CASES:

Rule 65- Certiorari

Rolan Jeff A. Lancion


Arellano University School of Law Page 53
Remedial Law 2- Brondial Notes AY 2017- 2018

- A petition for certiorari is an original action


designed to correct only errors of jurisdiction
and not of judgment. Held:

-Considering that Small Claims are within the NO- A petition for certiorari should be instituted
exclusive jurisdiction of MTC, MTCC, petition for within a 60-day period from notice of judgment,
order or resolution. The 60 day period is
certiorari assailing its disposition should be filed
with the RTC exercising concurrent jurisdiction. inextendible to avoid unreasonable delay that
would violate constitutional right.

-A judgment of acquittal in criminal case may be


Case: People vs Hon. Castaneda Jr. 712 SCRA assailed in a petition for certiorari, it must be
12/11/2013 shown that there was grave abuse of discretion
amounting to lack or excess of jurisdiction or
denial of due process.
Facts:

-On November 5, 2011, Garcia and Vestidas Jr


-The resolution issued by the CTA is in
were charged in the City of Manila for accordance with the rules of evidence and
unlawfully and fradulently import into the port jurisprudence.
of Manila 858 cartons of Anti-Virus Kaspersky.

Case: Maglalang vs PAGCOR, 712 SCRA


- On January 15, 2013, Garcia and Vestidas, Jr.
12/11/2013
filed their Omnibus Motion to File Demurrer to
Evidence with Leave of Court to Cancel Hearing
Scheduled on January 21, 2013,whichwas
grantedby the CTA. Thereafter, they filed Facts:
theDemurrer to Evidence, dated January 13, - Mark Magalalang was a teller at Casino Filipino
2012, claimingthat the prosecution failed to in Angeles City, Pampanga. On December 13,
prove their guilt beyond reasonable doubt. 2008, an incident happened wherein he and
customer Cecilia had a heated argument.

Issue: WON CTA abused its discretion in


dismissing the criminal case

Rolan Jeff A. Lancion


Arellano University School of Law Page 54
Remedial Law 2- Brondial Notes AY 2017- 2018

-Magalalang received a Memorandum from -One cannot file petition for certiorari where
PAGCOR informing him that he was charged appeal is available even if the ground availed of
with discourtesy towards a customer. He is grave abuse discretion.
received a penalty of 30-day suspension. He
filed a motion of appeal but it was denied by
the board. -The dismissal of the petition for certiorari on
the basis of non-exhausation of admin remedies
should be set aside.
-Magalalang filed a petition for certiorari
towards CA but it was denied on the ground of
failure to exhaust administrative remedies. Case: UP Board of Regents vs Judge Ligot-Telan
and Ramon Nadal 227 SCRA

Issue: WON the CA erred in its decision to


dismiss the certiorari Doctrine: “A university has an academic
freedom and court cannot interfere with it”

Held:
Facts:

YES- Under the doctrine of exhaustion of admin -Ramon Nadal applied for a STFAP or
remedies, a party should availd first of scholarship since he was enrolled in the college
administrative process before going to court of Law in UP.
intervention with the exception that no admin
review is provided by law
-The Scholarship office found discrepancy in the
STFAP application of Nadal. SDP rendered a
-Decision of administrative or quasi-admin decision against Nadal which is expulsion from
agencies which are declared by law final and the University.
unappelable are subject to judicial review if
they fail the test of arbitrariness or grave abuse
of discretion. -Nadal appealed this decision to the BOR and it
was reduced to Honorable dismissal.

Rolan Jeff A. Lancion


Arellano University School of Law Page 55
Remedial Law 2- Brondial Notes AY 2017- 2018

-Nadal filed with the RTC of QC a petition for -One day the discovered that they are no longer
Mandamus and TRO against UP to admit him to owner of the parcel of lots since Pres. Marcos
school. Judge Ligot-Telan granted the petition through PD 293 cancelled all the titles
and ordered UP to accept Nadal. belonging to occupants in Caloocan and was
transferred to the members of PSG.

Issue: WON Judge Ligot-Telan abused her


discretion in granting the petition of Nadal -After the EDSA Revolution, Sps Tuason filed a
petition for certiorari towards the Supreme
Court assailing the validity of such PD.
Held:

Issue: WON the petition for certiorari can be


YES- The judge abused its discretion when it applied to question the Presidential Decree
disregarded the issue of academic freedom issued by President Marcos
which provides ample justification for the
impostion of disciplinary sanction.
Held:

-By issuing a writ of Preliminary Injunction and


Mandamus, it premptorily curtailed the YES- A petition for certiorari may be issue to
academic freedom. nullify a judicial or quasi-judicial act.

Case: Sps Tuason vs Register of Deeds, - The Presidential Decree may be struck down
Caloocan City, 157 SCRA by the writ of certiorari because it is done by an
officer in the performance of a judicial function.

Facts:
- The acts of nullifying the title were completely
-Sps Tuason were retired public school alien to the executive acts since it is a judicial
teachers. The bought a land located in Caloocan function therefore petition for certiorari is the
City. proper remedy.

Rolan Jeff A. Lancion


Arellano University School of Law Page 56
Remedial Law 2- Brondial Notes AY 2017- 2018

- In order for the Court of Appeals to acquire


jurisdiction over the respondent: (a) service of
Province of Leyte vs Energy Development 760 the order or resolution indicating the CA's initial
SCRA 149 action or voluntary submission of the
respondent to the jurisdiction of CA.

Facts:

- In 2006 & 2007, the Province of Leyte issued 4 - The petition for shall indicate the material
franchise tax assessment against EDC. dates showing the notice of judgment, final
order or resolution was received and a Motion
for New Trial and Motion for Reconsideration
was filed and when notice of denial was
- Province of Leyte issued another tax
received.
assessment on 2008 against EDC. EDC filed a
Preliminary Injunction with a prayer for
assessing or collecting franchise tax but it was
denied by the RTC. -Failure of the petitioner to comply with the
requirements shall be a ground for the dismissal
of the petition.

- CA denied the petition for certiorari filed by


the Province of Leyte due to failure to provide
proof of service.
Rule 65- Prohibition

Issue: WON CA erred in dismissing the petition


for certiorari
Case: Vivas vs Monetary Board of BSP, 703
SCRA 8/7/2013

Held:
Facts:

YES- In a petition for certiorari, there is a need -Rural Bank of Faire was a registered bank in
for the Court of Appeals to acquire jurisdiction Cagayan Valley. Vivas and his principals
over the person of the parties before in can acquired the controlling interest over RBFI.
resolve the merits. -BSP issued a certificate of authority to RFBI for
another 50 years. Monetary Board issued a

Rolan Jeff A. Lancion


Arellano University School of Law Page 57
Remedial Law 2- Brondial Notes AY 2017- 2018

resolution placing RBFI under prompt corrective Facts:


action. Vivas moved for a reconsideration of
such resolution. -Corales was the duly elected Mayor of
Nagcarlan, Laguna from 1998 to 2004 and he
-Monetary Board issued a resolution placing appointed Dr. Angeles to the position of
ECBI under receivership. Vivas filed a petition Municipal Administrator. The Sangguniang
for prohibition on the ground of grave abuse of Bayan disapproved the resolution on the
discretion in prohibiting to continue its ground of nepotism.
business.

-Petitioner Corales filed a petition for


Issue: WON the petition for prohibition is prohibition against Andal and Sangguniang
proper Bayan to desist from collecting reimbursement
from the advances paid.

Held:
-OSG on behalf of Andal filed a Motion to
NO- Prohibition is a preventive remedy seeking Dismiss but it was denied by the RTC however
that a judgment be rendered which would CA reversed such decision.
direct the defendant from continuing with the
commission of an act.

Issue: WON the petition for prohibition filed by


Corales is premature
-The petitioner did not observe hierarchy of
courts since prohibition can be filed with the
RTC, CA or SC exercising concurrent jurisdiction.
Held:

-SC also ruled that this case is not of


transcendental importance since it did not YES- Prohibition being a preventive remedy to
involve any public interest issue. seek a judgment ordering the defendant to
desist from continuing the commission of an act
provided that there is no appeal or any other
plain, speedy and adequate remedy in the
Case: Corales vs Republic 703 SCRA 8/27/2013 course of law.

Rolan Jeff A. Lancion


Arellano University School of Law Page 58
Remedial Law 2- Brondial Notes AY 2017- 2018

-Petitioners have all the remedies available to Held:


them at the admin level but they failed to
exhaust the same and instead sought judicial NO- The writ of prohibition is a remedy to
intervention. Before a party may seek the prevent unlawful and oppressive exercise of
intervention of court, he should avail of all the legal authority. The writ shall not lie to correct
administrative process. errors of judgment but only errors of
jurisdiction.

Case: Tan vs Court of Appeals, 524 SCRA


-There was no evidence on record to support
petitioners claim that they were forced
arraigned. Absent proof of force or intimidation,
Doctrine: “Writ of Prohibition shall issue to cure the trial judge enjoys the presumption of
errors of jurisdiction” regularity.

Facts: Rule 65- Mandamus


-James Kind charged Roderick Go, et, al with
violation of BP 22 and estafa.
Case: Hipos Jr. vs Judge Bay, 581 SCRA
-Sps Go proposed to Mr. King a business 3/17/2009
transaction wherein they would borrow cash
from the latter in exchange of Roderick Go’s
postdated check.
Facts:
-King filed a criminal complaint in the fiscal’s
-On December 15, 2003, two information for
office and prosecutor Montero found probable
cause to file an information in court. the crime of rape and acts of lasciviousness
were filed against Hipos towards the Family
-Petitioner filed a TRO before the Court of Court.
Appeals to restrain from proceeding the case
but it was dismissed. -Private Complainants filed a motion for re-
investigation for the proper determination of
probable cause.

Issue: WON the remedy of prohibition is proper

Issue: WON the writ of mandamus is the proper


remedy to compel Judge Bay to dismiss the case

Rolan Jeff A. Lancion


Arellano University School of Law Page 59
Remedial Law 2- Brondial Notes AY 2017- 2018

-Petitioner a constable was discharged from the


service for allegedly losing his firearm while
Held: Meteoro were also discharged for being absent
No- Mandamus is an extraordinary writ without leave.
commanding to do an act required to be done, -Napolcom issued a resolution considering as
when the respondent unlawfully neglects the absorbed into police force those who had been
performance of an act which law specifically discharged by virtue of pending criminal or
enjoins. administrative case. Petitioner Sanchez and
Meteoro are on the list.

-Mandamus lies only to compel an officer to


perform a ministerial act and not discretionary. -Without any action from the PNP chief,
Mandamus is never available to direct the petitioner filed a petition for Mandumus in
exercise of judgment or discretion in a which the RTC granted. The petition contains
particular way or reversal of an action already
that the PNP chief is mandated to accept them
taken. back to service. The decision of the RTC was
reversed by the Court of Appeals.

-It was held in Crespo vs Mogul that once a


criminal complaint or information is filed in Issue: WON the cause of action filed by Sanchez
court, any disposition or dismissal of the and Metoro should be granted
case/accused rest with the jurisdiction or
discretion of the trial court. Held:

NO-Mandamus is employed only to compel the


performance, when refused of a ministerial but
not to require anyone to fulfill a discretionary.
Case: Sanchez vs Lastimosa 534 SCRA 9/25/07 -The writ can only be issued when the
applicants legal right to the performance of a
particular act sought to be compelled is clear
Doctrine: “The PNP absorption is not a and complete.
ministerial act by the office”
-The PNP Chief's issuance of the orders for
absorption of herein petitioners is not
compellable by mandamus since it does not
Facts:
involve a ministerial duty.

Rolan Jeff A. Lancion


Arellano University School of Law Page 60
Remedial Law 2- Brondial Notes AY 2017- 2018

Case: Social Justice System vs Mayor Atienza,


517 SCRA 3/07/2007
-The objective of the ordinance is to protect the
Facts: residence of Manila from possible terrorist
attack and Mayor Atienza should implement
-On November 2001, the Sangguiniang such ordinance immediately.
Panglungsod ng Maynila enacted an ordinance
on which the land in Pandacan shall be Case: Villanueva vs JBC, 755 SCRA 182
reclassified from industrial to commercial and
directed the oil operators to cease and desist its Doctrine: “Selection of nominees for judicial
operation. vacancy is discretionary”

Facts:

-The City of Manila and DOE entered into a - Villanuevas was appointed as the Presiding
Judge of MTC in Compostela Valley. After 1 year
Memorandum of Understanding that will
provide a better safety related measures. of office as MTC judge, he applied for the
vacant position of RTC Judge in Tagum City.

-JBC informed that Villanueva is not included in


-Social Justice filed a petition for Mandamus the short list of applicants since he did not
against Mayor Atienza to implement such possess the 5 year prescribed for being an RTC
ordinance. judge.

Issue: WON the petitioner has the legal duty to -He filed a petition for certiorari to prevent the
enforce such ordinance JBC from perfoming its function of selection and
mandamus to include his name in the short list.
Held:

YES- Mandamus can filed when ay tribunal,


corporation, board, officer or natural person Issue: WON Villanueva has a cause of action to
neglects the performance of an act which the file a petition for certiorari and mandamus in
law enjoins. this case

-The principal function of Mandamus is to Held:


command and expedite and in this case the
petitioners who are interested in the execution
of the laws shall be considered as real party in NO- The petition for certiorari and prohibition
interest. can only be applied to correct errors of

Rolan Jeff A. Lancion


Arellano University School of Law Page 61
Remedial Law 2- Brondial Notes AY 2017- 2018

jurisdiction committed by tribunal, board, exceptions: (a) grave violation of the


officer, corporation and natural person. JBC Constitution, (b) exceptional character of the
does not fall within the scope of those situation, (c) constitutional issue requires
mentioned. formulation of controlling principle and (d)
capable of repetition.
-Mandamus cannot be availed by Villanueva in
assailing the policy of JBC. It shall not apply -An allegation as serious as a violation of a
when such instrumentality is exercising constitutional or legal duty, coupled with the
discretionary function and the process of pressing public interest in the resolution of all
judicial position is discretionary and not related issues, prompts this Court to pursue a
ministerial. definitive ruling thereon, if not for the proper
guidance of the government or agency
Case: Funa vs Manila Economic and Cultural concerned, then for the formulation of
Office 715 SCRA controlling principles for the education of the
Facts: bench, bar and the public in general.

-Dennis Funa sent a letter to Commission on -Another issue is concerned with the petition is
Audt requesting for a copy of the latest financial non-observance of the principle of hierarchy of
and audit report for the purpose of his right to courts. The COA assails the filing of the instant
information towards matters of public concern. mandamus petition directly with this Court,
when such petition could have very well been
-Commissioner Narank revelead that MECO was presented, at the first instance, before the
not among the agencies audited by any Court of Appeals or any Regional Trial Court.
Corporate Sector.
The COA claims that the petitioner was not able
-Funa filed the instant petition for mandamus. to provide compelling reasons to justify a direct
He impleaded COA and MECO. resort to the Supreme Court. In view of the
transcendental importance of the issues raised
-COA on argue that the petition already become in the mandamus, petition, as earlier
moot and academic since chairperson Tan mentioned, this Court waives this last
directed a team of auditors to account of procedural issue in favor of a resolution on the
MECO.
merits.
Issue: WON the petition for Mandamus may be
denied on ground of moot & academic Rule 66- Quo Warranto

Held: Literal Meaning- “What Authority a person is


occupying an office or franchise”
NO- The Court should decide the case otherwise
moot & academic subject to the following

Rolan Jeff A. Lancion


Arellano University School of Law Page 62
Remedial Law 2- Brondial Notes AY 2017- 2018

Section 1. Action by Government against election. Dra Rodolfo filed a petition towards
individuals. — An action for the usurpation of a the COMELEC to disqualify Lonzanida on the
public office, position or franchise may be ground that he already served 4 consecutive
commenced by a verified petition brought in terms.
the name of the Republic of the Philippines
against:

(a) A person who usurps, intrudes into, or - Comelec 2nd division rendered a decision
unlawfully holds or exercises a public office, cancelling the COC fo Lonzanida. During the
position or franchise;
pendency of the action, Lonzanida and Aratea
won respectively.
(b) A public officer who does or suffers an act
which, by the provision of law, constitutes a
-Comelec en banc issued a resolution
ground for the forfeiture of his office; or
disqualifying Lonzanida.
(c) An association which acts as a corporation
Issue: WON Aratea the 2nd placer can assume
within the Philippines without being legally
incorporated or without lawful authority so to public office in lieu of the disqualified winner
act.
Held:
- It is a proceeding assailing the authority or by YES- Lonzanida lacks eligibility under the
what authority a person is occupying a position.
present constitution and statutory provision
-The subject matter is a public office. since he hold public office for 4 consecutive
terms.
Case Doctrine: Calleja vs Panday- The Quo
Warranto is dismissed because is settled is the -In the quo warranto petition, the ground to
rule that it shall only be applicable to public oust an elected official are ineligibility and
offices. disloyalty to the Republic. While quo warranto
and cancellation share the same ineligibility
Q: What kind of public office shall quo warranto grounds as they from the grounds cited. Under
apply? the election code, the disqualification for
elective office are misrepresented in the COC
A: It shall be applicable only to appointive office
while under the rules it must ineligibility and
since an elected office is governed by the
Omnibus Election Code. disloyalty to the Republic and must be initiated
within 10 days after proclamation.
Case: Aratea vs COMELEC 683 SCRA
Case: Lokin Jr vs COMELEC, 621 SCRA
Facts:
Facts:
-Lonzanida and Antipolo were candidates for
Mayor of San Antonio, Zambales in the 2010

Rolan Jeff A. Lancion


Arellano University School of Law Page 63
Remedial Law 2- Brondial Notes AY 2017- 2018

-CIBAC was a registered group under the party A: The Solicitor General or a public prosecutor
list system. Bro Eddie Villanueva, the president may, with the permission of the court in which
of CIBAC listed 5 nominees from which the the action is to be commenced, bring such an
representatives would be choosen and Lokin action at the request and upon the relation of
was the 2nd nominee. another person; but in such case the officer
bringing it may first require an indemnity for
the expenses and costs of the action in an
-CIBAC filed an ammended list of nominees amount approved by and to be deposited in the
wherein Lokin et,al were dropped. court by the person at whose request and upon
whose relation the same is brought. (Rule 66
Sec 3)

-CIBAC won in the party list and gained 2 seats Q: What is the issue of quo-warranto in an
in the congress. COMELEC en banc proclaimed election code?
Cruz-Gonzales as the 2nd official nominee.
A: The ineligibility of the public officer to hold
office under the election code but in Rule 66 it
-Lokin Jr filed a petition for mandamus to pertains to disqualification of public officer.
compel COMELEC to declare him as the 2nd
nominee. -When a person usurps a public office, he is
subject to Quo Warranto. He shall be ousted
from office and the one who has claim to that
office must be reinstated.
Issue: WON Quo Warranto under the Rule 66 is
applicable in a election protest Q: Which court shall have jurisdiction over a
petition of quo warranto?
Held:
A: Concurrent jurisdiction of RTC, CA and SC.
NO- Election protest is strictly between
defeated and winning candidate based on the Q: What shall be the venue in a petition for quo
grounds of electoral frauds and irregularity. warranto?

-Neither does an action for quo warranto will lie A: An action under the preceding six sections
since considering that the case does not involve can be brought only in the Supreme Court, the
the ineligibility and disloyalty of Cruz-Gonzales Court of Appeals, or in the Regional Trial Court
to the Republic of the Philippines or some other exercising jurisdiction over the territorial area
cause of disqualification of her. where the respondent or any of the
respondents resides, but when the Solicitor
Q: What instrumentality of the government can
General commences the action, it may be
file a case of quo warranto?

Rolan Jeff A. Lancion


Arellano University School of Law Page 64
Remedial Law 2- Brondial Notes AY 2017- 2018

brought in a Regional Trial Court in the City of -Mendoza filed a Motion for execution with
Manila, in the Court of Appeals, or in the court a quo but it was denied since Allas is no
Supreme Court. (Rule 66, Sec 7) longer holding public office.

Exception: A private individual can commence Issue: WON a petition for quo warranto can be
an action for quo warranto provided he has a applied against the new person occupying the
claim to the public office. public position

Held:
Q: Can a Third party who does not have a claim
to the public office file an action for quo NO- A petition for quo warranto is a proceeding
warranto? to determine the right of a person to the use of
exercise of a franchise or public office and to
A: YES, He is called as “Relator” oust its enjoyment.

-A third party complainant who has no claim to -A judgment in quo warranto does not bind the
the public office but can initiate a petition for respondent's successor in office even though
quo warranto. such successor may trace his title to the same.

-The OSG will help to facilitate the action. -A quo warranto is never directed against an
office but against the person to determine if he
Case: Mendoza vs Allas, 302 SCRA is constitutionally and legally authorized to
perform such act.
Facts:
Section 11. Limitations. — Nothing contained in
-Mendoza filed a petition for quo warranto this Rule shall be construed to authorize an
against Allas who was appointed by Former action against a public officer or employee for
President Ramos while he was assigned in his ouster from office unless the same be
Cagayan de Oro. commenced within one (1) year after the cause
of such ouster, or the right of the petitioner to
-The trial court ruled in favor of Mendoza and hold such office or position, arose, nor to
ordered Allas to be ousted from office. authorize an action for damages in accordance
with the provisions of the next preceding
-Allas appelead the decision towards the Court section unless the same be commenced within
of Appeals, during its pendency Allas was one (1) year after the entry of the judgment
promoted as the Deputy Commissioner establishing the petitioner's right to the office in
question.
therefore he filed a motion to dismiss the
appeal. Damages: Prescriptive Period

For petition: 1 year from the accrual of the


action

Rolan Jeff A. Lancion


Arellano University School of Law Page 65
Remedial Law 2- Brondial Notes AY 2017- 2018

For damages: 1 year from the entry of -An action for quo warranto is not applicable in
judgment usurpation of power under a private
corporation since it is only limited to persons
-Damages shall be filed in a separate action who usurp public office/corporation.
CASES Case: De Castro vs Carlos 696 SCRA 4/16/2013

Case: Calleja vs Panday 483 SCRA Facts:


Doctrine: “Quo Warranto is applicable only to - PGMA appointed Emmanuel De Castro as
usurpation of public office” AGMO of MMDA and it was affirmed by the
member of Metro Manila Council.
Facts:
-During the change of administration, De Castro
- Panday filed a petition for with the RTC of
was reassigned to the legal and legislative
Camarines Sur for quo warranto with damages.
affairs office and his service vehicle was
- Panday alleged that from 1985 up to the filing withdrawn.
of the petition with court that they were
- De Castro demanded payment of his salary
members of the board of directors of St. John
and reinstatement towards the Office of the
Hospital.
President.
-On May 2005, Callega et.,al, who were
-President Aquino appointed Emerson Carlos as
incorporators and stockholders of the
the new AGMO of MMDA that prompted De
corporation forcibly and with aid of armed men
Castro to file an action for quo warranto to the
usurped the power in such institution.
Supreme Court.
- The trial court granted the petition filed by
Issue: WON the petition for quo warranto is
Panday.
proper for his reinstatement

Held:
Issue: WON the petition for quo warranto is the
NO- A petition for quo warranto is a proceeding
proper remedy in a usurpation of power in a
to determine the rights of a person to use or
private corporation
exercise of a franchise or office and to oust the
Held: holder from the enjoyment thereof. When the
action is filed by a private person in his own
NO- Quo Warranto is limited only against name, he must prove that he is entitle to the
person who usurps a public office, position.
position/franchise, public officer who forfeit
their office.

Rolan Jeff A. Lancion


Arellano University School of Law Page 66
Remedial Law 2- Brondial Notes AY 2017- 2018

-Considering that De Castro is appointed by -Petition for Mandamus is proper in this case in
PGMA whose term already ended, his term of this case since the taking of oath of office is no
office also already ended. longer discretion since there was already a
judgment rendered by COMELEC that Reyes is
-The person suing must show that he has a clear disqualified to hold office.
right to the office but in this case it already
cease to exist.

Case: Velasco vs Speaker Belmonte 779 SCRA


Rule 67- Expropriation
81
Inherent Rights of the State
Facts:
-Eminent Domain
-Velasco and Reyes were candidates for the
position of Representative of Marinduque. -Police Power
Joseph Tan filed a petition to the COMELEC to
disqualify Reyes. -Power of Taxation

-Comelec grant the petition and cancelled the Constitutional basis: “No property shall be
COC of Reyes but in the election she won for taken without payment of just compensation”
the position of congress representative. The
Eminent Domain
Comelec En Banc affirmed the decision of the
division. Basis: Regalian Doctrine- All lands belong to the
state and he who claims otherwise has the
-Velasco filed a petition for Mandamus towards
burden of proof
Speaker Belmonte to administer the oath in his
favor. Substantive Law: CA 141  PD 1521

Issue: WON the petition filed by Velasco is Q: To what does the property refer in this
proper provision?

Held: A: Real or Personal Property can be subject of


expropriation.
YES- A petition for quo-warranto is a proceeding
to determine the right of a person to the use or -Personal Properties shall be governed by
exercise of a franchise or office. The special laws.
action/petition is the enforcement of clear legal
duties and not to try disputed title therefore it Example: Ill-Gotten Wealth Law or Anti Money
is not quo warranto. Laundering Act

Rolan Jeff A. Lancion


Arellano University School of Law Page 67
Remedial Law 2- Brondial Notes AY 2017- 2018

*The property provided in the Constitution is YES- under the ROC, it provides that there is no
not limited to real property but also includes requirement of filing of an answer or summary
Personal. judgment in expropriation case. Under this
case, there is no such thing as the matter of
Q: What is the concept of “Taking”? right to dismiss since the landowner has already
A: It is not limited to actual seizure or custody. suffered damage upon actual taking of the
There is already “taking” when there is property.
diminution of usefulness. If -The dismissal of expropriation case restores
utilization/usefulness of the property is gone,
the expropriated land but when possession of
there is actual taking therefore he is entitled to land cannot be turned over because it is neither
compensation. convenient nor feasible, the only remedy is to
Case: National Power Corporation vs CA, 436 pay just compensation and for this case the
SCRA property is no longer feasible for use or
occupancy.
Facts:
MIA CASE
-NPC is created to generate geothermal,
hydroelectric power while Antonio Pobre is an Manila International Airport expropriated
owner of 68,969 sq meter land located in Tiwi, several properties located within the airport
Albay. Pobre began developing the property as area. The adjacent owners of the properties
a resort subdivision which he named as Tiwi Hot also demand for compensation since they are
Spring Resort. owners of poultry farm and their chicken or
ducks cannot produce eggs.
-On 1977, NPC filed its expropriation case to
acquire an 8,311 sq meter of the property. In Ruling: There was already a taking of the
1979, NPC filed its 2nd expropriation case for property even if the adjacent lot was not
5,554 sq meters. covered of expropriation because its business
or livelihood of the owners situated already
-NPC filed a motion to dismiss to the 2nd case diminished therefore they are entitled to just
sunce they already found an alternative site for compensation.
such project. RTC rendered a decision in favor
of Pobre and it was affirmed by the Court of *The taking of the property has the purpose of
Appeals. public use

Issue: WON Pobre is entitled to just PUBLIC USE- It is not necessary beneficial to the
compensation for the entire property sought to entire public/community. Even only a portion of
be expropriated the membership in the community will be
benefited, it can be a subject of expropriation.
Held:

Rolan Jeff A. Lancion


Arellano University School of Law Page 68
Remedial Law 2- Brondial Notes AY 2017- 2018

As long as the common good is promoted it Case Doctrine: Asia Emerging Dragon vs DOTC
shall be for public use.
- Before the government can take
Example: Pink urinals constructed by former over/possession of the property, the have to
MMDA Chairman Bayani Fernando. pay 100% of the zonal valuation for judicial
deposit. It shall only be applicable towards
Formula for Just Compensation: infrastructure projects.
JC (Just Compensation) = FMV (Fair Market Example of Infrastructure Project: MRT/LRT
Value) + Consequential Damages –
Line Extension
Consequential benefit which should not be
more than the damages Note: A party cannot expropriate a government
property

FMV- Fair Market Value= The price where the Q: What if it is a government property but with
buyer is willing to buy and the seller is willing to individuals residing, can you still proceed with
sell. the expropriation?

Q: Can you apply the FMV strictly in the A; YES, Since based from Rule 67 Sec 1, all
payment of just compensation? person owning or claiming to own or occupying
any part thereof.
A: No, The owner shall be prejudiced of such
since the determination belongs to the Q: What is the jurisdiction of expropriation
government. case?

-Upon filing of the complaint, the government A: RTC- An action incapable of pecuniary
can immediately takeover provided they will estimation
pay deposit equivalent to the assessed value of
Q: Who can be the plaintiff in an action for
the land- AMMENDED
expropriation?
“The term deposit is already amended”-
A: The government can only be the plaintiff
BRONDIAL
under the exercise of eminent domain.
Q: What is the current value of judicial deposit
-The Local Government Code also grant that
for taking immediate takeover?
power to government owned and controlled
A: It must be 15% of the zonal valuation of the corp.
property.
Q: Can a Barangay council expropriate a
Zonal Value- Price declared per assessment of property?
the BIR

Rolan Jeff A. Lancion


Arellano University School of Law Page 69
Remedial Law 2- Brondial Notes AY 2017- 2018

A: NO. It must be with the concurrence of Bar Problem: Can Congress pass a law
Sangguniang panglungsod or pangbayan. expropriating certain portion of UST in Espana
to be paid through exchange or barter with
Q: What controls the power of GOCC? another 1 hectare property located in Calamba,
A: It will be their charter that will control their Laguna. The objective to build a statue of
power to expropriate. Arsenio Lacson.

Expropriation vs Reclamation Suggested Answer: NO, Since the


determination of just compensation is always a
Expropriation- Property shall be taken for judicial function.
public purpose provided that there is a payment
of just compensation. Case: City of Manila vs Serrano 359 SCRA

Reclamation- It is a mode of acquiring title over Doctrine: “The government should deposit an
the property amount equivalent to the assessed value of the
property”
Procedure in Court
Facts:
A) The court will issue an order of
condemnation -On December 21, 1993, the City Council of
- This order is a final order and the Manila enacted an ordinance authorizing the
remedy shall be record on appeal which expropriation of certain properties located in
is 30 days from notice. Tondo.
B) Determination of Just Compensation -One of the properties sought to be
- The Court is mandated to appoint 3 expropriated was derived from TCT issued in
Commissioners that will determine just the name of Feliza de Guia. It was later on
compensation. 1-Plaintiff 1-Defendant transferred to Lee Kuan Hui.
1- Judicial Court
- The members of the commission are -The trial court issued an order directing the
entitled to accept evidence and hear City of Manila to deposit the amount of Php
testimonies. 1,825,421.00 equivalent to the assessed value
- The Commission shall not issue a of the property. Serrano filed a petition for
judgment but only a report certiorari that the land sought to be
- The report shall not be conclusive upon expropriated are exempt from expropriation
the court but it was denied by the Court of Appeals.

*Determination of Just Compensation is always Issue: WON the property sought complied with
a judicial determination and it can never be a the procedure of expropriation
legislative or executive function.
Held:

Rolan Jeff A. Lancion


Arellano University School of Law Page 70
Remedial Law 2- Brondial Notes AY 2017- 2018

NO- The plaintiff shall have the right to take or Held:


enter upon the possession of the real property
if he deposited with the authorized government YES- A legal easement should be granted to the
depository an amount equivalent to the Republic since based on the title contained the
assessed value of the property. If personal reservation that the land is covered by LRA
property, its value shall be ascertained and the which can be subjected to right of way not
amount deposited shall be fixed by the court. exceeding 60 meters.

-A writ of execution may only be issued by a -The exercise of eminent domain occurs not
only when the government actually deprives
court upon the filing of a complaint sufficient in
form and substance and upon deposit made by the owner of his property of its ordinary use but
the government. also when there is a practical destruction or
material impairment.
Case: Republic vs Andaya, 524 SCRA
-Andaya is entitled to payment of just
Facts: compensation which is neither more or less
than the monetary value of the land.
-Andaya is the registered owner of two parcels
of land situated in Butuan City. These properties Case: NPC vs YCLA Sugar Developmet Corp 712
are subject to an easement for public highway, SCRA 550
irrigation and aqueducts.
Facts:
-The Republic instituted an action before the
RTC of Butuan City to enforce the easement of - In order to complete its Calapan-Mamburao
Island Grind Project. It shall construct a
right of way. Board of Commissioner rendered a
finding that the Easement would diminish the transmission line that would traverse several
value of the property. private properties.

-NPC filed a complaint for expropriation with


RTC against YCLA. It sought the expropriation of
-RTC rendered a decision that the plaintiff is the parcels of land owned by the defendants.
legally entitled to inherent right of
expropriation. CA imposed a 6% interest in the -RTC rendered a decision directing NPC to pay
consequential damage from the writ of Php 5 million representing the value of the land.
possession. It was affirmed by the Court of Appeals.

Issue: WON the RTC and CA correctly arrived at


Issue: WON the Republic is liable for just
compensation in enforcing the legal easement the amount of just compensation

Rolan Jeff A. Lancion


Arellano University School of Law Page 71
Remedial Law 2- Brondial Notes AY 2017- 2018

YES- LBP is tasked with the responsibility of


determining the value of the land placed under
Held: land reform and for its just compensation.
YES- Just Compensation is the full and fair -Basis of the just compensation for property
market value of the property taken from its under expropriation should either by the
owner by the expropriator. assessment from the government or sworn
-It shall also be ascertained as of the time of the valuation by the owner.
taking which is during the expropriation -The determination of just compensation is
proceeding. always a judicial determination.
-The amount of just compensation could only
be attained by a reliable and actual data. A
Commissioner's report of land shall be RULE 68- FORECLOSURE OF REAL ESTATE
considered as evidence in the determination of MORTGAGE
just compensation.
Sec 1- Complaint in action for foreclosure
-The trial court may accept or reject the report
submitted by the commissioner. -Date and due execution of the mortgage

Case: Limkaichong vs LBP, 799 SCRA 139 -Name and residence of the mortgagor

Facts: -Description of the mortgaged property

- Limkaichong was the registered owner of an -Statement of the date of the note
agricultural lands located in Negros Oriental. -Names and residence of all persons having or
DARAB sent to her notice for the land valuation claiming an interest in the property
and acquisition.
Two Types
-Limkaichong filed in the RTC of Dumaguete City
a complaint for the fixing of just compensation. - Judicial (Rule 68)
- Extra-judicial (Act 4118)
-RTC sitting as Special Agricultural Court
dismissed the complaint on the ground that it Applicability: When there is a mortgage of real
should be filed within 15 days from notice of property and always anchored on a loan.
assailed order.
Q: Who are the parties in a real estate
Issue: WON there was grave abuse of discretion mortgage?
in dismissing the complaint
A: *Mortgagee- Plaintiff *Mortgagor-
Held: Defendant

Rolan Jeff A. Lancion


Arellano University School of Law Page 72
Remedial Law 2- Brondial Notes AY 2017- 2018

-Considered to be necessary party in the action


of foreclosure
Last Par of Sec 1: The names and residences of
all persons having or claiming an interest in the If the Property is already foreclosed, it is already
property subordinate in right to that of the sold- WRONG IMPRESSION
holder of the mortgage, all of whom shall be
made defendants in the action. Section 2. Judgment on foreclosure for
payment or sale- If upon the trial in such action
-They shall be considered as necessary party, the court shall find the facts set forth in the
one to whom there can be no complete complaint to be true, it shall ascertain the
determination of the case. amount due to the plaintiff upon the mortgage
debt or obligation, including interest and other
-Without impleading all persons whose rights charges as approved by the court, and costs,
are subordinate to that of the mortgagor, there and shall render judgment for the sum so found
can be no complete determination of the case. due and order that the same be paid to the
court or to the judgment obligee within a
Q: Who are the necessary and indispensable
period of not less than ninety (90) days nor
party in an action for foreclosure of real estate
more than one hundred twenty (120) days
mortgage?
from the entry of judgment, and that in default
of such payment the property shall be sold at
A: The Owners of the property- Indispensable
public auction to satisfy the judgment.
Persons having a subordinate right- Necessary
-It pertains to judgment to pay
Party.

Time Frame: The same be paid to the court or


Q: Is the Mortgagor equivalent to the owner of
to the judgment oblige within a period of not
the property?
less than 90 days nor more than 120 days from
the entry of judgment.
A: Not necessarily since the owner can only give
his consent to effect such mortgage.
-Default of such payment, the property shall be
sold at a public auction.
Q: How about the possessor of the property?

Equity of Redemption- Period of not less than


A: It will depend if his rights are subordinate to
90 days nor more than 120 days from the entry
that of the holder.
of judgment to pay the amount of mortgage
Junior Encumbrances- Where the property debt.
subsequent to the mortgage.

Rolan Jeff A. Lancion


Arellano University School of Law Page 73
Remedial Law 2- Brondial Notes AY 2017- 2018

Right of Redemption- The 1 year period to -It is only through a motion filed in court.
redeem the property counted from the
registration of the certificate of sale. -When the sale of the property and the amount
is less than the judgment obligation. A party
-The judgment obligor can redeem the property shall file a motion in the court a quo for
along with the redemptioner deficiency judgment.

*If there is non-compliance of payment during -Once the deficiency judgment is granted, a
the period of equity of redemption, the party shall proceed using the remedies available
property can now be sold at the public auction. in Rule 39 (Sec 31,32 & 34)

Q: Who are the buyers of the property in the Q: What will happen if the property mortgaged
auction sale? is greater than the judgment obligation?

A: Usually the mortgagee A: It will return to the judgment obligor the


excess amount of the property but if there are
-The foreclosure sale must be confirmed by the junior encumbrances, it shall be given to them.
court ORDER OF CONFIRMATION and without
such confirmation, the equity shall not be cut. Q: is there a necessity of demand in real estate
mortgage?
-If a party appeals such judgment, the period
for equity of redemption shall not run. A: No need for a demand in foreclosure of real
estate.
Section 6 Deficiency Judgment- If upon the sale
of any real property as provided in the next Case Doctrine: Goldenway vs Equitable PCI
preceding section there be a balance due to the Bank: The notice is not necessary in extra-
plaintiff after applying the proceeds of the sale, judicial foreclosure.
the court, upon motion, shall render judgment
against the defendant for any such balance for Q: How do you file an application for extra-
which, by the record of the case, he may be judicial foreclosure of real mortgage?
personally liable to the plaintiff, upon which
execution may issue immediately if the balance A: A party can file an application towards the
is all due at the time of the rendition of the Branch Clerk of Court.
judgment; otherwise; the plaintiff shall be
entitled to execution at such time as the Q: Is there a right of redemption in extra-
balance remaining becomes due under the judicial foreclosure?
terms of the original contract, which time shall
A: It will depend on the contract between the
be stated in the judgment.
parties.

Rolan Jeff A. Lancion


Arellano University School of Law Page 74
Remedial Law 2- Brondial Notes AY 2017- 2018

Q:What is the period in extra-judicial NO- CA erred in ruling this case that failure of
foreclosure? Manila Banking Corp to personal notice
constitutes breach of contract.
A: It will depend on the nature of the contract
between the contract. -Personal Notice to the mortgagor in extra-
judicial foreclosure is not necessary but only
Writ of Possession- Not a separate action, it is requires the posting of the notice of sale in
only filed through a Motion and Summary three public places and the publication of that
Hearing is necessary. notice in a newspaper of general circulation.

Case: Sps. Marquez vs Alindog 714 SCRA


Exception: When the property belong to a third
party who is not a party to the mortgage Facts:
contract.
-Anita Marquez extended a loan in the amount
Cases: of Php 500,000 to certain Benjamin Gutierrez,
he executed a deed of Real Estate Mortgage
Case: Ramirez vs Manila Banking Corp, 712 contract located in Tagaytay.
SCRA, 12/20/2013
-Gutierrez defaulted in the payment of his loan
Facts: obligation. Anita sought the extra-judicial
foreclosure. At the public auction, Anita
-Ramirez mortgaged 2 parcels of land located at
emerged as the highest bidder.
Marikina City in favor of Manila Banking Corp.
-Sps. Alindog filed a civil case for annulment of
-Manila Bankers filed a request for extra-judicial
real estate mortgage and certification of sale.
foreclosure of real estate mortgage before Atty.
RTC issued a preliminary injunction enjoining
Sanez due to the failure of Ramirez to pay his
Sps. Marquez from taking possession of the
loan.
property.
-Ramirez filed an action for the annulment of
sale and it was granted by the RTC.
Issue: WON the Trial Court erred in granting the
-CA reversed the decision since of personal
writ of preliminary injunction after the lapse of
notice of foreclosure to Ramirez is not a ground
the redemption period.
to set aside the foreclosure sale.

Issue: WON personal notice is necessary in


extra-judicial foreclosure Held:

Held:

Rolan Jeff A. Lancion


Arellano University School of Law Page 75
Remedial Law 2- Brondial Notes AY 2017- 2018

YES- The purchaser in an extra-judicial Held:


foreclosure is entitled to the possession of the
property and can demand that he be place in YES- Personal notice to the mortgagor in extra-
possession during or after the expiration of judicial foreclosure is not necessary hence not a
redemption period. ground to set aside the foreclosure sale.

-A writ of possession in favor of the purchaser -Issue of lack of publication of notice cannot be
in a foreclosure sale is ministerial towards the raised for the first time on appeal.
RTC with the exception that a third party is
actually holding the property by adverse title or
right. Case: LZK Holdings vs Planters Development
Bank, 714 SCRA
-The issuance of a writ of possession is already a
ministerial function of the RTC. Facts:

Case: Ardiente vs Provincial Sheriff, 436 SCRA -LZK Holdings obtained a Php 40 million loan
from PDB and it was secured the same with a
Facts: real estate mortgage over its land located in La
-Sps Ardiente obtained a loan from Peninsula Union. The Lot was sold at public auction after
Development Bank secured by a real estate PDB extrajudicially foreclosed the property due
mortgage over the land. Ardiente purchased a to LZK's failure to pay the loan.
mini bus coming from the obtained loan. This -LZK Holdings filed before the RTC of Makati
mini bus figured in an accident and as a result City for annulment of extra-judicial foreclosure
Sps. Ardiente failed to pay the loan. of real properties. 3 days before the expiration
-PDB extra judicially foreclosed the mortgage of LZK's redemption, RTC issued a TRO.
and it was sold in a public auction. Two (2) days -RTC of Makati rendered a decision declaring as
before the expiration of the redemption period, null and void the consolidated title of PDB.
Sps. Ardiente filed a complaint for the
annulment of the auction sale. Issue: WON in a petition for issuance of writ of
possession may be dispensed when the court
-RTC rendered a decision dismissing the sale on already issued an order.
the ground that strict compliance on the
publication was not complied with. Held:

Issue: WON publication is necessary in sale of a YES- Under the principle of conclusiveness of
property in extra-judicial foreclosure of judgment, the right of PDB to a writ of
mortgage possession is binding and conclusive towards
the parties.

Rolan Jeff A. Lancion


Arellano University School of Law Page 76
Remedial Law 2- Brondial Notes AY 2017- 2018

-The proceeding in a petition for a writ of within Register of Deeds, which in no case shall
possession is ex-parte and summary in nature. be more than 3 months per foreclosure.
It is a judicial proceeding brought for the
benefit of one party and without notice by the Case: Robles vs Yapcinco, G.R. No. 169568,
court to any person adverse of interest. October 22, 2014.

Case: Goldenway Merchandising vs Equitable Facts:


PCI Banks, G.R. No. 195540 March 13, 2013 -Yapcinco constituted a mortgage on the
Facts: property located in Tarlac in favor of Marcelo to
secure the performance of his obligation. When
-Goldenway executed a real estate mortgage in Yapcinco failed to pay the obligation, Cruz
favor of Equitable PCI Bank over it properties brought an action for judicial foreclosure.
situated in Valenzuela City. When Goldenway
failed to settle its obligation, PCI Bank extra- -Apolinario Cruz donated the said property to
judicially foreclosed the mortgage. his grandchildren but Bernabe falsified a deed
of sale whereby he made it appear that
-The counsel of Goldenway offered to redeem Yapcinco sold the property to him.
the foreclosed property but PCI Bank
mentioned that such redemption is not possible -All heirs of Yapcinco instituted an action for
since the certificate of sale had already been annulment of TCT which Bernabe falsified, RTC
registered. rendered a decision in favor of the heirs.

-Goldenway filed a complaint for specific -CA reversed the decision of RTC stating that an
non-registration of its certificate of sale will
performance and damages but it was dismissed
by the RTC. have an effect that the period of redemption
shall not begin to run.
Issue: WON PCI Bank can be compelled to sell
the foreclosed property. Issue: WON the non-registration of certificate
of sale is applicable in judicial foreclosure
Held:
Held:
NO- The Right of Redemption being statutory
must be exercised in the manner prescribed by NO- Failure of Cruz to register the certificate of
the statue and within the prescribed time limit. sale was of no consequence since it is only
applicable in extra-judicial foreclosure.
-Under Act 3135, juridical person whose
property is being sold pursuant to an extra- -The effect of failure to obtain judicial
confirmation was only to prevent the title from
judicial foreclosure, shall have the right to
redeem the property until, but not later, the being transferred to him. Such right shall not
registration of the certificate of foreclosure sale give rise to any right in favor of the mortgagor.

Rolan Jeff A. Lancion


Arellano University School of Law Page 77
Remedial Law 2- Brondial Notes AY 2017- 2018

-Judicial confirmation operated only to divest inclusive of penalties at the time of the
the rights of all parties to the action and to best foreclosure.
their right in the purchaser subject to right of
redemption.

Case: Metrobank vs CPR Promotions and Rule 69- Partition


Marketing, G.R. No. 200567, June 22, 2015 -There should always co-ownership in order to
apply partition.
Facts:

-From Feb to Oct 2015, CPR Promotion Section 1. Complaint in action for partition of
obtained loans from Metrobank. The loans real estate. — A person having the right to
were covered by 15 Promissory Notes. compel the partition of real estate may do so as
provided in this Rule, setting forth in his
-To secure the loans, Sps. Reynoso executed complaint the nature and extent of his title and
an adequate description of the real estate of
two deeds of real estate mortgage. They also
which partition is demanded and joining as
executed a continuing surety over the loans
defendants all other persons interested in the
they have secured. property.
-Upon maturity of the loans, CPR defaulted in -A complaint shall set forth the nature and
payment prompted Metrobank to extra- extent of his title.
judicially foreclosure the real estate mortgage.
-Adequate description of the real estate of
-MBCTC filed a civil case to recover the which portion is demanded and joining of
deficiency balance and the trial court ruled in defendants.
their favor. It was reversed by the Court of
Appeals. Sec 2- Order for partition and partition by
agreement thereunder
Issue: WON CA erred in reversing the decision
of the Trial Court -If after the trial the court finds that the plaintiff
has the right thereto, it shall order the partition
Held: of the real estate among all the parties in
NO- In extra-judicial foreclosure of mortgage, interest.
the proceeds of the sale are insufficient to pay -If the parties are able to agree, make the
the debt, the mortgagee has the right to partition among themselves by proper
recover the deficiency from the debtor. instruments of conveyance and the court shall
-There can only be deficit when the proceeds of confirm the partition.
the sale is not sufficient to cover the foreclosure
proceeding and the amount due to the creditor,

Rolan Jeff A. Lancion


Arellano University School of Law Page 78
Remedial Law 2- Brondial Notes AY 2017- 2018

-A final order decreeing partition and property. The other sibling filed an action for
accounting may be appealed by any party partition based on the extra-judicial settlement
aggrieved thereby. of estate.

Sec 3- Commissioner to make partition when Issue: WON partition shall apply in the case and
parties fail to agree WON there is co-ownership in the property

-The court shall appoint NOT MORE THAN 3 Ruling: In a partition case, the court should
competent and DISINTERESTED persons as determine if there is co-ownership over the
commissioners to make the partition. property. In this case, there is no more co-
ownership from the time the bank consolidated
Sec 5- Assignment or sale of real estate by the title.
commissioners
Parts of Partition:
-The real estate or portion cannot be divided
without prejudice (a) Determination of co-ownership of the
property
-The court may order it assigned to one of the (b) Accounting of the expenses for the
parties willing to take the same, provided he partition, the proceeds of such property
pays to the other parties such amount as the
commissioners deem equitable -The accounting must be determined by a
commissioner.
Co-Ownership- When a property is owned by
two or more persons. Situation:

Case: Balus vs Balus H&W- Owned a parcel of land (1,000 sq meter)

Facts: Mr. Balus (Father) owned a piece of land Children: Andoy, Betty, Carla, Danilo and Eric
located in Tarlac and he secured a loan from a
bank. The property was subjected into a real Procedure: if the siblings cannot agree on extra-
judicial settlement, the can file an action for
estate mortgage. Mr Balus failed to pay the loan
and as a result the land was foreclosed. The partition.
Bank was the buyer in the auction sale and the (A) When a party file a petition for
title thereto was consolidated. The two children partition, the court must convience the
of Mr. Balus entered into an extra-judicial parties to enter into a “project of
settlement of estate of their father and they do partition” or Extra-judicial settlement of
not have an idea that the TCT of the lands were estate
already consolidated in favor of the bank. One -In case of co-ownership, only one party
of the children bought back the property from can file an action but if they are
the bank and wanted full ownership of the

Rolan Jeff A. Lancion


Arellano University School of Law Page 79
Remedial Law 2- Brondial Notes AY 2017- 2018

defendants all of them must be property is assigned to one of the parties upon
impleaded. his paying to the others the sum or sums
-If the other co-owners does not want ordered by the court, the judgment shall state
to join the action, they shall be the fact of such payment and of the assignment
of the real estate to the party making the
considered as “unwilling plaintiff”.
payment, and the effect of the judgment shall
be to vest in the party making the payment the
(B) The trial court will assign commissioners
whole of the real estate free from any interest
that will determine the accounting of the co- on the part of the other parties to the action. If
owned property the property is sold and the sale confirmed by
the court, the judgment shall state the name of
-The commissioner will insist another extra- the purchaser or purchasers and a definite
judicial settlement of estate. description of the parcels of real estate sold to
each purchaser, and the effect of the judgment
-The commissioner has the option of shall be to vest the real estate in the purchaser
assignment or Buy out of the property or purchasers making the payment or
payments, free from the claims of any of the
-If one of the co-owners opposed the parties to the action. A certified copy of the
assignment, it will be automatically judgment shall in either case be recorded in the
terminated. registry of deeds of the place in which the real
estate is situated, and the expenses of such
-The commissioner has the remedy of sale recording shall be taxed as part of the costs of
over the property and the proceeds will be the action.
partitioned.
Cases: Felciano vs Canoza, G.R. No. 161746,
Section 10. Costs and expenses to be taxed and September 1, 2010
collected. — The court shall equitably tax and
apportion between or among the parties the Facts:
costs and expenses which accrue in the action,
-When Antonio Felciano died on May 30, 1930,
including the compensation of the
commissioners, having regard to the interests he left behind his only property located in
of the parties, and execution may issue therefor Bustos, Bulacan.
as in other cases.
-On 1972, Leona, Maria and Pedro with the
Section 11. The judgment and its effect; copy exception of Salinas executed an extra-judicial
to be recorded in registry of deeds. — If actual settlement of estate belonging to Antonio.
partition of property is made, the judgment
shall state definitely, by metes and bounds and -Jacinto Feliciano applied for a free patent over
adequate description, the particular portion of the land.
the real estate assigned to each party, and the
effect of the judgment shall be to vest in each
party to the action in severalty the portion of
the real estate assigned to him. If the whole

Rolan Jeff A. Lancion


Arellano University School of Law Page 80
Remedial Law 2- Brondial Notes AY 2017- 2018

-Surviving heirs of late Esteban Felciano filed a -Hilaria prompted to seek the demolition of the
complaint alleging that the settlement of Estate house of Emilia which prompted the latter to
was done without their participation & consent. seek partition of the lots. RTC rendered a
decision that the partition was dismissed.
-CA reversed the decision alleging that
prescription already set in since the annulment -CA reversed the decision by the RTC since no
of extra-judicial settlement already lapsed necessity to place Lot 707 under judicial admin
which is 4 years from the issuance of free since Carolina sold her ½ share to Felipa and
patents. Hilaria.

Issue: WON Extra-Judicial partition of the estate Issue: WON there is still co-ownership over the
wherein some of the compulsory heirs were property
excluded can be assailed after 4 years.
Held:
Held:
YES- The first stage in an action for partition is
NO- The prescriptive period to institute the the settlement of co-ownership.
action to annul the deed of extra-judicial
settlement is 4 years counted from the -Mere issuance of a certificate of title in the
discovery of fraud. name of any person does not foreclose the
possibility that the real property maybe under
-The complaint was only filed 16 years after co-ownership with persons not named in the
Jacinto was issued free patents and petitioner certificate.
have obtained constructive notice of fraud upon
-Co-ownership of Lot 707 is established, the
the registration of Free Patent.
affidavit of self- adjudication did not prejudice
Case: Vda de Figuracion vs Figuracion-Gerilla, the share of Agripina. The deed of sale executed
G.R. No. 151334, February 13, 2013 by Carolina in favor of Hilaria and Felipa was
valid as to the share of Carolina in the co-
Facts: ownership.
-Parties are the heirs of Leandro Figuracion who
died intestate in 1958. Subject of the dispute
are two parcels of land situated in Pangasinan. Case: Mangahas vs Brobio, G.R. No. 183852,
October 20, 2010.
-On 1962, Carolina executed an Affidavit of Self-
Adjudication as the sole and exclusive heir of Facts:
the deceased. She also executed a deed of -Pacifico died intestate and was survived by his
absolute sale in favor of Hilaria and Felipa. wife (Euforcina) 4 legitimate and 3 illegitimate
child.

Rolan Jeff A. Lancion


Arellano University School of Law Page 81
Remedial Law 2- Brondial Notes AY 2017- 2018

-The heirs of Pacifico executed a deed of extra- Accion Interdictal- Recovery of Possession de
judicial settlement of estate and respondent facto
agreed to pay Carmela of her share in the estate
but the former failed to comply. -Unlawful Detainer -Forcible Entry
-Carmela Mangahas filed a complaint for Unlawful Detainer Forcible Entry
Specific Performance with Damages and the
trial court rendered a decision in her favor. Possession was lawful Possession was
-It was reversed by the Court of Appeals at the start and unlawful from the
become unlawful later start
considering that the waiver in the deed of
on.
extra-judicial may not considered as the
consideration in the Promissory Note. Demand to vacate the Demand to vacate is
property is necessary not necessary
Issue: WON an act of partition is the proper
Prescriptive Period: 1 Prescriptive Period: 1
remedy year from the last year from the last
demand entry in the property
Held:
NO- An action for partition implies that the -Upon the lapse of 1
year, it will be
property is still owned in common. Considering
converted into Accion
that the heirs had already executed a deed of Publiciana
extra-judicial and waived their shares in favor of
respondent. Grounds: Grounds:
-The properties are no longer under the state of
-Expiration of the FITTS- Fraud,
co-ownership, there is nothing more to be Contract (Written or Intimidation, Threat,
partitioned as ownership land already merged Oral) Strategy and Stealth
into one person.
-Violation of any
term/condition in the
contract

-Tolerance
Rule 70- Unlawful Detainer and Forcible Entry

3 Kinds of Action for Recovery of Real Property


-In Ejectment case, the rent control law
Accion Reinvindicatoria- Recovery of
provides for other grounds of assignment.
Ownership
Example:
Accion Publiciana- Recovery of Plenary
Possession -Assignment of lease or subleasing rights

-Arrears in rent payment for a total of 3 months

Rolan Jeff A. Lancion


Arellano University School of Law Page 82
Remedial Law 2- Brondial Notes AY 2017- 2018

-Legitimate need of the lessor/owner of his except lack of jurisdiction over the subject
property matter. Cross-claims and compulsory
counterclaims not asserted in the answer shall
-All cases of unlawful detainer & forcible entry be considered barred. The answer to
are within the jurisdiction of MTC or MTCC. counterclaims or cross-claims shall be served
and filed within ten (10) days from service of
Exception: On appeal to RTC, it is no longer the answer in which they are pleaded.
governed by summary procedure but regular
procedure. (Refuga vs CA) Section 8. Preliminary conference; appearance
of parties.
Q: What is the jurisdiction of Unlawful Detainer
and Forcible Entry Cases? Not later than thirty (30) days after the last
answer is filed, a preliminary conference shall
A: MTC or MTCC be held. The provisions of Rule 18 on pre-trial
shall be applicable to the preliminary
Q: What is the venue of ejectment case? conference unless inconsistent with the
provisions of this Rule.
A: Where the property is located since the
action is quasi in rem. The subject matter of the Section 13. Prohibited pleadings and motions.
case is real property The following petitions, motions, or pleadings
shall not be allowed:
Cases of Unlawful Detainer and Forcible Entry
is governed by Summary Procedure 1. Motion to dismiss the complaint
except on the ground of lack of
*No Hearing jurisdiction over the subject matter, or
failure to comply with section 12;
*Periods are shorter
2. Motion for a bill of particulars;
Section 4. Pleadings allowed.
3. Motion for new trial, or for
-Complaint, compulsory counterclaim and reconsideration of a judgment, or for
cross-claim reopening of trial;

-All pleadings shall be verified 4. Petition for relief from judgment;

Section 6. Answers. — 5. Motion for extension of time to file


pleadings, affidavits or any other paper;
Within ten (10) days from service of summons,
the defendant shall file his answer to the 6. Memoranda;
complaint and serve a copy thereof on the
plaintiff. Affirmative and negative defenses not 7. Petition for certiorari, mandamus, or
pleaded therein shall be deemed waived, prohibition against any interlocutory
order issued by the court;

Rolan Jeff A. Lancion


Arellano University School of Law Page 83
Remedial Law 2- Brondial Notes AY 2017- 2018

8. Motion to declare the defendant in -When the defendant raises the defense of
default; ownership, the question of possession cannot
be resolved without deciding the issue of
9. Dilatory motions for postponement; ownership. It shall be resolved only to
determine the issue of possession.
10. Reply;
-The issue of ownership shall not be res judicata
11. Third-party complaints; therefore litis pendentia shall not apply.

12. Interventions. INJUNCTIVE RELIEFS (Sec 15 & 20)

Motion to Dismiss Section 15. Preliminary injunction. — The court


may grant preliminary injunction, in accordance
Exceptions: -Lack of jurisdiction over the with the provisions of Rule 58 hereof, to
subject matter prevent the defendant from committing further
acts of dispossession against the plaintiff.
-Failure to comply with brgy conciliation
regardless of the residence of the parties. A possessor deprived of his possession through
forcible from the filing of the complaint, present
Exception to Exception: Corporation a motion in the action for forcible entry or
unlawful detainer for the issuance of a writ of
-There is no trial in a case of unlawful detainer preliminary mandatory injunction to restore
& forcible entry him in his possession. The court shall decide
the motion within thirty (30) days from the
-Only Submission of position papers filing thereof.

-There is Preliminary Conference- In compliance Section 20. Preliminary mandatory injunction


with (AM 07-03-09) in case of appeal. — Upon motion of the
plaintiff, within ten (10) days from the
Meditation-Arbitration-Submission of Pre-Trial perfection of the appeal to the Regional Trial
Order-Position Paper Court, the latter may issue a writ of preliminary
mandatory injunction to restore the plaintiff in
- If the judge would like to have clarificatory possession if the court is satisfied that the
questions, he will order to have a hearing. defendant's appeal is frivolous or dilatory or
that the appeal of the plaintiff is prima
*The only issue that will be resolved in an facie meritorious.
unlawful detainer & forcible entry is possession
de facto. *A judgment in unlawful detainer & forcible
entry is immediately executory.
Sec 16- Resolving defense of ownership
-Appeal shall not toll the execution of the
judgment unless it is coupled by sufficient

Rolan Jeff A. Lancion


Arellano University School of Law Page 84
Remedial Law 2- Brondial Notes AY 2017- 2018

supersedes bond and regularly monthly deposit Supersedes bond- Unpaid rentals or if there are
for the use & occupancy of the premises. no unpaid rentals, it shall not be necessary.

-Once the records of the case is elevated -Once the Regional Trial Court affirms the
towards the RTC, there should be a monthly decision of the MTC, there shall be execution of
deposit for the use & occupancy of the judgment.
premises.
Remedy: File a petition for review under Rule
Q: Where should a party file their notice of 42.
appeal & posting of supersedes bond?
-If a party wants to stall the execution of
A: MTC judgment, he shall file a preliminary injunction
or TRO.
Q: Where should a party pay the regular
monthly deposit for the use & occupancy? Case Doctrine Zacarrias vs Anacay- “In order
for tolerance to be a valid ground for unlawful
A: Appellate Court- RTC detainer, it must co-exist with possession”

Q: When should a party pay the regular -The act of tolerance must be together with the
monthly deposit for the use & occupancy? time of possession.

A: On or Before the 10th day of every Example #1: Mr A is an owner of a real property
succeeding month. and B is occupying the property of Mr. A. Mr. A
is tolerating the occupancy of B in his property,
Rationale: Under Rule 40 of ROC, the clerk of his possession is co-existence with tolerance. If
court shall have 15 days to which to transmit Mr.A will tell B to vacate the property and the
the records of the case from MTC to RTC. latter refuse, Mr. A can file a case of unlawful
detainer on the ground of tolerance.
Q: How much will be the regular monthly
deposit? Example #2: Mr A sold the property to Mr. Y,
the latter will now be the owner. He noticed
A: Amount based on the judgment that B is still in his property and thereafter
demanded to vacate the property. In the event
Example: A-lessor, B-lessee with a contract of Mr. Y will file a case against B, it will not prosper
lease in the house which will expire on Dec on the ground of tolerance since it is not co-
2018 with a monthly rental of Php 5,000. A existence with possession.
wrote a letter to B that the rental will increase
to Php 15,000. Rule 70- Unlawful Detainer and Forcible Entry

-In ejectment case, the court shall state the


reasonable amount of rentals.
Case: Suarez vs Emboy Jr, 718 SCRA 677

Rolan Jeff A. Lancion


Arellano University School of Law Page 85
Remedial Law 2- Brondial Notes AY 2017- 2018

possession under any contract, express or


implied. The two are distinguished from each
Facts: other in that in forcible entry, the possession of
-A house located in Cebu City occupied Sps. the defendant is illegal from the beginning, and
Felix and Maribu claim that their mother that the issue is which party has prior de facto
occupied the subjected land. possession while in unlawful detainer,
possession of the defendant is originally legal
-On 2004, they were asked by their cousins, but became illegal due to the expiration or
heirs of Vicente to vacate the subject lot. termination of the right to possess.
Respondents received a demand letter from
Suarez's counsel to vacate such land but they
did not follow such order. They filed a petition -The jurisdiction of these two actions, which are
for partition while Carmencita for unlawful summary in nature, lies in the proper municipal
detainer. trial court or metropolitan trial court. Both
-MTC rendered a decision in favor of actions must be brought within one year from
Carmencita and it was affirmed by RTC. CA the date of actual entry on the land, in case of
reversed the decision of the lower courts on the forcible entry, and from the date of last
ground that the case is not unlawful detainer. demand, in case of unlawful detainer. The issue
in said cases is the right to physical possession.

Issue: WON the Court of Appeals erred in its


decision -In a complaint for unlawful detainer, the
following key jurisdictional facts must be
alleged and sufficiently established: (1)initially,
possession of property by the defendant was by
Held:
contract with or by tolerance of the plaintiff; (2)
NO eventually, such possession became illegal upon
notice by plaintiff to defendant of the
-Accion interdictal comprises two distinct termination of the latter’s right of possession;
causes of action, namely, forcible entry (3) thereafter, the defendant remained in
(detentacion) and unlawful detainer possession of the property and deprived the
(desahuico). In forcible entry, one is deprived of plaintiff of the enjoyment thereof; and (4)within
physical possession of real property by means one year from the last demand on defendant to
of force, intimidation, strategy, threats, or vacate the property, the plaintiff instituted the
stealth whereas in unlawful detainer, one complaint for ejectment.
illegally withholds possession after the
expiration or termination of his right to hold

Rolan Jeff A. Lancion


Arellano University School of Law Page 86
Remedial Law 2- Brondial Notes AY 2017- 2018

- To give the court jurisdiction to effect the occupancy of the property. Failure of the
ejectment of an occupant or deforciant on the defendant to comply is a ground for the
land, it is necessary that the complaint must outright execution of the judgment.
sufficiently show such a statement of facts as to
bring the party clearly within the class of cases -In this case, the bond filed did not meet the
for which the statutes provide a remedy, legal requirement since the bond posted was a
without resort to parol testimony, as these property bond, not case not surety.
proceedings are summary in nature. In short, -Enforcement of ejectment cases requires the
the jurisdictional facts must appear on the face sheriff to give such writ and to demand from
of the complaint. defendant to vacate such property.
Atty. Alconera vs Pallanan, 714 SCRA 204

Facts: Case: Teodoro vs Espino, 715 SCRA 435


-Pending appeal, the apellate court of RTC
rendered a decision granting the motion for
execution. Rafols called Atty. Alconera to tell Facts:
that the sheriff was about to execute the
-A lot included in a property registered in the
judgment.
name of Genaro Teodoro wherein he had 5
-There was a heated argument between Atty. children.
Alconera and Sheriff Pallanan since the former
-Petra was the daughter of Genaro only
mentioned that they have a pending MR
occupied the property. In her will, she
therefore the execution is improper.
appointed Genaro as the devisee.

-Teodoro effected the demolition of the


Issue: WON an appeal or MR shall toll the ancestral house since he intends to use it for
execution pending appeal other purpose. Respondent who is living in the
subjected land erected a fence which forced
Teodoro to file forcible entry.
Held: NO- A judgment on forcible entry and -Both parties raised the issue of ownership via
unlawful detainer shall be immediately succession. MTC rendered a decision claiming
executory to avoid further injustice unless the that Teodoro cannot be disturbed of their
the defendant stayed the execution upon possession over the property.
completion of the following (a) notice of appeal,
(b) filing a supersedeas bond and (c) periodic -RTC reversed the decision in favor of Teodoro
deposit of rental or compensation for the use & Teodoro.

Rolan Jeff A. Lancion


Arellano University School of Law Page 87
Remedial Law 2- Brondial Notes AY 2017- 2018

Issue: WON exclusive ownership of a property YES- Execution shall issue immediately upon
subject of an action for forcible entry should be motion unless an appeal has been perfected
proven. and the defendant file a supersedeas bond and
payment of rentals, damages.

-Perfection of an appeal is not sufficient to stay


Held: the execution of the judgment in an ejectment
NO- Exclusive ownership are not among those case.
needed to be proved to be entitled to -In appeals by notice of appeal, the court loses
possession in forcible entry. jurisdiction over the case upon the perfection of
-The elements of forcible entry: (a) force, appeal filed in due time and expiration of the
intimidation, threat, strategy and stealth to time to appeal.
deprive another of physical possession, (b) Case: CGR Corp vs Treyes, 522 SCRA
plaintiff must allege and prove prior physical
possession, (c) physical/material possession and Facts:
(d) ejectment cases proceed independently of
claim of ownership. -Treyes allegedly forcibly and unlawfully
entered the leased properties and once inside
barracaded the entrance of the fishponds.

Case: Ferrer vs Judge Rabaca, 632 SCRA -CGR filed a complaint with the MTC for forcible
entry and damages with the RTC. Treyes filed a
motion to dismiss on the ground of litis
Facts: pendentia and res judicata.

- Complainant were the president of YMCA in -RTC dismissed the complaint on the ground of
an ejectment against Cano. They received a prematurity.
favorable judgment pending appeal they tried
to execute the judgment of MTC but it was
denied by Judge Rabaca. Issue: WON the separate complaints for forcible
entry and damages can independently
instituted
Issue: WON the issuance of writ of execution
pending appeal can be granted
Held:

YES- The only damages that may be recovered


Held: in an action for forcible entry is the fair rental

Rolan Jeff A. Lancion


Arellano University School of Law Page 88
Remedial Law 2- Brondial Notes AY 2017- 2018

value or reasonable compensation for the use & became illegal upon termination of such lawful
occupation of the property. possession.

-Other damages arising out of forcible -Tolerance must be present from the start of
entry/unlawful detainer may be filed separatelt possession sought to be recovered to as
and independently of the claim or restoration of unlawful detainer and not forcible entry.
possession.
-It is essential in unlawful detainer cases that
the plaintiff's act of tolerance must be present
from the start of possession which is sought to
be recovered.

Case: Manalang vs Bacani, 745 SCRA, January


12, 2015

Case: Zacarias vs Anacay, 736 SCRA 508 Facts:

- Petitioners Ruben Manalang, Amado


Manalang, Carlos Manalang, Concepcion M.
Facts: Gonzales, Ladislao Manalang and Luis Manalang
were the co-owners of Lot No 4236 with an
-A complaint for Ejectment with Damages was
area of 914 square meters of the Guagua
filed on 2008 by Amanda Zacarias against
Cadastre, and declared for taxation purposes in
Victoria Anacay. MTC rendered a decision in
the name of Tomasa B. Garcia.
favor of Anacay since the complaint failed to
state the elements of unlawful detainer. -A preliminary relocation survey conducted by
the Lands Management Section of the
-RTC the decision of the MTC on the ground
Department of Environment and Natural
that respondent's occupation of the property
Resources (DENR) confirmed the result on the
without Zacaria's consent that it can be
encroachment. When the respondents refused
converted into contract.
to vacate the encroached portion and to
Issue: WON tolerance can be converted into surrender peaceful possession thereof despite
one of the grounds of unlawful detainer demands, the petitioners commenced this
action for unlawful detainer on April 21, 1997 in
Held: the MTC of Guagua (Civil Case No. 3309), and
the case was assigned to Branch 2 of that court.
- The complaint of Zacarias failed to allege that
the possession by the respondent being initially - On September 17, 1998, the MTC (Branch 2)
legal or tolerated by the petitioner and which dismissed Civil Case No. 3309 for lack of
jurisdiction based on its finding that the action

Rolan Jeff A. Lancion


Arellano University School of Law Page 89
Remedial Law 2- Brondial Notes AY 2017- 2018

involved an essentially boundary dispute that that the RTC ultimately decided the appeal
should be properly resolved in an accion based on the survey and the surveyor's
reivindicatoria. testimony instead of the record of the
proceedings had in the court of origin.
-RTC reversed the MTC (Branch 2), and
remanded the case for further proceedings, -The MTC dismissed the action because it did
holding that because there was an apparent not have jurisdiction over the case. The
withholding of possession of the property and dismissal was correct. It is fundamental that the
the action was brought within one year from allegations of the complaint and the character
such withholding of possession the proper of the relief sought by the complaint determine
action was ejectment which was within the the nature of the action and the court that has
jurisdiction of the MTC; and that the case was jurisdiction over the action.
not a boundary dispute that could be resolved
in an accion reinvidicatoria, considering that it -To be clear, unlawful detainer is an action filed
involved a sizeable area of property and not a by a lessor, vendor, vendee, or other person
mere transferring of boundary. against whom the possession of any land or
building is unlawfully withheld after the
Issue: WON RTC has the authority to receive expiration or termination of the right to hold
additional evidence on appeal in an ejectment possession by virtue of any contract, express or
case. implied.To vest in the MTC the jurisdiction to
effect the ejectment from the land of the
respondents as the occupants in unlawful
Held: detainer, therefore, the complaint should
embody such a statement of facts clearly
NO- Under Rule 70, Sec 18 of the ROC it showing the attributes of unlawful detainer.
provides that: “The judgment or final order shall
be appealable to the appropriate Regional Trial Case: De la Cruz vs Hermano, 754 SCRA 231
Court which shall decide the same on the basis Facts:
of the entire record of the proceedings had in
the court of origin and such memoranda and/or -Respondents Antonio and Remedios Hermano
briefs as may be submitted by the parties or are the registered owners of a house and lot
required by the Regional Trial Court.” situated in P.B. Constantino Subdivision,
Tagaytay City, covered by Transfer Certificate of
-RTC violated the foregoing rule by ordering the Title (TCT) No. T-24503. On 13 June 2002,
conduct of the relocation and verification Antonio sued petitioner before the MTCC of
survey in aid of its appellate jurisdiction and by Tagaytay City, Branch 1, for ejectment and
hearing the testimony of the surveyor, for its damages.
doing so was tantamount to its holding of a trial
de novo. The violation was accented by the fact

Rolan Jeff A. Lancion


Arellano University School of Law Page 90
Remedial Law 2- Brondial Notes AY 2017- 2018

-petitioner admitted the existence of TCT No. T- possession was obtained by unlawful means.
24503, but she contended that the true and Further, the complainant must allege and prove
actual owner of the property was Don Enciso prior physical possession of the property in
Benitez (Benitez). Allegedly, Antonio and his litigation until he or she was deprived thereof
wife, respondent Remedios Hermano, had by the defendant. The one-year period within
already sold the property to Benitez; the latter, which to bring an action for forcible entry is
in turn, sold it to petitioner by virtue of a Deed generally counted from the date of actual entry
of Absolute Sale,which they executed on 1 into the land, except when entry was made
March 2001. through stealth; if so, the one-year period
would be counted from the time the plaintiff
-The MTCC found that Antonio had, indeed,
learned about it.
executed a Deed of Absolute Sale over the
subject property in favor of Benitez. -The complaint subject of this case was
captioned as ejectment. From a reading of the
allegations of the subject Complaint, we find
The RTC opined that respondents, Complaint that the action is one for forcible entry.
did not clearly show whether it was one for Petitioner alleged that he is the owner of the
forcible entry or for unlawful detainer. Because property registered under TCT No. T-24503;
it appeared to be an ejectment case, the MTCC that the possession thereof by respondent on 1
took cognizance of it. The parties subsequent
September 2001 was pursuant to an alleged
pleadings revealed, however, that the case was Memorandum of Agreement between her and a
actually an accion reivindicatoria. certain Don Mario Enciso, without the authority
Issue: WON the complaint sufficiently suffice and consent of the petitioner; and that he has
the requisites of unlawful detainer served written demands, dated 27 September
2001 and 24 October 2001, but that respondent
Held: refused to vacate the property. According to
petitioner, the Complaint, which was filed on 13
YES- Section 1, Rule 70 of the Rules of Court,
June 2002, was filed within one year from the
requires that in actions for forcible entry, it
occupation of the property.
must be alleged that the complainant was
deprived of the possession of any land or
building by force, intimidation, threat, strategy,
or stealth, and that the action was filed anytime -The Court agrees with the CA's findings that
within one year from the time the unlawful the Complaint was timely filed. It is settled that
deprivation of possession took place. This where forcible entry occurred clandestinely, the
requirement implies that in those cases, one-year prescriptive period should be counted
possession of the land by the defendant has from the time the person who was deprived of
been unlawful from the beginning, as the possession demanded that the deficient desist

Rolan Jeff A. Lancion


Arellano University School of Law Page 91
Remedial Law 2- Brondial Notes AY 2017- 2018

from dispossession when the former learned -On June 4, 2007, the RTC decided in the
about it. Spouses Dumlao’s favor. It ordered the
defendants (1) to immediately vacate the
property and turn it over to the Spouses
Dumlao, and (2) to pay accumulated rentals,
damages, and attorney’s fees. The RTC also
prohibited the defendants from accepting
enrolees to the San Mariano Academy.
Case: Erorita vs Dumlao, 781 SCRA 551
-The CA ruled that the applicable law on
Facts:
jurisdiction when the complaint was filed, was
-Spouses Antonio and Ligaya Dumlao(Spouses Republic Act No. 7691 (RA 7691). This law
Dumlao)are the registered owners of a parcel of provides that in civil actions involving a real
land located at Barangay San Mariano, Roxas, property’s title or possession, jurisdiction
Oriental Mindoro, and covered by TCT No. T- depends on the property’s assessed value and
53000. The San Mariano Academy structures location – if the assessed value exceeds fifty
are built on the property. thousand pesos (P50,000.00) in Metro Manila,
and twenty thousand pesos (P20,000.00)
-The Spouses Dumlao agreed to allow the outside of Metro Manila, the RTC has
petitioners to continue to operate the school on jurisdiction.
the property. The Spouses Erorita appointed
Hernan and Susan Erorita as the San Mariano
Academy’s administrators.
Issue: WON RTC has jurisdiction over the
-On December 16, 2002, the Spouses Dumlao complaint.
asked the petitioners to vacate the property.
Although the Spouses Erorita wanted to
comply, they could not immediately close the Held:
school without clearance from the Department
of Education, Culture, and Sports to whom they NO- Jurisdiction does not depend on the
are accountable. complaint’s caption.Nor is jurisdiction changed
by the defenses in the answer; otherwise, the
-On March 4, 2004, the Spouses Dumlao filed a defendant may easily delay a case by raising
complaint for recovery of possession before the other issues, then, claim lack of jurisdiction.
Regional Trial Court (RTC) against the
defendants Hernan, Susan, and the Spouses -To make a case for unlawful detainer, the
Erorita. complaint must allege that: (a) initially, the
defendant lawfully possessed the property,
either by contract or by plaintiff’s tolerance; (b)

Rolan Jeff A. Lancion


Arellano University School of Law Page 92
Remedial Law 2- Brondial Notes AY 2017- 2018

the plaintiff notified the defendant that his right personalities toward others, or refusal to be
of possession is terminated; (c) the defendant sworn or to answer as a witness, or to subscribe
remained in possession and deprived plaintiff of an affidavit or deposition when lawfully
its enjoyment; and (d) the plaintiff filed a required to do so, may be summarily adjudged
in contempt by such court and punished by a
complaint within one year from the last
fine not exceeding two thousand pesos or
demand on defendant to vacate the property. A imprisonment not exceeding ten (10) days, or
complaint for accion publiciana or recovery of both, if it be a Regional Trial Court or a court of
possession of real property will not be equivalent or higher rank, or by a fine not
considered as an action for unlawful detainer if exceeding two hundred pesos or imprisonment
any of these special jurisdictional facts is not exceeding one (1) day, or both, if it be a
lower court.
omitted.

-The complaint did not state that (i) possession Section 3. Indirect contempt to be punished
after charge and hearing. — After a charge in
was unlawfully withheld and (ii) the complaint
writing has been filed, and an opportunity given
was filed within a year from the last demand. to the respondent to comment thereon within
Because these special jurisdictional facts for an such period as may be fixed by the court and to
unlawful detainer case were lacking, we held be heard by himself or counsel, a person guilty
that the case should be accion publiciana over of any of the following acts may be punished for
which the RTC has jurisdiction indirect contempt;

In the present case, however, the complaint (a) Misbehavior of an officer of a court in the
clearly contained the elements of an unlawful performance of his official duties or in his
detainer case. Thus, the case should have been official transactions;
filed with the MTC. The RTC had no jurisdiction
(b) Disobedience of or resistance to a lawful
over this case. writ, process, order, or judgment of a court,
including the act of a person who, after being
dispossessed or ejected from any real property
by the judgment or process of any court of
competent jurisdiction, enters or attempts or
Rule 71- Contempt
induces another to enter into or upon such real
property, for the purpose of executing acts of
Kinds:
ownership or possession, or in any manner
Direct and Indirect disturbs the possession given to the person
adjudged to be entitled thereto;
Section 1. Direct contempt punished
(c) Any abuse of or any unlawful interference
summarily. — A person guilty of misbehavior in
with the processes or proceedings of a court
the presence of or so near a court as to obstruct
not constituting direct contempt under section
or interrupt the proceedings before the same,
1 of this Rule;
including disrespect toward the court, offensive

Rolan Jeff A. Lancion


Arellano University School of Law Page 93
Remedial Law 2- Brondial Notes AY 2017- 2018

(d) Any improper conduct tending, directly or property involved or such amount as may be
indirectly, to impede, obstruct, or degrade the alleged and proved.
administration of justice;
The writ of execution, as in ordinary civil
(e) Assuming to be an attorney or an officer of a actions, shall issue for the enforcement of a
court, and acting as such without authority; judgment imposing a fine unless the court
otherwise provides. (6a)
(f) Failure to obey a subpoena duly served;
Section 8. Imprisonment until order obeyed. —
(g) The rescue, or attempted rescue, of a person When the contempt consists in the refusal or
or property in the custody of an officer by virtue omission to do an act which is yet in the power
of an order or process of a court held by him. of the respondent to perform, he may be
imprisoned by order of the court concerned
-In Indirect Contempt Proceedings, it partakes until he performs it. (7a)
of criminal proceedings. Notice & Hearing is
mandatory. Section 9. Proceeding when party released on
bail fails to answer. — When a respondent
Jurisdiction: Regional Trial Court released on bail fails to appear on the day fixed
for the hearing, the court may issue another
Penalties: order of arrest or may order the bond for his
appearance to be forfeited and confiscated, or
-Fine both; and, if the bond be proceeded against,
the measure of damages shall be the extent of
-Ceisure the loss or injury sustained by the aggrieved
party by reason of the misconduct for which the
-Reprimand contempt charge was prosecuted, with the
costs of the proceedings, and such recovery
Section 7. Punishment for indirect contempt. shall be for the benefit of the party injured. If
— If the respondent is adjudged guilty of there is no aggrieved party, the bond shall be
indirect contempt committed against a Regional liable and disposed of as in criminal cases.
Trial Court or a court of equivalent or higher
rank, he may be punished by a fine not Judgment
exceeding thirty thousand pesos or Section 11. Review of judgment or final
imprisonment not exceeding six (6) months, or order; bond for stay. — The judgment or final
both. If he is adjudged guilty of contempt order of a court in a case of indirect contempt
committed against a lower court, he may be may be appealed to the proper court as in
punished by a fine not exceeding five thousand
criminal cases. But execution of the judgment
pesos or imprisonment not exceeding one (1)
month, or both. If the contempt consists in the or final order shall not be suspended until a
violation of a writ of injunction, temporary bond is filed by the person adjudged in
restraining order or status quo order, he may contempt, in an amount fixed by the court from
also be ordered to make complete restitution to which the appeal is taken, conditioned that if
the party injured by such violation of the

Rolan Jeff A. Lancion


Arellano University School of Law Page 94
Remedial Law 2- Brondial Notes AY 2017- 2018

the appeal be decided against him he will abide Case: Judge Espanol vs Atty. Formoso, 522
by and perform the judgment or final order SCRA

Case: Yasay vs Recto 313 SCRA Facts:

Facts: -On January 25, 2000, Sharcons filed with the


Regional Trial Court (RTC), Branch 90,
-Sec Perfecto Yasay upon request of certain Dasmariñas, Cavite a complaint for quieting of
stockholders of Interport Resources issued a title, docketed as Civil Case No. 2035-00.
TRO enjoining them from holding the scheduled Impleaded as defendants were spouses Mapua,
annual meeting. Evanswinda Morales, and the Register of Deeds
of Trece Martires City.
-The stockholders proceeded with such meeting
presided by Manalaysay, SEC declared that the
stockholders meeting as null and void. -Judge Dolores L. Español (petitioner), issued an
Manalaysay questioned the validity of the TRO Order stating that Benito See and Marly See,
but the SEC ordered that Manalaysay and president and treasurer, respectively, of
Ricalde guilty of contempt. Sharcons, and its counsel, Atty. Benjamin
Issue: WON the contempt institutred motu Formoso, respondents, have used a spurious
proprio hold the respondents guilty of certificate of title and tax declaration when it
contempt (Sharcons) filed with the RTC its complaint for
quieting of title.
Held:
- On July 12, 2001, petitioner issued warrants of
NO- The charge of contempt partakes of the arrest against respondents. They were confined
nature of a criminal offense. Civil contempt is in the municipal jail of Dasmariñas, Cavite. That
the failure to do something ordered by a court same day, respondents filed a motion for bail
for the benefit of a party while criminal and a motion to lift the order of arrest. But they
contempt is any conduct directed against the were denied outright by petitioner.
authority or dignity of the court. Civil contempt
are generally held to be remedial and civil in - Court of Appeals reversed the decision of
their nature. Judge Espanol on the ground that direct
contempt of court is criminal in nature,
-SEC order directing respondent to show cause petitioner should have conducted a hearing.
why they should not be cited in contempt was Thus, she could have determined whether
highly improper. respondents are guilty as charged.

Issue: WON Atty Formoso is guilty of direct


contempt of court for using falsified documents

Rolan Jeff A. Lancion


Arellano University School of Law Page 95
Remedial Law 2- Brondial Notes AY 2017- 2018

Held: to the accused to be heard by himself and


counsel. Moreover, settled is the rule that a
NO- In this jurisdiction, it is now recognized
contempt proceeding is not a civil action, but a
that courts have the inherent power to punish
separate proceeding of a criminal nature in
for contempt on the ground that respect for
which the court exercises limited jurisdiction.
the courts guarantees the very stability of the
Thus, the modes of procedure and the rules of
judicial institution. Such stability is essential to
evidence in contempt proceedings are
the preservation of order in judicial
assimilated as far as practicable to those
proceedings, to the enforcement of judgments,
adapted to criminal prosecutions.Perforce,
orders, and mandates of the courts, and,
petitioner judge erred in declaring summarily
consequently, to the very administration of
that respondents are guilty of direct contempt
justice.
and ordering their incarceration. She should
-This Court characterized direct contempt as have conducted a hearing with notice to
one done "in the presence of or so near the respondents.
court or judge as to obstruct the administration
of justice." It is a contumacious act done facie
curiae and may be punished summarily without Case: Sison vs Judge Caoibes Jr., 429 SCRA
hearing.
Facts:
-Indirect or constructive contempt, in turn, is
-Sison an MMDA traffic officer apprehended the
one perpetrated outside of the sitting of the
son of Judge Caoibes for violation a no right
court and may include misbehavior of an
turn while the latter was driving along EDSA.
officer of a court in the performance of his
Caoibes III introduced himself as the son of
official duties or in his official transactions,
Judge Caoibes and pleaded that he is in an
disobedience of or resistance to a lawful writ,
official judicial errand but Sison did not heed to
process, order, judgment, or command of a
the explanation of Caoibes III.
court, or injunction granted by a court or a
judge, any abuse or any unlawful interference -Judge Caoibes issued an order requiring Sison
with the process or proceedings of a court not to appear before him to explain the traffic
constituting direct contempt, or any improper inciden involving his son and to show cause on
conduct tending directly or indirectly to why he should not be cited in Contempt.
impede, obstruct or degrade the administration -Sison filed an admin case against Judge Caoibes
of justice. on the ground that indirect contempt constitute
- Following Section 3, Rule 71, a contemner may grave abuse of authority.
be punished only after a charge in writing has Issue: WON the act of MMDA officer Sison
been filed, and an opportunity has been given constitutes an offense of contempt

Rolan Jeff A. Lancion


Arellano University School of Law Page 96
Remedial Law 2- Brondial Notes AY 2017- 2018

Held: statements and improper conduct tending


directly or indirectly to impede, obstruct or
NO- There is no contempt of court which
degrade the administration of justice. He argues
requires that the person obstructed should be
that their pronouncements and malicious
performing duty connected with judicial
comments delved not only on the supposed
function.
inaction of the Court in resolving the petitions
-Respondent Judge Caoibes was not justified to filed, but also on the merits of the criminal
consider the act and remarks of Sison but cases before the RTC and prematurely
displaying arrogance towards and deliberate concluded that he and his co-accused are guilty
disregard of the usual respect and courtesy. of murder.
Case: Marantan vs Atty Jose Diokno, 716 SCRA Issue: WON respondents are liable for indirect
Facts: contempt for making improper conduct to
obstruct the administration of justice
-The criminal cases involve an incident which
transpired on November 7, 2005, where Anton Held:
Cu-Unjieng (son of respondent La’O), Francis NO- For a comment to be considered as
Xavier Manzano, and Brian Anthony Dulay, contempt of court "it must really appear" that
were shot and killed by police officers in front of such does impede, interfere with and
the AIC Gold Tower at Ortigas Center, which embarrass the administration of justice. What
incident was captured by a television crew from is, thus, sought to be protected is the all-
UNTV 37 (Ortigas incident). important duty of the court to administer
justice in the decision of a pending case. The
-On January 6, 2013, a shooting incident specific rationale for the sub judice rule is that
occurred in Barangay Lumutan, Municipality of courts, in the decision of issues of fact and law
Atimonan, Province of Quezon, where should be immune from every extraneous
Marantan was the ground commander in a influence; that facts should be decided upon
police-military team, which resulted in the evidence produced in court; and that the
death of thirteen (13) men (Atimonan incident). determination of such facts should be
This encounter, according to Marantan, elicited uninfluenced by bias, prejudice or sympathies.
much negative publicity for him. -The "clear and present danger" rule means
that the evil consequence of the comment must
-Marantan submits that the respondents be "extremely serious and the degree of
violated the sub judice rule, making them liable imminence extremely high" before an utterance
for indirect contempt under Section 3(d) of Rule can be punished. There must exist a clear and
71 of the Rules of Court, for their contemptuous present danger that the utterance will harm

Rolan Jeff A. Lancion


Arellano University School of Law Page 97
Remedial Law 2- Brondial Notes AY 2017- 2018

the administration of justice. Freedom of petitioners counsel and ordering the transmittal
speech should not be impaired through the of the records of the case to the Office of the
exercise of the power of contempt of court Clerk of Court for re-raffle to another sala. The
unless there is no doubt that the utterances in case was subsequently re-raffled to RTC Branch
question make a serious and imminent threat to 90 presided by Judge Reynaldo B. Daway, who
the administration of justice. It must constitute likewise voluntarily recused himself from the
an imminent, not merely a likely, threat. case per Order dated July 13, 2007.

- As to the merits, the comments seem to be -During the January 11, 2007 inspection, the
what the respondents claim to be an expression only document produced by the Acting
of their opinion that their loved ones were Corporate Secretary, Atty. Antonio V. Meriz,
murdered by Marantan. This is merely a and one of the staff, Malou Santos, was the
reiteration of their position in G.R. No. 199462, Stock and Transfer Book of the Corporation.
which precisely calls the Court to upgrade the They alleged that they could not find from the
charges from homicide to murder. The Court corporate records the copies of the proxies
detects no malice on the face of the said submitted by the stockholders, including the
statements. tape recordings taken during the stockholder's
meeting, and that they needed more time to
Case: Capitol Hills Gold and Country Club vs
locate and find the list of stockholders as of
Sanchez, 717 SCRA
March 2002, which was in the bodega of the
Facts: Corporation.
-On July 1, 2002, respondent Manuel O. Issue: WON the petitioners are guilty of
Sanchez (respondent), a stockholder of contempt charges
petitioner Capitol Hills Golf & Country Club, Inc.
Held:
(Corporation) filed a petition for the
nullification of the annual meeting of NO- In contempt proceedings, the prescribed
stockholders of May 21, 2002 and the special procedure must be followed. Sections 3 and 4,
meeting of stockholders of April 23, 2002. Rule 71 of the Rules of Court provide the
procedure to be followed in case of indirect
-On August 12, 2002, respondent filed a Motion
contempt. First, there must be an order
for Production and Inspection of Documents,
requiring the respondent to show cause why he
which the court granted in an Order dated
should not be cited for contempt.Second, the
September 10, 2002.
respondent must be given the opportunity to
-On February 9, 2007, Judge Hernando issued comment on the charge against him. Third,
an Order, inhibiting himself from handling the there must be a hearing and the court must
case in view of his close friendship relation with investigate the charge and consider

Rolan Jeff A. Lancion


Arellano University School of Law Page 98
Remedial Law 2- Brondial Notes AY 2017- 2018

respondent's answer. Finally, only if found the long overdue order to produce and make
guilty will respondent be punished accordingly. available for inspection and photocopying of
the requested records/documents.
-In all other cases, charges for indirect
contempt shall be commenced by a verified
petition with supporting particulars and
Case: Tormis vs Paredes, 749 SCRA 505
certified true copies of documents or papers
involved therein, and upon full compliance Facts:
with the requirements for filing initiatory -Jill added that Judge Paredes included Judge
pleadings for civil actions in the court Tormis in his discussions not only once but
concerned. If the contempt charges arose out several times. In one session, Judge Paredes
of or are related to a principal action pending was even said to have included in his discussion
in the court, the petition for contempt shall Francis Mondragon Tormis(Francis), son of
allege that fact but said petition shall be Judge Tormis, stating that he was a court-noted
docketed, heard and decided separately, addict. She was absent from class at that time,
unless the court in its discretion orders the but one of her classmates who was present,
consolidation of the contempt charge and the Rhoda L. Litang(Rhoda), informed her about the
principal action for joint hearing and decision. inclusion of her brother. To avoid humiliation in
school, Jill decided to drop the class under
-In contempt proceedings, the respondent Judge Paredes and transfer to another law
must be given the right to defend himself or school in Tacloban City.
herself and have a day in court a basic Judge Paredes denied the accusations of Jill. He
requirement of due process. This is especially stated that Judge Tormis had several
so in indirect contempt proceedings, as the administrative cases, some of which he had
court cannot decide them summarily pursuant investigated; that as a result of the
to the Rules of Court. investigations, he recommended sanctions
-In this case, the proceedings for indirect against Judge Tormis; that Judge Tormis used
contempt have not been initiated. To the Jill, her daughter, to get back at him; that he
Court's mind, the September 3, 2007 Resolution discussed in his class the case of Lachica v.
could be treated as a mere reiteration of the Tormis, but never Judge Tormis involvement in
September 10, 2002 Order. It is not yet a the marriage scams nor her sanctions as a result
judgment or final order of a court in a case of of the investigation conducted by the Court;
indirect contempt as contemplated under the that he never personally attacked Judge Tormis
Rules. The penalty mentioned therein only dignity and credibility.
serves as a reminder to caution petitioners of
the consequence of possible non-observance of

Rolan Jeff A. Lancion


Arellano University School of Law Page 99
Remedial Law 2- Brondial Notes AY 2017- 2018

-Justice Diy came up with the following choose his words and exercise more caution
recommendations, The undersigned and control in expressing himself. In other
Investigating Justice finds that indeed Judge words, a judge should possess the virtue of
Paredes is guilty of conduct unbecoming of a gravitas. Furthermore, a magistrate should not
judge. Conduct unbecoming of a judge is descend to the level of a sharp-tongued, ill-
classified as a light offense under Section 10, mannered petty tyrant by uttering harsh words,
Rule 140 of the Revised Rules of Court. snide remarks and sarcastic comments. He is
required to always be temperate, patient and
Issue: WON Judge Paredes is guilty of conduct
courteous, both in conduct and in language.
unbecoming of a judge

Held:

YES- Misconduct is defined as a transgression of


some established and definite rule of action,
more particularly, unlawful behavior or gross
SPECIAL PROCEEDINGS (Rule 72-109)
negligence by a public officer. The misconduct is
grave if it involves any of the additional Rule 72- General Provisions
elements of corruption, willful intent to violate
Still applicable Not Anymore
the law, or to disregard established rules, which applicable
must be established by substantial evidence. As A. Settlement of Constitution of
distinguished from simple misconduct, the estate of Family Home
elements of corruption, clear intent to violate deceased person
the law, or flagrant disregard of established
B. Escheat Voluntary
rule, must be manifest in a charge of grave
dissolution of
misconduct. corporations
-The subjudice rule restricts comments and C. Guardianship Rescission &
and custody of Revocation of
disclosures pertaining to the judicial children adoption
proceedings in order to avoid prejudging the
issue, influencing the court, or obstructing the D. Trustees Hospitalization of
administration of justice. Insane person
E. Adoption Declaration of
-A judge should always conduct himself in a absence and death
manner that would preserve the dignity, H. Habeas Corpus
independence and respect for himself, the F. Cancellation or
Court and the Judiciary as a whole. He must correction of
entries in the
exhibit the hallmark judicial temperament of
utmost sobriety and self-restraint. He should

Rolan Jeff A. Lancion


Arellano University School of Law Page 100
Remedial Law 2- Brondial Notes AY 2017- 2018

civil registry 4. The issues are determined by law

G. Change of Name -The petitioner must allege such law since it is


jurisdictional.
K. Judicial approval of
voluntary recognition of Q: When does an estate come to existence?
minor children
A: If a person is deprived of life or someone
died. (Actual or Presumptive Death)
Prerogative Writs- Habeas Data, Writ of
Amparo and Writ of Kalikasan.
Actual Death- It is evidence by the death
certificate of the decedent
Special Proceedings- A remedy by which a
party seeks to establish a status, right or
Presumptive Death- Basis: Rule 131 Sec 3
particular fact. (Rule 1 Sec 3)
GR: When a person is absent for 7 years
GENERAL CHARACTERISTICS OF SPECIAL
without any news of that person being alive,
PROCEEDINGS
he/she shall be considered dead.

1. There is only one party (petitioner)


XPN: For purposes of opening succession- 10
years
Exception: Habeas Corpus, Writ of Amparo,
Writ of Kalikasan & Writ of Habeas Data
XPN to XPN: 75 years old- absence of 5 years
shall be sufficient to open his succession.
2. The court acquires jurisdiction over the party
through publication.
Qualification other instances:

-The publication can be once a week for 3


-A person on board lost during a sea voyage or
consecutive weeks.
aircraft which is missing- 4 years from the loss
of vessel or aircraft
-For escheat proceedings, once a week for 6
consecutive weeks.
-Member of the armed forces who has taken
part in armed hostilities- 4 years
-No need for a valid service of summons or
payment of the correct docket fee
-Person who has been in danger of death- 4
years
3. It is imprescriptible or special proceedings do
not prescribe
-For purposes of marriage- 2 years when there
is danger of death

Rolan Jeff A. Lancion


Arellano University School of Law Page 101
Remedial Law 2- Brondial Notes AY 2017- 2018

Estate- Extension of the decedent Contents of the application

-The totality of the decedent’s assets & liability -Personal Circumstance of the petitioner

Settlement of Estate- (IAL)- Inventory, -Marriage contract of the parents


Administration and Liquidation
-Birth certificate of the petitioner

-What things that the decedent left behind? -Evidence of ownership (TCT/OCT)

-Who will administer the properties? B. Judicial

1. Summary Settlement of Estate of Small


-To whom these properties shall be
Value (Rule 74 Sec 2)
transferred?
-Not practical in actual practice
Kinds of Settlement
-The gross value of the estate of a deceased
A. Extra-Judicial person, whether intestate or testate shall not
exceed Ten Thousand Pesos (Php 10,000)
(1) Agreement/Extra-Judicial partition
(Rule 69) 2. Judicial Partition (Rule 69)

-Project of Partition, Appointment of


(2) Affidavit of Self-Adjudication
Commissioner, Assignment/buyout and Sale
- It usually happen when a decedent left only 3. Escheat (Rule 91)
with one heir, no will and with no debts.
Requirements:
-A party cannot execute an affidavit of self-
adjudication if he has legitimate/illegitimate -Person dies intestate
sibling alive. -Seized real/personal property

-Application shall be made in the Register of -Leaving no heir


Deeds where the property is locared.
-No Will
-There will be a publication of the application,
*The state shall settle the estate for the
once a week for three (3) consecutive weeks in
decedent in favor of the state.
a newspaper of general circulation.
Q: What is the jurisdiction and venue?
-There shall be posting the public place where
the property is located.

Rolan Jeff A. Lancion


Arellano University School of Law Page 102
Remedial Law 2- Brondial Notes AY 2017- 2018

A:Jurisdiction - In all matters of probate, both patent, the state is the proper party in interest
testate and intestate, where the gross value of to file Reversion.
the estate exceeds Three hundred thousand
pesos (P300,000.00) or, in probate matters in Reversion- An action for taking back the
Metro Manila, where such gross value exceeds property of the state, it is only initiated by the
Four Hundred Thousand pesos (P400,000.00).- State in a judicial action.
RTC Case: Maltos vs Heirs of Eusebio Borromeo,
Venue-Where the property is located 770 SCRA 397

Example: Properties belonging to the estate of Facts:


Mr. A is located in Manila, QC, CDO and Davao. -On February 13, 1979, Eusebio Borromeo was
-Escheat proceeding can be filed anywhere issued Free Patent No. 586681 over a piece of
wherein the property is located. agricultural land located in San Francisco,
Agusan del Sur, covered by Original Certificate
-The distribution of properties shall be based on of Title No. P-9053.
its location for public schools, charitable
institution and centers. -On June 15, 1983, well within the five-year
prohibitory period, Eusebio Borromeo sold the
Period of Publication: Once a week for Six (6) land to Eliseo Maltos. Eusebio Borromeo died
consecutive weeks in a newspaper of general on January 16, 1991. His heirs claimed that prior
circulation. to his death, he allegedly told his wife, Norberta
Borromeo and his children to nullify the sale
Case: Alvarico vs Sola, 382 SCRA
made to Eliseo Maltos and have the Transfer
Facts: The property was owned through patent Certificate of Title No. T-5477 cancelled because
issued by DENR. Under CA 141, a patent issued the sale was within the five-year prohibitory
cannot be disposed until 25 years unless there period.
is permission from the secretary of the DENR. -On June 23, 1993, Norberta Borromeo and her
During the pendency of the sales application of children (heirs of Borromeo) filed a Complaint
the patent, Alvarico caused the transfer of the for Nullity of Title and Reconveyance of Title
title to its niece. A patent was issued in favor of
against Eliseo Maltos, Rosita Maltos, and the
the niece of Alvarico. The sister of the decedent Register of Deeds of Agusan del Sur. The trial
questioned the transfer of the patent based on court dismissed the Complaint on the ground of
a deed of assignment over the property. failure to state a cause of action. Also, the heirs
Ruling: The case was dismissed on the ground of Borromeo did not have a right of action
that the sister of the decedent is not a proper because they were unable to establish their
party in interest. In cases involving issuance of status as heirs of the late Eusebio Borromeo.

Rolan Jeff A. Lancion


Arellano University School of Law Page 103
Remedial Law 2- Brondial Notes AY 2017- 2018

Issue: WON the petitioners are the real-party in reversion must first be filed by the Office of
interest in this case the Solicitor General.

Held: 4. Conventional Settlement of Estate (Rule 73-


90)
-The five-year period prohibiting the sale of
land obtained under homestead or free patent Two Kinds:
is provided under Section 118 of the Public Land
Testate Settlement- A decedent left a will
Act: Except in favor of the Government or any
of its branches, units, or institutions, or legally Intestate Settlement- A decedent left without a
constituted banking corporations, lands will
acquired under free patent or homestead
provisions shall not be subject to encumbrance Q: What is the jurisdiction?
or alienation from the date of the approval of
A: According to RA 7691. In all matters of
the application and for a term of five years
probate, both intestate and testate, where the
from and after the date of issuance of the
gross value of the estate exceeds Three
patent or grant, nor shall they become liable to
Hundred Thousand Pesos (Php 300,000) outside
the satisfaction of any debt contracted prior to
Metro Manila or Four Hundred Thousand Pesos
the expiration of said period.
(Php 400,000) within Metro Manila. It shall be
- Any acquisition, conveyance, alienation, cognizable by RTC.
transfer, or other contract made or executed in
Q: What is the venue of such action?
violation of any of the provisions of sections
one hundred and eighteen, one hundred and A: Under Rule 73, Sec 1, it must be the last
twenty, one hundred and twenty-one, one residence of the decedent.
hundred and twenty-two, and one hundred
and twenty-three of this. Act shall be unlawful Case Doctrine: Fule vs CA- The venue for the
and null and void from its execution and shall settlement of estate shall be the last residence
produce the effect of annulling and cancelling of the decedent.
the grant, title, patent, or permit originally
Rule 75- Production of Will: Allowance of Will
issued, recognized or confirmed, actually or
Necessary
presumptively, and cause the reversion of the
property and its improvements to the State.
Section 1. Allowance necessary. Conclusive as
-There is sufficient cause to revert the property to execution.
in favor of the state. However, this court
No will shall pass either real or personal estate
cannot declare reversion of the property in unless it is proved and allowed in the proper
favor of the state in view of the limitation court. Subject to the right of appeal, such
imposed by Section 101 that an action for

Rolan Jeff A. Lancion


Arellano University School of Law Page 104
Remedial Law 2- Brondial Notes AY 2017- 2018

allowance of the will shall be conclusive as to its -The rule provides that whoever that has
due execution. possession of the will must surrender it to court
within 20 days of death or knowledge of it.
PROBATE OF A WILL IS MANDATORY
-2 months after the death of Mr. X, Mrs X and
-Any court that takes cognizance of a probate of Mr B (Second son) filed for the probate of the
a will shall have limited jurisdiction. will annexed in the RTC of Makati City.

-Limited jurisdiction is a misnomer Q: What shall be the proper venue of the


probate of the will?
-Settlement of estate shall begin with the
probate of a will or appointment of A: It shall be Makati City since it is the last
administrator residence of the decedent.

-Limited jurisdiction is when the court is acting Q: Where does Mr. A should file his opposition
on a probate of a will. over the probate of the will?

Section 2. Custodian of will to deliver. — The A: It shall be in Cebu RTC since Makati City is
person who has custody of a will shall, within not the proper jurisdiction
twenty (20) days after he knows of the death
of the testator, deliver the will to the court Q: Is there a probate of the will in Cebu when
having jurisdiction, or to the executor named in Mr. A surrendered it to the court?
the will.
A: YES, the mere surrender of the will in court
-Whoever is in possession of a will, it has the connotes that it can now set the hearing for the
duty to surrender such will. If that person does probate.
not surrender the will, he can be penalized by
court. Exclusionary Rule- The court that take
cognizance of the case, it takes it to the
-That person should surrender such will to the exclusion of all other courts.
court within 20 days from death of decedent or
knowledge of the death. Rule 76- Allowance or Disallowance of Will

Example: Mr. X resides in Makati City, he has 5 Section 1. Who may petition for the allowance
children and he executed a will in Makati City. of will. — Any executor, devisee, or legatee
Upon the execution of the will, he gave it to his named in a will, or any other person interested
eldest son Mr. A. 2 years later Mr. A got married in the estate, may, at any time after the death
and transferred residence in Cebu City. Mr X of the testator, petition the court having
died a year after the marriage of Mr. A in jurisdiction to have the will allowed, whether
Makati City. Mr A complied with the provision the same be in his possession or not, or is lost
in Rule 75 and surrenders a copy of the will with or destroyed.
the RTC of Cebu City.

Rolan Jeff A. Lancion


Arellano University School of Law Page 105
Remedial Law 2- Brondial Notes AY 2017- 2018

DELTA- Devisee, Executor, Legatee, Testator, Rule 77- Allowance of Will Proved Outside of
Any person interested Philippines and Administration of Estate
Thereunder
No Witness Rule- The application for
allowance/disallowance of a will is filed by the
Section 1. Will proved outside Philippines may
testator himself
be allowed here. — Wills proved and allowed in
a foreign country, according to the laws of such
1-Witness Rule- A Notarial Will which is country, may be allowed, filed, and recorded by
uncontested and no oppositor the proper Court of First Instance in the
Philippines.
2 witness rule- lost/destroyed will, the
petitioner should prove its existence Q: Can a will executed outside the Philippines
executed in the Phil?
3 witness rule- contested holographic will
A: NO, Since it shall be considered as a foreign
4 witness rule- Notarial Will Contested; Present judgment
Three (3) witnesses including the notary public
Establishment of foreign judgment/reprobate
Section 9. Grounds for disallowing will. — The of a will
will shall be disallowed in any of the following
cases: Requisites:
(a) If not executed and attested as required by
-The testator should be domiciled outside the
law; Phil.
(b) If the testator was insane, or otherwise -The testator executed a will outside the Phil.
mentally incapable to make a will, at the time of
its execution;
-According to the law of the country executing
(c) If it was executed under duress, or the
-It is probated of a court of competent
influence of fear, or threats;
jurisdiction
(d) If it was procured by undue and improper
Q: What shall be the jurisdiction for the
pressure and influence, on the part of the
reprobate of the will?
beneficiary, or of some other person for his
benefit;
A: RTC since it is a court of general jurisdiction
and it is action incapable of pecuniary
(e) If the signature of the testator was procured
estimation
by fraud or trick, and he did not intend that the
instrument should be his will at the time of
Q: Where shall be the venue of the action?
fixing his signature thereto
A: It shall be the location/place of the estate

Rolan Jeff A. Lancion


Arellano University School of Law Page 106
Remedial Law 2- Brondial Notes AY 2017- 2018

-The reprobate court should render a valid (a) To the surviving husband or wife, as the case
judgment or issue a certificate of finality. may be, or next of kin, or both, in the discretion
of the court, or to such person as such surviving
-The court shall appoint ancillary administrator husband or wife, or next of kin, requests to
which shall take charge of the property in the have appointed, if competent and willing to
Philippines. serve;

Rule 78- Letters Testamentary and of (b) If such surviving husband or wife, as the case
Administration, when and to whom issued may be, or next of kin, or the person selected
by them, be incompetent or unwilling, or if the
For Letters Testamentary- Discretionary husband or widow, or next of kin, neglects for
towards the court. thirty (30) days after the death of the person to
apply for administration or to request that
-The executor must post a Bond and Accept the administration be granted to some other
trust. person, it may be granted to one or more of the
principal creditors, if may be granted to one or
Section 1. Who are incompetent to serve as more of the principal creditors, if competent
executors or administrators. — No person in and willing to serve;
competent to serve as executor or
administrator who: (c) If there is no such creditor competent and
willing to serve, it may be granted to such other
(a) Is a minor; person as the court may select.

(b) Is not a resident of the Philippines; and RULE 80-Special Administrator

(c) Is in the opinion of the court unfit to execute Section 1. Appointment of special
the duties of the trust by reason of administrator. — When there is delay in
drunkenness, improvidence, or want of granting letters testamentary or of
understanding or integrity, or by reason of administration by any cause including an appeal
conviction of an offense involving moral from the allowance or disallowance of a will,
turpitude. the court may appoint a special administrator
to take possession and charge of the estate of
*DELTA can file a petition for letters of the deceased until the questions causing the
testamentary delay are decided and executors or
administrators appointed.
Section 6. When and to whom letters of
administration granted. — If no executor is -There should be delay in the appointment of a
named in the will, or the executor or executors regular administrator/executor.
are incompetent, refuse the trust, or fail to give
bond, or a person dies intestate, administration Section 2. Powers and duties of special
shall be granted: adminsitrator. — Such special administrator
shall take possession and charge of the goods,
chattels, rights, credits, and estate of the

Rolan Jeff A. Lancion


Arellano University School of Law Page 107
Remedial Law 2- Brondial Notes AY 2017- 2018

deceased and preserve the same for the Grandchildren are not entitled to the fruits of
executors or administrator afterwards the estate. (Heirs of Hilario Ruiz vs Edmond
appointed, and for that purpose may Ruiz)
commence and maintain suits as administrator.
He may sell only such perishable and other
property as the court orders sold. A special
administrator shall not be liable to pay any RULE 85- Accountability and Compensation of
debts of the deceased unless so ordered by the Executors and Administrators
court.
Section 1. Executor or administrator
Q: What are the rights & obligations of the chargeable with all estate and income. —
administrator/executor? Except as otherwise expressly provided in the
following sections, every executor or
A: -Payment/Posting of bonds administrator is chargeable in his account with
the whole of the estate of the deceased which
-Submission of inventory within 3 months from has come into his possession, at the value of
appointment the appraisement contained in the inventory;
with all the interest, profit, and income of such
-Accounting of the estate within 1 year from estate; and with the proceeds of so much of the
appointment or order of probate court estate as is sold by him, at the price at which it
was sold.
-Payment of debts
-A lawyer and Executor at the same time cannot
-Sale, mortgage or encumbrance of the estate charge attorney’s fee.

-Distribution & partition of the estate- END of Remedy: Charge such expenses with all the
the settlement of estate estate and income

RULE 83-Inventory and Appraisal. Provision for Rule 86- Claims Against estate
Support of Family
Section 1. Notice to creditors to be issued by
Section 1. Inventory and appraisal to be court. — Immediately after granting letters
returned within three months. — Within three testamentary or of administration, the court
(3) months after his appointment every shall issue a notice requiring all persons having
executor or administrator shall return to the money claims against the decedent to file
court a true inventory and appraisal of all real them in the office of the clerk of said court.
and personal estate of the deceased which has
come into his possession or knowledge. In the Section 2. Time within which claims shall be
appraisement of such estate, the court may filed. — In the notice provided in the preceding
order one or more of the inheritance tax section, the court shall estate the time for the
appraisers to give his or their assistance. filing of claims against the estate, which shall
not be more than twelve (12) not less than six
(6) months after the date of the first

Rolan Jeff A. Lancion


Arellano University School of Law Page 108
Remedial Law 2- Brondial Notes AY 2017- 2018

publication of the notice. However, at any time Q: Which should prevail statute non-claims or
before an order of distribution is entered, on statute of limitation?
application of a creditor who has failed to file
his claim within the previously limited, the court A: Statute of non-claims since if a party do not
may, for cause shown and on such terms as are file against the estate within the period, it shall
equitable, allow such claim to be filed within a be barred forever.
time not exceeding one (1) month.
-A party should comply with the period of
Statute of non-claims- Claims against estate statute of non-claims rather than statute of
which shall not be more that twelve (12) not limitation.
less than six (6) months after the date of the
first publication of the notice. Example of Contingent Claim

Q: What are claimable against estate? -Mr. Mayaman promised Juan that if he will
pass the bar examination, he will give him Php 5
A: The basis shall be Rule 86 Sec 5: million pesos. Before the release of the bar
exams results, Mr Mayaman died but upon the
-Claims for money against the decedent based release of the result Juan passed the bar
on contract. (Due, Not Due or Contingent) examination.

-Funeral Expenses -Juan can now file a contingent claim against


the estate of Mr. Mayaman within the period of
-Expenses for last sickness statue of non-claims even if he passed or not
passed the bar exams.
-Judgment for money
Example of Funeral Expenses
Exception: Counter-claims brought by the
executor/administrator against claimants -Mr. Sakit died in St. Luke’s Hospital (BGC) due
to colon cancer, he stayed in that hospital for 6
Q: What is a statute of non-claims? months before he passed away. St. Lukes must
file such unpaid hospital expenses to the estate
A: Period wherein any person that has an of Mr. Sakit. In the woek of Mr. Sakit, Arlington
interest over the estate can file their claim. funeral services catered the funeral. In the
event the heirs of Mr. Sakit failed to pay the
Q: What is period of statute of non-claims? funeral expenses, Arlington can file a claim
against the estate.
A: It shall not be more than 12 months nor less
than 6 months after the date of its publication. Section 7. Mortgage debt due from estate. — A
creditor holding a claim against the deceased
Q: Is there a conflict between statute of non- secured by mortgage or other collateral
claims against statute of limitations? security, may abandon the security and
prosecute his claim in the manner provided in
A: NO this rule, and share in the general distribution

Rolan Jeff A. Lancion


Arellano University School of Law Page 109
Remedial Law 2- Brondial Notes AY 2017- 2018

of the assets of the estate; or he may foreclose No on-going settlement of estate- A Party can
his mortgage or realize upon his security, by start the filing of such complaint for the
action in court, making the executor or settlement of estate
administrator a party defendant, and if there is
a judgment for a deficiency, after the sale of -The executor/administrator shall file an answer
the mortgaged premises, or the property within 15 days against the person interested
pledged, in the foreclosure or other proceeding
to realize upon the security, he may claim his -If a party has a claim for recovery of
deficiency judgment in the manner provided in real/personal property, recovery of interest
the preceding section or he may rely upon his therein, claim for damages against the estate.
mortgage or other security alone, and
foreclosure the same at any time within the -A party can file a separate action against the
period of the statute of limitations, and in that executor/administrator under Rule 87
event he shall not be admitted as a creditor,
and shall receive no share in the distribution of Q: What if the estate is not sufficient to cover
the other assets of estate; but nothing herein the payment of debts and no specific provision
contained shall prohibit the executor or in the will as to what property shall be used,
administrator from redeeming the property what will be the proper remedy?
mortgaged or pledged, by paying the debt for
which it is held as security, under the direction A: The personal property shall be the first in
of the court, if the court shall adjudge it to be priority, if the personal property is not
for the best interest of the estate that such sufficient, a party can proceed to the real
redemption shall be made. property.

Q: How can a party can claim against the estate -Only real property can be the subject of
under a mortgage property? mortgage, encumbrances and levy. Personal
properties can only be sold ahead of the real
A: A party has 3 options: (a) file it as a claim properties.
against the estate, (b) apply Rule 68-
foreclosure of real estate mortgage, (c) avail of Rule 87- Actions By and Against Executors and
(RA 3135)- extra-judicial foreclosure of real Administrators
estate mortgage
Section 1. Actions which may and which may
-The following remedies are not successive but not be brought against executor or
alternative in nature administrator. — No action upon a claim for
the recovery of money or debt or interest
Q: How do a party can claim against the estate? thereon shall be commenced against the
executor or administrator; but to recover real
A: It will depend if there is an on-going or personal property, or an interest therein,
settlement of estate- File a motion and Present from the estate, or to enforce a lien thereon,
evidence and actions to recover damages for an injury to
person or property, real or personal, may be
commenced against him.

Rolan Jeff A. Lancion


Arellano University School of Law Page 110
Remedial Law 2- Brondial Notes AY 2017- 2018

Section 2. Executor or administrator may bring mortgaged, up to the value thereof;


or defend actions which survive. — For the
recovery or protection of the property or rights (5) Credits for the making, repair,
of the deceased, an executor or administrator safekeeping or preservation of personal
may bring or defend, in the right of deceased, property, on the movable thus made,
actions for causes which survive. repaired, kept or possessed;

Rule 88- Payment of Debts of the Estate (6) Claims for laborers' wages, on
the goods manufactured or the work done;
The Executor/Administrator shall be guided
with preference of credit under the Civil Code: (7) For expenses of salvage, upon the goods
salvaged;
Art. 2241. With reference to specific
movable property of the debtor, the following (8) Credits between the landlord and the
claims or liens shall be preferred: tenant, arising from the contract of tenancy
on shares, on the share of each in the fruits
(1) Duties, taxes and fees due thereon or harvest;
to the State or any subdivision thereof;
(9) Credits for transportation, upon the
(2) Claims arising from misappropriation, goods carried, for the price of the contract
breach of trust, or malfeasance by public and incidental expenses, until their delivery
officials committed in the performance of and for thirty days thereafter;
their duties, on the movables, money or
securities obtained by them; (10) Credits for lodging and supplies
usually furnished to travelers by hotelkeepers,
(3) Claims for the unpaid price of movables on the movables belonging to the guest as long
sold, on said movables, so long as they are as such movables are in the hotel, but not for
in the possession of the debtor, up to the money loaned to the guests;
value of the same; and if the movable has been
resold by the debtor and the price is still (11) Credits for seeds and expenses for
unpaid, the lien may be enforced on the price; cultivation and harvest advanced to the debtor,
this right is not lost by the immobilization of upon the fruits harvested;
the thing by destination, provided it has not
lost its form, substance and identity; neither is (12) Credits for rent for one year, upon
the right lost by the sale of the thing the personal property of the lessee existing on
together with other property for a lump the immovable leased and on the fruits of the
sum, when the price thereof can be same, but not on money or instruments of
determined proportionally; credit;

(4) Credits guaranteed with a pledge so (13) Claims in favor of the depositor if
long as the things pledged are in the hands the depositary has wrongfully sold the thing
of the creditor, or those guaranteed by a deposited, upon the price of the sale.
chattel mortgage, upon the things pledged or

Rolan Jeff A. Lancion


Arellano University School of Law Page 111
Remedial Law 2- Brondial Notes AY 2017- 2018

Art. 2242. With reference to specific pecuniary charges or other conditions imposed
immovable property and real rights of upon the donee, upon the immovable donated;
the debtor, the following claims, mortgages
and liens shall be preferred, and shall constitute (10) Credits of insurers, upon the property
an encumbrance on the immovable or real insured, for the insurance premium for two
right: years.

(1) Taxes due upon the land or building;

(2) For the unpaid price of real property CASES:


sold, upon the immovable sold;
Case: San Luis vs San Luis, 514 SCRA, Feb 2007
(3) Claims of laborers, masons,
mechanics and other workmen, as well as Facts:
of architects, engineers and contractors,
The parties involved in this case are the heirs of
engaged in the construction, reconstruction or
repair of buildings, canals or other works, former governor of Laguna. During his lifetime,
upon said buildings, canals or other works; Governor San Luis contracted three marriages.
On his first marriage, he had three (3) legitimate
(4) Claims of furnishers of materials children but the spouse died eventually. In his
used in the construction, reconstruction, or
second marriage, Governor San Luis obtained a
repair of buildings, canals or other works,
upon said buildings, canals or other works; divorce decree in the United States, On his third
marriage, Gov. San Luis married Feliscitas.
(5) Mortgage credits recorded in the During the death of the decedent, they were
Registry of Property, upon the real estate living in Alabang, Muntinlupa. Feliscitas filed a
mortgaged;
petition for the administration of the estate of
(6) Expenses for the preservation or Gov. San Luis towards the Regional Trial Court
improvement of real property when the law of Makati. The children of Gov. San Luis
authorizes reimbursement, upon the challenged the said petition on the ground of
immovable preserved or improved; wrong venue since it should be filed in Laguna
since the decedent is the governor of the
(7) Credits annotated in the Registry of
Property, in virtue of a judicial order, by province for a long time. They also questioned
attachments or executions, upon the property Feliscitas as not the proper party in interest
affected, and only as to later credits; since the marriage between Gov. San Luis and
Feliscitas is void ab initio.
(8) Claims of co-heirs for warranty in the
partition of an immovable among them, Issue: WON RTC of Makati is the proper venue
upon the real property thus divided; for the settlement of estate

(9) Claims of donors or real property for Held:

Rolan Jeff A. Lancion


Arellano University School of Law Page 112
Remedial Law 2- Brondial Notes AY 2017- 2018

YES- Based on the doctrine laid down in Fule an Opposition/Motion to Dismiss. The
“The venue for the settlement of estate shall be petitioners asserted that as shown by his Death
the last residence of the decendent”. Residence Certificate, Eliseo was a resident of Capas,
pertains to the actual, personal and physical Tarlac and not of Las Piñas City, at the time of
abode of the decedent. his death. Pursuant to Section 1, Rule 73 of the
Revised Rules of Court, the petition for
-The court also ruled that the requirement for
settlement of decedent’s estate should have
venue under the settlement of estate shall be
been filed in Capas, Tarlac and not in Las Piñas
residence and not domicile. Domicile is defined
City.
the residence of someone with intent to remain
so for the purposes of election, since the -The lower court ruled that the venue of the
requirement is residence and not domicile petition was properly laid in Las Piñas City,
thereby discrediting the position taken by the
Case: Garcia-Quiazon vs Belen, 702 SCRA
petitioners that Eliseo’s last residence was in
July 31,2013 Capas, Tarlac, as hearsay.
Facts:

- On 12 September 1994, Maria Lourdes Elise Issue: WON the petition for letters of
Quiazon (Elise), represented by her mother, Ma. administration if properly filed in Las Pinas as
Lourdes Belen (Lourdes), filed a Petition for venue of the action
Letters of Administration before the Regional
Trial Court (RTC) of Las Piñas City. In her
Petition docketed as SP Proc. No. M-3957, Elise Held:
claims that she is the natural child of Eliseo
YES- Rule 73 Sec. 1. Where estate of deceased
having been conceived and born at the time
persons settled. – If the decedent is an
when her parents were both capacitated to
inhabitant of the Philippines at the time of his
marry each other. Insisting on the legal capacity
death, whether a citizen or an alien, his will
of Eliseo and Lourdes to marry, Elise impugned
shall be proved, or letters of administration
the validity of Eliseo’s marriage to Amelia by
granted, and his estate settled, in the Court of
claiming that it was bigamous for having been
First Instance now Regional Trial Court in the
contracted during the subsistence of the latter’s
province in which he resides at the time of his
marriage with one Filipito Sandico (Filipito).
death, and if he is an inhabitant of a foreign
-Claiming that the venue of the petition was country, the Court of First Instance now
improperly laid, Amelia, together with her Regional Trial Court of any province in which
children, Jenneth and Jennifer, opposed the he had estate.
issuance of the letters of administration by filing

Rolan Jeff A. Lancion


Arellano University School of Law Page 113
Remedial Law 2- Brondial Notes AY 2017- 2018

- The term "resides" connotes ex vi termini properly laid in Las Piñas City. It is evident from
"actual residence" as distinguished from "legal the records that during his lifetime, Eliseo
residence or domicile." This term "resides," like resided at No. 26 Everlasting Road, Phase 5,
the terms "residing" and "residence," is elastic Pilar Village, Las Piñas City. For this reason, the
and should be interpreted in the light of the venue for the settlement of his estate may be
object or purpose of the statute or rule in which laid in the said city.
it is employed. In the application of venue
Case: Agtarap vs Agtarap, 651 SCRA, October
statutes and rules – Section 1, Rule 73 of the
2012
Revised Rules of Court is of such nature –
residence rather than domicile is the Facts:
significant factor. Even where the statute uses -On September 15, 1994, Eduardo filed with the
word "domicile" still it is construed as meaning Regional Trial Court (RTC), Branch 114, a
residence and not domicile in the technical verified petition for the judicial settlement of
sense. Some cases make a distinction between the estate of his deceased father Joaquin
the terms "residence" and "domicile" but as Agtarap (Joaquin).
generally used in statutes fixing venue, the
terms are synonymous, and convey the same -The petition alleged that Joaquin died intestate
meaning as the term "inhabitant." In other on November 21, 1964 in without any known
words, "resides" should be viewed or debts or obligations. During his lifetime, Joaquin
understood in its popular sense, meaning, the contracted two marriages, first with Lucia
personal, actual or physical habitation of a Garcia (Lucia), and second with Caridad Garcia
person, actual residence or place of abode. It (Caridad). Lucia died on April 24, 1924.
signifies physical presence in a place and actual -Joseph, Gloria, and Teresa filed their
stay thereat. Venue for ordinary civil actions answer/opposition. They alleged that the two
and that for special proceedings have one and subject lots belong to the conjugal partnership
the same meaning. As thus defined, of Joaquin with Lucia, and that, upon Lucias
"residence," in the context of venue death in April 1924, they became the pro
provisions, means nothing more than a indiviso owners of the subject properties. They
person’s actual residence or place of abode, said that their residence was built with the
provided he resides therein with continuity exclusive money of their late father Jose, and
and consistency. the expenses of the extensions to the house
were shouldered by Gloria and Teresa, while
the restaurant (Manongs Restaurant) was built
-The Court of Appeals cannot be faulted for with the exclusive money of Joseph and his
affirming the ruling of the RTC that the venue business partner.
for the settlement of the estate of Eliseo was

Rolan Jeff A. Lancion


Arellano University School of Law Page 114
Remedial Law 2- Brondial Notes AY 2017- 2018

-RTC issued a resolution and order of partition administered, and liquidated, and the debts
of the property involved. thereof paid; in the testate or intestate
proceedings of the deceased spouse, and if
Issue: WON the court acquired jurisdiction over
both spouses have died, the conjugal
the distribution of inheritance
partnership shall be liquidated in the testate or
Held: intestate proceedings of either. Thus, the RTC
YES had jurisdiction to determine whether the
properties are conjugal as it had to liquidate
-The general rule is that the jurisdiction of the the conjugal partnership to determine the
trial court, either as a probate or an intestate estate of the decedent.
court, relates only to matters having to do with
the probate of the will and/or settlement of the
estate of deceased persons, but does not Case: Suntay III vs Cojuangco-Suntay 683 SCRA
extend to the determination of questions of October 2012
ownership that arise during the proceedings.
Facts:
The patent rationale for this rule is that such
court merely exercises special and limited - The decedent Cristina Aguinaldo-Suntay
jurisdiction. (Cristina) died intestate on 4 June 1990. Cristina
was survived by her spouse, Dr. Federico Suntay
-The probate court may provisionally pass upon
(Federico) and five grandchildren: three
in an intestate or a testate proceeding the
legitimate grandchildren, including herein
question of inclusion in, or exclusion from, the
respondent, Isabel; and two illegitimate
inventory of a piece of property without
grandchildren, including petitioner Emilio III, all
prejudice to the final determination of
by Federico’s and Cristina’s only child, Emilio A.
ownership in a separate action. Second, if the
Suntay (Emilio I), who predeceased his parents.
interested parties are all heirs to the estate, or
the question is one of collation or -Isabel’s parents, along with her paternal
advancement, or the parties consent to the grandparents, were involved in domestic
assumption of jurisdiction by the probate court relations cases, including a case for parricide
and the rights of third parties are not impaired, filed by Isabel Cojuangco against Emilio I. Emilio
then the probate court is competent to resolve I was eventually acquitted.
issues on ownership.
-On 27 September 1993, more than three years
-Section 2, Rule 73 of the Rules of Court after Cristina’s death, Federico adopted his
provides that when the marriage is dissolved illegitimate grandchildren, Emilio III and Nenita.
by the death of the husband or the wife, the On 26 October 1995, respondent Isabel, filed
community property shall be inventoried, before the Regional Trial Court (RTC), Malolos,

Rolan Jeff A. Lancion


Arellano University School of Law Page 115
Remedial Law 2- Brondial Notes AY 2017- 2018

Bulacan, a petition for the issuance of letters of days after the death of the person to apply for
administration over Cristina’s estate. administration or to request that
administration be granted to some other
-Federico filed a Motion to Dismiss Isabel’s
person, it may be granted to one or more of
petition for letters of administration on the
the principal creditors, if competent and
ground that Isabel had no right of
willing to serve; (c) If there is not such creditor
representation to the estate of Cristina, she
competent and willing to serve, it may be
being an illegitimate grandchild of the latter as
granted to such other person as the court may
a result of Isabel’s parents’ marriage being
select.
declared null and void.
-The paramount consideration in the
-The trial court rendered a decision appointing
appointment of an administrator over the
Emilio III as administrator of decedent Cristina’s
estate of a decedent is the prospective
intestate estate. The Court of Appeals reversed
administrator’s interest in the estate. This is the
and set aside the decision of the RTC, revoked
same consideration which Section 6, Rule 78
the Letters of Administration issued to Emilio III,
takes into account in establishing the order of
and appointed respondent as administratrix of
preference in the appointment of administrator
the subject estate
for the estate. The rationale behind the rule is
Issue: WON Emilio III is the proper that those who will reap the benefit of a wise,
administrator of the estate of the decedent speedy and economical administration of the
Held: estate, or, in the alternative, suffer the
consequences of waste, improvidence or
NO-Rule 78 SEC. 6 When and to whom letters mismanagement, have the highest interest and
of administration granted. – If no executor is most influential motive to administer the
named in the will, or the executor or executors estate correctly.
are incompetent, refuse the trust, or fail to give -Under certain circumstances and for various
bond, or a person dies intestate, administration reasons well-settled in Philippine and American
shall be granted:(a) To the surviving husband or jurisprudence, we have upheld the
wife, as the case may be, or next of kin, or appointment of co-administrators: (1) to have
both, in the discretion of the court, or to such the benefits of their judgment and perhaps at
person as such surviving husband or wife, or all times to have different interests
next of kin, requests to have appointed, if represented;(2) where justice and equity
competent and willing to serve; (b) If such demand that opposing parties or factions be
surviving husband or wife, as the case may be, represented in the management of the estate
or next of kin, or the person selected by them, of the deceased; (3) where the estate is large
be incompetent or unwilling, or if the husband or, from any cause, an intricate and perplexing
or widow, or next of kin, neglects for thirty (30) one to settle; (4) to have all interested persons

Rolan Jeff A. Lancion


Arellano University School of Law Page 116
Remedial Law 2- Brondial Notes AY 2017- 2018

satisfied and the representatives to work in -On June 29, 1992, four years after the testators
harmony for the best interests of the estate; death, it was private respondent Maria Pilar
and when a person entitled to the Ruiz Montes who filed before the Regional Trial
administration of an estate desires to have Court, Branch 156, Pasig, a petition for the
another competent person associated with him probate and approval of Hilario Ruizs will and
in the office. for the issuance of letters testamentary to
Edmond Ruiz. Edmond opposed the petition on
-it should be noted that on the matter of
the ground that the will was executed under
appointment of administrator of the estate of
undue influence.
the deceased, the surviving spouse is preferred
over the next of kin of the decedent. When the -On January 19, 1993, the probate court
law speaks of "next of kin," the reference is to ordered to deposit with the Branch Clerk of
those who are entitled, under the statute of Court the rental deposit and payments totalling
distribution, to the decedent's property; one P540,000.00 representing the one-year lease of
whose relationship is such that he is entitled to the Valle Verde property. In compliance, on
share in the estate as distributed, or, in short, January 25, 1993, turned over the amount of
an heir. In resolving, therefore, the issue of P348,583.56, representing the balance of the
whether an applicant for letters of rent after deducting P191,416.14 for repair and
administration is a next of kin or an heir of the maintenance expenses on the estate.
decedent, the probate court perforce has to
-On May 14, 1993, Edmond withdrew his
determine and pass upon the issue of filiation.
opposition to the probate of the will.
Case: Heirs of Hilario Ruiz vs Edmond Ruiz, 252 Consequently, the probate court on May 18,
SCRA 1993, admitted the will to probate and ordered
the issuance of letters testamentary to
Facts:
conditioned upon the filing of a bond in the
-On June 27, 1987, Hilario M. Ruiz executed a amount of P50,000.00. The letters testamentary
holographic will naming as his heirs his only son, were issued on June 23, 1993.
Edmond Ruiz, his adopted daughter, private
Issue: WON the estate shall be liable for the
respondent Maria Pilar Ruiz Montes, and his
support of the testator's grandchildren
three granddaughters, private respondents
Maria Cathryn, Candice Albertine and Maria Held:
Angeline, all children of Edmond Ruiz. The
NO- Section 3 of Rule 83 of the Revised Rules of
testator bequeathed to his heirs substantial
Court provides: Sec. 3. Allowance to widow and
cash, personal and real properties and named
family. - The widow and minor or incapacitated
Edmond Ruiz executor of his estate.
children of a deceased person, during the
settlement of the estate, shall receive

Rolan Jeff A. Lancion


Arellano University School of Law Page 117
Remedial Law 2- Brondial Notes AY 2017- 2018

therefrom under the direction of the court, legitimate children (Rafael, Jose and Antonio
such allowance as are provided by law. Ortaez) and five illegitimate children by Ligaya
Novicio (herein private respondent Ma. Divina
-It is settled that allowances for support under
Ortaez-Enderes and her siblings Jose, Romeo,
Section 3 of Rule 83 should not be limited to the
Enrico Manuel and Cesar, all surnamed Ortaez).
minor or incapacitated children of the
deceased. Article 188 of the Civil Code of the -Juliana S. Ortaez, claiming that she owned
Philippines, the substantive law in force at the 1,014 Philinterlife shares of stock as her
time of the testators death, provides that conjugal share in the estate, sold said shares
during the liquidation of the conjugal with right to repurchase in favor of herein
partnership, the deceaseds legitimate spouse petitioner Filipino Loan Assistance Group
and children, regardless of their age, civil (FLAG), represented by its president, herein
status or gainful employment, are entitled to petitioner Jose C. Lee. Juliana Ortaez failed to
provisional support from the funds of the repurchase the shares of stock within the
estate. The law is rooted on the fact that the stipulated period, thus ownership thereof was
right and duty to support, especially the right to consolidated by petitioner FLAG in its name.
education, subsist even beyond the age of
-On July 12, 1995, herein private respondent
majority.
Ma. Divina Ortaez-Enderes and her siblings filed
-Grandchildren are not entitled to provisional a motion for appointment of special
support from the funds of the decedents administrator of Philinterlife shares of stock.
estate. The law clearly limits the allowance to This move was opposed by Special
widow and children and does not extend it to Administrator Jose Ortaez. On November 8,
the deceaseds grandchildren, regardless of their 1995, the intestate court granted the motion of
minority or incapacity. private respondents Enderes et al. and
appointed private respondent Enderes special
Case: Lee vs RTC of Quezon City Branch 85, 423 administratrix of the Philinterlife shares of
SCRA stock.

Facts: -On December 20, 1995, Special Administratrix


Enderes filed an urgent motion to declare void
-Dr. Juvencio P. Ortaez incorporated the
ab initio the memorandum of agreement dated
Philippine International Life Insurance
March 4, 1982. On January 9, 1996, she filed a
Company, Inc. on July 6, 1956. At the time of
motion to declare the partial nullity of the
the companys incorporation, Dr. Ortaez owned
extrajudicial settlement of the decedents
ninety percent (90%) of the subscribed capital
estate.
stock. On July 21, 1980, Dr. Ortaez died. He left
behind a wife (Juliana Salgado Ortaez), three

Rolan Jeff A. Lancion


Arellano University School of Law Page 118
Remedial Law 2- Brondial Notes AY 2017- 2018

-Our jurisprudence is therefore clear that (1)


Issue: WON the sale of the shares of stock be any disposition of estate property by an
rendered void ab initio since it done without administrator or prospective heir pending final
court approval adjudication requires court approval and (2)
any unauthorized disposition of estate
Held:
property can be annulled by the probate court,
YES -it is clear that Juliana Ortaez, and her three there being no need for a separate action to
sons, Jose, Rafael and Antonio, all surnamed annul the unauthorized disposition.
Ortaez, invalidly entered into a memorandum
-The sale of any property of the estate by an
of agreement extrajudicially partitioning the
administrator or prospective heir without
intestate estate among themselves, despite
order of the probate or intestate court is void
their knowledge that there were other heirs or
and passes no title to the purchaser.
claimants to the estate and before final
settlement of the estate by the intestate court.
Since the appropriation of the estate properties
Case: Unionbank vs Santibanez, 452 SCRA Feb
by Juliana Ortaez and her children (Jose, Rafael
2005
and Antonio Ortaez) was invalid, the
subsequent sale thereof by Juliana and Jose to a Facts:
third party (FLAG), without court approval, was -On May 31, 1980, the First Countryside Credit
likewise void. Corporation (FCCC) and Efraim M. Santibaez
-An heir can sell his right, interest, or entered into a loan agreement in the amount of
participation in the property under P128,000.00. The amount was intended for the
administration under Art. 533 of the Civil Code payment of the purchase price of one (1) unit
which provides that possession of hereditary Ford 6600 Agricultural All-Purpose Diesel
property is deemed transmitted to the heir Tractor. In view thereof, Efraim and his son,
without interruption from the moment of Edmund, executed a promissory note in favor of
death of the decedent. However, an heir can the FCCC, the principal sum payable in five
only alienate such portion of the estate that equal annual amortizations of P43,745.96 due
may be allotted to him in the division of the on May 31, 1981 and every May 31st thereafter
estate by the probate or intestate court after up to May 31, 1985.
final adjudication, that is, after all debtors shall -Sometime in February 1981, Efraim died,
have been paid or the devisees or legatees shall leaving a holographic will. Subsequently in
have been given their shares.This means that March 1981, testate proceedings commenced
an heir may only sell his ideal or undivided before the RTC of Iloilo City, Branch 7, docketed
share in the estate, not any specific property as Special Proceedings No. 2706. On April 9,
therein.

Rolan Jeff A. Lancion


Arellano University School of Law Page 119
Remedial Law 2- Brondial Notes AY 2017- 2018

1981, Edmund, as one of the heirs, was person to dispose of his property by will may
appointed as the special administrator of the be rendered nugatory. The authentication of a
estate of the decedent. will decides no other question than such as
touch upon the capacity of the testator and the
-On August 20, 1981, a Deed of Assignment
compliance with those requirements or
with Assumption of Liabilities was executed by
solemnities which the law prescribes for the
and between FCCC and Union Savings and
validity of a will.
Mortgage Bank, wherein the FCCC as the
assignor, among others, assigned all its assets -The probate proceeding had already acquired
and liabilities to Union Savings and Mortgage jurisdiction over all the properties of the
Bank. deceased, including the three (3) tractors. To
dispose of them in any way without the probate
-Demand letters for the settlement of his
courts approval is tantamount to divesting it
account were sent by petitioner Union Bank of
with jurisdiction which the Court cannot allow.
the Philippines (UBP) to Edmund, but the latter
Every act intended to put an end to indivision
failed to heed the same and refused to pay.
among co-heirs and legatees or devisees is
Thus, on February 5, 1988, the petitioner filed a
deemed to be a partition, although it should
Complaint for sum of money against the heirs of
purport to be a sale, an exchange, a
Efraim Santibaez, Edmund and Florence, before
compromise, or any other transaction. Thus, in
the RTC of Makati City, Branch 150.
executing any joint agreement which appears
Issue: WON the partition in the agreement to be in the nature of an extra-judicial
executed by the heirs is valid partition, as in the case at bar, court approval
Held: is imperative, and the heirs cannot just divest
the court of its jurisdiction over that part of
NO the estate. Moreover, it is within the
-A probate court has the jurisdiction to jurisdiction of the probate court to determine
determine all the properties of the deceased, to the identity of the heirs of the decedent. In the
determine whether they should or should not instant case, there is no showing that the
be included in the inventory or list of properties signatories in the joint agreement were the only
to be administered. heirs of the decedent. When it was executed,
the probate of the will was still pending before
-In testate succession, there can be no valid
the court and the latter had yet to determine
partition among the heirs until after the will
who the heirs of the decedent were. Thus, for
has been probated. The law enjoins the
Edmund and respondent Florence S. Ariola to
probate of a will and the public requires it,
adjudicate unto themselves the three (3)
because unless a will is probated and notice
tractors was a premature act, and prejudicial to
thereof given to the whole world, the right of a
the other possible heirs and creditors who may

Rolan Jeff A. Lancion


Arellano University School of Law Page 120
Remedial Law 2- Brondial Notes AY 2017- 2018

have a valid claim against the estate of the -Maximino was married to Donata but their
deceased. union did not produce any children. When
Maximino died on 1 May 1952, Donata
-The filing of a money claim against the
instituted intestate proceedings to settle her
decedents estate in the probate court is
husbands estate with the Cebu City Court of
mandatory. This requirement is for the purpose
First Instance (CFI), 14th Judicial District.
of protecting the estate of the deceased by
informing the executor or administrator of the -Donata died on 1 November 1977. Erlinda, one
claims against it, thus enabling him to examine of Donatas nieces, instituted with the RTC a
each claim and to determine whether it is a petition for the administration of the intestate
proper one which should be allowed. The plain estate of Donata. Erlinda and her husband,
and obvious design of the rule is the speedy Gregorio, were appointed by the RTC as
settlement of the affairs of the deceased and administrators of Donatas intestate estate.
the early delivery of the property to the Controversy arose among Donatas heirs when
distributees, legatees, or heirs. `The law strictly Erlinda claimed exclusive ownership of three
requires the prompt presentation and parcels of land.
disposition of the claims against the decedent's
-On 3 March 1987, the heirs of Maximino filed a
estate in order to settle the affairs of the estate
Complaint with the RTC against the heirs of
as soon as possible, pay off its debts and
Donata for the partition, annulment, and
distribute the residue.
recovery of possession of real property. RTC
Case: Pilapil vs Heirs of Maximo Briones, 514 rendered a decision in favor of the heirs of
SCRA, Feb 2007 Maximino.

Facts: Issue: WON there was fraud in the acquisition


of the decision made by the lower court
-Petitioners are the heirs of the late Donata
Ortiz-Briones (Donata), consisting of her Held:
surviving sister, Rizalina Ortiz-Aguila (Rizalina);
NO- The existence of any trust relations
Rizalinas daughter, ErlindaPilapil (Erlinda); and
between petitioners and respondents shall be
the other nephews and nieces of Donata, in
examined in the light of Article 1456 of the New
representation of her two other sisters who had
Civil Code, which provides that, [i]f property is
also passed away. Respondents, on the other
acquired through mistake or fraud, the person
hand, are the heirs of the late Maximino
obtaining it is, by force of law, considered a
Briones (Maximino), composed of his nephews
trustee of an implied trust for the benefit of the
and nieces, and grandnephews and
person from whom the property comes. Hence,
grandnieces, in representation of the deceased
the foremost question to be answered is still
siblings of Maximino.
whether an implied trust under Article 1456 of

Rolan Jeff A. Lancion


Arellano University School of Law Page 121
Remedial Law 2- Brondial Notes AY 2017- 2018

the New Civil Code had been sufficiently Case: Heirs of Maglasang vs MBC, 706 SCRA
established in the present case. 235

-While it is true that since the CFI was not Facts:


informed that Maximino still had surviving
-On June 16, 1975, spouses Flaviano and Salud
siblings and so the court was not able to order
Maglasang (Sps.Maglasang) obtained a credit
that these siblings be given personal notices of
line from respondent in the amount of
the intestate proceedings, it should be borne in
₱350,000.00 which was secured by a real estate
mind that the settlement of estate, whether
mortgage executed over seven of their
testate or intestate, is a proceeding in rem, and
properties located in Ormoc City and the
that the publication in the newspapers of the
Municipality of Kananga, Province of Leyte.
filing of the application and of the date set for
They availed of their credit line by securing
the hearing of the same, in the manner
loans in the amounts of ₱209,790.50 and
prescribed by law, is a notice to the whole
₱139,805.83 on October 24, 1975 and March
world of the existence of the proceedings and
15, 1976, respectively, both of which becoming
of the hearing on the date and time indicated in
due and demandable within a period of one
the publication. The publication requirement of
year.
the notice in newspapers is precisely for the
purpose of informing all interested parties in -After Flaviano Maglasang (Flaviano) died
the estate of the deceased of the existence of intestate on February 14,1977, his widow Salud
the settlement proceedings, most especially Maglasang (Salud) and their surviving children,
those who were not named as heirs or filed a verified petition for letters of
creditors in the petition, regardless of whether administration of the intestate estate of
such omission was voluntarily or involuntarily Flaviano before the then Court of First Instance
made. of Leyte, Ormoc City, Branch 5 (probate court).

-Although Donata may have alleged before the -In view of the issuance of letters of
CFI that she was her husbands sole heir, it was administration, the probate court, on August
not established that she did so knowingly, 30, 1977, issued a Notice to Creditors for the
maliciously and in bad faith, so as for this Court filing of money claims against Flaviano’s estate.
to conclude that she indeed committed fraud. -The probate court terminated the proceedings
This Court again brings to the fore the delay by with the surviving heirs executing an extra-
which respondents filed the present case, when judicial partition of the properties of Flaviano’s
the principal actors involved, particularly, estate. The loan obligations owed by the estate
Donata and Maximinos siblings, have already to respondent, however, remained unsatisfied
passed away and their lips forever sealed as to due to respondent’s certification that Flaviano’s
what truly transpired between them. account was undergoing a restructuring.

Rolan Jeff A. Lancion


Arellano University School of Law Page 122
Remedial Law 2- Brondial Notes AY 2017- 2018

-RTC rendered a decision directing the heirs of without the right to file a claim for any
Maglasang to pay the amount of ₱434,742.36 deficiency.
with interest at the rate of 12%. CA affirmed the
-In our jurisdiction, the remedies available to
decision made by the RTC.
the mortgage creditor are deemed alternative
Issue: WON CA erred in affirming the RTC's and not cumulative. Notably, an election of one
award of deficiency judgment remedy operates as a waiver of the other. For
this purpose, a remedy is deemed chosen upon
Held:
the filing of the suit for collection or upon the
YES- Claims against deceased persons should be filing of the complaint in an action for
filed during the settlement proceedings of their foreclosure of mortgage, pursuant to the
estate. Such proceedings are primarily governed provision of Rule 68 of the 1997 Rules of Civil
by special rules found under Rules 73 to 90 of Procedure.
the Rules, although rules governing ordinary
-Section 7, Rule 86 governs the parameters and
actions may, as far as practicable, apply
the extent to which a claim may be advanced
suppletorily.
against the estate, whereas Act No. 3135 sets
-A creditor holding a claim against the deceased out the specific procedure to be followed when
secured by a mortgage or other collateral the creditor subsequently chooses the third
security" as above-highlighted, it may be option – specifically, that of extra-judicially
reasonably concluded that the aforementioned foreclosing real property belonging to the
section covers all secured claims, whether by estate. The application of the procedure under
mortgage or any other form of collateral, which Act No. 3135 must be concordant with Section
a creditor may enforce against the estate of the 7, Rule 86 as the latter is a special rule
deceased debtor. applicable to claims against the estate, and at
-Jurisprudence breaks down the rule under the same time, since Section 7, Rule 86 does
Section 7, Rule 86 and explains that the secured not detail the procedure for extra-judicial
creditor has three remedies/options that he foreclosures, the formalities governing the
may alternatively adopt for the satisfaction of manner of availing of the third option – such as
his indebtedness. In particular, he may choose the place where the application for extra-
to: (a) waive the mortgage and claim the entire judicial foreclosure is filed, the requirements of
debt from the estate of the mortgagor as an publication and posting and the place of sale –
ordinary claim; (b) foreclose the mortgage must be governed by Act No. 3135.
judicially and prove the deficiency as an -Having unequivocally opted to exercise the
ordinary claim; and (c) rely on the mortgage third option of extra-judicial foreclosure under
exclusively, or other security and foreclose the Section 7, Rule 86, respondent is now precluded
same before it is barred by prescription,

Rolan Jeff A. Lancion


Arellano University School of Law Page 123
Remedial Law 2- Brondial Notes AY 2017- 2018

from filing a suit to recover any deficiency - The Petitioner argues that the assailed
amount as earlier discussed. Decision and Order of the Court a quo, supra,
should be annulled and set aside on the
grounds of extrinsic fraud and lack of
Case: Butiong vs Plazo, 765 SCRA 227 jurisdiction.
Facts: Issue: WON RTC of Nasugbu Batangas has
-On November 16, 1989, Pedro L. Rifioza died jurisdiction over the settlement of estate
intestate, leaving several heirs, including his_ Held:
children with his first wife, respondents Ma.
- Considering that settlement of estate is a
Gracia R. Plazo and Ma. Fe Alaras, as well as
special proceeding cognizable by a probate
several properties including a resort covered by
court of limited jurisdiction while judicial
Transfer Certificates of Title (TCT) No. 51354
partition with annulment of title and recovery
and No. 51355, each with an area of 351 square
of possession are ordinary civil actions
meters, and a family home, the land on which it
cognizable by a court of general jurisdiction, the
stands is covered by TCT Nos. 40807 and 40808,
trial court exceeded its jurisdiction in
both located in Nasugbu, Batangas.
entertaining the latter while it was sitting
-Respondents alleged that sometime in March merely in its probate jurisdiction. This is in view
1991, they discovered that their co-heirs, of the prohibition found in the Rules on the
Pedro’s second wife, Benita"Tenorio and other joiner of special civil actions and ordinary civil
children, had sold the subject properties to actions.
petitioners, spouses Francisco Villafria and
Rule 74 Sec 1-. Extrajudicial settlement by
Maria Butiong, who are now deceased and
agreement between heirs. - If the decedent left
substituted by their son, Dr. Ruel B. Villafria,
no will and no debts and the heirs are all of
without their knowledge and consent.
age 5 or the minors are represented by their
-On October 1, 2001, the trial court nullified the judicial or legal representatives duly
transfer of the subject Properties to petitioners authorized for the purpose, the parties may
and spouses Bondoc due to irregularities in the without securing letters of administration,
Documents of conveyance offered by divide the estate among themselves as they
petitioner’s .as well as the circumstances see fit by means of a public instrument filed in
Surrounding the execution of the same. the office of the register of deeds, and should
Specifically, the Extra-Judicial Settlement was they disagree, they may do so in an ordinary
notarized by a notary public that was not duly action of partition. If there is only one heir, he
commissioned as such on the date it was may adjudicate to himself the entire estate by
executed. means of an affidavit filled in the office of the

Rolan Jeff A. Lancion


Arellano University School of Law Page 124
Remedial Law 2- Brondial Notes AY 2017- 2018

register of deeds. The parties to an the Rules of Court. An exception to this rule,
Extrajudicial settlement, whether by public however, is found in the aforequoted Section 1
instrument or by stipulation in a pending of Rule 4 wherein the heirs of a decedent, who
action for partition, or the sole heir who left no will and no debts due from is estate,
adjudicates the entire estate to himself by may divide the estate either extrajudicially or
means of an affidavit shall file, simultaneously in an ordinary action or partition without
with and as a condition precedent to the filing submitting the same for judicial administration
of the public instrument, or stipulation in the nor applying for the appointment of an
action for partition, or of the affidavit in the administrator by the court.
office of the register of deeds, a bond with the
-Section 1, Rule 74 of the Revised Rules of
said register of deeds, in an amount equivalent
Court, however, does not preclude the heirs
to the value of the personal property involved
from instituting administration proceedings,
as certified to under oath by the parties
even if the estate has no· debts or obligations, if
concerned and conditioned upon the payment
they do not desire to resort for good reasons to
of any just claim that may be filed under section
an ordinary action for partition. he complaint
4 of this rule. It shall be presumed that the
contained allegations inherent in an action for
decedent left no debts if no creditor files a
settlement of estate does not. Mean that there
petition for letters of administration within
was a prohibited joined of causes of action for
two (2) years after the death of the decedent.
questions as to the estate's properties as well as
-The fact of the Extrajudicial settlement or a determination of the heirs, their status as
administration shall be Published in a such, and the nature and extent of their titles to
newspaper of general circulation in the manner the estate, may also be properly ventilated in
provided in the next succeeding section; but no partition proceedings alone.
Extrajudicial settlement shall be binding upon
any person who has not participated therein or GUARDIANSHIP (Rule 92-97) AM No. 03-02-05
had no notice thereof. SC

-The general rule is that when a person dies Q: What are the three kinds of guardianship?
intestate, or, if testate, failed to name an
executor in his will or the executor o named is A: Natural, Judicial and Guardianship ad litem
incompetent, or refuses the trust, or. Fails to
Natural Guardian- Parents of the Ward
furnish the bond equipped by the Rules of
Court, then the decedent's estate shall be Judicial Guardian- Any person qualified to be
judicially administered and the competent appointed as guardian of the person of the
court shall appoint a qualified administrator ward or its property.
the order established in Section 6 of Rule 78 of

Rolan Jeff A. Lancion


Arellano University School of Law Page 125
Remedial Law 2- Brondial Notes AY 2017- 2018

Guardian Ad Litem- Any person qualified to be -Deaf and dumb who are unable to read and
appointed as guardian but has limited duty write
since his duty is for a particular purpose.
-Unsound mind even though they have lucid
Object of guardianship interval

-Person of the ward -Persons having a disease, weak mind and of


age
-Property of the ward
Sec 3- Venue
-Person & property of the ward
-Where the prospective ward resides
Kinds:
*Under the new rules, guardians are mandated
-Guardianship over the minor to post a bond.

-Guardianship over the incompetent -The natural guardians are not entitled to sell
the property of their wards.
*Under the new rules on guardianship, mere
minority is not a ground for him to qualify. -Their remedy is to apply for the appointment
as guardian ad litem in order for them to sell
(Example: Orphan, one of the parents died and such property
remarried)
Q: When is guardianship is terminated?
Rule 92- Venue
A: The petition for guardianship ceases to exist
Sec 1- Where to institute proceeding upon the death of the ward or guardian.
(Jurisdiction)
Case Doctrine: Caniza as Represented by
Minor- Family Court Amparo Evangelista vs CA

Incompetent & Minor- Family Court “Guardianship was terminated upon the death
of Caniza but the case against Sps. Estrada shall
Incompetent- RTC not be dismissed on the ground of substitution”

Sec 2- Meaning of word “incompetent” Rule 93- Appointment of Guardians

-Person suffering the penalty of civil interdiction -In default of parents or court appointed
guardian, the court may appoint a guardian.
-Hospitalized lepers
-Surviving grandparents
-Prodigals

Rolan Jeff A. Lancion


Arellano University School of Law Page 126
Remedial Law 2- Brondial Notes AY 2017- 2018

-Oldest brother or sister of the minor over 21 incapable of discharging his trust or unsuitable
years of age unless fit or disqualified therefor, or has wasted or mismanaged the
estate, or failed for thirty (30) days after it is
-Actual custodian of the minor over 21 years of due to render an account or make a return, the
age court may, upon reasonable notice to the
guardian, remove him, and compel him to
-Any other person who is the sound discretion surrender the estate of the ward to the person
of the court that would serve the best interest found to be lawfully entitled thereto. A
of the minor. guardian may resign when it appears proper to
allow the same; and upon his resignation or
Section 4. Opposition to petition. — Any removal the court may appoint another in his
interested person may, by filing a written place.
opposition, contest the petition on the ground
of majority of the alleged minor, competency of Section 3. Other termination of guardianship.
the alleged incompetent, or the insuitability of — The marriage or voluntary emancipation of
the person for whom letters are prayed, and a minor ward terminates the guardianship of
may pray that the petition be dismissed, or that the person of the ward, and shall enable the
letters of guardianship issue to himself, or to minor to administer his property as though he
any suitable person named in the opposition. were of age, but he cannot borrow the money
or alienate or encumber real property without
Rule 96- General Power and Duties of the consent of his father or mother, or
Guardians guardian. He can sue and be sued in court only
with the assistance of his father, mother or
Q: What is the authority of the guardian? guardian. The guardian of any person may be
discharged by the court when it appears, upon
A: A guardian has full authority to take the application of the ward or otherwise, that
possession of the property of the ward, to pay the guardianship is no longer necessary.
the debts of the ward, to settle accounts,
CASES:
collect debts, and appear in actions for the
ward, Estate to be managed frugally, and Goyena vs Ledesma-Gustillo, Jan 13, 2003
proceeds applied to maintenance of ward,
Guardian may be authorized to join in partition Facts:
proceedings after hearing, Inventories and
-On July 8, 1996, respondent filed at the RTC of
accounts of guardians, and appraisement of
Makati a PETITION FOR LETTERS OF
estates.
GUARDIANSHIP over the person and properties
of her sister Julieta.
Rule 97- Termination
-(a)That for the most part during the year 1995
Section 2. When the guardian removed or and 1996, Julieta Ledesma has been a patient in
allowed to resign. New appointment. — When the Makati Medical Center where she is under
a guardian becomes insane or otherwise
medical attention for old age, general debility,

Rolan Jeff A. Lancion


Arellano University School of Law Page 127
Remedial Law 2- Brondial Notes AY 2017- 2018

and a mini-stroke which she suffered in the observation by the trial court, cited in the
United States in early 1995; (b) That Julieta present petition, that Julieta was still placed
Ledesma is confined to her bed and cannot get under the care of doctors after she checked out
up from bed without outside assistance, and and was returned to the hospital when she
she has to be moved by wheel chair; (c) That suffered another stroke.
Julieta Ledesma owns real estate and personal
properties in Metro Manila and in Western - SC noted two undisputed facts in the case at
Visayas, with an aggregate estimated assessed bar, to wit: 1) Petitioner opposed the petition
and par value of P1 Million Pesos for the appointment of respondent as guardian
before the trial court because, among other
-Petitioner filed an Opposition to the petition reasons, she felt she was disliked by
for letters of guardianship. She later filed an respondent, a ground which does not render
Amended Opposition on August 15, 1996. respondent unsuitable for appointment as
-The trial court found Julieta incompetent and guardian, and 2) Petitioner concealed the
incapable of taking care of herself and her deteriorating state of mind of Julieta before the
property and appointed respondent as guardian trial court, which is reflective of a lack of good
of her person and properties. faith.

Issue: WON Amparo Gustillo is the proper Case: Neri vs Heirs of Hadji Yusop Uy, 683
guardian in this case SCRA

Doctrine: “Parents should apply for judicial


Held:
guardianship in order for them to sell properties
YES- As a rule, when it appears that the judge of their children.”
has exercised care and diligence in selecting
the guardian, and has given due consideration
Facts:
to the reasons for and against his action which
are urged by the interested parties, his action -During the lifetime of Petitoner’s mother,
should not be disturbed unless it is made very Anunciacion, she and her 2nd husband, Enrique,
clear that he has fallen into grievous error. acquired several homestead properties. When
Anunciacion died, however, Enrique in his
personal capacity and as natural guardian of his
minor children Rosa and Douglas, together
-The claim that respondent is hostile to the best with, Napoleon, Alicia and Visminda executed
interests of Julieta also lacks merit. That an Extra-Judicial Settlement of the Estate with
respondent removed Julieta from the Makati Absolute Deed of Sale (1979) adjudicating
among themselves the said homestead
Medical Center where she was confined after
properties, and thereafter, sold the properties
she suffered a stroke does not necessarily show to the late spouses Uy for a consideration of
her hostility towards Julieta, given the 80,000.

Rolan Jeff A. Lancion


Arellano University School of Law Page 128
Remedial Law 2- Brondial Notes AY 2017- 2018

-On 1996, the children of Enrique filed a Held: No, as to the shares of the minor children
complaint for annulment of the said sale against because as a natural guardian, he is merely
spouses Uy, assailing the validity of the sale for clothed with powers of administration.
having been sold within the prohibited period.
And, also, for having been executed without the With respect to Rosa and Douglas who were
consent or approval of Eutropia, Victoria, Rosa minors at the time of the execution of the
and Douglas; thus, depriving the latter siblings settlement and sale, their NATURAL GUARDIAN
of their legitime. and father, Enrique, represented them in the
transaction. However, on the basis of the laws
-Uy countered that the sale took place beyond prevailing at that time, Enrique was merely
the 5 year prohibitory period from the issuance clothed with POWERS OF ADMINISTRATION and
of the homestead patents. They also denied bereft of any authority to dispose of their 2/16
that Eutropia and Victoria were excluded from shares in the estate of their mother,
the Extra-judicial settlement and sale of the Anunciacion.
subject properties, and interposed further the
defense of prescription and laches. Power of dominion, is granted by law only to a
JUDICIAL GUARDIAN of the ward’s property
RTC rendered a Decision annulling the Extra- and even then only with court’s prior approval
judicial settlement of estate with Absolute Deed secured in accordance with the proceedings
of Sale. It ruled that the sale is void because set forth by the Rules.
Eutropia and Victoria were deprived of their
hereditary rights and that Enrique had no Exception: RATIFICATION Consequently, the
judicial authority to sell the shares of his minor disputed sale entered into by Enrique in behalf
children, Rosa and Douglas. of his minor children without the proper judicial
authority, unless ratified by them upon reaching
On appeal, however, CA reserved and set aside the age of majority, is unenforceable in
RTC decision. accordance with Art. 1317 and 1403(1) of the
Civil Code.
Issue: WON Enrique, as guardian of his children Records, however, show that Rosa had ratified
and co-owner (with his children), sell their co- the extrajudicial settlement of the estate with
owned property? absolute deed of sale. The same, however, is
not true with respect to Douglas for lack of
evidence showing ratification.

Rolan Jeff A. Lancion


Arellano University School of Law Page 129
Remedial Law 2- Brondial Notes AY 2017- 2018

THEREFORE, the extrajudicial settlement with -During the course of the trial, respondent filed
sale is invalid and not binding on Eutropia, demurrer to evidence which was granted by the
Victoria and Douglas. trial court. It was declared that petitioner failed
to provide documentary and testimonial
Case: Oropesa vs Oropesa, 671 SCRA (4/2012) evidence to establish that General Oropesa is
incompetent to run his personal affairs.
Facts:
Issue: WON respondent is considered as
-On January 23, 2004, the (petitioner) filed with incompetent person who should be placed
the Regional Trial Court of Paraaque City, a under guardianship
petition for him and a certain Ms. Louie Ginez
to be appointed as guardians over the property Held:
of his father, the (respondent) Cirilo Oropesa.
-A guardianship is a trust relation of the most
-It is alleged among others that the sacred character, in which one person, called a
(respondent) has been afflicted with several guardian acts for another called the ward whom
maladies and has been sickly for over ten (10) the law regards as incapable of managing his
years already having suffered a stroke on April own affairs. A guardianship is designed to
1, 2003 and June 1, 2003, that his judgment and further the wards well-being, not that of the
memory [were] impaired and such has been guardian. It is intended to preserve the wards
evident after his hospitalization; that even property, as well as to render any assistance
before his stroke, the (respondent) was that the ward may personally require. It has
observed to have had lapses in memory and been stated that while custody involves
judgment, showing signs of failure to manage immediate care and control, guardianship
his property properly; that due to his age and indicates not only those responsibilities, but
medical condition, he cannot, without outside those of one in loco parentis as well.
aid, manage his property wisely, and has
become an easy prey for deceit and exploitation -Persons who, though of sound mind but by
by people around him, particularly Ms. Ma. reason of age, disease, weak mind or other
Luisa Agamata, his girlfriend. similar causes, are incapable of taking care of
themselves and their property without outside
- On July 6, 2004, respondent filed his aid are considered as incompetents who may
Opposition to the petition for guardianship. properly be placed under guardianship. A

Rolan Jeff A. Lancion


Arellano University School of Law Page 130
Remedial Law 2- Brondial Notes AY 2017- 2018

finding that a person is incompetent should be -RTC reversed the decision "action by which the
anchored on clear, positive and definite issue of defendants' possession should be
evidence. resolved is accion publiciana, the obtaining
factual and legal situation, demanding
-With the failure of petitioner to formally offer adjudication by such plenary action for recovery
his documentary evidence, his proof of his of possession cognizable in the first instance by
fathers incompetence consisted purely of the Regional Trial Court."
testimonies given by himself and his sister.
These testimonies, which did not include any Issue: WON Evangelista, as Caiza's legal
expert medical testimony, were insufficient to guardian had authority to bring said action
convince the trial court of petitioners cause of
action and instead lead it to grant the demurrer Held:
to evidence that was filed by respondent.
-Amparo Evangelista was appointed by a
Case: Caniza vs CA, G.R. No. 110427 February competent court the general guardian of both
24, 1997 the person and the estate of her aunt, Carmen
Caiza. Her Letters of Guardianship dated
Facts: December 19, 1989 clearly installed her as the
"guardian over the person and properties of the
-On November 20, 1989, being then ninety-four incompetent CARMEN CAIZA with full authority
(94) years of age, Carmen Caiza, a spinster, a to take possession of the property of said
retired pharmacist, and former professor of the incompetent in any province or provinces in
College of Chemistry and Pharmacy of the which it may be situated and to perform all
University of the Philippines, was declared other acts necessary for the management of her
incompetent by judgment of the Regional Trial properties By that appointment, it became
Court of Quezon City, Branch 107, in a Evangelista's duty to care for her aunt's person,
guardianship proceeding instituted by her to attend to her physical and spiritual needs, to
niece, Amparo A. Evangelista. assure her well-being, with right to custody of
her person in preference to relatives and
-Judgment was rendered by the MetroTC on friends.
April 13, 1992 in Caiza's favor, the Estradas
being ordered to vacate the premises and pay -Evangelista was merely discharging the duty to
Caiza P5,000.00 by way of attorney's fees. attend to "the comfortable and suitable

Rolan Jeff A. Lancion


Arellano University School of Law Page 131
Remedial Law 2- Brondial Notes AY 2017- 2018

maintenance of the ward" explicitly imposed on letters of guardianship shall be issued only upon
her by Section 4, Rule 96 of the Rules of Court: “ the submission of the bond, conditioned on the
following provisions of the Rule 94 Section 1, of
A guardian must manage the estate of his
the 1997 Rules of Civil Procedure.
ward frugally and without waste, and apply
the income and profits thereof, so far as -Abad filed an appeal to the CA. He argued that
maybe necessary, to the comfortable and the RTC erred in disqualifying him from being
suitable maintenance of the ward and his appointed as Maura’s guardian despite the fact
family, if there be any; and if such income and that he has all the qualifications stated under
the Rules. That he was not a resident of
profits be insufficient for that purpose, the
Mangaldan, Pangasinan should not be a ground
guardian may sell or encumber the real estate,
for his disqualification as he had actively and
upon being authorized by order to do so, and
efficiently managed the affairs and properties
apply to such of the proceeds as may be
of his aunt even if he is residing in Metro
necessary to such maintenance."
Manila. Court of Appeals affirmed the decision
of the Regional Trial Court.
Case: Abad vs Biazon, 687 SCRA (12/5/2012)
Issue: WON Biason is qualifed to become the
guardian of Maura
Facts:
Held:
-On March 19, 2007, petitioner Eduardo Abad
(Abad) filed a petition for guardianship over the -In his petition, Abad prayed for the nullification
person and properties of Maura B. Abad of the CA Decision dated August 28, 2009 and
(Maura) with the Regional Trial Court (RTC), Resolution dated April 19, 2010, which
Dagupan City, Branch 42. He averred that dismissed his appeal from the Decision dated
Maura, who is single, more than ninety (90) September 26, 2007 of the RTC and denied his
years old and a resident of Rizal Street, motion for reconsideration, respectively.
Poblacion, Mangaldan, Pangasinan, is in dire Basically, he was challenging Biason’s
need of a guardian who will look after her and qualifications and the procedure by which the
her business affairs. Due to her advanced age, RTC appointed him as guardian for Maura.
Maura is already sickly and can no longer However, with Biason’s demise, it has become
manage to take care of herself and her impractical and futile to proceed with resolving
properties unassisted thus becoming an easy the merits of the petition. It is a well-
prey of deceit and exploitation. established rule that the relationship of
guardian and ward is necessarily terminated
-RTC ordered and appointing Biason as Maura's
by the death of either the guardian or the
guardian. The Court hereby fixes the
ward.The supervening event of death rendered
guardianship bond at Php500,000.00 and the

Rolan Jeff A. Lancion


Arellano University School of Law Page 132
Remedial Law 2- Brondial Notes AY 2017- 2018

it pointless to delve into the propriety of Situation: Liza, 19 years of age and earning Php
Biason’s appointment since the juridical tie 200,000 per month. She wants to adopt Little
between him and Maura has already been Mario.
dissolved. The petition, regardless of its
Q: Is Liza qualified to adopt Little Mario?
disposition, will not afford Abad, or anyone else
for that matter, any substantial relief. A: NO, Legal age is not only a requirement in
the petition for Adoption. Liza is not entitled to
adopt since Liza does not have full civil capacity.
ADOPTION AND CUSTODY OF MINORS Under the Civil Code, a person shall have the
full capacity when he/she reached the age of
Jurisdiction- Family Court
21.
Venue- The residence of the adopter
-If she obtained consent from the biological
Q: What is Adoption? parents of Little Mario, Liza is entitled or has
A: Adoption is a juridical act, a proceeding in the right to adoption.
rem which creates a relationship of two person -The prospective adoptee must be one that is
similar to that of a legitimate paternity and legally available for adoption.
filiation.
Q: When is a person legally available for
Juridical Act- It cannot be a subject of a adoption?
contract since there must be a judicial order.
A: In petition for adoption, there must be an
Proceeding in Rem- It binds the whole world indication that a person is legally available for
which creates a relationship between two adoption.
persons similar to that of a legitimate paternity
Q: What are the kinds of legally available
and filiation.
persons for adoption?
*Adoption has taken a lot of changes
A: Voluntarily committed- The Biological
throughout the years.
parents renounces their right/parental
RA 8552- Domestic Adoption Act authority.
RA 8043- Inter-Country Adoption Act Involuntary committed- The government insists
AM 02-6-02- Rules on Adoption that a person should be adopted

Child- Anyone below: Kinds: -Abandoned -Neglected

18 years of age (Domestic Adoption) Child Placement Agency vs Child Caring Agency

15 years of age (Inter- Country Adoption Act)

Rolan Jeff A. Lancion


Arellano University School of Law Page 133
Remedial Law 2- Brondial Notes AY 2017- 2018

Child Placement Agency- They authorize to take Section 7. Who May Adopt. – The following
care of a child and to file an adoption may adopt:
proceeding.
(a) Any Filipino citizen of legal age, in possession
Child Caring Agency- An organization that take of full civil capacity and legal rights, of good
care of a child support but not authorize to file moral character, has not been convicted of any
crime involving moral turpitude, emotionally
adoption proceeding.
and psychologically capable of caring for
Simulation of Birth- When a party tries to children, at least sixteen (16) years older than
tamper the birth certificate towards the civil the adoptee, and who is in a position to support
and care for his/her children in keeping with the
registry to make it appear that they are the real means of the family. The requirement of sixteen
parents of a child who is his own biological (16) year difference between the age of the
adopter and adoptee may be waived when the
-Considered as Criminal Offense
adopter is the biological parent of the adoptee,
Case: In Re Petition for Adoption of Michelle or is the spouse of the adoptee's parent;
and Michael Lim
(b) Any alien possessing the same qualifications
Facts: Mr & Mrs Lim wanted to adopt Michelle as above stated for Filipino nationals: Provided,
and Michael. One day, a woman went to the That his/her country has diplomatic relations
with the Republic of the Philippines, that he/she
clinic of Mrs. Lim asking if they can adopt her 11
has been living in the Philippines for at least
day old child. Sps Lim took care of the children three (3) continuous years prior to the filing of
and even enrolled them in an exclusive school. the application for adoption and maintains such
Mrs. Lim simulated the birth certificates of residence until the adoption decree is entered,
Michelle and Michael. When Mrs. Lim filed the that he/she has been certified by his/her
diplomatic or consular office or any appropriate
petition, Michelle was already married and
government agency that he/she has the legal
Michael was of legal age. Mrs. Lim was able to capacity to adopt in his/her country, and that
get the consent of her husband, the children his/her government allows the adoptee to enter
and it was granted by the lower court. his/her country as his/her adopted
son/daughter: Provided, Further, That the
Issue: WON Mrs. Lim validly adopt Michelle and requirements on residency and certification of
Michael the alien's qualification to adopt in his/her
country may be waived for the following:
Held: NO- The Supreme Court denied the
petition on the ground that petition for (i) a former Filipino citizen who seeks to adopt a
adoption must be filed jointly by spouses even if relative within the fourth (4th) degree of
there is a prospective divorce between them. consanguinity or affinity; or

Procedure for RA 8552- Domestic Adoption Act (ii) one who seeks to adopt the legitimate
son/daughter of his/her Filipino spouse; or

Rolan Jeff A. Lancion


Arellano University School of Law Page 134
Remedial Law 2- Brondial Notes AY 2017- 2018

(iii) one who is married to a Filipino citizen and (d) A person of legal age if, prior to the
seeks to adopt jointly with his/her spouse a adoption, said person has been consistently
relative within the fourth (4th) degree of considered and treated by the adopter(s) as
consanguinity or affinity of the Filipino spouse; his/her own child since minority;
or
(e) A child whose adoption has been previously
(c) The guardian with respect to the ward after rescinded; or
the termination of the guardianship and
clearance of his/her financial accountabilities. (f) A child whose biological or adoptive
parent(s) has died: Provided, That no
Husband and wife shall jointly adopt, except in proceedings shall be initiated within six (6)
the following cases: months from the time of death of said
parent(s).
(i) if one spouse seeks to adopt the legitimate
son/daughter of the other; or Section 9. Whose Consent is Necessary to the
Adoption. – After being properly counseled and
(ii) if one spouse seeks to adopt his/her own informed of his/her right to give or withhold
illegitimate son/daughter: Provided, However, his/her approval of the adoption, the written
that the other spouse has signified his/her consent of the following to the adoption is
consent thereto; or hereby required:

(iii) if the spouses are legally separated from (a) The adoptee, if ten (10) years of age or over;
each other.
(b) The biological parent(s) of the child, if
In case husband and wife jointly adopt, or one known, or the legal guardian, or the proper
spouse adopts the illegitimate son/daughter of government instrumentality which has legal
the other, joint parental authority shall be custody of the child;
exercised by the spouses.
(c) The legitimate and adopted sons/daughters,
Section 8. Who May Be Adopted. – The ten (10) years of age or over, of the adopter(s)
following may be adopted: and adoptee, if any;

(a) Any person below eighteen (18) years of age (d) The illegitimate sons/daughters, ten (10)
who has been administratively or judicially years of age or over, of the adopter if living with
declared available for adoption; said adopter and the latter's spouse, if any; and

(b) The legitimate son/daughter of one spouse (e) The spouse, if any, of the person adopting or
by the other spouse; to be adopted.

(c) An illegitimate son/daughter by a qualified Kinds of Individual that can adopt:


adopter to improve his/her status to that of
legitimacy; -Filipino Citizen

Rolan Jeff A. Lancion


Arellano University School of Law Page 135
Remedial Law 2- Brondial Notes AY 2017- 2018

-Alien Citizen womanizer therefore Mrs. Cang wanted to


separate from him. Mr Cang left for United
-Guardian
States and later on filed a divorce within that
RA 8043- Inter Country Adoption Act of 2005 country. He left behind in the Phil. 3 minor
Qualification for alien/foreigner adopter children. The brother in law wanted to adopt
the three children of Mrs. Cang who were
-Legal Age childless. A petition for adoption was filed
-Full Civil Capability before the trial court. Mr. Cang discovered the
petition for adoption and he come to the Phil
-Qualified emotionally/psychologically/mentally
and opposed on the petition on the ground that
-Resident of the Phil for 3 consecutive years his consent was not given. Petitioner argued
that consent of Mr. Cang is not necessary on
-His country must have diplomatic ties with the
the ground that he already abandoned the
Phil
children.
-Duly certified by his own country to be eligible
Issue: WON the consent of Mr. Cang is
to adopt
necessary in the adoption proceeding
Exception: Residency Requirement can be
Held:
waived
YES- Consent of Mr. Cang is necessary. The
-Former Filipino who seeks to adopt a relative
defense of abandonment is not proper since it
within 4th degree of consanguinity.
should be total/absolute. The records will show
-Married to a Filipino who seeks to adopt a that there is no abandonment on the part of
relative within 4th degree of consanguinity or Mr. Cang of his parental authority since he
legitimate child from former marriage. regularly communicate with the children.
Consent

Q: whose consent is required in adoption *While adoption is for the best interest of
proceeding? prospective adoptee, that should not deprive
A: Adoptee must be at least 10 years of age & the biological parent of their parental authority.
Parents of the prospective adoptee, children of Case: Castro vs Gregorio 738 SCRA
the adopter (whether legitimate/illegitimate),
Facts: Atty Jose Castro of Laoag City and also a
spouses
practioner of law in Manila. He was married to
Case: Cang vs Court of Appeals, 296 SCRA 128 Mrs Castro and during their marriage they had
one child but died during the pregnancy. Atty
Facts: -Sps Cang were residents of the
Philippines, Mr Cang was found to be a Castro and Mrs Castro got separated but later

Rolan Jeff A. Lancion


Arellano University School of Law Page 136
Remedial Law 2- Brondial Notes AY 2017- 2018

on reconciled to each other and had one Ruling: The Supreme Court held that there is no
daughter named Joanne. Mrs Castro separated reason or prohibition for the adoptee to use the
again from Atty. Castro on the ground that the middle name of her former natural parent.
latter has homosexual tendencies. Atty Castro Considering that the identity of a person is not
sought to adopt the 2 illegitimate child to whom only dependent on the paternal side but also on
he had with their housemate. The petition for the maternal side.
adoption was granted by the trial court. Mrs
Case: Petition for Change of Name of Julian Lim
Castro discovered the alleged adoption after
Carulasan Wang 588 SCRA 98
the death of Atty Castro and argued that her
consent was not secured during the adoption Facts: A petition was filed by Anna Lisa Wang
proceeding. The adoption proceeding was held for the change of name and/or
in the trial court of Batac which not the correction/cancellation of entry in the Civil
residence of Atty. Castro during his lifetime.
Registry of her son, a minor, Julian Lin Carulasan
Mrs. Castro availed of annulment of judgment
Wang before the RTC of Cebu City.
on the ground of extrinsic fraud.
Julian was the son of Anna Lisa Wang and Sing-
Issue: WON the consent of Mrs. Castro is Foe Wang. They were not yet married to each
necessary in the adoption proceeding other when Julian was born. Subsequently,
Held: YES- The two children which Atty Castro when Julian’s parents got married, the latter
sought to adopt is not his illegitimate children executed a deed of legitimation of their son so
to the house maid but to Larry her common law that the child’s name was changed from Julian
spouse. It was also ruled that Atty. Castro is Lin Carulasan to Julian Lin Carulasan Wang.
allegedly the lover of Larry. The consent of Mrs.
Castro is also necessary in order for adoption to
Reason: Since the family plans to stay in
be valid.
Singapore and, since in Singapore middle names
Case: IN Re Matter of Adoption of Stephanie or the maiden surname of the mother are not
Nathy Astorga Garcia 454 SCRA carried in a persons name, they anticipated that
Facts: Stephanie was the illegitimate child of Julian will be discriminated against because of
Mr. Catindig and he wanted to adopt her. In the his current registered name which carries a
petition for adoption, the petitioner asked that middle name. Also, the spouses’ daughter and
the child will retain Garcia as her middle which Julian might get confused if they are really
is the family name of the biological mother. This brothers and sisters because they have
petition was opposed by the OSG on the ground different surnames. Lastly, Carulasan sounds
that it should be through a separate petition
funny in Singapore’s Mandarin language since
under change of name Rule 103.
they do not have the letter “R” but if there is,

Rolan Jeff A. Lancion


Arellano University School of Law Page 137
Remedial Law 2- Brondial Notes AY 2017- 2018

they pronounce it as “L”. It is for these reasons A PRIVILEGE AND NOT A RIGHT, so that before a
why the name of Julian Lin Carulasan Wang is person can be authorized to change his name
requested to be changed to Julian Lin Wang. given him either in his certificate of birth or civil
RTC: denied the petition. It found that the registry, he must show PROPER AND
reasons abovementioned does not fall within REASONABLE CAUSE, or ANY COMPELLING
the grounds recognized by law. It further ruled REASON which may justify such change.
that the real reason behind is only convenience. Otherwise, the request should be denied.
MR: Denied. The Singaporean practice of not
carrying a middle name does not justify the VALID GROUNDS FOR CHANGE OF NAME:
dropping of the middle name of a legitimate 1. When the name is ridiculous, dishonorable
Filipino child who intends to study there. The or extremely difficult to write or pronounce;
dropping of the middle name would be 2. When the change results as a legal
tantamount to giving due recognition to or consequence, as in legitimation;
application of the laws of Singapore instead of 3. When the change will avoid confusion;
Philippine law which is controlling. 4. When one has continuously used and been
Hence, this Appeal. SC required the OGS to known since childhood by a Filipino name,
comment on the petition. and was unaware of alien parentage;
5. A sincere desire to adopt a Filipino name to
OSG: Trial Court is correct. legitimate children erase signs of former alienage, all in good
have the right to bear the surnames of both faith and without prejudicing anybody; and
their mother and father, and such right cannot 6. When the surname causes embarrassment
be denied by the mere expedient of dropping and there is no showing that the desired
the same (Family Code). Mere convenience is change of name was for a fraudulent
not sufficient to support a petition for change of purpose or that the change of name would
name and/or cancellation of entry. prejudice public interest.
Issue: Whether the name mother’s surname IN GRANTING/DENYING:
should be dropped in the instant case because The question of proper and reasonable cause is
it is a common practice in Singapore to omit left to the sound discretion of the court. The
said surname? evidence presented need only be satisfactory to
Decision: No. Petition is denied. the court and not all the best evidence
The State has an interest in the names borne by available.
individuals and entities for purposes of
identification, and that A CHANGE OF NAME IS

Rolan Jeff A. Lancion


Arellano University School of Law Page 138
Remedial Law 2- Brondial Notes AY 2017- 2018

What is involved is not a mere matter of Held: The Supreme Court ruled that after
allowance or disallowance of the request, but a adoption, the adopter died. Upon his death, the
JUDICIOUS evaluation of the sufficiency and biological mother took care of the deceased
propriety of the justifications advanced in son. The parental authority of the biological
support thereof, mindful of the consequent parents has returned upon the death of the
results in the event of its grant and with the adopter therefore she is not entitled to the
sole prerogative for making such determination benefits of SSS.
being lodged in the courts.
Inter-Country Adoption Law

Rescission of Adoption- The adopter can no -It can be availed of Filipino Citizens living
longer rescind adoption and the right to rescind abroad or those permanently residing abroad.
shall belong to the adoptee
-A party can file a petition for adoption in the
Q: What is the effect of the Family court in Phil or Inter-Country Adoption
rescission/termination of adoption? Board.

A: The adoptee shall go back to his biological Q: Can a foreigner avail of domestic adoption
parents along with all the legal ties that were act?
severed.

A: YES, the law does not prohibit a foreigner to


Case: Bartolome vs SSS 740 SCRA avail of domestic adoption act.

Facts: A certain seaman that is earning well Exceptions: -Seeks to adopt a relative within 4th
meet an accident that caused his death. The civil degree
biological mother filed an application with the
Compensation Act as beneficiary of his -Seeks to adopt her own illegitimate child
deceased son. This application was disapproved
by SSS on the ground that during the childhood -Seeks to adopt the legitimate child of the
of the deceased, he was adopted by his spouse
grandfather. The grandfather already died
therefore the adoption relationship was already Cases:
terminated.

Rolan Jeff A. Lancion


Arellano University School of Law Page 139
Remedial Law 2- Brondial Notes AY 2017- 2018

Case: Vda. de Jacob vs Court of Appeals- 312 Pedro Pilapil as the legally adopted son of
Alfredo.
SCRA 772
Issue: WON Pedro Pilapil is legally adopted
son of Alfredo Jacob
-Petitioner claimed to be the surviving spouse
of deceased Dr. Alfredo E. Jacob and was Held:
appointed Special Administratix for the various NO- The factual findings of the trial court
estates of the deceased by virtue of are accorded great weight and respect by
appellate courts, because it had the
a reconstructed Marriage Contract between opportunity to observe the demeanor of
herself and the deceased. witnesses and to note telltale signs
indicating the truth or the falsity of a
testimony. The rule, however, is not
-During the proceeding for the settlement of
applicable to the present case, because it
the estate of the deceased Alfredo in Case No. was Judge Augusto O. Cledera, not
T-46 (entitled Tomasa vda. de Jacob v. Jose the ponente, who heard the testimonies of
Centenera, et al) herein defendant-appellee the two expert witnesses.
Pedro sought to intervene therein claiming his -No proof was presented that Dr. Jacob had
share of the deceaseds estate as Alfredos treated him as an adopted child. Likewise, both
adopted son and as his sole surviving the Bureau of Records Management in Manila
and the Office of the Local Civil Registrar of
heir. Pedro questioned the validity of the
Tigaon, Camarines Sur, issued Certifications
marriage between appellant Tomasa and his
that there was no record that Pedro Pilapil had
adoptive father Alfredo. been adopted by Dr. Jacob. Taken together,
these circumstances inexorably negate the
- The trial court found some irregularities alleged adoption of respondent.
in the execution of the Marriage Contract.
(a) No copy of the Marriage Contract was -The burden of proof in establishing adoption
sent to the local civil registrar by the is upon the person claiming such relationship.
solemnizing officer thus giving the
implication that there was no copy of the Case: Republic vs Hon. Jose R. Hernandez,
253 SCRA 509
marriage contract sent to, nor a record
existing in the civil registry of Manila, (b) In
signing the Marriage Contract, the late
Alfredo Jacob merely placed his Facts:
thumbmark on said contract. -On March 10, 1994, herein private
-Appellee presented the Order dated 18 respondent spouses, Van Munson y
July 1961 in Special Proceedings No. 192 Navarro and Regina Munson y Andrade,
issued by then Presiding Judge Moya filed a petition to adopt the minor Kevin
granting the petition for adoption filed by Earl Bartolome Moran.
deceased Alfredo which declared therein

Rolan Jeff A. Lancion


Arellano University School of Law Page 140
Remedial Law 2- Brondial Notes AY 2017- 2018

-At the hearing on April 18, 1994, - The law allows the adoptee, as a matter of
petitioner opposed the inclusion of the right and obligation, to bear the surname of
relief for change of name in the same the adopter, upon issuance of the decree of
petition for adoption. In its formal adoption. It is the change of the
opposition dated May 3, 1995, petitioner adoptees surname to follow that of the
reiterated its objection to the joinder of adopter which is the natural and necessary
the petition for adoption and the petitions consequence of a grant of adoption and must
for change of name in a single proceeding, specifically be contained in the order of the
arguing that these petitions should be court, in fact, even if not prayed for by
conducted and pursued as two separate petitioner.
proceedings.
However, the given or proper name, also known
-The trial court granted the petition for
as the first or Christian name, of the adoptee
adoption filed by herein petitioners.
must remain as it was originally registered in
the civil register. The creation of an adoptive
relationship does not confer upon the adopter a
Issue: WON the court a quo erred in
license to change the adoptees registered
granting the prayer for the change of the
Christian or first name. The automatic change
registered proper or given name of the
thereof, premised solely upon the adoption
minor adoptee embodied in the petition
thus granted, is beyond the purview of a decree
for adoption
of adoption. Neither is it a mere incident in nor
Held: an adjunct of an adoption proceeding, such that
a prayer therefor furtively inserted in a petition
Art. 189 of the Family Code enumerate in no for adoption, as in this case, cannot properly be
uncertain terms the legal effects of adoption: granted.

(1) For civil purposes, the adopted shall be -The name of the adoptee as recorded in the
deemed to be a legitimate child of the adopters civil register should be used in the adoption
and both shall acquire the reciprocal rights and proceedings in order to vest the court with
obligations arising from the relationship of jurisdiction to hear and determine the
parent and child, including the right of the same and shall continue to be so used until the
adopted to use the surname of the adopters; court orders otherwise. Changing the given or
proper name of a person as recorded in the civil
(2) The parental authority of the parents by register is a substantial change in one’s official
nature over the adopted shall terminate and be or legal name and cannot be authorized without
vested in the adopters, except that if the a judicial order.
adopter is the spouse of the parent by nature of - Petition for adoption and a petition for change
the adopted, parental authority over the of name are two special proceedings which, in
adopted shall be exercised jointly by both substance and purpose, are different from each
spouses; and other. Each action is individually governed by
particular sets of laws and rules. These two
(3) The adopted shall remain an intestate heir proceedings involve disparate issues. In a
of his parents and other blood relatives. petition for adoption, the court is called upon to

Rolan Jeff A. Lancion


Arellano University School of Law Page 141
Remedial Law 2- Brondial Notes AY 2017- 2018

evaluate the proposed adopters fitness and Held:


qualifications to bring up and educate the
NO -Under the Domestic Adoption Act
adoptee properly. On the other hand, in a
provision, which Sampana suggested, the alien
petition for change of name, no family relations
adopter can jointly adopt a relative within the
are created or affected for what is looked into is
fourth degree of consanguinity or affinity of
the propriety and reasonableness of the
his/her Filipino spouse, and the certification of
grounds supporting the proposed change of
the alien’s qualification to adopt is waived.
name. The individual merits of each issue must
be separately assessed and determined for -Having no valid reason not to file the petition
neither action is dependent on the other. for adoption, Sampana misinformed Nery of the
status of the petition.

Case: Nery vs Sampana, 734 SCRA


Rule 102- Habeas Corpus
-On 14 February 2009, Sampana sent a text
message informing Nery that he already filed
Section 1. To what habeas corpus extends. —
the petition for adoption and it was already
Except as otherwise expressly provided by law,
published. Sampana further informed Nery that
they needed to rehearse before the hearing. the writ of habeas corpus shall extend to all
cases of illegal confinement or detention by
Subsequently, Sampana told Nery that the
which any person is deprived of his liberty, or
hearing was set on 5 March 2010 in Branch 11
of Malolos, Bulacan. by which the rightful custody of any person is
withheld from the person entitled thereto.
-On 11 March 2010, Nery inquired from Branch
11 of Malolos, Bulacan about the status of the Section 3. Requisites of application therefor. —
petition for adoption and discovered that there Application for the writ shall be by petition
was no such petition filed in the court. Thus, in signed and verified either by the party for
the afternoon of the same day, Nery met whose relief it is intended, or by some person
Sampana and sought the reimbursement of the on his behalf, and shall set forth:
₱100,000.00 she paid him. Sampana agreed,
but said that he would deduct the filing fee (a) That the person in whose behalf the
worth ₱12,000.00. application is made is imprisoned or restrained
-Commissioner Antiquiera found Sampana on his liberty;
guilty of malpractice for making Nery believe
that he already filed the petition for adoption (b) The officer or name of the person by whom
and for failing to file the petition despite he is so imprisoned or restrained; or, if both are
receiving his legal fees. Thus, Commissioner unknown or uncertain, such officer or person
Antiquiera recommended a penalty of three (3) may be described by an assumed appellation,
months suspension from the practice of law. and the person who is served with the writ shall
be deemed the person intended;
Issue: WON there was a valid adoption
proceeding (c) The place where he is so imprisoned or
restrained, if known;

Rolan Jeff A. Lancion


Arellano University School of Law Page 142
Remedial Law 2- Brondial Notes AY 2017- 2018

(d) A copy of the commitment or cause of Section 10. Contents of return. — When the
detention of such person, if it can be procured person to be produced is imprisoned or
without impairing the efficiency of the remedy; restrained by an officer, the person who makes
or, if the imprisonment or restraint is without the return shall state therein, and in other cases
any legal authority, such fact shall appear. the person in whose custody the prisoner is
found shall state, in writing to the court or
Section 4. When writ not allowed or discharge judge before whom the writ is returnable,
authorized. — If it appears that the person plainly and unequivocally:
alleged to be restrained of his liberty is in the
custody of an officer under process issued by a (a) Whether he has or has not the party in his
court or judge or by virtue of a judgment or custody or power, or under restraint;
order of a court of record, and that the court
or judge had jurisdiction to issue the process, (b) If he has the party in his custody or power,
render the judgment, or make the order, the or under restraint, the authority and the true
writ shall not be allowed; or if the jurisdiction and whole cause thereof, set forth at large, with
appears after the writ is allowed, the person a copy of the writ, order execution, or other
shall not be discharged by reason of any process, if any, upon which the party is held;
informality or defect in the process, judgment,
or order. Not shall anything in this rule be held (c) If the party is in his custody or power or is
to authorize the discharge of a person charged restrained by him, and is not produced,
with or convicted of an offense in the particularly the nature and gravity of the
Philippines, or of a person suffering sickness or infirmity of such party by reason of
imprisonment under lawful judgment. which he cannot, without danger, be bought
before the court or judge;

(d) If he has had the party in his custody or


Section 6. To whom writ directed, and what to power, or under restraint, and has transferred
require. — In case of imprisonment or restraint such custody or restraint to another,
by an officer, the writ shall be directed to him, particularly to whom, at what time, for what
and shall command him to have the body of cause, and by what authority such transfer was
the person restrained of his liberty before the made.
court or judge designated in the writ at the
time and place therein specified. In case of Section 12. Hearing on return. Adjournments.
imprisonment or restraint by a person not an — When the writ is returned before one judge,
officer, the writ shall be directed to an officer, at a time when the court is in session, he may
and shall command him to take and have the forthwith adjourn the case into the court, there
body of the person restrained of his liberty to be heard and determined. The court or judge
before the court or judge designated in the before whom the writ is returned or adjourned
writ at the time and place therein specified, must immediately proceed to hear and examine
and to summon the person by whom he is the return, and such other matters as are
restrained then and there to appear before said properly submitted for consideration, unless for
court or judge to show the cause of the good cause shown the hearing is adjourned, in
imprisonment or restraint. which event the court or judge shall make such

Rolan Jeff A. Lancion


Arellano University School of Law Page 143
Remedial Law 2- Brondial Notes AY 2017- 2018

order for the safekeeping of the person Country Club when he was in Baguio City. On
imprisoned or restrained as the nature of the the other hand, Erlinda lived in Antipolo City.
case requires. If the person imprisoned or
restrained is not produced because of his -On February 25, 1998, Erlinda filed with the
alleged sickness or infirmity, the court or judge Regional Trial Court, Antipolo City a petition for
must be satisfied that it is so grave that such guardianship over the person and property of
person cannot be produced without danger, Potenciano Ilusorio due to the latters advanced
before proceeding to hear and dispose of the age, frail health, poor eyesight and impaired
matter. On the hearing the court or judge shall judgment.
disregard matters of form and technicalities in
respect to any warrant or order of commitment -Erlinda filed with the Court of Appeals a
of a court or officer authorized to commit by petition for habeas corpus to have the custody
law. of lawyer Potenciano Ilusorio. She alleged that
respondents refused petitioners demands to
Section 13. When the return evidence, and see and visit her husband and prohibited
when only a plea. — If it appears that the Potenciano from returning to Antipolo City but
prisoner is in custody under a warrant of it was denied by the court of appeals.
commitment in pursuance of law, the return
shall be considered prima facie evidence of the Issue: WON Habeas Corpus shall prosper in this
cause of restraint, but if he is restrained of his case
liberty by any alleged private authority, the
return shall be considered only as a plea of the Held:
facts therein set forth, and the party claiming
the custody must prove such facts. NO- Writ of habeas corpus extends to all cases
of illegal confinement or detention, or by
Cases: Ilusorio vs Bildner, 512 SCRA 169 which the rightful custody of a person is
withheld from the one entitled thereto. It is
Facts: available where a person continues to be
unlawfully denied of one or more of his
- Potenciano Ilusorio is about 86 years of age constitutional freedoms, where there is denial
possessed of extensive property valued at of due process, where the restraints are not
millions of pesos. For many years, lawyer merely involuntary but are unnecessary, and
Potenciano Ilusorio was Chairman of the Board where a deprivation of freedom originally valid
has later become arbitrary. It is devised as a
and President of Baguio Country Club.
speedy and effectual remedy to relieve persons
from unlawful restraint, as the best and only
-Erlinda Kalaw and Potenciano Ilusorio sufficient defense of personal freedom.
contracted matrimony and lived together for a
period of thirty (30) years. In 1972, they - The essential object and purpose of the writ
separated from bed and board for undisclosed of habeas corpus is to inquire into all manner of
reasons. Potenciano lived at Urdaneta involuntary restraint, and to relieve a person
Condominium, Ayala Ave., Makati City when he therefrom if such restraint is illegal.
was in Manila and at Ilusorio Penthouse, Baguio

Rolan Jeff A. Lancion


Arellano University School of Law Page 144
Remedial Law 2- Brondial Notes AY 2017- 2018

-The evidence shows that there was no actual injunction, mandamus, and habeas corpus (with
and effective detention or deprivation of lawyer an urgent application for the issuance of
Potenciano Ilusorios liberty that would justify temporary restraining order and/or writ of
the issuance of the writ. The fact that lawyer preliminary injunction) filed by Panfilo M.
Potenciano Ilusorio is about 86 years of age, or Lacson, Michael Ray B. Aquino, and Cezar O.
under medication does not necessarily render Mancao; (2) G.R. No. 147781
him mentally incapacitated. Soundness of mind for mandamus and/or review of the factual
does not hinge on age or medical condition but basis for the suspension of the privilege of the
on the capacity of the individual to discern his writ of habeas corpus, with prayer for a
actions. temporary restraining order filed by Miriam
Defensor-Santiago; (3) G.R. No. 147799 for
-Being of sound mind, he is thus possessed with prohibition and injunction with prayer for a writ
the capacity to make choices. In this case, the of preliminary injunction and/or restraining
crucial choices revolve on his residence and the order filed by Rolando A. Lumbao; and (4) G.R.
people he opts to see or live with. The choices No. 147810 for certiorari and prohibition filed
he made may not appeal to some of his family by the political party Laban ng Demokratikong
members but these are choices which Pilipino.”
exclusively belong to Potenciano. He made it
clear before the Court of Appeals that he was Issue: WON Habeas Corpus shall prosper in this
not prevented from leaving his house or seeing case
people.
Held:
Case: Lacson vs Perez, 357 SCRA
NO- The rule requires that a party must show a
Facts: personal stake in the outcome of the case or an
injury to himself that can be redressed by a
-On May 1, 2001, President Macapagal-Arroyo, favorable decision so as to warrant an
faced by an angry and violent mob armed with invocation of the court’s jurisdiction and to
explosives, firearms, bladed weapons, clubs, justify the exercise of the courts remedial
stones and other deadly weapons assaulting powers in his behalf.
and attempting to break into Malacaang, issued
Proclamation No. 38 declaring that there was a -At best, the instant petition may be considered
state of rebellion in the National Capital as an action for declaratory relief, petitioner
Region. She likewise issued General Order No. 1 claiming that its right to freedom of expression
directing the Armed Forces of the Philippines and freedom of assembly is affected by the
and the Philippine National Police to suppress declaration of a state of rebellion and that said
the rebellion in the National Capital proclamation is invalid for being contrary to the
Region. Warrantless arrests of several alleged Constitution.
leaders and promoters of the rebellion were
thereafter effected. Case: Sangca vs City Prosecutor of Cebu, 524
SCRA 610
-There are 4 petitions filed before the Supreme
Court:” (1) G.R. No. 147780 for prohibition, Facts:

Rolan Jeff A. Lancion


Arellano University School of Law Page 145
Remedial Law 2- Brondial Notes AY 2017- 2018

-On July 7, 2006, at about 2:00 P.M., Yap and Issue: WON Habeas Corpus is the proper
Tuliao were able to contact the informant and remedy is already moot
inquired from him if he was really sincere with
his words and the latter replied Held:
affirmatively.Ligan immediately composed a
team and planned for an entrapment operation YES- Writ of habeas corpus extends to all cases
against respondent and her cohorts. of illegal confinement or detention in which
any person is deprived of his liberty, or in
- Yap told her no problem as long as she has the which the rightful custody of any person is
item, he will give her the money. Respondent withheld from the person entitled to it. Its
instructed Yap to go with her at the parking essential object and purpose is to inquire into
area so that she could give it to him and there, all manner of involuntary restraint and to
she got inside her car. She took the shabu inside relieve a person from it if such restraint is
the compartment of her Toyota Fortuner with illegal. The singular function of a petition for
plate number YCX 965 and handed to him one habeas corpus is to protect and secure the basic
(1) packed medium size of heat sealed freedom of physical liberty.
transparent plastic sachet filled with white
crystalline substance believed to be -In the instant case, records show that Adam
shabu. Upon receiving the said has been released upon order of the trial judge
item, Yap pressed it to determine if it was really on January 26, 2007. Therefore, the petition has
shabu or not and when he noticed that it was become moot.
shabu, he immediately miscalled the members
of the team informing them that the transaction Case: TUJANMILITANTE IN BEHALF OF THE
was consummated and subsequently held MINOR CRISELDA M. CADA vs CADA-DEAPARA,
respondent. He then introduced himself as G.R. No. 210636, July 28, 2014
PDEA 7 operative. Tuliao, who was just at the
side of the car, assisted Yap in apprehending Facts:
the suspect. They also seized her cellular phone
and the Toyota Fortuner which she used in -On March 24, 2011, respondent Raquel M.
delivering and transporting illegal drugs. Cada-Deapera filed before the RTC-Caloocan a
verified petition for writ of habeas corpus. In
-The inquest prosecutor recommended the the said petition, respondent demanded the
dismissal of the case but was disapproved by immediate issuance of the special writ,
the City Prosecutor. Consequently, an directing petitioner Ma. Hazelina Tujan-
information charging Adam with violation of Militante to produce before the court
Section 5, Article 2 of R.A. No. 9165 was filed. respondent's biological daughter, minor
Criselda M. Cada (Criselda), and to return to her
-On petition for review before the Department the custody over the child.
of Justice, Secretary Raul M. Gonzalez found no
probable cause to hold Adam liable for the - The next day, on March 25, 2011, the RTC-
offense charged. Caloocan issued a writ of habeas corpus,
ordering petitioner to bring the child to court
on March 28, 2011. Despite diligent efforts and

Rolan Jeff A. Lancion


Arellano University School of Law Page 146
Remedial Law 2- Brondial Notes AY 2017- 2018

several attempts, however, the Sheriff was -The petition may likewise be filed with the
unsuccessful in personally serving petitioner Supreme Court, Court of Appeals, or with any
copies of the habeas corpus petition and of the of its members and, if so granted, the writ shall
writ. Instead, on March 29, 2011, the Sheriff left be enforceable anywhere in the Philippines.
copies of the court processes at petitioner’s
The writ may be made returnable to a Family
Caloocan residence, as witnessed by
respondent’s counsel and barangay officials. Court or to any regular court within the region
where the petitioner resides or where the
-Petitioner filed a Petition for Guardianship over minor may be found for hearing and decision
the person of Criselda before the RTC, Branch on the merits. Upon return of the writ, the
89 in Quezon City (RTC-Quezon City). court shall decide the issue on custody of
Respondent filed a Motion to Dismiss the
minors. The appellate court, or the member
petition for guardianship on the ground of litis
thereof, issuing the writ shall be furnished a
pendentia, among others.
copy of the decision.
- Raquel moved for the ex parte issuance of an
alias writ of habeas corpus before the RTC- -In view of the afore-quoted provision, it is
Caloocan, which was granted by the trial court indubitable that the filing of a petition for the
on August 8, 2011 but it was denied by the RTC issuance of a writ of habeas corpus before a
and it was affirmed by the Court of Appeals. family court in any of the cities enumerated is
proper as long as the writ is sought to be
Issue: WON RTC has jurisdiction over the
habeas corpus case enforced within the National Capital Judicial
Region.
Held:
Rule 103- Change of Name
Section 20. Petition for writ of habeas corpus.-
A verified petition for a writ of habeas corpus Rule 108- Cancellation/Correction of Entries in
involving custody of minors shall be filed with the Civil Registry
the Family Court. The writ shall been
enforceable within its judicial region to which RA 9048- AN ACT AUTHORIZING THE CITY OR
the Family Court belongs. However, the MUNICIPAL CIVIL REGISTRAR OR THE CONSUL
petition may be filed with the regular court in GENERAL TO CORRECT A CLERICAL OR
the absence of the presiding judge of the Family TYPOGRAPHICAL ERROR IN AN ENTRY AND/OR
Court, provided, however, that the regular CHANGE OF FIRST NAME OR NICKNAME IN THE
court shall refer the case to the Family Court as CIVIL REGISTER WITHOUT NEED OF A JUDICIAL
soon as its presiding judge returns to duty. ORDER, AMENDING FOR THIS PURPOSE
ARTICLES 376 AND 412 OF THE CIVIL CODE OF
-The petition may also be filed with the THE PHILIPPINES
appropriate regular courts in places where
there are no Family Courts. Example: In one compound, there are 2 persons
named as Juan de la Cruz. The difference that
they have is the middle name. (Juan Reyes Dela

Rolan Jeff A. Lancion


Arellano University School of Law Page 147
Remedial Law 2- Brondial Notes AY 2017- 2018

Cruz/Juan Robles Dela Cruz). Whenever Juan Q: What is the jurisdiction under Rule 108?
Reyes Dela-Cruz receives a letter or packages, it
is intended for another. In order avoid A: RTC
confusion, Juan Reyes dela cruz filed a petion
for change of name Q: What is the venue in this petition?

-The only record that should be changed is the A: The location of the civil registrar keeping the
name in the Birth Certificate. records of the subject matter a person want to
correct/cancel an entry
Rule 103- Change of Name
Q: What are the entries that is the subject of
Q: What is the Jurisdiction? cancellation or correction of entries?

A: RTC A: Under Sec 2, (a) births: (b) marriage; (c)


deaths; (d) legal separations; (e) judgments of
Q: What is the venue of this action? annulments of marriage; (f) judgments
declaring marriages void from the beginning; (g)
A: It is where the petitioner resides
legitimations; (h) adoptions; (i)
Q: What are the contents of the petition under acknowledgments of natural children; (j)
Rule 103? naturalization; (k) election, loss or recovery of
citizenship; (l) civil interdiction; (m) judicial
A: (a) That the petitioner has been a bona determination of filiation; (n) voluntary
fide resident of the province where the petition emancipation of a minor; and (o) changes of
is filed for at least three (3) years prior to the name.
date of such filing;
Q: Who are the parties to the petition?
(b) The cause for which the change of the
petitioner's name is sought; A: Petitioner, Civil Registrar and any person
interested of be affected
(c) The name asked for.
Two Procedures:
Q: How the court acquire jurisdiction over the
petition?
Adversial Procedure- Substantial error
A: Upon publication of the notice of at least Summary Procedure- Typographical
once a week for three (3) consecutive weeks in error/clerical error
a newspaper of general circulation.
Case Doctrine: Eleosida vs Civil Registrar of QC-
Rule 108- Cancellation/Correction of Entries in
“ If a Party wants to correct substantial/clerical
the Civil Registry error, the remedy is Rule 108. If the cause of

Rolan Jeff A. Lancion


Arellano University School of Law Page 148
Remedial Law 2- Brondial Notes AY 2017- 2018

action is adversial, it shall be summary in Q: What are the grounds?


nature.”
A: (1) The petitioner finds the first name or
-RA 9048 did not repeal Rule 103 and 108 but nickname to be ridiculous, tainted with
only amendatory. dishonor or extremely difficult to write or
pronounce. (2) The new first name or nickname
Case Doctrine: Onde vs Civil Registrar of Las
has been habitually and continuously used by
Pinas- “If a party sought for amendment of
typographical error in their record, the remedy the petitioner and he has been publicly known
is RA 9048” by that by that first name or nickname in the
community: or (3) The change will avoid
Q: What is a typographical/clerical error? confusion.

A: An error which is harmless, visible to the Section 6. Duties of the City or Municipal Civil
eyes and of understanding. Registrar or the Consul General. – The city or
municipal civil registrar or the consul general to
Example: Matio Reyes---- Matio Reyes
whom the petition is presented shall examine
Huanito Santos----- Juanito Santos the petition and its supporting documents. He
shall post the petition in a conspicuous place
July 2, 1890-------- July 2, 1990 provided for that purpose for ten (10)
consecutive days after he finds the petition and
*No Correction of Error but change of name as its supporting documents sufficient in form and
a remedy.
substance.
-If in the Birth Certificate, the sex of a party is
Q: Did RA 9048 repealed Rule 103 and 108?
not indicated. The remedy is not correction of
entries but late registration.
A: No, it only amended the rule pursuant to
case of Onde.

RA 9048- Change of Name -It only pertains to correction but not


cancellation. It sought to correct clerical/
Section 3. Who May File the Petition and typographical error.
Where. – Any person having direct and personal
interest in the correction of a clerical or
-The Judicial proceeding is adversial and the
typographical error in an entry and/or change
of first name or nickname in the civil register party must comply with the notices.
may file, in person, a verified petition with the
local civil registry office of the city or Procedure:
municipality where the record being sought to
be corrected or changed is kept.

Rolan Jeff A. Lancion


Arellano University School of Law Page 149
Remedial Law 2- Brondial Notes AY 2017- 2018

-File a petition/application with the office of the *If a party is a resident of Manila and the entry
civil registrar where the records are kept. sought to be cancelled/corrected is in Davao.
He/she can file his petition in Civil Registrar of
-Posting of Notice in the place of the petition Manila and not in Davao.
for 10 consecutive days.
-Posting of Notice should be in Manila and
– The city or municipal civil registrar or the Davao.
consul general to whom the petition is
presented shall examine the petition and its Q: What is the remedy if the Civil Registrar
supporting documents. He shall post the denies the petition?
petition in a conspicuous place provided for
that purpose for ten (10) consecutive days after A: A party can file an appeal towards the Civil
he finds the petition and its supporting Registrar General. If the Civil Registrar denied
documents sufficient in form and substance. the petition, a party can proceed with judicial
remedy under Rule 103/108.
- The city or municipal civil registrar or the
consul general shall act on the petition and shall Case: Eleosida vs Civil Registrar, G.R. No.
render a decision not later than five (5) working 130277. May 9, 2002
days after the completion of the posting and/or
publication requirement. Facts:

The civil registrar general shall, within ten (10) -On January 30, 1997, petitioner Ma. Lourdes
working days from receipt of the decision Eleosida filed a petition before the Regional
granting a petition, exercise the power to Trial Court of Quezon City seeking to correct the
impugn such decision by way of an objection following entries in the birth certificate of her
based on the following grounds: (1) The error is son, Charles Christian: first, the surname
not clerical or typographical; (2) The correction
"Borbon" should be changed to "Eleosida;"
of an entry or entries in the civil register is
substantial or controversial as it affects the civil second, the date of the parents' wedding
status of a person; or (3) The basis used in should be left blank; and third, the informant's
changing the first name or nickname of a name should be "Ma. Lourdes B. Eleosida,"
person does not fall under Section 4. instead of "Ma. Lourdes E. Borbon." In support
of her petition, petitioner alleged that she gave
- Where the petition is denied by the city or birth to her son out of wedlock on May 24,
municipal civil registrar or the consul general, 1992; that she and the boy's father, Carlos
the petitioner may either appeal the decision to Borbon, were never married; and that the child
the civil registrar general or file the appropriate is therefore illegitimate and should follow the
petition with the proper court. mother's surname.

Rolan Jeff A. Lancion


Arellano University School of Law Page 150
Remedial Law 2- Brondial Notes AY 2017- 2018

-The trial court dismissed the petition for lack of where the trial court has conducted
merit. proceedings where all relevant facts have been
fully and properly developed, where opposing
Issue: WON corrections of entries in the counsel have been given opportunity to
certificate of live birth pursuant to Article 412 of demolish the opposite party's case, and where
the Civil Code, in relation to Rule 108 of the the evidence has been thoroughly weighed and
Rules of Court may be allowed even if the errors considered.
to be corrected are substantial and not merely
clerical errors of a harmless and innocuous -If all these procedural requirements have been
nature. followed, a petition for correction and/or
cancellation of entries in the record of birth
Held: even if filed and conducted under Rule 108 of
the Revised Rules of Court can no longer be
YES- Rule 108 of the Revised Rules of Court described as 'summary.'
provides the procedure for cancellation or
correction of entries in the civil registry. The Case: Republic vs Kho, G.R. No. 170340, June
proceedings under said rule may either be 29, 2007
summary or adversary in nature. If the
correction sought to be made in the civil Facts:
register is clerical, then the procedure to be
adopted is summary. If the rectification affects -On February 12, 2001, Carlito and his siblings
the civil status, citizenship or nationality of a Michael, Mercy Nona and Heddy Moira filed
party, it is deemed substantial, and the before the RTC of Butuan City a verified petition
procedure to be adopted is adversary. for correction of entries in the civil registry
of Butuan City to effect changes in their
-The persons who must be made parties to a respective birth certificates. Carlito also asked
proceeding concerning the cancellation or the court in behalf of his minor children, Kevin
correction of an entry in the civil register are-- and Kelly, to order the correction of some
(1) the civil registrar, and (2) all persons who entries in their birth certificates.
have or claim any interest which would be
affected thereby. -Carlito requested the correction in his birth
certificate of the citizenship of his mother to
-Even substantial errors in a civil registry may Filipino instead of Chinese, as well as the
be corrected and the true facts established deletion of the word married opposite the
under Rule 108 provided the parties aggrieved phrase Date of marriage of parents because his
by the error avail themselves of the parents, Juan Kho and Epifania Inchoco
appropriate adversary proceeding. An (Epifania), were allegedly not legally married.
appropriate adversary suit or proceeding is one

Rolan Jeff A. Lancion


Arellano University School of Law Page 151
Remedial Law 2- Brondial Notes AY 2017- 2018

-Carlitos filed an amended petition that his -A petition for correction is an action in rem,
second name be deleted from his record of an action against a thing and not against a
birth and that the citizenship be corrected from person. The decision on the petition binds not
Filipino to Chinese. only the parties thereto but the whole world.
An in rem proceeding is validated essentially
-The trial court directed the local civil registrar through publication. Publication is notice to the
of Butuan City to correct the entries in the whole world that the proceeding has for its
record of birth of Carlito, as follows: (1) change object to bar indefinitely all who might be
the citizenship of his mother from Chinese to minded to make an objection of any sort
Filipino; (2) delete John from his name; and (3) against the right sought to be established. It is
delete the word married opposite the date of the publication of such notice that brings in the
marriage of his parents. The last correction was whole world as a party in the case and vests the
ordered to be effected likewise in the birth court with jurisdiction to hear and decide it.
certificates of respondents Michael, Mercy
Nona, and Heddy Moira. -The correction of the mother’s citizenship from
Chinese to Filipino as appearing in Carlitos birth
Issue: WON the case involve is substantial in record was also proper. Of note is the fact that
nature during the cross examination by the city
prosecutor of Epifania, he did not deem fit to
Held: question her citizenship. Such failure to oppose
the correction prayed for, which certainly was
If the entries in the civil register could be not respondents fault, does not in any way
corrected or changed through mere summary change the adversarial nature of the
proceedings and not through appropriate proceedings.
action wherein all parties who may be affected
by the entries are notified or represented, the Case: Republic vs Silverio, G.R. No. 174689
door to fraud or other mischief would be set October 22, 2007
open, the consequence of which might be
detrimental and far reaching. Facts:

-the obvious effect of Republic Act No. 9048 is -Petitioner Rommel Jacinto Dantes Silverio filed
to make possible the administrative correction a petition for the change of his first name and
of clerical or typographical errors or change of sex in his birth certificate in the Regional Trial
first name or nickname in entries in the civil Court of Manila, Branch 8.
register, leaving to Rule 108 the correction of
substantial changes in the civil registry in -He further alleged that he is a male
appropriate adversarial proceedings. transsexual, that is, "anatomically male but
feels, thinks and acts as a female" and that he

Rolan Jeff A. Lancion


Arellano University School of Law Page 152
Remedial Law 2- Brondial Notes AY 2017- 2018

had always identified himself with girls since be prejudiced by the use of his true and official
childhood. Petitioner lived as a female and was name. In this case, he failed to show, or even
in fact engaged to be married. He then sought allege, any prejudice that he might suffer as a
to have his name in his birth certificate changed result of using his true and official name.
from "Rommel Jacinto" to "Mely," and his sex
from "male" to "female." -Under RA 9048, a correction in the civil
registry involving the change of sex is not a
-The trial court rendered a decision in favor of mere clerical or typographical error. It is a
the petitioner granting the change of name and substantial change for which the applicable
sex. OSG, filed a petition for certiorari in the procedure is Rule 108 of the Rules of Court.
Court of Appeals. It alleged that there is no law
allowing the change of entries in the birth -There is no special law in the Philippines
certificate by reason of sex alteration. governing sex reassignment and its effects.

-Court of Appeals reversed the decision of the Case: Republic vs Cagandahan, G.R. No.
trial court since there is no law allowing the 166676, September 12, 2008
change of either name or sex in the certificate Facts:
of birth on the ground of sex reassignment
through surgery. -Jennifer Cagandahan as born on January 13,
1981 and was registered as a female in the
Issue: WON the petitioner has a legal ground to Certificate of Live Birth but while growing up,
change its name and sex in the birth certificate. she developed secondary male characteristics
and was diagnosed to have Congenital Adrenal
Held: Hyperplasia (CAH) which is a condition where
persons thus afflicted possess both male and
NO- A change of name does not alter one’s female characteristics. She further alleged that
legal capacity or civil status. RA 9048 does not she was diagnosed to have clitoral hyperthropy
sanction a change of first name on the ground in her early years and at age six, underwent an
of sex reassignment. Rather than avoiding ultrasound where it was discovered that she has
confusion, changing petitioner’s first name for small ovaries.
his declared purpose may only create grave
complications in the civil registry and the -t age thirteen, tests revealed that her ovarian
public interest. structures had minimized, she has stopped
growing and she has no breast or menstrual
-Before a person can legally change his given development. She then alleged that for all
name, he must present proper or reasonable interests and appearances as well as in mind
cause or any compelling reason justifying such and emotion, she has become a male
change. In addition, he must show that he will person. Thus, she prayed that her birth

Rolan Jeff A. Lancion


Arellano University School of Law Page 153
Remedial Law 2- Brondial Notes AY 2017- 2018

certificate be corrected such that her gender be with. And accordingly, he has already ordered
changed from female to male and her first his life to that of a male. Respondent could have
name be changed from Jennifer to Jeff. undergone treatment and taken steps, like
taking lifelong medication, to force his body
-RTC granted the petition filed by Cagandahan into the categorical mold of a female but he did
on the ground that he is suffering from CAH. not. He chose not to do so. Nature has instead
taken its due course in respondent’s
Issue: WON the petition for correction of entry development to reveal more fully his male
for sex should be granted characteristics.

Held:

YES- Under Rep. Act No. 9048, a correction in Case: Onde vs Civil Registrar of Las Pinas City,
the civil registry involving the change of sex is G.R. No. 197174 September 10, 2014
not a mere clerical or typographical error. It is a
substantial change for which the applicable Facts:
procedure is Rule 108 of the Rules of Court.
-Petitioner filed a petition for correction of
-Respondent undisputedly has CAH. This entries in his certificate of live birth before the
condition causes the early or inappropriate R TC and named respondent Office of the Local
appearance of male characteristics. A person, Civil Registrar of Las Pinas City as sole
like respondent, with this condition produces respondent. Petitioner alleged that he is the
too much androgen, a male hormone. A illegitimate child of his parents Guillermo A.
newborn who has XX chromosomes coupled Onde and Matilde DC Pakingan, but his birth
with CAH usually has a (1) swollen clitoris with certificate stated that his parents were
the urethral opening at the base, an ambiguous married.
genitalia often appearing more male than
female; (2) normal internal structures of the -RTC dismissed the petition for correction of
female reproductive tract such as the ovaries, entries on the ground thatit is insufficient in
uterus and fallopian tubes; as the child grows form and substance. It ruled that the
older, some features start to appear male, such proceedings must be adversarial since the first
as deepening of the voice, facial hair, correction is substantial in nature and would
and failure to menstruate at puberty. About 1 in affect petitioner’s status as a legitimate child. It
10,000 to 18,000 children are born with CAH. was further held that the correction in the first
name of petitioner and his mother can be done
-Respondent here has simply let nature take its by the city civil registrar under Republic Act
course and has not taken unnatural steps to (R.A.) No. 9048.
arrest or interfere with what he was born

Rolan Jeff A. Lancion


Arellano University School of Law Page 154
Remedial Law 2- Brondial Notes AY 2017- 2018

Issue: WON RTC erred in its decision in the required by Section 3, Rule 108 of the Rules of
correction on the first name of the petitioner Court.

Held: Case: Republic vs Olaybar, G.R. No. 189538


February 10, 2014
No- Under Section 1 of R.A. No. 9048, clerical or
typographical errors on entries in a civil register Facts:
can be corrected and changes of first name can
be done by the concerned city civil registrar -Respondent requested from the National
without need of a judicial order. Statistics Office (NSO) a Certificate of No
Marriage (CENOMAR) as one of the
- Under R.A. No. 9048, jurisdiction over requirements for her marriage with her
applications for change of first name is now boyfriend of five years. Upon receipt thereof,
primarily lodged with administrative officers. she discovered that she was already married to
The intent and effect of said law is to exclude a certain Ye Son Sune, a Korean National, on
the change of first name from the coverage of June 24, 2002, at the Office of the Municipal
Rules 103 (Change of Name) and 108 Trial Court in Cities (MTCC), Palace of Justice.
(Cancellation or Correction of Entries in the Civil
Registry) of the Rules of Court, until and unless -She, thus, filed a Petition for Cancellation of
an administrative petition for change of name is Entries in the Marriage Contract, especially the
first filed and subsequently denied. entries in the wife portion thereof. Respondent
impleaded the Local Civil Registrar of Cebu City,
-Corrections of entries in the civil register as well as her alleged husband, as parties to the
including those on citizenship, legitimacy of case.
paternity or filiation, or legitimacy of marriage,
involve substantial alterations. Substantial -RTC granted the petition and ordered the Civil
errors in a civil registry may be corrected and Registrar to cancel the marriage contract.
the true facts established provided the parties
aggrieved by the error avail themselves of the Issue: WON Rule 108 is applicable in this case
appropriate adversary proceedings.
Held:
- Petition seeking a substantial correction of an
entry in a civil register must implead as parties YES
to the proceedings not only the local civil
-Rule 108 of the Rules of Court provides the
registrar, as petitioner did in the dismissed
procedure for cancellation or correction of
petition for correction of entries, but also all
entries in the civil registry. The proceedings
persons who have or claim any interest which
may either be summary or adversary. If the
would be affected by the correction. This is

Rolan Jeff A. Lancion


Arellano University School of Law Page 155
Remedial Law 2- Brondial Notes AY 2017- 2018

correction is clerical, then the procedure to be allowing the correction of the subject certificate
adopted is summary. If the rectification affects of marriage by cancelling the wife portion
the civil status, citizenship or nationality of a thereof, the trial court did not, in any way,
party, it is deemed substantial, and the declare the marriage void as there was no
procedure to be adopted is adversary. marriage to speak of.

-It is true that in special proceedings, formal


pleadings and a hearing may be dispensed with,
and the remedy granted upon mere application Case: Braza vs City Civil Registrar of
or motion. However, a special proceeding is not Himamaylan, G.R. No. 181174, December 4,
always summary. The procedure laid down in 2009
Rule 108 is not a summary proceeding per se.
It requires publication of the petition; it Facts:
mandates the inclusion as parties of all
persons who may claim interest which would -On April 15, 2002, Pablo Braza died in a
be affected by the cancellation or correction; it vehicular accident in Bandung, West Java,
also requires the civil registrar and any person Indonesia.
in interest to file their opposition, if any; and it
states that although the court may make -During the wake following the repatriation of
orders expediting the proceedings, it is after his remains to the Philippines, respondent
hearing that the court shall either dismiss the
Lucille Titular (Lucille) began introducing her co-
petition or issue an order granting the same.
respondent minor Patrick Alvin Titular Braza
-Aside from the certificate of marriage, no such (Patrick) as her and Pablo's son. Ma. Cristina
evidence was presented to show the existence thereupon made inquiries in the course of
of marriage.1âwphi1 Rather, respondent which she obtained Patrick's birth certificate
showed by overwhelming evidence that no
from the Local Civil Registrar of Himamaylan
marriage was entered into and that she was not
City, Negros Occidental.
even aware of such existence. The testimonial
and documentary evidence clearly established
-Ma. Cristina likewise obtained a copy of a
that the only "evidence" of marriage which is
marriage contract showing that Pablo and
the marriage certificate was a forgery.
Lucille were married on April 22, 1998, drawing
-Respondent indeed sought, not the her and her co-petitioners to file on December
nullification of marriage as there was no 23, 2005 before the Regional Trial Court of
marriage to speak of, but the correction of the Himamaylan City, Negros Occidental a
record of such marriage to reflect the truth as
set forth by the evidence. Otherwise stated, in

Rolan Jeff A. Lancion


Arellano University School of Law Page 156
Remedial Law 2- Brondial Notes AY 2017- 2018

petition to correct the entries in the birth contentious alterations may be allowed only in
record of Patrick in the Local Civil Register. adversarial proceedings, in which all interested
parties are impleaded and due process is
-The trial court dismissed the petition without
properly observed.
prejudice, it holding that in a special proceeding
for correction of entry, the court, which is not -Their cause of action is actually to seek the
declaration of Pablo and Lucilles marriage as
acting as a family court under the Family Code, void for being bigamous and impugn Patricks
has no jurisdiction over an action to annul the legitimacy, which causes of action are governed
marriage of Lucille and Pablo, impugn the not by Rule 108 but by A.M. No. 02-11-10-SC
which took effect on March 15, 2003, and Art.
legitimacy of Patrick, and order Patrick to be 171 of the Family Code, respectively, hence, the
subjected to a DNA test, hence, the controversy petition should be filed in a Family Court as
should be ventilated in an ordinary adversarial expressly provided in said Code.
- Validity of marriages as well as legitimacy and
action. filiation can be questioned only in a direct
action seasonably filed by the proper party,
Issue: WON the trial court has jurisdiction and not through collateral attack such as the
under Rule 108 to nullify marriages and rule on petition filed before the court a quo.
legitimacy and filiation.

Held:

Rule 108 of the Rules of Court vis a vis Article


412 of the Civil Code charts the procedure by
which an entry in the civil registry may be
cancelled or corrected. The proceeding
contemplated therein may generally be used
EVIDENCE- “The most practical among Remedial
only to correct clerical, spelling, typographical
Law”
and other innocuous errors in the civil
registry. A clerical error is one which is visible Rule 128- General Provisions
to the eyes or obvious to the understanding;
an error made by a clerk or a transcriber; a -Two things to consider in Evidence: -Relevancy
–Credibility
mistake in copying or writing, or a harmless
change such as a correction of name that is -In order for an evidence to be credible, it must
clearly misspelled or of a misstatement of the also come from a credible witness.
occupation of the parent. Substantial or

Rolan Jeff A. Lancion


Arellano University School of Law Page 157
Remedial Law 2- Brondial Notes AY 2017- 2018

Example: Cardinal Tagle or Pope Francis Sanction- It is allowed by law or Rules


(Competent)
Q: Where were on June 5, 2000?
-There can be evidence which is material and
A: On that day, I am on my way to the moon. relevant but not competent.

-In this example, Cardinal Tagle is credible but Case Doctrine: Zulueta vs CA- “The evidence
his testimony is irrelevant. presented by might be material & relevant to
the case but they are not sanctioned by the
Definition: “Evidence is the means sanctioned rules. The constitutional rights was violated in
by the rules of ascertaining in a judicial obtaining the pieces of evidence which is the
proceeding the truth respecting as a matter of right to privacy”
fact” (R 128,Sec 1)
Example of laws/statute:
According to Wigmore: Requirement for
Admissibility of Evidence -Anti-Wiretapping law

-Relevancy -Competency -Hearsay Rule

Means=Relevancy Case: OngChia vs Republic

Q: What is the meaning of relevancy or means? Facts: Ongchia a China man filed a petition
before the court for naturalization. His petition
A: It connects immediately two things: Cause & for naturalization was granted by the trial court
Effect. but the OSG appealed such decision on the
ground that some documentary evidence were
Q: What is the definition of Relevancy? not offered in court.

A: That which tends to prove. Ruling: The rules of court shall not be applicable
in a petition for naturalization therefore the
Q: How does a matter (word, object, document)
presentation of evidence shall not apply.
to prove something?
Case Doctrine: Sasan Jr vs NLRC-“ In Labor
A: It bridges the cause & effect of the evidence.
Cases, evidence not presented before the Labor
Arbiter may still be presented on appeal and it is
-Evidence must be relevant to the case
still admissible. Labor cases are not covered by
the Rules of Court.”
Example: A caliber 45 gun is material in killing a
person.

Rolan Jeff A. Lancion


Arellano University School of Law Page 158
Remedial Law 2- Brondial Notes AY 2017- 2018

To Ascertain- To make sure that the evidence is Objects which are not identifiable
the real thing (Authentication)
Example: Blood, oil, shabu
Nature of Evidence:

-Object/Real Evidence
Case: Ong Chia vs Republic, 328 SCRA
-Documentary Evidence
Facts:
- Testimonial Evidence
 Petitioner was born on January 1, 1923
Steps for Authentication in Amoy, China. In 1932, he arrived at
the port of Manila on vessel "Angking”
-Marking of Evidence
and found employment and eventually
-Identification of Evidence started his own business, married a
Filipina, with whom he had four
Rationale: To prepare the evidence during the
trial to which it will be offered. children. At the age of 66, he filed a
verified petition to be admitted as a
Documentary Evidence- It must be supported
with testimonial evidence. Filipino citizen under C.A. No. 473,
otherwise known as the Revised
Q: How do you authenticate testimonial
Naturalization Law, as amended.
evidence?
Petitioner, after stating his
A: Before a witness is allowed to testify, he
qualifications as required in and lack of
must take an OATH.
the disqualifications enumerated in of
-When a witness does not want to take an oath
the law.
during his presentation, his testimony cannot
be authenticated therefore it is inadmissible.

Q: How do you authenticate object evidence?  During the hearings, petitioner testified

A: It will depend if the object evidence is easily as to his qualifications and presented
identifiable since by their very nature, they are three witnesses to corroborate his
already marked.
testimony. Prosecutor Moran remarked
Example: Gun used in a killing incident (Serial that petitioner seems to be well-versed
Number of the Gun)

Rolan Jeff A. Lancion


Arellano University School of Law Page 159
Remedial Law 2- Brondial Notes AY 2017- 2018

with the major portion of the history of failed to support his petition with the
the Philippines and is convinced that appropriate documentary evidence.
petitioner really deserves to be Issue: WON CA erred in its decision on denying
admitted as a citizen of the Philippines. the presentation of documentary evidence
And for this, do not wish to present any
Held: NO
evidence to counteract or refute the
testimony of the witnesses for the - The contention has no merit. Petitioner failed
petitioner, as well as the petitioner to note Rule 143 of the Rules of Court which
himself. provides that “These rules shall not apply to
land registration, cadastral and election cases,
naturalization and insolvency proceedings, and
 August 25, 1999, the trial court
other cases not herein provided for, except by
granted the petition and
analogy or in a suppletory character and
admitted petitioner to Philippine
whenever practicable and convenient.”
citizenship.

-Rule on formal offer of evidence (Rule 132, Sec


 The State through the Office of the
34) now being invoked by petitioner is clearly
Solicitor General, appealed contending
not applicable to the present case involving a
that petitioner: (1) failed to state all the
petition for naturalization. The only instance
names by which he is or had been
when said rules may be applied by analogy or
known; (2) failed to state all his former
suppletorily in such cases is when it is
places of residence in violation of C.A.
"practicable and convenient." That is not the
No. 473; (3) failed to conduct himself in
case here, “since reliance upon the documents
a proper and irreproachable manner
presented by the State for the first time on
during his entire stay in the Philippines,
appeal, in fact, appears to be the more practical
in violation of; (4) has no known
and convenient course of action considering
lucrative trade or occupation and his
that decisions in naturalization proceedings are
previous incomes have been
not covered by the rule on r e s judicta”.
insufficient or misdeclared and (5)

Rolan Jeff A. Lancion


Arellano University School of Law Page 160
Remedial Law 2- Brondial Notes AY 2017- 2018

-A final favorable judgment does not Held:


preclude the State from later on moving for a
NO- Indeed the documents and papers in
revocation of the grant of naturalization on
question are inadmissible in evidence. The
the basis of the same documents.
constitutional injunction declaring the privacy
of communication and correspondence
Case: Zulueta vs. CA 253 SCRA
inviolable is no less applicable simply because it
Facts: is the wife who is the party against whom the
constitutional provision is to be enforced. The
-Petitioner Cecilia Zulueta is the wife of private
only exception to the prohibition in the
respondent Alfredo Martin. On March 26, 1982,
Constitution is if there is a lawful order from a
petitioner entered the clinic of her husband, a
court or when public safety or order requires
doctor of medicine, and in the presence of her
otherwise, as prescribed by law.
mother, a driver and private respondents
secretary, forcibly opened the drawers and -Any violation of this provision renders the
cabinet in her husband’s clinic and took 157 evidence obtained inadmissible for any purpose
documents consisting of private in any proceeding.
correspondence between Dr. Martin and his
Case: People vs Yatar, 428 SCRA
alleged paramours, greetings cards, cancelled
checks, diaries, Dr. Martins passport, and FACTS
photographs. Accused-appellant was sentenced to death for
the special complex crime of Rape with
-The documents and papers were seized for use
Homicide, and ordering him to pay the heirs of
in evidence in a case for legal separation and for
the victim. Appellant was charged to have had
disqualification from the practice of medicine
carnal knowledge of a certain Kathylyn Uba
which petitioner had filed against her husband.
against her will, and with the use of a bladed
Issue: WON the documentary evidence weapon, stabbed the latter inflicting upon her
presented during the trial are competent fatal injuries resulting in her untimely demise.
evidence

Rolan Jeff A. Lancion


Arellano University School of Law Page 161
Remedial Law 2- Brondial Notes AY 2017- 2018

In the instant case, appellant raises the issue of the witnesses and observing their deportment,
credibility of witnesses, specifically assigning as conduct and attitude. Absent any showing that
error on the part of the trial court, the latter’s the trial judge overlooked, misunderstood, or
giving of much weight to the evidence misapplied some facts or circumstances of
presented by the prosecution notwithstanding weight which would affect the result of the
their doubtfulness. case, the trial judge’s assessment of credibility
deserves the appellate court’s highest
ISSUE (1) respect. Where there is nothing to show that
Whether appellant’s contentions as regards the the witnesses for the prosecution were
witnesses’ credibility are meritorious. actuated by improper motive, their testimonies
are entitled to full faith and credit.
HELD: NO.
The issue regarding the credibility of the The weight of the prosecution’s evidence must
prosecution witnesses should be resolved be appreciated in light of the well-settled rule
against appellant. This Court will not interfere which provides that an accused can be
with the judgment of the trial court in convicted even if no eyewitness is available, as
determining the credibility of witnesses unless long as sufficient circumstantial evidence is
there appears in the record some fact or presented by the prosecution to prove beyond
circumstance of weight and influence which has doubt that the accused committed the crime.
been overlooked or the significance of which
has been misinterpreted. ISSUE (2)
Sufficiency of Circumstantial Evidence
Well-entrenched is the rule that the findings of
the trial court on credibility of witnesses are HELD: Circumstantial evidence, to be sufficient
entitled to great weight on appeal unless to warrant a conviction, must form an unbroken
cogent reasons are presented necessitating a chain which leads to a fair and reasonable
reexamination if not the disturbance of the conclusion that the accused, to the exclusion of
same; the reason being that the former is in a others, is the perpetrator of the crime. To
better and unique position of hearing first hand determine whether there is sufficient

Rolan Jeff A. Lancion


Arellano University School of Law Page 162
Remedial Law 2- Brondial Notes AY 2017- 2018

circumstantial evidence, three requisites must by mortgaging the subject property for the
concur: (1) there is more than one purpose of helping her defray her business
circumstance; (2) facts on which the inferences expenses; she later discovered that Nena did
are derived are proven; and (3) the not secure any loan nor mortgage the property;
combination of all the circumstances is such as she wants the title in the name of Nena
to produce a conviction beyond reasonable cancelled and the subject property reconveyed
doubt. to her. Daniela died on July 29, 1988 leaving her
Case: Tating vs Marcella, 519 SCRA children as her heirs.

Facts: -In a letter dated March 1, 1989, Carlos


informed Nena that when Daniela died they
-On October 14, 1969, Daniela sold the subject
discovered the sworn statement she executed
property to her granddaughter, herein
on December 28, 1977 and, as a consequence,
petitioner Nena Lazalita Tating.
they are demanding from Nena the return of
-The contract of sale was embodied in a duly their rightful shares over the subject property
notarized Deed of Absolute Sale executed by as heirs of Daniela. Nena did not reply. Efforts
Daniela in favor of Nena. Subsequently, title to settle the case amicably proved futile. Hence,
over the subject property was transferred in the her son filed a complaint with the RTC praying
name of Nena. She declared the property in her for the nullification of the Deed of Absolute
name for tax purposes and paid the real estate Sale. RTC decide in favour of the plaintiff and
taxes due thereon for the years 1972, 1973, was affirmed by the CA.
1975 to 1986 and 1988. However, the land
Issue: WON the Sworn Statement should have
remained in possession of Daniela.
been rejected outright by the lower courts.
=On December 28, 1977, Daniela executed a
Held:
sworn statement claiming that she had actually
no intention of selling the property; the true There is no issue on the admissibility of the
agreement between her and Nena was simply subject sworn statement. However, the
to transfer title over the subject property in admissibility of evidence should not be
favor of the latter to enable her to obtain a loan equated with weight of evidence. The

Rolan Jeff A. Lancion


Arellano University School of Law Page 163
Remedial Law 2- Brondial Notes AY 2017- 2018

admissibility of evidence depends on its simulated and that, as a consequence, a trust


relevance and competence while the weight of relationship was created between them.
evidence pertains to evidence already
Case: People vs Salafranca, 666 SCRA
admitted and its tendency to convince and
persuade. Thus, a particular item of evidence
Facts: After trial, the RTC convicted Salafranca,
may be admissible, but its evidentiary weight
stating: “The evidence is clear that it was
depends on judicial evaluation within the
Rodrigo Salafranca who delivered two (2)
guidelines provided by the rules of evidence. It
stabbing blows to the victim while holding
is settled that affidavits are classified as
Johnny Bolanon with his left arm encircled
hearsay evidence since they are not generally
around Bolanon’s neck stabbing the latter with
prepared by the affiant but by another who
the use of his right hand at the right sub costal
uses his own language in writing the affiant’s
area which caused Bolanon’s death. Not only
statements, which may thus be either omitted
because it was testified to by Augusto Mendoza
or misunderstood by the one writing them.
but corroborated by Rodolfo Estaño, the
Moreover, the adverse party is deprived of the
victim’s uncle who brought Bolanon to the
opportunity to cross-examine the affiant. For
hospital and who relayed to the court that
this reason, affidavits are generally rejected for
when he aided Bolanon and even on their way
being hearsay, unless the affiants themselves
to the hospital while the latter was suffering
are placed on the witness stand to testify
from hard breathing, victim Bolanon was able to
thereon.
say that it was Rodrigo Salafranca who stabbed
The Court finds that both the trial court and the him.”
CA committed error in giving the sworn
-Salafranca has come to the Court on a final
statement probative weight. Since Daniela is no
appeal, continuing to challenge the credibility of
longer available to take the witness stand as she
the witnesses who had incriminated him.
is already dead, the RTC and the CA should not
have given probative value on Daniela's sworn Issue: WON Salafranca is guilty beyond
statement for purposes of proving that the reasonable doubt.
contract of sale between her and petitioner was
Held:

Rolan Jeff A. Lancion


Arellano University School of Law Page 164
Remedial Law 2- Brondial Notes AY 2017- 2018

YES. recognized that the statement of the victim an


hour before his death and right after the
-Salafranca’s denial and alibi were worthless in
hacking incident bore all the earmarks either of
the face of his positive identification by
a dying declaration or part of the res gestae
Mendoza as the assailant of Bolanon. The lower
either of which was an exception to the hearsay
courts properly accorded full faith to such
rule.
incrimination by Mendoza considering that
Salafranca did not even project any ill motive Rule 129- WHAT NEED TO BE PROVED
that could have impelled Mendoza to testify
Section 1. Judicial notice, when mandatory. —
against him unless it was upon the truth.
A court shall take judicial notice, without the
introduction of evidence, of the existence and
It appears from the foregoing testimony that territorial extent of states, their political history,
Bolanon had gone to the residence of Estaño, forms of government and symbols of
nationality, the law of nations, the admiralty
his uncle, to seek help right after being stabbed and maritime courts of the world and their
by Salafranca; that Estaño had hurriedly dressed seals, the political constitution and history of
the Philippines, the official acts of legislative,
up to bring his nephew to the Philippine executive and judicial departments of the
General Hospital by taxicab; that on the way to Philippines, the laws of nature, the measure of
time, and the geographical divisions.
the hospital, Estaño had asked Bolanon who
had stabbed him, and the latter had told Estaño Judicial Notice- It is a process whereby the
court takes note of certain facts which are
that his assailant had been Salafranca; that at capable of being known to a veritable certainty
the time of the utterance Bolanon had seemed by consulting sources of indisputable accuracy,
thereby relieving one party of the burden of
to be having a hard time breathing, causing producing evidence to prove these facts.
Estaño to advise him not to talk anymore; and
-Judicial Notice can be mandatory or
that about ten minutes after his admission at discretionary.
the emergency ward of the hospital, Bolanon
Q: When is judicial notice mandatory?
had expired and had been pronounced dead.
A: A court shall take judicial notice, without the
-Such circumstances qualified the utterance of introduction of evidence of: State, International
Bolanon as both a dying declaration and as part Law, Philippines, Laws of Nature, Measures of
Time, and Geographical Divisions.
of the res gestae, considering that the Court has
Q: When is judicial notice mandatory?

Rolan Jeff A. Lancion


Arellano University School of Law Page 165
Remedial Law 2- Brondial Notes AY 2017- 2018

A: A court may take judicial notice of matter pleading is deemed by the adverse party if he
which are of public knowledge, or are capable does not specifically deny it under oath and set
of unquestionable demonstration, or ought to forth what he claims to be the facts.
be known to because of their judicial functions.
-Material averment in the complaint, other than
Q: When to take judicial notice? those as to the amount of unliquidated
damages, shall be deemed admitted when not
A: During the trial, the court, on its own specifically denied.
initiative, or on request of a party, may
announce its intention to take judicial notice of -Stipulation of facts
any matter and allow the parties to be heard
-Admission in the pleadings
thereon. After the trial, and before judgment or
on appeal, the proper court, on its own Q: May a judicial admission be contradicted?
initiative or on request of a party, may take
judicial notice of any matter and allow the A: NO, except by showing that it was made
parties to be heard thereon if such matter is through palpable mistake (R129 Sec 4)
decisive of a material issue in the case. (Rule
129 Sec 3) Case: LBP vs Banal, 434 SCRA

Facts:
Q: Can a court take judicial notice of foreign
laws? - Spouses Vicente and Leonidas Banal,

A: Courts cannot take judicial notice of foreign respondents, are the registered owners of
law. The parties may however stipulate on what agricultural land situated in San Felipe, Basud,
the foreign law is which case the same are
Camarines Norte covered by Transfer Certificate
judicial admissions binding upon them and they
cannot take a contrary stance. of Title No. T-6296. A portion of the land
consisting of 6.2330 hectares (5.4730 of which
Section 4. Judicial admissions. — An admission, is planted to coconut and 0.7600 planted to
verbal or written, made by the party in the
course of the proceedings in the same case, palay) was compulsorily acquired by the
does not require proof. The admission may be Department of Agrarian Reform (DAR) pursuant
contradicted only by showing that it was made
through palpable mistake or that no such to Republic Act (R.A.) No. 6657, as amended,
admission was made. otherwise known as the Comprehensive

Examples of judicial admission: Agrarian Reform Law of 1988.

- The genuineness and due execution of an -Spouses Vicente and Leonidas Banal rejected
actionable document copied in or attached to a
the above valuation. Thus, pursuant to Section

Rolan Jeff A. Lancion


Arellano University School of Law Page 166
Remedial Law 2- Brondial Notes AY 2017- 2018

16(d) of R.A. 6657, as amended, a summary intention to take judicial notice of any
administrative proceeding was conducted matter and allow the parties to be heard
before the Provincial Agrarian Reform thereon.”
Adjudicator (PARAD) to determine the valuation
After the trial, and before judgment or on
of the land. Eventually, the PARAD rendered its
appeal, the proper court, on its own initiative or
Decision affirming the Land bank’s valuation.
on request of a party, may take judicial notice
Issue: WON the valuation of respondent’s of any matter and allow the parties to be heard
property take judicial notice
thereon if such matter is decisive of a material
Held: issue in the case.

-Courts are not authorized to take judicial


Case: People vs Kulais 292 SCRA
notice of the contents of the records of other
FACTS:
cases even when said cases have been tried or
are pending in the same court or before the -Five Information for the crime of kidnapping

same judge. They may only do so in the for ransom were filed before the RTC of

absence of objection and with the knowledge Zamboanga City against the accused. That being

of the opposing party, which are not obtaining all private individuals, conspiring and

here. confederating together, mutually aiding and


assisting one another, with threats to kill Felix
Furthermore, as earlier stated, the Rules of
Rosario, who is a male public officer of the City
Court shall apply to all proceedings before the
Government of Zamboanga, who was then
Special Agrarian Courts. In this regard, Section
aboard a Cimarron vehicle which was being
3, Rule 129 of the Revised Rules on Evidence is
ambushed by the herein accused at the
explicit on the necessity of a hearing before a
highway of Sitio Tigbao Lisomo, Zamboanga
court takes judicial notice of a certain matter,
City, and brought Rosario to different
thus:
mountainous places of Zamboanga City and

“SEC. 3. Judicial notice, when hearing necessary. Zamboanga Del Sur, where he was detained and

During the trial, the court, on its own initiative, deprived of his liberty.

or on request of a party, may announce its

Rolan Jeff A. Lancion


Arellano University School of Law Page 167
Remedial Law 2- Brondial Notes AY 2017- 2018

-Appellant Kulais argues that he was denied due against the appellant. Hence, the appellant was
process when the trial court took judicial notice not denied due process. His conviction was
of the testimony given in another case by one based mainly on the positive identification
Lt. Melquiades Feliciano, who was the team made by some of the kidnap victims. These
leader of the government troops that captured witnesses were subjected to meticulous cross-
them and his purported cohorts. Because he examinations conducted by appellant's counsel.
was allegedly deprived of his right to cross-
Case: Laureno vs CA, 324 SCRA
examine a material witness in the person of
Lieutenant Feliciano, he contends that the Facts:
latter's testimony should not be used against
-In 1978, Menandro Laureano was hired as a
him.
pilot by the Singapore Airlines Limited (SAL). In
ISSUE: WON the trial court erred in taking 1982 however, SAL was hit by recession and so
judicial notice of a material testimony given in it had to lay off some employees.
another case by Lt. Melquiades Feliciano.
-Laureano was one of them. Laureano asked for

HELD: reconsideration but it was not granted.


Aggrieved, Laureano filed a labor case for illegal
-No, As a general rule, courts should not take
dismissal against SAL. But in 1987, he withdrew
judicial notice of the evidence presented in
the labor case and instead filed a civil case for
other proceedings, even if these have been
damages due to illegal termination of contract
tried or are pending in the same court, or have
against SAL. Laureano filed the case here in the
been heard and are actually pending before
Philippines. SAL moved for the dismissal of the
the same judge. This is especially true in
case on the ground of lack of jurisdiction. The
criminal cases, where the accused has the
motion was denied. On trial, SAL alleged that
constitutional right to confront and cross-
the termination of Laureano is valid pursuant to
examine the witnesses against him.
Singaporean law.

-Even if the court a quo did take judicial notice -The trial court ruled in favor of Laureano. SAL
of the testimony of Lieutenant Feliciano, it did appealed the case raising the issue of lack of
not use such testimony in deciding the cases jurisdiction, non-applicability of Philippine laws,

Rolan Jeff A. Lancion


Arellano University School of Law Page 168
Remedial Law 2- Brondial Notes AY 2017- 2018

and estoppel, among others. The Court of On 14 February 2005, an RRCG bus was plying
Appeals reversed the trial court. its usual southbound route. According to Elmer
Andales, bus conductor, two men insisted on
ISSUE: WON Singaporean Law is applicable to
getting on the bus. He then let them in. As soon
this case.
as the bus reached the stoplight at the corner of
HELD:
Ayala Avenue the two insisted on getting off the
No. -The specific Singaporean Law which holds bus. According to Andales, the bus driver
valid the dismissal of Laureano is not proved in initially did not want to let them off the bus,
court. As such, the trial court cannot make a because of an ordinance prohibiting the
determination if the termination is indeed valid unloading except at designated bus stops.
under Singaporean Law. Philippine courts do Moments after, a fire quickly engulfed the bus.
not take judicial notice of the laws of The prosecution presented documents
Singapore. SAL has the burden of proof. SAL furnished by the Department of Justice,
failed to prove such law hence Philippine law confirming that shortly before the explosion,
shall apply. However, the case must be the spokesperson of the Abu Sayyaf Group
dismissed on the ground of estoppel. Under announced over radio station that the group
our laws, all money claims arising from had a Valentine's Day "gift" for former
employer-employee relationships must be filed President Gloria Macapagal-Arroyo. Accused
within three years from the time the cause of Trinidad gave ABS-CBN News Network an
action accrued. Laureano’s cause of action exclusive interview some time after the
accrued in 1982 when he was terminated but incident, confessing his participation in the
he only filed the money claim in 1987 or more bombing incident. The bus conductor identified
than three years from 1982. Hence he is already Baharan and Trinidad, and confirmed that they
barred by prescription. were the two men who had entered the RRCG
busy. Members of the Abu Sayyaf Group
were then charged with multiple murder and
Case: People vs Baharan, 639 SCRA
multiple frustrated murder. On their
Facts: arraignment for the multiple murder charge,
Baharan, Trinidad, and Asali all entered a plea

Rolan Jeff A. Lancion


Arellano University School of Law Page 169
Remedial Law 2- Brondial Notes AY 2017- 2018

of guilty. On the other hand, upon arraignment Trinidad previously pled guilty to another
for the multiple frustrated murder charge, charge - multiple murder - based on the same
accused Asali pled guilty. Accused Trinidad and act relied upon in the multiple frustrated
Baharan pled not guilty. Rohmat pled not murder charge. The Court further notes that
guilty to both charges. prior to the change of plea to one of guilt,
accused Baharan and Trinidad made two other
ISSUES: confessions of guilt - one through
WON the trial court gravely erred in accepting an extrajudicial confession (exclusive television
plea of guilt despite insufficiency of searching interviews, as stipulated by both accused during
inquiry into the voluntariness and full pretrial), and the other via judicial admission
comprehension of the consequences of the said (pretrial stipulation). Considering the foregoing
plea; and circumstances, we deem it unnecessary to rule
WON the trial court gravely erred in finding that on the sufficiency of the "searching inquiry"
the guilt of accused-appellants for the crimes in this instance. Remanding the case for re-
charged had been proven beyond reasonable arraignment is not warranted, as the accused's
doubt. plea of guilt was not the sole basis of the
condemnatory judgment under consideration.
RULING:
(B) NO. Insofar as appellants Baharan and
(A) NO. This court have reiterated in a long line Trinidad are concerned, the evidence for the
of cases that the conduct of a searching inquiry prosecution, in addition to that which can be
remains the duty of judges, as they are drawn from the stipulation of facts, primarily
mandated by the rules to satisfy themselves consisted of the testimonies of the bus
that the accused had not been under coercion conductor, who positively identified accused
or duress; mistaken impressions; or a Baharan and of the accused-turned-state-
misunderstanding of the significance, effects, witness. The guilt of the accused Baharan and
and consequences of their guilty plea. This Trinidad was sufficiently established by these
requirement is stringent and mandatory. The corroborating testimonies, coupled with their
Court observes that accused Baharan and respective judicial admissions (pretrial

Rolan Jeff A. Lancion


Arellano University School of Law Page 170
Remedial Law 2- Brondial Notes AY 2017- 2018

stipulations) and extrajudicial confessions witness stand. It is admissible in evidence


(exclusive television interviews, as they both against appellant Palijon. Moreover, where
stipulated during pretrial) that they were several accused are tried together for the same
indeed the perpetrators of the Valentine's Day offense, the testimony of a co-accused
bombing. It is true that under the rule, implicating his co-accused is competent
statements made by a conspirator against a co- evidence against the latter.
conspirator are admissible only when made
during the existence of the conspiracy.
Case: Ligtas vs People 767 SCRA
However, as the Court ruled in People v.
Buntag, if the declarant repeats the statement Facts:
in court, his extrajudicial confession becomes
- On or about the 29th day of June 2000 at Sitio
a judicial admission, making the testimony Lamak, Barangay San Juan, Municipality of
admissible as to both conspirators. Thus, in Sogod, Province of Southern Leyte, Philippines
and within the jurisdiction of this Honorable
People v. Palijon, the Court held the following:
Court, the above-named accused, with intent of
… [W]e must make a distinction between gain, entered into the abaca plantation
extrajudicial and judicial confessions. An belonging to one Anecita Pacate, and once
inside the plantation, did then and there
extrajudicial confession may be given in
willfully, unlawfully and feloniously harvested
evidence against the confessant but not against 1,000 kilos of abaca fibers, valued at
his co-accused as they are deprived of the Php29,000.00 at Php29.00 per kilo.
opportunity to cross-examine him. A judicial
- On June 29, 2000, Cabero, the plantation's
confession is admissible against the declarant's administrator, and several men, including
co-accused since the latter are afforded Cipres, went to the plantation to harvest abaca
upon Anecita Pacate's instructions. At about
opportunity to cross-examine the former.
10:00 a.m., Cabero and his men were surprised
Section 30, Rule 130 of the Rules of Court to find Ligtas harvesting abaca at the plantation.
applies only to extrajudicial acts or admissions Ligtas was accompanied by three (3)
unidentified men.
and not to testimony at trial where the party
adversely affected has the opportunity to cross- - RTC convict accused Monico Ligtas guilty
beyond reasonable doubt of the crime of Theft.
examine the declarant. Mercene's admission
implicating his co-accused was given on the

Rolan Jeff A. Lancion


Arellano University School of Law Page 171
Remedial Law 2- Brondial Notes AY 2017- 2018

- The Court of Appeals affirmed the ruling of the Object Evidence-1 section
trial court.
Q: What is object evidence?
Issue: WON Ligtas is guilty beyond reasonable
doubt for the crime of theft? A: It is which is directly addressed to the senses
of the court like tangible things exhibited or
Held:
demonstrated in open court.
NO
Documentary Evidence- (Best Evidence, Parole
-Decisions in administrative cases are not
Evidence & Electronic Evidence)
binding on criminal proceedings. This court has
ruled in a number of cases that it is indeed a
Testimonial Evidence- The rest of the sections
fundamental principle of administrative law
under Rule 130.
that administrative cases are independent from
criminal actions for the same act or omission. -The strongest kind of evidence is object
evidence, the 2nd will be documentary and the
Findings of fact of administrative agencies in
weakest is testimonial.
the exercise of their quasi-judicial powers are
entitled to respect if supported by substantial -Object and Documentary Evidence must also
evidence. This court is not tasked to weigh be testified and supported in court.
again "the evidence submitted before the
administrative body and to substitute its own Example: Presentation of Mahal & Mura to
judgment as to the sufficiency of evidence.” prove that there is dwarfism in the country.

- The landowner's share in the produce depends -In view of the presentation of Mahal & Mura, a
on the agreement between the parties. Hence, person should testify for their existence.
the harvesting done by the tenant is with the
Corpus Delicti- Fact of the crime
landowner’s consent.
Q: What do you mean of to the sense of court?

Rule 130- Rules of Admissibility A: It is addressed to the senses of the judge (5


senses of Man)
Section 1 Object as Evidence- Objects as
evidence are those addressed to the senses of The kinds of Object Evidence shall depend on
what sense of man it is addressed.
the court. When an object is relevant to the
facts in issue, it may be exhibited, examined or Visual Evidence- Pertains to Visual Sense
viewed by the court.
Audio Evidence- Pertains to Hearing Sense

Rolan Jeff A. Lancion


Arellano University School of Law Page 172
Remedial Law 2- Brondial Notes AY 2017- 2018

Gustatory Evidence- Pertains to Sense of Taste - On cross, he admitted that it was he alone
who conducted the search because Bolanos
Olfactory Evidence- Pertains to the Sense of
Smell was standing behind him in the living room
portion of the house and that petitioner
Tactil Evidence- Pertains to the Sense of Touch
handed to him the things to be searched,
*Frequently used in practice is Audio-Visual which included the pillow in which the two
Evidence.
sachets of shabu were kept.
Cases: People vs Malinlin 553 SCRA - On 20 June 2004 the trial court rendered

Facts: its Decision declaring petitioner guilty


beyond reasonable doubt of the offense
-On the strength of a warrant of search and
charged.
seizure issued by the RTC of Sorsogon City,
- The Court of Appeals rendered the assailed
Branch 52, a team of five police officers
decision affirming the judgment of the trial
raided the residence of petitioner in
court but modifying the prison sentence to an
Barangay Tugos, Sorsogon City on 4
indeterminate term of twelve (12) years as
February 2003. The team was headed by
minimum to seventeen (17) years as maximum.
P/Insp. Catalino Bolanos (Bolanos), with
PO3 Roberto Esternon (Esternon), SPO1
Pedro Docot, Issue: WON the shabu confiscated shall be
admissible evidence against the petitioner?

- SPO1 Danilo Lasala and SPO2 Romeo Held:

Gallinera (Gallinera) as members. The


- Prosecutions for illegal possession of
search conducted in the presence of
prohibited drugs necessitates that the
Barangay Kagawad.
elemental act of possession of a prohibited

- Petitioner was charged with violation of substance be established with moral certainty,

Section 11 of Republic Act No. 9165, together with the fact that the same is not

otherwise known as The Comprehensive authorized by law. The dangerous drug itself

Dangerous Drugs Act of 2002. constitutes the very corpus delicti of the

Rolan Jeff A. Lancion


Arellano University School of Law Page 173
Remedial Law 2- Brondial Notes AY 2017- 2018

offense and the fact of its existence is vital to illegal drug actually recovered from the
a judgment of conviction. Pagaduan. Strict compliance with the
prescribed procedure is required because of the
Case: People vs Pagaduan, 627 SCRA illegal drug’s unique characteristic rendering it
Facts: indistinct, not readily identifiable, and easily
open to tampering alteration or substitution
-Buy-bust operation was conducted by PO3 either by accident or otherwise.
Almarez, SPO1 Balido and Captain de Vera.
Ruper Pagaduan was arrested and plastic sachet Case: Salas vs Matusalem 705 SCRA
of what appears to be shabu was marked,
Facts:
request for laboratory examination was done
the same day. - On May 26, 1995, Annabelle Matusalem
(respondent) filed a complaint for
-The plastic sachet was turned over to PNP
Support/Damages against Narciso Salas
Crime Laboratory two days after. He was found
(petitioner) in the Regional Trial Court of
guilty by the court and the same was affirmed
Cabanatuan City.
by CA
- Respondent claimed that the petitioner is the
-Pagaduan contents among others that the father of her son Christian Paulo Salas who was
prosecution failed to show an unbroken chain born on December 28, 1994. Petitioner, already
of custody in the handling of the seized drug. 56 years old at the time, enticed her as she was
He claims that there was no evidence to show then only 24 years old, making her believe that
when the marking were done. he is a widower.

Issue: WON the integrity of the evidence was - Petitioner rented an apartment where
preserved respondent stayed and shouldered all expenses
in the delivery of their child, including the cost
Held:
of caesarian operation and hospital
confinement. However, when respondent
NO- The prosecution failed to show that the
refused the offer of petitioner’s family to take
illegal drug presented in the court is the same

Rolan Jeff A. Lancion


Arellano University School of Law Page 174
Remedial Law 2- Brondial Notes AY 2017- 2018

the child from her, petitioner abandoned the the mother, doctor, registrar, or other person is
respondent and her child and left them to the incompetent evidence of paternity.
mercy of relatives and friends. Respondent
-While baptismal certificates may be considered
further alleged that she attempted suicide due
public documents, they can only serve as
to depression but still petitioner refused to
evidence of the administration of the
support her and their child.
sacraments on the dates so specified. They are
- The testimonial evidence of the owner of the not necessarily competent evidence of the
apartment where petitioner allegedly housed veracity of entries therein with respect to the
respondent when she was pregnant was one of child’s paternity.
the basis for respondent’s claim for support.
- Pictures taken of the mother and her child
- The RTC ruled in favor of respondent to which together with the alleged father are
the CA affirmed on appeal. inconclusive evidence to prove paternity. As to
the handwritten notes are not sufficient to
Issue: WON the trial and appellate court erred
establish Christian Paulo’s filiation to petitioner.
in ruling that respondent’s evidence sufficiently
Thus, even these notes were authentic; they do
proved that her son Christian Paulo is the
not qualify as competent evidence of
illegitimate child of petitioner
illegitimate filiation an admission of filiation in a
Held: private handwritten instrument signed by the
parent concerned.

Yes -The SC held that a certificate of live birth


purportedly identifying the putative father is
not competent evidence of paternity when
Case: People vs Posing, 703 SCRA
there is no showing that the putative father
had a hand in the preparation of the Facts:

certificate. Thus, if the father did not signed in - SPO1 Angeles together with PO1 Jesus Cortez
the birth certificate, the placing of his name by (PO1 Cortez), PO1 Ralph Nicart (PO1 Nicart) and
the informant were dispatched to Makabayan
St., Brgy. Obrero, Kamuning, Quezon City, and

Rolan Jeff A. Lancion


Arellano University School of Law Page 175
Remedial Law 2- Brondial Notes AY 2017- 2018

upon arrival, the informant and SPO1 Angeles their duties in a regular manner, unless there is
proceeded to the squatter’s area. evidence to the contrary.

- SPO1 Angeles met Posing beside the - But time and again, jurisprudence is consistent
basketball court, where he was introduced by in stating that less than strict compliance with
the informant as a buyer of shabu. The former the procedural aspect of the chain of custody
asked if he could buy one hundred peso rule does not necessarily render the seized drug
(P100.00) worth of shabu for personal use. items inadmissible.
Posing then pulled out one (1) transparent
plastic sachet from his pocket and gave it to - RA 9165 and its subsequent Implementing
SPO1 Angeles in exchange for the buy-bust Rules and Regulations (IRR) do not require strict
money. Afterwards, SPO1 Angeles took out his compliance as to the chain of custody rule.
cap to alert his companions that the deal was
already concluded. PO1 Cortez and PO1 Nicart - As to the charge of illegal possession of
rushed to the scene and introduced themselves dangerous drugs, the prosecution must
as police officers. establish the following elements: (1) the
accused is in possession of an item or object,
-The trial court rendered a decision founding which is identified to be a prohibited or
Posing guilty of Sec 5 & 11 of RA 9165. CA regulated drug; (2) such possession is not
affirmed the decision of the trial court. authorized by law; and (3) the accused freely
and consciously possessed the drug.
Issue: WON accused Posing is guilty beyond
reasonable doubt for violation of RA 9165. Case: People vs Mercury Dela Cruz, G.R. No.
212171 September 7, 2016
Held:

YES- For the successful prosecution of offenses


Facts:
involving the illegal sale of drugs under Section
5, Article II of R.A. No. 9165, the following - The accused-appellant Mercury Dela Cruz alias
elements must be proven: (1) the identity of the
"Deday" was found guilty of illegal sale of shabu
buyer and seller, object and consideration; and
(2) the delivery of the thing sold and the in violation of Section 5, Article II of R.A. 9165
payment therefor. What is material to the
and sentenced him to suffer the penalty of life
prosecution for illegal sale of dangerous drugs is
the proof that the transaction or sale actually imprisonment and to pay a fine of P500, 000.00.
took place, coupled with the presentation in
court of evidence of corpus delicti.
-The RTC gave full credence to the testimonies
- In cases involving violations of Dangerous of the police officers who conducted the buy-
Drugs Act, credence should be given to the
narration of the incident by the prosecution bust operation against the accused-appellant,
witnesses especially when they are police and rejected the self-serving defenses of denial
officers who are presumed to have performed

Rolan Jeff A. Lancion


Arellano University School of Law Page 176
Remedial Law 2- Brondial Notes AY 2017- 2018

and alibi of the accused-appellant and her live- on the seized item nor was it photographed, the
in partner. The RTC noted the the categorical court find it untenable. The alleged non-
affirmation of accused-appellant and her live-in compliance with Section 21 of R.A. 9165 was
partner that the arresting officers did not not fatal to the prosecution's case because the
demand anything from them in exchange for apprehending team properly preserved the
the accused-appellant's liberty created the integrity and evidentiary value of the seized
presumption that the arresting officers were drugs.
performing their official functions regularly. - Although ideally the prosecution should offer
a perfect chain of custody in handling of
- On intermediate appellate review, the CA evidence, "substantial compliance with the
affirmed in toto the Roc's ruling. The CA agreed legal requirements on the handling of the
with the RTC in giving weight to the testimonies seized item" is sufficient. The court ruled that
of the prosecution witnesses, and held that the even if the arresting officers failed to strictly
arresting officer's complied with the proper comply with the requirements under Section 21
procedure in the custody and disposition of the of R.A. 9165, such procedural lapse is not fatal
seized drugs. and will not render the items seized
inadmissible in evidence.
Issue: WON the confiscated dangerous drug
constitutes the corpus delicti of the crime. - To be admissible in evidence, the prosecution
must be able to present through records or
Held: testimony, the whereabouts of the dangerous
Yes. drugs from the time these were seized from
-The court considered by the courts in arriving the accused by the arresting officers; turned
at the decision despite the fact that the forensic over to the investigating officer; forwarded to
chemist who examined it did not testify in the laboratory for determination their
court. The accused-appellant's contention that composition; and up to the time these are
the drugs were marked not at the place where offered in evidence. For as long as the chain of
she was apprehended but at the police station custody remains unbroken, as in this case,
and that there was no physical inventory made even though the procedural requirements

Rolan Jeff A. Lancion


Arellano University School of Law Page 177
Remedial Law 2- Brondial Notes AY 2017- 2018

provided for in Section 21 R.A. 9165 were not Public Record- (d) When the original is a public
record in the custody of a public officer or is
faithfully observed, the guilt of the accused will
recorded in a public office.
not be affected.
-The content of the document is the subject of
inquiry in a trial.
Section 2 Documentary Evidence- Documents
as evidence consist of writing or any material
Q: What is secondary evidence under the best
containing letters, words, numbers, figures,
evidence rule?
symbols or other modes of written expression
offered as proof of their contents.
A: Secondary evidence is that which may be
used to prove the contents of a document
Three Rules: Best Evidence Rule, Parole
where the original is lost, destroyed, or cannot
Evidence Rule & Electronic Evidence Rule
be produced or where the original is in the
custody or control of the party against whom
Best Evidence Rule
the evidence is offered and who fails to produce
if after reasonable notice.
Section 3. Original document must be
produced; exceptions. — When the subject of
Q: What does secondary evidence consist of?
inquiry is the contents of a document, no
evidence shall be admissible other than the
A: Secondary evidence consist of: (i) copy of the
original document itself, except in the following
original, (ii) a recital of its contents in an
cases:
authentic document (iii) testimony of witnesses.
Exceptions:
Q: What is the proper foundation for the
introduction of secondary evidence in the case
Custody/Control- (a) When the original has
of loss, destruction or inability to produce the
been lost or destroyed, or cannot be produced
original?
in court, without bad faith on the part of the
offeror;
A: The offeror must show: (i) proof of the
execution or existence of the document, (ii)
Lost or destroyed without bad faith-(b) When
proof of the cause of its unavailability, without
the original is in the custody or under the
bad faith on his part, (iii) proof of reasonable
control of the party against whom the evidence
diligence and good faith on the part of the
is offered, and the latter fails to produce it after
offeror in the search for or attempt.
reasonable notice;
Case: Loon vs Power Master Inc, 712 SCRA
Accounts (numerous)- (c) When the original
consists of numerous accounts or other Facts:
documents which cannot be examined in court
without great loss of time and the fact sought Respondents Power Master, Inc. and Tri-C
to be established from them is only the general General Services employed and assigned the
result of the whole; and
petitioners as janitors and leadsmen in various

Rolan Jeff A. Lancion


Arellano University School of Law Page 178
Remedial Law 2- Brondial Notes AY 2017- 2018

Philippine Long Distance Telephone Company


(PLDT) offices in Metro Manila area.
Subsequently, the petitioners filed a complaint Issue: WON the respondents were estopped
for money claims against Power Master, Inc., from submitting pieces of evidence for the first
Tri-C General Services and their officers, the time on appeal;
spouses Homer and Carina Alumisin Held:
(collectively, the respondents). The petitioners
alleged in their complaint that they were not Yes.
paid minimum wages, overtime, holiday,
premium, service incentive leave, and
A party may only adduce evidence for the first
thirteenth month pays. They further averred
time on appeal if he adequately explains his
that the respondents made them sign blank
delay in the submission of evidence and he
payroll sheets. On June 11, 2001, the
sufficiently proves the allegations sought to be
petitioners amended their complaint and
proven.In labor cases, strict adherence to the
included illegal dismissal as their cause of
technical rules of procedure is not required.
action. They claimed that the respondents
Time and again, we have allowed evidence to
relieved them from service in retaliation for the
be submitted for the first time on appeal with
filing of their original complaint.
the NLRC in the interest of substantial justice.
The Labor Arbiter awarded the petitioners Thus, we have consistently supported the rule
salary differential, service incentive leaves and that labor officials should use all reasonable
13th month pays. In awarding these claims the means to ascertain the facts in each case
LA stated that the burden in proving the speedily and objectively, without regard to
payment of these money claims rests with the technicalities of law or procedure, in the
employer. However, they were not awarded interest of due process. However, this liberal
backwages, overtime, holiday and premium policy should still be subject to rules of reason
pays for failure to show that they rendered and fairplay. The liberality of procedural rules
overtime work and worked on holidays. is qualified by two requirements: (1) a party
Moreover, it was not decided that they were should adequately explain any delay in the
illegally dismissed for failure to show notice of submission of evidence; and (2) a party should
termination of employment. sufficiently prove the allegations sought to be
proven. The reason for these requirements is
Both parties appealed to the ruling of the LA. that the liberal application of the rules before
NLRC allowed the respondents to present quasi-judicial agencies cannot be used to
pieces of evidence for the first time on appeal perpetuate injustice and hamper the just
on the ground that they have been deprived of resolution of the case. Neither is the rule on
due process. The Court of Appeals affirmed the liberal construction a license to disregard the
ruling of the NLRC. rules of procedure.

Rolan Jeff A. Lancion


Arellano University School of Law Page 179
Remedial Law 2- Brondial Notes AY 2017- 2018

Guided by these principles, the CA grossly erred Parole Evidence=Evidence Aliunde


in ruling that the NLRC did not commit grave
abuse of discretion in arbitrarily admitting and -Any evidence outside document cannot
giving weight to the respondents’ pieces of establish the terms and conditions of the
contract.
evidence for the first time on appeal. The
respondents failed to adequately explain their -A party cannot proof to establish a contract
delay in the submission of evidence. through a new contract since it will be
inadmissible as evidence.

Rationale: When 2 persons enter into a


contract and they put in writing the terms &
conditions of such, it shall be binding among
Parole Evidence Rule them.

Section 9. Evidence of written agreements. — Q: What is parole evidence rule?

When the terms of an agreement have been A: It is the rule which provides that once the
reduced to writing, it is considered as terms of an agreement have been reduced to
containing all the terms agreed upon and there writing, it is considered as containing all the
can be, between the parties and their terms agreed upon and there can be between
successors in interest, no evidence of such the parties and their successor in interest, no
terms other than the contents of the written evidence of such terms other than the contents
agreement. of the written agreement.

However, a party may present evidence to Q: What is parole evidence?


modify, explain or add to the terms of written
agreement if he puts in issue in his pleading: A: It is any evidence whether oral or written,
outside of the written agreement, which is
(a) An intrinsic ambiguity, mistake or offered to modify, explain or add to the terms
imperfection in the written agreement; of the written agreement.

(b) The failure of the written agreement to Q: What is the rationale behind parole
express the true intent and agreement of the evidence?
parties thereto;
A: Under the Integration Rule, it is presumed
(c) The validity of the written agreement; or that the parties to a written contract intended it
to integrate all the terms of their agreement.
(d) The existence of other terms agreed to by
the parties or their successors in interest after Q: What are the exceptions to the parole
the execution of the written agreement. evidence rule?

The term "agreement" includes wills. A: (VISA)

Rolan Jeff A. Lancion


Arellano University School of Law Page 180
Remedial Law 2- Brondial Notes AY 2017- 2018

1. Validity of the written agreement and conditions and conditions


2. Failure of the written agreement to express through parole through parole
the true Intent and agreement of the evidence, it should be evidence, it should be
parties. set forth in the set forth in the
3. Existence of Subsequent terms pleadings. pleadings.
4. Intrinsic Ambiguity, mistake or -In order for the -In order for the
imperfection exception to prosper, exception to prosper,
it should be alleged in it should be alleged in
Exceptions: the pleadings. the pleadings.
-Can be invoked by -Can be invoked only
Validity of the written agreement non-parties to the by a party to the
document written contract
Q: What if the contract is void or invalid?

A: There is no contract to speak of since it is


void ab initio. Case: Ortanez vs Court of Appeals, 266 SCRA

Intrinsic Ambiguity in the contract- In which it Facts:


has double interpretation Private respondent Rafael Ortanez filed with
the Quezon City RTC a complaint for annulment
Example: Dollar= US Dollar or Singaporean
of marriage with damages against petitioner
Dollar
Teresita Salcedo-Ortanez, on grounds of lack of
marriage license and/or psychological
Mistake- It refers to an error in the drafting or
wording of the written agreement which incapacity of the petitioner.
changes the intended meaning thereof.
Among the exhibits offered by private
respondent were three (3) cassette tapes of
Best Evidence Rule vs Parole Evidence Rule
alleged telephone conversations between
BEST EVIDENCE RULE PAROLE EVIDENCE petitioner and unidentified persons.
RULE
Teresita submitted her Objection/Comment to
Applies to all Applies only to written
Rafael’s oral offer of evidence. However, the
documentary evidence contracts (including
trial court admitted all of private respondent’s
wills)
Original is not Original is available in offered evidence and later on denied her
available in court court motion for reconsideration, prompting
-In order for the -In order for the petitioner to file a petition for certiorari with
exception to prosper, exception to prosper, the CA to assail the admission in evidence of the
it should be alleged in it should be alleged in aforementioned cassette tapes.
the pleadings. the pleadings. These tape recordings were made and obtained
-When a party tries to -When a party tries to when private respondent allowed his friends
contest certain terms contest certain terms

Rolan Jeff A. Lancion


Arellano University School of Law Page 181
Remedial Law 2- Brondial Notes AY 2017- 2018

from the military to wire-tap his home or arrangement, to secretly overhear, intercept,
telephone. or record such communication or spoken word
by using a device commonly known as a
CA denied the petition because (1) Tape dictaphone or dictagraph or detectaphone or
recordings are not inadmissible per se. They walkie-talkie or tape-recorder, or however
and any other variant thereof can be admitted otherwise described.”
in evidence for certain purposes, depending on
how they are presented and offered and on “Sec. 4. Any communication or spoken word, or
how the trial judge utilizes them in the interest the existence, contents, substance, purport, or
of truth and fairness and the even handed meaning of the same or any part thereof, or any
administration of justice; and (2) A petition information therein contained, obtained or
for certiorari is notoriously inappropriate to secured by any person in violation of the
rectify a supposed error in admitting evidence preceding sections of this Act shall not be
adduced during trial. The ruling on admissibility admissible in evidence in any judicial, quasi-
is interlocutory; neither does it impinge on judicial, legislative or administrative hearing or
jurisdiction. If it is erroneous, the ruling should investigation.”
be questioned in the appeal from the judgment
on the merits and not through the special civil Absent a clear showing that both parties to the
action of certiorari. The error, assuming telephone conversations allowed the recording
gratuitously exists, cannot be any more than an of the same, the inadmissibility of the subject
error of law, properly correctible by appeal and tapes is mandatory under Rep. Act No. 4200.
not by certiorari.
Petitioner then filed the present petition for
review under Rule 45 of the Rules of Court.
Case: Lapu-Lapu Foundation vs CA, 421 SCRA

Petitioner Elias Q. Tan, then President of the co-


Issue: petitioner Lapulapu Foundation, Inc., obtained
WON the recordings of the telephone four loans from the respondent Allied Banking
conversations are admissible in evidence Corporation covered by four promissory notes
in the amounts of P100,000 each. The
No. Rep. Act No. 4200 entitled “An Act to
petitioners failed to pay the same. The
Prohibit and Penalize Wire Tapping and Other
respondent Bank was constrained to file a
Related Violations of the Privacy of
Communication, and for other purposes” complaint seeking payment by the petitioners,
expressly makes such tape recordings jointly and solidarily, of the sum of P493,566.61
inadmissible in evidence thus: representing their loan obligation, exclusive of
“Sec. 1. It shall be unlawful for any person, not interests, penalty charges, attorney’s fees and
being authorized by all the parties to any costs
private communication or spoken word, to tap
any wire or cable, or by using any other device

Rolan Jeff A. Lancion


Arellano University School of Law Page 182
Remedial Law 2- Brondial Notes AY 2017- 2018

In its answer to the complaint, the petitioner there was an unwritten agreement between
Foundation denied incurring indebtedness from him and the respondent Bank that he would pay
the respondent Bank alleging that the loans the loans from the proceeds of his shares of
were obtained by petitioner Tan in his personal stocks in the Lapulapu Industries Corp.
capacity. The petitioner Foundation maintained
that it never authorized petitioner Tan to co-
sign in his capacity as its President any Issue: WON petitioner Tans claim regarding the
promissory note and that the petitioner purported unwritten agreement between him
Foundation never benefited, directly or and the respondent Bank on the payment of the
indirectly, therefrom. For his part, petitioner obligation should be upheld.
Tan admitted that he contracted the loans from
the respondent Bank in his personal capacity.
The parties, however, agreed that the loans
were to be paid from the proceeds of petitioner
Tans shares of common stocks in the Lapulapu Held:
Industries Corporation, a real estate firm. The
loans were covered by promissory notes which No.
were automatically renewable (rolled-over)
-The parole evidence rule constrains this Court
every year at an amount including unpaid
to reject petitioner Tans claim regarding the
interests, until such time as petitioner Tan was
purported unwritten agreement between him
able to pay the same from the proceeds of his
and the respondent Bank on the payment of the
aforesaid shares.
obligation. Section 9, Rule 130 of the of the
According to petitioner Tan, the respondent Revised Rules of Court provides that [w]hen the
Banks employee required him to affix two terms of an agreement have been reduced to
signatures on every promissory note, assuring writing, it is to be considered as containing all
him that the loan documents would be filled the terms agreed upon and there can be,
out in accordance with their agreement. between the parties and their successors-in-
However, after he signed and delivered the loan interest, no evidence of such terms other than
documents to the respondent Bank, these were the contents of the written agreement.
filled out in a manner not in accord with their
In this case, the promissory notes are the law
agreement, such that the petitioner Foundation
between the petitioners and the respondent
was included as party thereto. Further, prior to
Bank. These promissory notes contained
its filing of the complaint, the respondent Bank
maturity dates as follows: February 5, 1978,
made no demand on him.
March 28, 1978, April 11, 1978 and May 5,
Applying the parole evidence rule, the CA 1978, respectively. That these notes were to be
likewise rejected petitioner Tans assertion that paid on these dates is clear and explicit.

Rolan Jeff A. Lancion


Arellano University School of Law Page 183
Remedial Law 2- Brondial Notes AY 2017- 2018

Nowhere was it stated therein that they would document under Best Evidence Rule if it is a
be renewed on a year-to-year basis or rolled- printout or output readable by sight or other
over annually until paid from the proceeds of means, shown to reflect the data accurately.
petitioner Tans shares in the Lapulapu
Industries Corp. Accordingly, this purported Necessity of Authentication
unwritten agreement could not be made to vary Q: How does a party authenticate electronic
or contradict the terms and conditions in the evidence?
promissory notes.
A: To authenticate electronic evidence, it must
Evidence of a prior or contemporaneous verbal be made by someone privy to the document. A
agreement is generally not admissible to vary, party who is not privy to the document has no
contradict or defeat the operation of a valid right to authenticate such evidence.
contract. While parole evidence is admissible to
explain the meaning of written contracts, it Q: What is the exception under this rule?
cannot serve the purpose of incorporating into
A: Expert witness
the contract additional contemporaneous
conditions which are not mentioned at all in Example: Text Message- A party can present
writing, unless there has been fraud or expert witness that is working under such
mistake. No such allegation had been made by telecommunication company.
the petitioners in this case.
Situation:
Electronic Evidence (AM No. 01-7-01 SC)
Mr. X texted Mr. Y on January 1, 2017 and this is
Electronic Evidence- information or the content of the text message: “Mr Y, I will kill
representation of information, data figures, you!”
symbols or other modes of written expression
by which a right is established or an obligation -Mr A (son) of Mr Y cannot use such evidence
extinguished, or by which fact may be proved, since he is not privy to the contract.
which is received, recorded, transmitted,
-Only Mr X and Mr. Y can testify on the context
stored, processed, retrieved or produced
of the text message.
electronically.
Rule 4- Best Evidence Rule
Examples:
Sec 1- An electronic document shall be
-Email, recorded or saved text messages,
regarded as the equivalent of an original
computer generated documents, Facebook chat
document under the Best Evidence Rule if it is a
or similar.
printout or output readable by sight or other
Original of an electronic document- It shall be means.
regarded as the equivalent of an original

Rolan Jeff A. Lancion


Arellano University School of Law Page 184
Remedial Law 2- Brondial Notes AY 2017- 2018

Electronic Message vs Electronic Document Marcos Saezs son, Adolfo Saez, the husband of
Gloria Leano Saez, about his problem. Out of
Electronic Document- it consist of electronic pity and for humanitarian consideration, Adolfo
data or message that creates RIGHTS or allowed Francisco Comorposa to occupy the
EXTINGUISHES OBLIGATION. land of Marcos Saez. Hence, his nipa hut was
carried by his neighbors and transferred to a
Electronic Signature- It shall be considered as portion of the land subject matter of this
admissible in evidence as the functional case. Such transfer was witnessed by several
equivalent of the signature of a person on a people, among them, Gloria Leano and Noel
Oboza. Francisco Comorposa occupied a portion
written document.
of Marcos Saez property without paying any
Q: How do you authenticate electronic rental.
signature? - Francisco Comorposa left for Hawaii, U.S.A. He
was succeeded in his possession by the
A: -Method or process was utilized to establish respondents who likewise did not pay any
a digital signature rental and are occupying the premises through
petitioners tolerance.
-Other means provided by law
- On 7 May 1998, a formal demand was made
-Other means satisfactory to the judge upon the respondents to vacate the premises
establishing the genuineness of electronic but the latter refused to vacate the same and
signature. claimed that they were the legitimate claimants
and the actual and lawful possessor[s] of the
premises. A complaint was filed with the
barangay office of Sta. Cruz[,] Davao del Sur,
but the parties failed to arrive at an amicable
settlement. Thus, the corresponding Certificate
Case: Heirs of Sabanpan vs Comorposa, 408 to File Action was issued by the said barangay
SCRA and an action for unlawful detainer was filed by
petitioners against respondents.
- The complaint alleged that Marcos Saez was
the lawful and actual possessor of Lot No. 845, - Respondents, in their Answer, denied the
Land 275 located at Darong, Sta. Cruz, Davao material allegations of the complaint and
del Sur with an area of 1.2 hectares. In 1960, he alleged that they entered and occupied the
died leaving all his heirs, his children and premises in their own right as true, valid and
grandchildren. lawful claimants, possessors and owners of the
said lot way back in 1960 and up to the present
- In 1965, Francisco Comorposa who was time; that they have acquired just and valid
working in the land of Oboza was terminated ownership and possession of the premises by
from his job. The termination of his ordinary or extraordinary prescription, and that
employment caused a problem in relocating his the Regional Director of the DENR, Region XI
house. Being a close family friend of Marcos has already upheld their possession over the
Saez, Francisco Comorposa approached the late land in question when it ruled that they were

Rolan Jeff A. Lancion


Arellano University School of Law Page 185
Remedial Law 2- Brondial Notes AY 2017- 2018

the rightful claimants and possessors and, Held:


therefore, entitled to the issuance of a title.
-If the Certification were a sham as petitioner
- The Municipal Trial Court of Sta. Cruz, Davao claims, then the regional director would not
del Sur rendered judgment in favor of have used it as reference in his Order. Instead,
petitioners but the Regional Trial Court of Digos, he would have either verified it or directed the
Davao del Sur, on appeal, reversed and set CENR officer to take the appropriate action, as
aside the said decision. the latter was under the formers direct control
and supervision.
- Affirming the Regional Trial Court (RTC), the
CA upheld the right of respondents as claimants - Petitioners claim that the Certification was
and possessors. The appellate court held that -- raised for the first time on appeal is incorrect.
although not yet final -- the Order issued by the AS early as the pretrial conference at the
regional executive director of the Department Municipal Trial Court (MTC), the CENR
of Environment and Natural Resources (DENR) Certification had already been marked as
remained in full force and effect, unless evidence for respondents as stated in the Pre-
declared null and void. The CA added that the trial Order. The Certification was not formally
Certification issued by the DENRs community offered, however, because respondents had not
environment and natural resources (CENR) been able to file their position paper.
officer was proof that when the cadastral
survey was conducted, the land was still -Neither the rules of procedure nor
alienable and was not yet allocated to any jurisprudence would sanction the admission of
person. According to the CA, respondents had evidence that has not been formally offered
the better right to possess alienable and during the trial. But this evidentiary rule is
disposable land of the public domain, because applicable only to ordinary trials, not to cases
they have suffiently proven their actual, covered by the rule on summary procedure --
physical, open, notorious, exclusive, continuous cases in which no full-blown trial is held.
and uninterrupted possession thereof since
-The admissibility of evidence should not be
1960. The appellate court deemed as self-
confused with its probative value.Admissibility
serving, and therefore incredible, the Affidavits
refers to the question of whether certain pieces
executed by Gloria Leano Saez, Noel Oboza and
of evidence are to be considered at all, while
Paulina Paran.
probative value refers to the question of
whether the admitted evidence proves an
issue. Thus, a particular item of evidence may
ISSUE: WON the Court of Appeals gravely abuse be admissible, but its evidentiary weight
its discretion and err in sustaining the Regional depends on judicial evaluation within the
Trial Courts ruling giving weight to the CENR guidelines provided by the rules of evidence.
Officers Certification, which only bears the
facsimile of the alleged signature of a certain -While in summary proceedings affidavits are
Jose F. Tagorda and, it is a new matter raised admissible as the witnesses respective
for the first time on appeal? testimonies, the failure of the adverse party to
reply does not ipso facto render the facts, set
forth therein, duly proven. Petitioners still

Rolan Jeff A. Lancion


Arellano University School of Law Page 186
Remedial Law 2- Brondial Notes AY 2017- 2018

bear the burden of proving their cause of


action, because they are the ones asserting an
affirmative relief. Issue: WON the facsimile sent by the petitioner
should be admissible as evidence under our law

Case: Torres vs PAGCOR, 661 SCRA Held:

- Petitioner was a Slot Machine Operations NO


Supervisor (SMOS) of respondent Philippine -Petitioner claims that that the sending of
Amusement and Gaming Corporation
documents thru electronic data message,
(PAGCOR).
which includes facsimile, is sanctioned under
- On the basis of an alleged intelligence report, Republic Act No. 8792, the Electronic
PAGCOR's Corporate Investigation Unit (CIU) Commerce Act of 2000.
allegedly conducted an investigation to verify
the veracity of such report. -A facsimile is not a genuine and authentic
- The CIU served petitioner with a pleading. It is, at best, an exact copy
Memorandum of Charges for dishonesty, preserving all the marks of an original.
serious misconduct, fraud and violation of office Without the original, there is no way of
rules and regulations.
determining on its face whether the facsimile
- On August 4, 2007, petitioner received a letter pleading is genuine and authentic and was
dated August 2, 2007 from Atty. Lizette F. originally signed by the party and his counsel.
Mortel, Managing Head of PAGCOR's Human It may, in fact, be a sham pleading.
Resource and Development Department,
dismissing him from the service. - A facsimile transmission is not considered as
- On September 14, 2007, petitioner filed with an electronic evidence under the Electronic
the CSC a Complaint against PAGCOR and its Commerce Act.
Chairman Efraim Genuino for illegal dismissal,
non-payment of backwages and other benefits.
Case: Ang vs Republic, 618 SCRA
- CSC denied the complaint filed by petitioner
Torres on the ground of Dishonesty, Gross Facts: Irish Sagud (Irish) and Rustan were
Misconduct, Serious Violation of Office Rules classmates at Wesleyan University in Aurora
and Regulations, Conduct Prejudicial to the Best Province. Rustan courted Irish and they became
Interest of the Service and Loss of Trust and "on-and-off" sweethearts towards the end of
Confidence. 2004. When Irish learned afterwards that
Rustan had taken a live-in partner (now his
- CA affirmed the decision issued by the CSC on
the ground that the petition lacks merit.

Rolan Jeff A. Lancion


Arellano University School of Law Page 187
Remedial Law 2- Brondial Notes AY 2017- 2018

wife), whom he had gotten pregnant, Irish Aurora who referred her to the police. Under
broke up with him. police supervision, Irish contacted Rustan
through the cellphone numbers he used in
Before Rustan got married, however, he got in sending the picture and his text messages. Irish
touch with Irish and tried to convince her to asked Rustan to meet her at the Lorentess
elope with him, saying that he did not love the Resort in Brgy. Ramada, Maria Aurora, and he
woman he was about to marry. Irish rejected did. He came in a motorcycle. After parking it,
the proposal and told Rustan to take on his he walked towards Irish but the waiting police
responsibility to the other woman and their officers intercepted and arrested him. They
child. Irish changed her cellphone number but searched him and seized his Sony Ericsson P900
Rustan somehow managed to get hold of it and cellphone and several SIM cards. While Rustan
sent her text messages. Rustan used two was being questioned at the police station, he
cellphone numbers for sending his messages, shouted at Irish: "Malandi ka kasi!"
namely, 0920-4769301 and 0921-8084768. Irish
replied to his text messages but it was to ask Issue: WON accused Rustan sent Irish by
him to leave her alone. cellphone message the picture with her face
pasted on the body of a nude woman, inflicting
In the early morning of June 5, 2005, Irish anguish, psychological distress, and humiliation
received through multimedia message service on her in violation of Section 5(h) of R.A. 9262.
(MMS) a picture of a naked woman with spread
legs and with Irish’s face superimposed on the Held:
figure (Exhibit A). The sender’s cellphone
Rustan argues that, since he was arrested and
number, stated in the message, was 0921-
certain items were seized from him without any
8084768, one of the numbers that Rustan used.
warrant, the evidence presented against him
Irish surmised that he copied the picture of her
should be deemed inadmissible. But the fact is
face from a shot he took when they were in
that the prosecution did not present in
Baguio in 2003 (Exhibit B).
evidence either the cellphone or the SIM cards
that the police officers seized from him at the
After she got the obscene picture, Irish got
time of his arrest. The prosecution did not need
other text messages from Rustan. He boasted
such items to prove its case. Exhibit C for the
that it would be easy for him to create similarly
prosecution was but a photograph depicting the
scandalous pictures of her. And he threatened
Sony Ericsson P900 cellphone that was used,
to spread the picture he sent through the
which cellphone Rustan admitted owning
internet. One of the messages he sent to Irish,
during the pre-trial conference.
written in text messaging shorthand, read:
"Madali lang ikalat yun, my chatrum ang tarlac
Though, the bulk of the evidence against him
rayt pwede ring send sa lahat ng chatter." Irish
consisted in Irish’s testimony that she received
sought the help of the vice mayor of Maria

Rolan Jeff A. Lancion


Arellano University School of Law Page 188
Remedial Law 2- Brondial Notes AY 2017- 2018

the obscene picture and malicious text able to escape. The policemen became
messages that the sender’s cellphone numbers suspicious of Enojas’ involvement in the
belonged to Rustan with whom she had been robbery and monitored his mobile phone which
previously in communication. Indeed, to prove he inadvertently left in the patrol car. The police
that the cellphone numbers belonged to conducted an entrapment, posing as Enojas in
Rustan, Irish and the police used such numbers communicating with the other accused. Enojas
to summon him to come to Lorentess Resort and another accused Gomez were arrested. The
and he did. Consequently, the prosecution did prosecution presented the transcripts of the
not have to present the confiscated cellphone mobile phone text messages between Enojas
and SIM cards to prove that Rustan sent those and some of his co-accused. The accused
messages. asserted that they were entitled to an acquittal
because they were illegally arrested and the
Moreover, Rustan admitted having sent the evidence of the text messages were
malicious text messages to Irish. His defense inadmissible, not having been properly
was that he himself received those messages identified.
from an unidentified person who was harassing
Irish and he merely forwarded the same to her, Issue: WON the text messages were admissible
using his cellphone. But Rustan never presented as evidence against the accused
the cellphone number of the unidentified
Held:
person who sent the messages to him to
authenticate the same. The RTC did not give YES- Text messages are admissible according to
credence to such version and neither will this the Rules on Electronic Evidence. Text
Court. Besides, it was most unlikely for Irish to messages are to be proved by the testimony of
pin the things on Rustan if he had merely tried a person who was a party to the same or has
to help her identify the sender. personal knowledge of them. The text
messages to and from the mobile phone left at
Case: People vs Enojas, GR No. 204894, March the scene by accused Enojas provided strong
10, 2014 leads on the participation and identities of the
accused. Indeed, the police caught them in an
Facts:
entrapment using this knowledge.
-Enojas was riding a taxi parked in front of
Aguila Auto Glass shop. He was approached by
patrolling policemen who found the taxi Testimonial Evidence
suspicious. He was asked to come with them to
the police station. However, on the way to the Section 20. Witnesses; their qualifications. —
police station, they encountered robbers. Except as provided in the next succeeding
During suchencounter with robbers, Enojas was section, all persons who can perceive, and

Rolan Jeff A. Lancion


Arellano University School of Law Page 189
Remedial Law 2- Brondial Notes AY 2017- 2018

perceiving, can make their known perception to incapable of perceiving the facts which they are
others, may be witnesses. examined and of relating the truthfully
disqualified to be a witness.
Religious or political belief, interest in the
outcome of the case, or conviction of a crime Q: Who is a child witness?
unless otherwise provided by law, shall not be
ground for disqualification. A: One who at the time of giving testimony is
below 18 years.
Requirements:
Section 21. Disqualification by reason of
-A person that can perceive- Ability to observe mental incapacity or immaturity. — The
and remember following persons cannot be witnesses:

- A person that can make known of his (a) Those whose mental condition, at the time
perception- Ability to communicate what he of their production for examination, is such that
perceived they are incapable of intelligently making
known their perception to others;
Q: Who are disqualified to testify in open court?
(b) Children whose mental maturity is such as to
A: Sec 21- Mental Incapacity or immaturity render them incapable of perceiving the facts
respecting which they are examined and of
Sec 22- Reason of Marriage relating them truthfully.

Sec 23- Reason of death or insanity of adverse -Children are presumed to be competent
party witness.

Sec 24- Privileged Communication Basis: Rule on the Examination of Child Witness

Q: May an insane person qualify as a witness? -If a party wants to assail the competency of
child witness, he shall have the burden of proof.
A: Yes, provided that he is capable or
intelligently making known his perception to Competency Test- Regular observation through
others. His insanity would affect the weight of searching question of the judge to simply
his testimony. establish whether the child is competent.

Q: Is a child qualified to be a witness? -The judge must determine if the child knows
the difference between truth and falsity. There
A: YES, every child is presumed qualified to be a is No Hard and Fast Rule.
witness. The court may conduct a competency
examination if substantial doubts exists as to -The Test must be done to the exclusion of the
the child’s competency as to the child’s public.
competency to testify. EXCEPTION- a child
whose mental capacity is such as to render him Q: What is Live-Link testimony?

Rolan Jeff A. Lancion


Arellano University School of Law Page 190
Remedial Law 2- Brondial Notes AY 2017- 2018

A: A child witness is excluded from the general A: The rule provides that during their marriage,
public in giving his testimony. a spouse may not testify for or against the other
spouse without the latter’s consent.
Q: What are testimonial aids?
Q: What are the exceptions to the marital
A: Object/Person that will help a child disqualification rule?
(psychological/emotional) in the course of
testimony. A: A spouse may testify against the other in:

Support Persons- Natural Persons that will -Civil case by one against the other
accompany the child in the course of his
testimony. ( Mother, Guardian, Guardian Ad -Criminal case for a crime committed by one
Litem) spouse against the other or the latter’s direct
descendants or ascendants.
Q: Who is a facilitator?
Case: Marcos vs Heirs of Andres Navarro, 700
A: A facilitator is one that propounds the SCRA
question to the child given by the parties
concerned. Facts:

Q: What are the question that are barred to ask Spouses Navarro passed away and left behind
towards a child witness? several parcels of land. They were survived by
their daughters, Luisa Navarro Marcos
A: -Sexual propensity of the child (petitioner) and Lydia Navarro Grageda, and the
heirs of their only son, Andres Navarro. The
-Sexual encounters of the child heirs of Andres are the respondents in this case.

Petitioner and her sister Lydia discovered that


Exception: If a party will establish that the
respondents are claiming exclusive ownership
seamen found in the vagina of the victim, he
of the lot based on an Affidavit of Transfer of
can ask specific sexual encounter of the child.
Real Property where Andres, Sr. donated the lot
to Andres, Jr. The sisters requested a
Section 22. Disqualification by reason of
handwriting examination of the affidavit. The
marriage. — During their marriage, neither the
PNP handwriting expert, PO2 Mary Grace
husband nor the wife may testify for or against
Alvarez, found that Andres, Sr.’s signature on
the other without the consent of the affected
the affidavit and the submitted standard
spouse, EXCEPT in a civil case by one against
signatures of Andres, Sr.were not written by the
the other, or in a criminal case for a crime
same person.
committed by one against the other or the
latter's direct descendants or ascendants.
The sisters sued the respondents for annulment
of the deed of donation. Respondents moved to
Q: What is marital disqualification rule?
disqualify PO2 Alvarez as a witness.

Rolan Jeff A. Lancion


Arellano University School of Law Page 191
Remedial Law 2- Brondial Notes AY 2017- 2018

RTC granted respondents’ motion and Under Section 49 of Rule 130, PO2 Alvarez is
disqualified Alvarez as a witness, ruling that her allowed to render an expert opinion.
supposed testimony would be hearsay as she
has no personal knowledge of the alleged Although the word “may” in Sec. 49 of Rule 130
handwriting of Andres, Sr. Also, there is no need signifies that the use of the opinion of an expert
for PO2 Alvarez to be presented, if she is to be witness is permissive and not mandatory on the
presented as an expert witness, because her part of the courts, handwriting experts are
testimony is not yet needed. CA dismissed the often offered as expert witnesses considering
sisters’ petition for certiorari. the technical nature of the procedure in
ISSUE: WON PO2 Alvarez should be disqualified examining forged documents.
as a witness
RTC shouldn’t have disqualified Alvarez as a
HELD: witness. She has the qualifications and none of
the disqualifications of a witness under the
No. Section 21 disqualifies a witness by reason Rules. The analysis of the signature in the deed
of mental incapacity or immaturity. Section 22 of donation executed by the late Andres, Sr. is
disqualifies a witness by reason of marriage. crucial to the resolution of the case.
Section 23 disqualifies a witness by reason of
Case: People vs Golimlim, 427 SCRA
death or insanity of the adverse party. Section
People vs Golimlim, 427 SCRA
24 disqualifies a witness by reason of privileged
communication. Facts:

The specific enumeration of disqualified


witnesses excludes the operation of causes of
- Salvador Golimlim (“Accused”) was charged
disability other than those mentioned in the
for the rape of Evelyn Canchela (“Victim”), a
Rules. The Rules should not be interpreted to
mental retardate, sometime in August 1996
include an exception not embodied therein.
while the Victim was placed in the Accused’s
custody by her mother who in turn left for
As a handwriting expert of the PNP, PO2 Alvarez
abroad to work as a domestic helper. Accused
can surely perceive and make known her
was the husband of the mother’s sister.
perception to others. Respondents’ motion to
disqualify her should have been denied by the
RTC for it was not based on any of these - Victim was able to file a complaint against the
grounds for disqualification. The RTC confused Accused only after several months. Accused
the qualification of the witness with the merely denied the allegations of the Victim and
credibility and weight of her testimony. blamed her mental incapacity in casting doubt
in the accusation against him, stating that she
Moreover, Section 49, Rule 130 of the Rules of mentioned many other names of men who had
Evidence is clear that the opinion of an expert intercourse with her. RTC ruled for conviction,
witness may be received in evidence. PO2 hence the instant appeal.
Alvarez’s testimony would not be hearsay.

Rolan Jeff A. Lancion


Arellano University School of Law Page 192
Remedial Law 2- Brondial Notes AY 2017- 2018

Issue: WON Evelyn Canchela, being a mental (a) The husband or the wife, during or after the
retardate, is disqualified from testifying as marriage, cannot be examined without the
witness to the case. consent of the other as to any communication
received in confidence by one from the other
Held: during the marriage except in a civil case by one
against the other, or in a criminal case for a
No. Evelyn Canchela is still qualified as witness crime committed by one against the other or
and may testify accordingly. the latter's direct descendants or ascendants.

- Mental retardation does not equate/translate Q: What us the marital communication


to the incapacity contemplated in Sections 20 & privilege?
21 of Rule 130. Thus, the Court cited the
definition of a mental retardate: A: It is the privilege which provides that the
“A mental retardate or a feebleminded husband or wife cannot be examined without
person is not, per se, disqualified from the consent of the other as to any confidential
being a witness, her mental condition communication received from the other during
the marriage except in a civil case by one
not being a vitiation of her credibility. It
against the other or in criminal case for a crime
is now universally accepted that committed by one against the other or the
intellectual weakness, no matter what latter’s direct as ascendants or descendants.
form it assumes, is not a valid objection
to the competency of a witness so long Marital Marital
as the latter can still give a fairly Disqualification Rule Communication
intelligent and reasonable narrative of Privilege
the matter testified to.” Blanket The privilege applies
disqualification of a only to confidential
- The Victim was fully capable of making her spouse from testifying communications
perceptions known to others since she was able for or against the
to respond to various questions framed in other
different forms so as to have her narrate the Effective only during Effective during and
whole incident and still give coherent facts. the existence of the after the marriage
Further, mere denial of the Accused is a weak marriage
defense and could not controvert the positive Can be invoked only if It may be invoked
testimony of the Victim herself. one of the spouses is a whether or not a
party to the action spouse is a party to
the action
Section 24. Disqualification by reason of
privileged communication. — The following AS TO PERIOD OF EFFECTIVITY- The marital
persons cannot testify as to matters learned in disqualification rule is effective only during the
confidence in the following cases: existence of the marriage while the marital
communication rule subsists even after the
termination of the marriage.

Rolan Jeff A. Lancion


Arellano University School of Law Page 193
Remedial Law 2- Brondial Notes AY 2017- 2018

AS TO SCOPE- The marital disqualification rule against him pursuant to Rule 130 of the Revised
is a total prohibition against any testimony by Rules of Court on marital disqualification.
one spouse for or against the other, while the
marital communication privilege applies only Respondent filed an opposition to the motion.
confidential communication between the Pending resolution of the motion, the trial court
spouses. directed the prosecution to proceed with the
presentation of the other witnesses. On
AS TO SPOUSE BEING A PARTY TO THE ACTION-
September 2, 1999, the trial court issued the
The marital disqualification rule can be invoked
only if one of the spouses is a party to the questioned Order disqualifying Esperanza
action, while the marital communication Alvarez from further testifying and deleting her
privilege may be invoked whether or not a testimony from the records. The prosecution
spouse is a party to the action. filed a motion for reconsideration but was
denied in the other assailed Order dated
Q: If a confidential communication by one
October 19, 1999. This prompted respondent
spouse to the other comes into the hands of a
to file with the Court of Appeals a petition for
third person, may a third person testify
thereon? certiorari with application for preliminary
injunction and temporary restraining order. On
A: YES, where the privileged communication May 31, 2000, the Appellate Court rendered a
comes into the hands of a third person, the Decision nullifying and setting aside the assailed
marital communications privilege does not Orders issued by the trial court. Hence, this
apply unless the aggrieved spouse shows that
petition for review on certiorari.
there was collusion between his spouse and
third person. ISSUE: WON Esperanza can testify over the
objection of her estranged husband on the
Case: Alvarez vs Ramirez, 473 SCRA
ground of marital privilege.
Doctrine: If the spouses are separated in fact,
HELD:
the marital disqualification rule shall not apply.
Yes, Esperanza may testify over the objection of
Facts:
her husband. The disqualification of a witness
Respondent Susan Ramirez was the complaining by reason of marriage under Sec. 22, Rule 130
witness in a criminal case or arson pending of the Revised Rules of Court has its exceptions
before the RTC. The accused was petitioner as where the marital relations are so strained
Maximo Alvarez, stranged husband of that there is no more harmony to be preserved.
Esperanza Alvarez, sister of respondent. On The acts of the petitioner stamp out all major
June 21, 1999, Esperanza Alvarez was called to aspects of marital life. On the other hand, the
the witness stand as the first witness against State has an interest in punishing the guilty
petitioner, her husband. Petitioner filed a and exonerating the innocent, and must have
motion to disqualify Esperanza from testifying the right to offer the testimony of Esperanza

Rolan Jeff A. Lancion


Arellano University School of Law Page 194
Remedial Law 2- Brondial Notes AY 2017- 2018

over the objection of her husband. The Q: Will the disqualification rule extend to the
purpose of marital disqualification rule shall children in the dinner?
not apply on the reason that: commonality of
interest no longer holds, their marriage is no A: YES, since the communication was given in
trust and confidence and the children belongs
longer protected since they are already
to the family.
separated in fact.
B. Attorney-Client Relationship

Case: People vs Judge Castaneda, 88 SCRA (b) An attorney cannot, without the consent of
his client, be examined as to any
Facts: A spouse owned a parcel of land and the communication made by the client to him, or
his advice given thereon in the course of, or
husband sell the parcel of land without the
with a view to, professional employment, nor
consent of his wife. The husband forged the
can an attorney's secretary, stenographer, or
signature of the wife on the marital conformity clerk be examined, without the consent of the
of the sale of the land. During the trial for the client and his employer, concerning any fact the
case of falsification, the wife is called to take knowledge of which has been acquired in such
the witness stand to testify but the adverse capacity
party objected on the ground of marital
-The client is being protected by this privilege
disqualification and it was sustained by Judge
communication in order for him to divulge
Castaneda.
certain information without uncertainty.
Held: The marriage is still subsisting and the
Rationale: To Protect the Legal Profession
spouses are not separated in fact. Considering
that the property involved is a conjugal -The privilege communication only covers legal
property and there is falsification of the matter which is the subject matter of the
signature, the marital disqualification shall not lawyer-client relationship.
be applicable.
-The communication also extends to its
-When a father commits a crime against the secretary, stenographer or clerk.
daughter, it is in effect a crime against mother
therefore the latter can testify against the Q: May the attorney himself be the one to
father. invoke the privilege?

Marital Privilege Communication Rule- It was A: YES, either the client or his lawyer may raise
intended not to be known by a Third Person or such privilege.
other people.
-The attorney-client privilege bars only the
Example: One evening during a family dinner, attorney, secretary, stenographer or clerk from
Father X confessed to his family that he killed A. being examined but the privilege does not
extend to third persons who obtain knowledge

Rolan Jeff A. Lancion


Arellano University School of Law Page 195
Remedial Law 2- Brondial Notes AY 2017- 2018

of the communication between the lawyer and -It is only applicable in Catholic Religion.
client.
E. Public interest privilege
C. Doctor-Client Relationship
(e) A public officer cannot be examined during
(c) A person authorized to practice medicine, his term of office or afterwards, as to
surgery or obstetrics cannot in a civil case, communications made to him in official
without the consent of the patient, be confidence, when the court finds that the public
examined as to any advice or treatment given interest would suffer by the disclosure.
by him or any information which he may have
acquired in attending such patient in a -It pertains to custodial of public records or
professional capacity, which information was communications
necessary to enable him to act in capacity, and
which would blacken the reputation of the -A public officer cannot be examined during his
patient. term of office or afterwards, as to
communication made to him in official
-A doctor cannot without the consent of the confidence and if the court finds that public
client be examined on communication or interest would be prejudiced.
information disclose in the course of medical
services. Case: Chan vs Chan, 702 SCRA

- It is applicable in civil or administrative cases Facts: Josielene filed a petition for the
declaration of nullity of her marriage to
Q: Are autopsies conducted by medico-legal respondent Johnny Chan (Johnny), the
officer covered by the privilege?
dissolution of their conjugal partnership of
gains, and the award of custody of their
A: NO, since there is patient or treatment
involved, the autopsy having been conducted children to her.
on a dead person.
During the pre-trial conference, Josielene pre-
D. PRIEST-CONFESSION RELATIONSHIP marked the Philhealth Claim Form1 that Johnny
attached to his answer as proof that he was
(d) A minister or priest cannot, without the forcibly confined at the rehabilitation unit of a
consent of the person making the confession, hospital. The form carried a physician’s
be examined as to any confession made to or handwritten note that Johnny suffered from
any advice given by him in his professional
“methamphetamine and alcohol abuse.”
character in the course of discipline enjoined by
the church to which the minister or priest Following up on this point, Josielene filed with
belongs. the RTC a request for the issuance of a
subpoena duces tecum addressed to Medical
-A minister or priest cannot, without the City, covering Johnny’s medical records when
confessant’s consent be examined as to any he was there confined. The request was
confession made to or advice.

Rolan Jeff A. Lancion


Arellano University School of Law Page 196
Remedial Law 2- Brondial Notes AY 2017- 2018

accompanied by a motion to “be allowed to communications made between a patient and


submit in evidence” the records sought by his doctor. Hence, the physician-patient
subpoena duces tecum. RTC and CA denied privilege does not cover information discovered
Josielene’s motion. under Rule 28. This procedure is availed with
the intention of making the results public during
ISSUE: WON the CA erred in ruling that the trial trial. Along with other modes of discovery, this
court correctly denied the issuance of a would prevent the trial from being carried on in
subpoena duces tecum covering Johnny’s the dark.
hospital records on the ground that these are
covered by the privileged character of the Case: CLARITA J. SAMALA vs. ATTY. LUCIANO
physician-patient communication. D. VALENCIA
A.C. No. 5439, January 22, 2007
HELD: NO. Issuance of a subpoena duces tecum
for its production will violate the physician- FACTS: Complainant Clarita J. Samala filed
patient privilege rule under Rule 130, Sec. 24(c) against Atty. Luciano D. Valencia for
of the Rules of Civil Procedure. However, this Disbarment. One of the grounds for disbarment
privilege is not absolute. The request of was the act of Atty. Valencia of serving on two
petitioner for a copy of the medical records has separate occasions as counsel for contending
not been properly laid. Instead of a request for parties. Atty. Valencia then filed his Comment.
the issuance of a subpoena duces tecum,
After which, the Court referred the case to the
Josielene Lara Chan should avail of the mode of Integrated Bar of the Philippines for
discovery under Rule 28 of Rules of Civil investigation, report and recommendation.
Procedure. Rule 28 pertains to the physical or After a series of hearings, the parties filed their
mental examination of persons. This may be respective memoranda and the case was
ordered by the court, in its discretion, upon deemed submitted for resolution. The
motion and showing of good cause by the Commissioner found respondent guilty of
requesting party, in cases when the mental violating Canons 15 and 21 of the Code of
and/or physical condition of a party is in Professional Responsibility. The IBP Board of
controversy. Aside from showing good cause,
Governors adopted and approved the report
the requesting party needs only to notify the and recommendation of Commissioner Reyes.
party to be examined (and all other parties) and
specify the time, place, manner, conditions, and ISSUE: WON Atty. Valencia violated the
scope of the examination, including the name of privilege communication by serving as counsel
the physician who will conduct the examination. for contending parties
RULING: Yes. Records show that Atty. Valencia
Discovery procedures provide a balance serves as a counsel for contending parties. The
between the need of the plaintiff or claimant to Presiding Judge even warned him to refrain
fully and fairly establish her case and the policy
to protect ― to a certain extent ―

Rolan Jeff A. Lancion


Arellano University School of Law Page 197
Remedial Law 2- Brondial Notes AY 2017- 2018

from repeating the act of being counsel of the previous litigation in which he appeared for
record of both parties. the former client.

Rule 15.03, Canon 15 of the Code of Executive Privilege


Professional Responsibility provides that a
lawyer shall not represent conflicting interests Q: What is executive privilege?
except by written consent of all concerned
A: It is the privilege which protects the
given after a full disclosure of the facts. A
confidentiality of conversation that takes place
lawyer may not, without being guilty of in the president’s performance of official duty.
professional misconduct, act as counsel for a
person whose interest conflicts with that of his Case: Almonte vs Vasquez, 244 SCRA
present or former client. He may not also
undertake to discharge conflicting duties any Facts: General Almonte was appointed by
more than he may represent antagonistic President Ramos as the head of EEIP. Almonte
was later on asked to divulge certain
interests.
information which is of national interest but he
invoke that if he disclosed such information, it
The stern rule against representation of will be a threat to national security and safety.
conflicting interests is founded on principles of
public policy and good taste. It springs from the Held: The manifestation made by General
attorney's duty to represent his client with Almonte is valid on the ground of executive
undivided fidelity and to maintain inviolate the privilege since it involves national security,
client's confidence as well as from the diplomatic matter and national safety.
injunction forbidding the examination of an
Case Doctrine: Neri vs Senate Committee “The
attorney as to any of the privileged president of the Republic of the Philippines
communications of his client. An attorney owes cannot be called into a congressional hearing
loyalty to his client not only in the case in which and the executive privilege given to the
he has represented him but also after the President is extended to its alter-ego”
relation of attorney and client has
terminated. The bare attorney-client Section 23. Disqualification by reason of death
or insanity of adverse party. — Parties or
relationship with a client precludes an attorney
assignor of parties to a case, or persons in
from accepting professional employment from whose behalf a case is prosecuted, against an
the client's adversary either in the same case or executor or administrator or other
in a different but related action. A lawyer is representative of a deceased person, or against
forbidden from representing a subsequent a person of unsound mind, upon a claim or
client against a former client when the subject demand against the estate of such deceased
matter of the present controversy is related, person or against such person of unsound mind,
cannot testify as to any matter of fact occurring
directly or indirectly, to the subject matter of

Rolan Jeff A. Lancion


Arellano University School of Law Page 198
Remedial Law 2- Brondial Notes AY 2017- 2018

before the death of such deceased person or Admissions and Confessions


before such person became of unsound mind.
Admission- A statement or confirmation about
Deadman’s Statute- It is for the protection of a fact.
the estate.
Confession- More than a statement of fact
Q: Who are disqualified to testify? which leads to an acknowledgment of guilt.

A: Plaintiff or assignor of plaintiff Two Types of Confession

Q: Against whom they are prohibited to testify? Judicial- The plea of guilty of an accused
pending trial in criminal cases.
A: Executor or Administrator of the estate on
claim against the estate or ante lite motam- Extra-judicial-
before the occurrence of the event.
Requirements: -To establish voluntariness of
PAP-EAR the accused

-If the case is against the executor or -it should be within the time frame of custodial
administrator of the estate, it is not a claim interrogation.
against the estate.
*If the confession is without the assistance of
-The claim must be on matter before the death lawyer, it shall be inadmissible as evidence
of the party.
-It must be in writing
Claim against the estate- The liability must not
be personal to the executor/administrator but Extra-Judicial Judicial Admission
chargeable to the estate. Admission
Made outside of the Made in the course of
Rationale: To avoid Perjury- When death closes proceeding in the the proceeding in the
the lip of someone, the law must come to the same case same case
other and close his lip as well. Must be offered in Need not be offered in
evidence in order to evidence since they
Case Doctrine: Sunga-Chan vs Chan “The dead’s considered by the already form part of
man statute will no longer apply since the court the records.
executor on the counter-claim is the plaintiff” May be contradicted May not be
by the admitter contradicted by the
Section 25. Parental and filial privilege. — No admitter except by
person may be compelled to testify against his showing that the
parents, other direct ascendants, children or admission was made
other direct descendants. through palpable
mistake or that no
-Applicable in civil and criminal cases

Rolan Jeff A. Lancion


Arellano University School of Law Page 199
Remedial Law 2- Brondial Notes AY 2017- 2018

such admission was A: Accused’s silence when his wife’s nephew


made. asked him why he killed his is an admission by
silence.
Q: What are the forms which ad admission may
take? Doctrine of adoptive admission- it is where a
party, by his words or conduct, voluntarily
A: An admission may be in the form of an act, a adopts or ratifies another’s statement. Evidence
declaration or statement, or an omission. of the statement would then be admissible
against the party.
Q: Give examples of acts as admission
Res inter alios acta rule- The rights of a party
A: (1) Voluntary participation in the police re- cannot be prejudiced by an act declaration or
enactment, (2) Flight from the scene of the omission of another.
crime, (3) Changing one’s appearance or name.
(4) Transfer of property. Exceptions:

Q: May an offer of compromise be considered 1. Co-partner


as admission of liability by the offeror?
2. Agent
A: Civil Case- an offer of compromise is not an
admission of liability and it not admissible in 3. Joint owner, joint debtor or other person
evidence against the offeror. interested with the party

Criminal Case- an offer of compromise by the 4. Co-conspirator


accused may be received in evidence as an
admission of guilt. 5. Privies

-A plea of guilty later withdrawn or an Rule on admission by a co-conspirator- The act


unaccepted offer of a guilty plea to a lesser or declaration of a conspirator relating to the
offense, as in plea bargaining, is not admissible conspiracy and during its existence, may be
in evidence against the accused who made the given in evidence against the co-conspirator
plea or offer. after the conspiracy is shown by evidence other
than such act or declaration.
Q: What is the rule on admission by silence?
Q: May an extrajudicial confession be given in
A: Any act or declaration made in the presence evidence against a person other than the
and within the hearing or observation of a party confessant?
who does or says nothing.
A: No on the ground that a person may not be
Q: Give an example of admission by silence? bound by the act or declaration of another. It
cannot be admitted against his or her co-
accused an it considered against them. The

Rolan Jeff A. Lancion


Arellano University School of Law Page 200
Remedial Law 2- Brondial Notes AY 2017- 2018

exception is when the confession is used as “Pagmamana sa Labas ng Hukuman” dated 10


corroborative or circumstantial evidence. August 1992.

Q: What is an interlocking confession?


-A pre-trial conference was conducted wherein
the parties entered into stipulations and
A: There are several confessant. Extra judicial
admissions as well as identification of the issues
confession made without collusion which are to be litigated.
identical with each other in their essential
details and corroborated by other evidence on -RTC – As a result of “Extrajudicial Settlement
record are admissible as circumstantial with Waiver” executed by the heirs of Pedro
evidence. Constantino Jr. and the subsequent execution
of another deed denominated as “Pagmamana
Q: May an extrajudicial confession made by an sa Labas ng Hukuman” executed by the heirs of
accused be sufficient ground for conviction? Santiago and Bruno Constantino (sons of Pedro
Sr.) to the exclusion of the other heirs, both
A: NO, unless it is corroborated by evidence of plaintiffs and defendants acted equally at fault.
corpus delicti. They are in pari declito.

Case: Constantino vs Heirs of Constantino Jr, -CA – in favor or the respondent heirs of Pedro
Jr., declaring that the “Extrajudicial Settlement
706 SCRA with Waiver” covering the 192 sq. m lot actually
belongs to Pedro Jr., hence, not part of the
Facts: estate of Pedro Sr. It is erroneous for the trial
court to declare the parties in pari delicto.
-This involves a controversy over a parcel of
land claimed to be part of an estate, which
ISSUE: WON the CA committed error by
needed to be proportionally subdivided among
disregarding the stipulations and admissions
heirs.
during the pre-trail conference on which the
-Pedro Constantino, Sr. ancestor of the application of the doctrine of in pari delicto was
petitioners and respondents, owned several based?
parcels of land one of which is an unregistered
parcel of land consisting of 240 sq. meters
situated at Sta. Monica, Hagonoy, Bulacan. HELD:
-Upon his death, he was survived by six (6) YES
children.
-On 17 June 1999, respondents Asuncion -The CA actually contradicted the admissions
Laquindanum (Asuncion) and Josefina Cailipan made no less by the respondents during the
(Josefina), great grandchildren of Pedro, Sr. in pre-trial conference where they stipulated that
representation of Pedro, Jr. filed a complaint the land belongs to Pedro Sr. The respondent’s
against petitioners: grandchildren of Pedro, Sr. admissions is an admission against the
for the nullification of a document entitled as respondent’s interest of the fact of ownership
by Pedro Sr. of the 192 sq. m lot which was

Rolan Jeff A. Lancion


Arellano University School of Law Page 201
Remedial Law 2- Brondial Notes AY 2017- 2018

transferred to respondent’s mother, the made cash withdrawals from the municipality’s
daughter of Pedro Sr. deposit account and such withdrawal,
purportedly for salaries, wages, allowances and
mid-year bonuses of municipal officers and
-Judicial admissions are legally binding on the
employees, had not been recorded in the
party making the admissions. Pre-trial
General Fund Cashbook; (3) Doldol made
admission in civil cases is one of the instances
adjustments in the said cashbook. In a Letter,
of judicial admissions explicitly provided for
the State Auditors demanded the immediate
under Section 7, Rule 19 of the Rules of Court,
refund of the said amount, and for Doldol to
which mandates that the contents of the pre-
submit a written explanation. Doldol failed to
trial order shall control the subsequent course
respond and was, thereafter, relieved of his
of the action, thereby, defining and limiting
duties. He was directed to transfer the account
the issues to be tried. Once the the stipulations
to Assistant Municipal Treasurer.
are reduced into writing and signed by the
parties and their counsels, they become binding The State Auditors then conducted another
on the parties who made them. They become audit. They discovered that Doldol incurred an
judicial admissions of the fact or facts added cash shortage. In a Letter to Doldol, the
stipulated. Even if placed at a disadvantageous State Auditors demanded the immediate
position, a party may not be allowed to rescind restitution of the missing fund, and directed
them unilaterally; it must assume the him to submit another written explanation.
consequences of the disadvantage. Again, Doldol failed to respond. The State
Auditors submitted their Report on their
-Respondents failed to refute the earlier examinations. The State Auditors submitted
admissions/stipulation before and during the their Memorandum on the result of the audits
trial. While denying ownership by Pedro Sr. of to the Provincial Auditor. On the same day,
the 192 sq. m lot, respondent Asuncion, when Doldol wrote the Provincial Treasurer
placed on the stand, offered a vague requesting that a re-audit be conducted on his
explanation as to how such parcel of land was cash and cash account, taking exception to the
acquired by Pedro Jr. findings of the State Auditors. Instead of
pursuing his request for a re-audit, Doldol opted
Case: Doldol vs People, 470 SCRA to refund the missing funds. He remitted certain
amount to the Acting Municipal Treasurer.
FACTS: Doldol promised to pay the balance of his
shortage. However, he reneged on his promise.
A team of State Auditors conducted an audit of
the cash and cash account of Conrado C. Doldol, The Provincial Auditor transmitted the
the Municipal Treasurer of Urbiztondo, Memorandum and Consolidated Report of the
Pangasinan. The audit covered the General State Auditors to the Ombudsman, and
Fund, Special Education Fund and Trust Fund in requested that Doldol be charged for
his custody. Doldol and the Municipal malversation of public funds. Despite the
Accountant were present during the audit. The extensions given to him, Doldol failed to file his
State Auditors discovered and noted the counter-affidavit. Two information for
following: (1) Doldol had a shortage; (2) Doldol malversation of public funds were then filed
against Doldol. Doldol testified that the funds

Rolan Jeff A. Lancion


Arellano University School of Law Page 202
Remedial Law 2- Brondial Notes AY 2017- 2018

which the State Auditors found missing were, in The defense counsel made an admission as to
fact, cash advances availed of by the municipal the authorship, authenticity, and voluntariness
employees. He insisted that not a single centavo of the execution of the counter-affidavit of
was used for his personal benefit. He averred accused Ladiana, which was subscribed and
that the charges lodged against him were sworn to before Cortez. In said counter-
premature because the same were based on an affidavit, accused Ladiana allegedly admitted to
incomplete audit. making the fatal shots on Francisco. However,
accused Ladiana allegedly did so in self-defense
In a Joint Decision, the trial court convicted the as Francisco was then purportedly attacking
accused of the crimes charged. Doldol appealed accused Ladiana and had, in fact, already
to the Court of Appeals (CA). The CA rendered inflicted a stab wound on the arm of accused
judgment affirming the appealed decision, and, Ladiana.
likewise, denied Doldol’s motion for
reconsideration thereof. However, Cortez emphasized that he was not
the one who conducted the preliminary
Issue: WON the partial restitution of the cash investigation of the complaint which led to the
shortage by Doldol can be considered as filing of the subject case. Additionally, Cortez
implied admission of misappropriation of the testified that he would not be able to anymore
missing funds recognize the face of the affiant in the said
Held: counter-affidavit, but maintained that there
was a person who appeared and identified
Yes. Except for his bare testimony, the Doldol himself as Josue Ladiana before he affixed his
offered no competent and credible evidence to signature on the counter-affidavit.
prove that the missing funds were actually cash
advances of employees in the municipality. The Sandiganbayan ruled that the prosecution
had been able to establish the guilt of petitioner
The Doldol could have offered in evidence the beyond reasonable doubt. The court a quo held
documents evidencing the names of the that his Counter-Affidavit, in which he had
recipients and amounts of the cash advances, admitted to having fired the fatal shots that
but failed to do so. Moreover, the petitioner caused the victim’s death, may be used as
wrote the Provincial Auditor and offered to evidence against him.
refund the missing funds. He was able to pay
only for certain amount, and failed to remit the Issue: WON the Counter-Affidavit he executed
balance of his shortage. Such partial restitution during the preliminary investigation of this case
of the cash shortage is an implied admission of is admissible proof showing his complicity in the
misappropriation of the missing funds. crime

Held:

YES.
Case: Ladiana vs People, 393 SCRA
-It is well-settled that the foregoing legal
Facts: formalities required by the fundamental law of
the land apply only to extra-judicial confessions
or admissions obtained during custodial

Rolan Jeff A. Lancion


Arellano University School of Law Page 203
Remedial Law 2- Brondial Notes AY 2017- 2018

investigations. Indeed, the rights enumerated in Before the Court on automatic review is the
the constitutional provision “exist only in Decision dated December 17, 1997 of the
custodial interrogations, or in-custody Regional Trial Court of Makati City, Branch 62,
interrogation of accused persons.” in Criminal Cases Nos. 97-385 to 97-388 finding
In the present case, petitioner admits that the appellant Feliciano Ulit y Tampoy guilty beyond
questioned statements were made during the reasonable doubt of two counts of qualified
preliminary investigation, not during the rape. During the hearing on October 20, 1997,
custodial investigation. Evidently, a person the prosecution presented Lucelle anew to
undergoing preliminary investigation before the
continue with her testimony on direct
public prosecutor cannot be considered as
being under custodial investigation. In fact, this examination.
Court has unequivocally declared that a She declared that the appellant raped her in
defendant on trial or under preliminary November 1996 and many other times
investigation is not under custodial
thereafter in her residence at No. 7104 San
interrogation.
Maximo Street, Makati City. Instead of asking
There is no question that even in the absence of questions to elicit the facts and circumstances
counsel, the admissions made by petitioner in before and during the commission of the
his Counter-Affidavit are not violative of his
crimes, the prosecutor asked Lucelle to identify
constitutional rights. It is clear from the
her signature in her sworn statement and to
undisputed facts that it was not exacted by the
police while he was under custody or affirm the truth of its contents. She did so. The
interrogation. Hence, the constitutional rights public prosecutor then marked the sworn
of a person under custodial investigation as statement in evidence as Exhibit H, and then
embodied in Article III, Section 12 of the 1987 manifested to the court that he had no more
Constitution, are not at issue in this case. questions for the witness on direct
In a confession, there is an acknowledgment of examination.
guilt; in an admission, there is merely a -On cross-examination, Lucelle testified that the
statement of fact not directly involving an appellant was her mother’s older brother. In
acknowledgment of guilt or of the criminal November 1996, she was not enrolled in any
intent to commit the offense with which one is school. Her father was working at a
charged. Thus, in the case at bar, a statement construction firm, the appellant was employed
by the accused admitting the commission of the at the Department of Environment and
act charged against him but denying that it was Sanitation in Makati City, while her
done with criminal intent is an admission, not a grandmother, who lived with her, worked as a
confession. maid in Bel Air Subdivison. Her mother worked
for one of her father’s cousins. On re-direct
Case: People vs Ulit, 423 SCRA
examination, the prosecution elicited from
Facts: Lucelle that the appellant raped her in
November 1996 at 11:00 p.m. inside the room

Rolan Jeff A. Lancion


Arellano University School of Law Page 204
Remedial Law 2- Brondial Notes AY 2017- 2018

of her aunt Marina in her grandmother’s house shall enjoy the right to confront and cross-
at No. 7104 San Maximo Street, Olympia, examine the witness testifying against
Makati City, and that her aunt, Marina, and her him. Generally, the affidavits of persons who
Ate Sharon were inside the room. When asked are not presented to testify on the truth of the
where her aunt and Ate Sharon were when she contents thereof are hearsay evidence. Such
was being raped in her aunts room, Lucelle did affidavit must be formally offered in evidence
not respond. When asked why she did not and accepted by the court; otherwise, it shall
respond to the questions propounded to her not be considered by the court for the simple
during the previous hearings and why she had reason that the court shall consider such
been crying in open court, Lucelle replied that evidence formally offered and accepted.
she was afraid of her uncle, the appellant.
In this case, Lucelle testified on and affirmed
Issue: WON the testimony given is admissible the truth of the contents of her sworn
statement which she herself had given. As
Held: gleaned from the said statement, she narrated
NO- Supreme Court do not agree with the ruling how and when the appellant raped and
of the trial court that the contents of the sworn subjected her to lascivious acts. She was cross-
statement of Lucelle are hearsay, simply examined by the appellants counsel and
because she did not testify thereon and merely answered the trial courts clarificatory
identified her signatures therein. By hearsay questions. The prosecution offered her sworn
evidence is meant that kind of evidence which statement as part of her testimony and the
does not derive its value solely from the court admitted the same for the said purpose
credence to be attributed to the witness herself without objection on the part of the appellant.
but rests solely in part on the veracity and
Case: Carlos L. Tanenggee vs People of the
competence of some persons from whom the Philippines GR No. 179448, June 26, 2013
witness has received the information. It signifies
all evidence which is not founded upon the Facts:
personal knowledge of the witness from whom
it is elicited, and which, consequently, is not -There were five separate information for estafa
subject to cross-examination. The basis for the through falsification of commercial documents
exclusion appears to lie in the fact that such were filed against petitioner. The said
testimony is not subject to the test which can information portray the same mode of
ordinarily be applied for the ascertainment of commission of the crime as in Criminal case No.
truth of testimony, since the declarant is not 98-163806 but differ with respect to the
present and available for cross-examination. In numbers of the checks and promissory notes
criminal cases, the admission of hearsay involved and the dates and amounts.
evidence would be a violation of the -In January 1998, two Metrobank auditors
constitutional provision while the accused conducted an audit of the Commercio Branch

Rolan Jeff A. Lancion


Arellano University School of Law Page 205
Remedial Law 2- Brondial Notes AY 2017- 2018

for more than a week. Therefore, an appellant Yes.


was asked by Elvira Ong-Chan, senior vice
president of Metrobank, to report to the Head -Petitioner’s written statement is admissible as
Office on the following day. evidence. Here, petitioner’s written statement
was given during an administrative inquiry
-In his surprise, there were seven other people conducted by his employer in connection with
present in the said office: two senior branch an anomaly or irregularity he allegedly
officers, two bank lawyers, two policemen, and committed in the course of his employment. No
a representative of the internal affairs unit of error can therefore be attributed to the courts
the bank, Valentino Elevado. below in admitting in evidence and in giving due
consideration to petitioner’s written statement
-Appellant claimed that Elevado asked him to as there is no constitutional impediment to its
sign a paper in connection with the audit admissibility.
investigation; that he inquired what he was
made to sign but was not offered any - Petitioner’s written statement was given
explanation; that he was intimidated to sign voluntarily, knowingly and intelligently. His
and was threatened by the police that he will be attempts to convince us that he signed, under
brought to the precinct if he will not sign; that duress and intimidation, an already prepared
he was not able to consult a lawyer since he typewritten statement. However, his claims
was not apprised of the purpose of the lacks sustainable basis and his supposition is
meeting; and that just to get it over with he just an afterthought for there is nothing in the
signed the paper which turned out to be a records that would support his claim of duress
confession. and intimidation.

-After the said meeting, appellant wanted to -It is settled that a confession or admission is
see Tan but was unable to find the latter and he presumed voluntarily until the contrary is
cannot even contact him. He asserts that the proved and the confessant bears the burden of
said written statement was taken in violation of proving the contrary.
his rights under Section 12, Article III of the
Constitution, particularly of his right to remain -Petitioner’s failed to overcome this
silent, right to counsel, and the right to be presumption. His written statement was found
informed of the first two rights. to have been executed freely and consciously.
The pertinent details he narrated in his
-Hence, the same should not have been statement were of such nature and quality that
admitted in evidence against him. only a perpetrator of the crime could finish.

Issue: WON the written statement executed by Case: People vs Santos, 221 SCRA 715
the appellant is admissible in evidence?
Facts:
Held:

Rolan Jeff A. Lancion


Arellano University School of Law Page 206
Remedial Law 2- Brondial Notes AY 2017- 2018

-Cupcupin and Bautista were ambushed while in Held:


a jeepney by 2 persons using unlicensed
firearms (.45 caliber pistols) at around 11:45am Yes. Guerrero’s testimony is admissible.
in Navotas, resulting to the death of Cupcupin
(driver) and serious physical injuries to Bautista; -The trial court correctly considered the
testimony of Guerrero as admissible for being
covered by the exception of proving knowledge,
-Witness Police Aide Bohol, the officer on duty plan, scheme, and the like, pursuant to Section
and directing traffic at that time, witnessed the 34, Rule 130. Thus, the Supreme Court affirmed
incident and identified Cupcupin and Bautista as the same and held as follows:
the victims, as well as Santos as the one who “We consider that the trial court did
shot the said victims. He also stated in his not commit reversible error in admitting
testimony that there were 2 other persons the Guerrero affidavit for the limited
shooting at the passengers of the above- purpose for proving knowledge or plan
mentioned vehicle;
or scheme, and more specifically, that
appellant knew that the particular
-During trial, the criminal propensity of the corner of two (2) particular streets in
accused Santos was being proven by the Manila was a good place to ambush a
prosecution when it submitted the testimony of vehicle and its passengers. Appellant
a certain Guerrero as part of its evidence, the also had waived the hearsay character
latter being a witness to another case of murder
of this evidence by failure seasonably to
imputed to the accused which took place at the
exact same place of the instant case; object to the admission of the affidavit;
it is too late in that day to raise the
hearsay rule in the appellant's
memorandum after prosecution and
defense had presented their respective
cases and had made their respective
-Santos was therefore identified by victim
Bautista and witness Bohol positively, leading to offers of evidence.”
his conviction before the trial court;
Case: Republic vs Heirs of Alejaga Sr, 393 SCRA

-On appeal, Santos argues among others that Facts:


the testimony of Guerrero is hearsay for not
having presented the latter as witness during Felipe Alejaga Sr. filed with the District Land
the trial of the instant case; Office of Roxas City, Iloilo a Free Patent
Application covering a parcel of land located at
Dumolog, Roxas City. When the application was
Issue: WON the testimony of Guerrero is executed under oath, Efren Recio, Land
admissible; and Inspector of Bureau of Lands, submitted a
report of his investigation and verification of
. the land to the latter’s District Land Office. The

Rolan Jeff A. Lancion


Arellano University School of Law Page 207
Remedial Law 2- Brondial Notes AY 2017- 2018

application of the respondent was approved by considered as null and void ab initio. However,
the District Land Officer of Roxas City and was the Court of Appeals reversed the ruling of the
issued a Free Patent. Thereafter, an Original RTC, which the latter failed to prove its
Certificate of Title was issued to the respondent allegation that respondents had obtained the
by the Register of Deeds. free patent and the Certificate of Title through
fraud and misrepresentation.
The heirs of Ignacio Arrobang, through counsel
in a letter-complaint requested the Director of Issue: WON the respondents have acquired a
Lands, Manila, for an investigation of the vested right over the parcel of land.
District Land Officer, Roxas City and the
Regional Office of Iloilo City, for irregularities in Held:
the issuance of the title of a foreshore land in NO.
favor of respondent. Isagani Cartagena, the -The claim of the respondents that an actual
Supervising Special Investigator, Legal Division investigation was conducted is not sustained by
of the Land Management Bureau submitted his the Verification and Investigation Report itself,
Report. The Chief of the Legal Division of the which bears no signature of Efren Recio. Their
Land Management Bureau (formerly Bureau of reliance on the presumption of regularity in the
Lands) recommended to the Director of Lands performance of official duty is thus misplaced.
appropriate civil proceeding for the cancellation Therefore, there can be no presumption that an
of Free Patent and the corresponding Original investigation and verification of the parcel of
Certificate of Title in the name of the land was actually conducted. Strangely,
respondent. respondents do not proffer any explanation
why the Verification and Investigation Reports
In the meantime, respondent obtained was not signed by Efren Recio. Even more
a NACIDA loan under the Cottage Industry important and as will later on be explained, this
Guarantee and Loan Fund by the defendant alleged presumption of regularity – assuming it
PNB executed in Cebu City in the amount of ever existed – is overcome by the evidence
P10,000.00. The loan was secured by a real presented by petitioner.
estate mortgage in favor of defendant PNB.

The government through the Solicitor HEARSAY RULE AND ITS EXCEPTIONS
General instituted an action for
Annulment/Cancellation of Patent and Title and -Hearsay is an out of court statement offered
Reversion against respondent, the PNB of Roxas for the truth of the matter asserted.
City and defendant Register of Deeds of Roxas -Hearsay is evidence of a fact which the witness
City. While the case is pending hearing, the does not know of his personal knowledge, that
respondent died. He was substituted by his wife is, something not derived from his own
Roqueta Alejaga and his seven (7) children. perception.

The RTC declared that by means of Exceptions:


fraud, hence, the approval of Free Patent
Application and issuance of Original Certificate Sec 37- Dying Declaration
of Title in the name of Felipe Alejaga be Sec 38- Declaration against Interest

Rolan Jeff A. Lancion


Arellano University School of Law Page 208
Remedial Law 2- Brondial Notes AY 2017- 2018

Sec 39- Act or Declaration about pedigree A: The purpose of X of his testimony is to
Sec 40- Family Reputation ascertain that B is alive.
Sec 41- Common Reputation -The statement of X is not hearsay therefore it
Sec 42- Part of the Res gestae is admissible.
Sec 43- Entries in the course of business
Sec 44-Entries in official records
Sec 45- Commercial list and like Independently relevant statement- It is an out
Sec 46- Learned Treaties of court statement which is relevant not for the
Sec 47- Testimony or deposition at a former truth of a matter asserted but for something
proceeding else.

-Hearsay Rule in Legal concept are those that


Q: What are the reasons for excluding hearsay?
are inadmissible as evidence since such
testimony is incompetent. A: 1) lack of opportunity, 2) Statement or
declaration not made under oath, 3) Court does
Q: What is the purpose of such hearsay rule? not have the opportunity to observe the
A: If the purpose is not to establish a statement demeanor of the declarant.
from which a person heard from a Third Person,
it shall not be hearsay.
Q: May hearsay be in written form?
Requirements:
-There must be an out of court statement or A: YES, If a party will present a letter to prove
declaration. the truth of the contents thereof without
-Such testimony or declaration must be presenting the letter-writer, the letter would be
repeated in court. hearsay.
-The purpose of such testimony is to establish
the truth of the out of court declaration. Section 37. Dying declaration. — The
declaration of a dying person, made under the
consciousness of an impending death, may be
Example: Mr X is on the witness stand, he
received in any case wherein his death is the
testified that “Mr.A told him that B killed C” subject of inquiry, as evidence of the cause and
surrounding circumstances of such death.
Q: Who made the out of court declaration?
A: it was A that made the declaration. -The declaration of dying person made under
the consciousness of an impending death, may
Q: Is that a hearsay statement? be received in any case wherein death is the
A: Not yet since the purpose of X is not yet subject of inquiry.
given.
Rationale: A person that is in the brink of
death, he naturally will tell the truth.
Q: What is the purpose of X in his testimony
that A mentioned to him that B killed C?

Rolan Jeff A. Lancion


Arellano University School of Law Page 209
Remedial Law 2- Brondial Notes AY 2017- 2018

-There must be a consciousness of impending or -24hrs interval from time of the commission of
a person is aware that he will actually die NOW. the crime and reporting to police authorities is
not anymore considered part of the res gestae.
-The basis should be the nature & gravity of the
wound. Verbal Acts Doctrine

-The counsel must establish relevance and Example: Juan saw Mr. Katol giving an envelope
competency of such testimony. to Mr. Balot with a statement, “Pare eto na
yung utang ko sa iyo na Php 100,000”. If Juan
-The subject matter of the case must be the will be called to the witness stand testifying
death of the declarant. such event, will it be admissible as evidence?
-It is applicable in criminal and civil cases. Answer: Yes, under the doctrine of verbal acts
docrtrine it shall form part of the res gestae.
Section 42. Part of res gestae. — Statements
made by a person while a starting occurrence is
Section 38. Declaration against interest. — The
taking place or immediately prior or subsequent
declaration made by a person deceased, or
thereto with respect to the circumstances
unable to testify, against the interest of the
thereof, may be given in evidence as part of res
declarant, if the fact is asserted in the
gestae. So, also, statements accompanying an
declaration was at the time it was made so far
equivocal act material to the issue, and giving it
contrary to declarant's own interest, that a
a legal significance, may be received as part of
reasonable man in his position would not have
the res gestae.
made the declaration unless he believed it to be
true, may be received in evidence against
Res Gestae= Things Done
himself or his successors in interest and against
third persons.
Excited utterances- Statement made by a
person while a starling occurrence is taking
Requirements:
place or immediately prior or subsequent
thereto regarding the circumstances. -The declarant must be dead or unable to testify
-Acts or Statements which are uttered in a very -The declaration must be against the interest of
dire situation. the declarant
-Statements made by a person while a -Fact asserted in the declaration was at the time
STARLING OCCURRENCE is taking place or it was made far contrary to declarant’s own
immediately or prior or subsequent thereto interest.
with respect to the circumstances.
-It covers all kinds of interest.
-There is NO HARD AND FAST RULE for the time
period in order to appreciate Res Gestae. Section 39. Act or declaration about pedigree.
— The act or declaration of a person deceased,

Rolan Jeff A. Lancion


Arellano University School of Law Page 210
Remedial Law 2- Brondial Notes AY 2017- 2018

or unable to testify, in respect to the pedigree Hearsay but Not hearsay and this
of another person related to him by birth or admissible as an admissible
marriage, may be received in evidence where it exception to the
occurred before the controversy, and the hearsay rule
relationship between the two persons is shown
by evidence other than such act or declaration. The declarant must be No requirement that
The word "pedigree" includes relationship, dead or unable to the admitter is dead
family genealogy, birth, marriage, death, the testify or unable to testify
dates when and the places where these fast
occurred, and the names of the relatives. It Admissible against Admissible only
embraces also facts of family history intimately declarant and third against the admitter
connected with pedigree. persons

Requirements:

-The declarant is dead or unable to testify Section 40. Family reputation or tradition
regarding pedigree. — The reputation or
-The act or declaration is about pedigree of tradition existing in a family previous to the
another person related to the declarant by birth controversy, in respect to the pedigree of any
one of its members, may be received in
or marriage.
evidence if the witness testifying thereon be
also a member of the family, either by
-The relationship between the relative and the
consanguinity or affinity. Entries in family bibles
declarant is shown by evidence other than such
or other family books or charts, engravings on
act or declaration. rings, family portraits and the like, may be
received as evidence of pedigree.
-The act or declaration occurred before the
controversy (ante lite motam) -The reputation or tradition existing in a family
previous to the controversy regarding the
Q: What does pedigree?
pedigree of any one of its members, may be
A: The word pedigree includes relationship, received in evidence if the witness testifying
family genealogy, birth, marriage, death, the thereon be also a member of the family either
date when and the places were these facts by consanguity or affinity.
occurred and the names of the relatives.
Section 41. Common reputation. — Common
Declaration Against Party Admission reputation existing previous to the controversy,
Interest respecting facts of public or general interest
more than thirty years old, or respecting
Made by a non-party Made by a party marriage or moral character, may be given in
evidence. Monuments and inscriptions in public
Must be against the Need not be against
places may be received as evidence of common
declarant’s interest the admitter’s interest
reputation.

Rolan Jeff A. Lancion


Arellano University School of Law Page 211
Remedial Law 2- Brondial Notes AY 2017- 2018

Q: IS a statement regarding common reputation Business- It extends to all case were the entries
admissible as a hearsay exception? were made in a professional capacity or in the
performance of a duty.
A: YES, provided that: (i) existed previous to the
controversy or ante litem motam and (ii) facts Examples: Book of account, invoices, statement
of public or general interest more than 30 years of account, bill of lading, log-book of security
or about marriage or moral character. guard, medical charts

-Common reputation which is generally Electronic business records- it is a


understood as the reputation existing in the memorandum, report, record or data
community in which the fact to be proven is compilation of acts, conditions, events,
known or exists. diagnoses or opinions made by electronic,
optical or other similar means.
Section 43. Entries in the course of business. —
Entries made at, or near the time of Requirements:
transactions to which they refer, by a person
deceased, or unable to testify, who was in a -The electronic business records is made at or
position to know the facts therein stated, may near the time of the transaction.
be received as prima facie evidence, if such
person made the entries in his professional -The electronic business record is made by or
capacity or in the performance of duty and in from transmission or supply of information by a
the ordinary or regular course of business or person with knowledge of the transaction.
duty.
-The electronic business record is kept in the
Requirements: regular course of conduct of a business activity.

-The entries must be made at or near the time -It is the regular practice to keep electronic
of the transaction to which they refer. business records.

-The entrant must be dead or unable to testify Section 44. Entries in official records. — Entries
in official records made in the performance of
-The entrant was in a position to know the facts his duty by a public officer of the Philippines, or
stated therein by a person in the performance of a duty
specially enjoined by law, are prima
-The entrant must have made the entries in his facie evidence of the facts therein stated.
professional capacity or in the performance of a
duty. Requirements:

-The entries must have been made in the - The entries must have been made in official
ordinary or regular course of business or duty. records.

-The entries must have been made by a public


officer of the Philippines in the performance of

Rolan Jeff A. Lancion


Arellano University School of Law Page 212
Remedial Law 2- Brondial Notes AY 2017- 2018

his duty or by a person in the performance of a Requirements:


duty enjoined by law.
-There is a published treatise, periodical or
-The entrant must have personal knowledge of pamphlet.
the facts stated by him or such facts were
acquired by him from reports made by person -It is on the subject of law, art, science or
under a legal duty to submit the same. history.

Section 45. Commercial lists and the like. — -The writer of the statement in the treatise,
Evidence of statements of matters of interest to periodical or pamphlet is recognized in his
persons engaged in an occupation contained in profession or calling as expert in the subject.
a list, register, periodical, or other published
compilation is admissible as tending to prove Section 47. Testimony or deposition at a
the truth of any relevant matter so stated if that former proceeding. — The testimony or
compilation is published for use by persons deposition of a witness deceased or unable to
engaged in that occupation and is generally testify, given in a former case or proceeding,
used and relied upon by them therein. judicial or administrative, involving the same
parties and subject matter, may be given in
Requirements: evidence against the adverse party who had the
opportunity to cross-examine him.
-There is a list, register, periodical or other
published compilation Requirements:

-Such published compilation contains -The witness or deponent is dead or unable to


statements or matters of interest to persons testify
engaged in an occupation.
-The former case or proceeding involves the
-The compilation is published for use by persons same parties and subject matter
engaged in that occupation
-The adverse party had the opportunity to
-The compilation is generally used and relied cross-examine the witness or deponent in the
upon by the person engaged in that occupation former case or proceeding.

Section 46. Learned treatises. — A published Double/Multiple hearsay- When a hearsay


treatise, periodical or pamphlet on a subject of declaration is embedded in another with the
history, law, science, or art is admissible as result that there are two or more levels of
tending to prove the truth of a matter stated hearsay.
therein if the court takes judicial notice, or a
witness expert in the subject testifies, that the
writer of the statement in the treatise,
periodical or pamphlet is recognized in his Opinion Rule
profession or calling as expert in the subject.

Rolan Jeff A. Lancion


Arellano University School of Law Page 213
Remedial Law 2- Brondial Notes AY 2017- 2018

General Rule: The opinion of a witness is not -The witness may also testify on his impression
admissible of the condition, appearance, behaviors or
emotions of a person.
Exception:

-Opinion of expert witness


Character Evidence
-Opinion of ordinary witness
Section 51. Character evidence not generally
Q: What is an expert opinion? admissible; exceptions: —

A: The opinion of a witness on a matter (a) In Criminal Cases:


requiring special knowledge or skill, experience,
training. (1) The accused may prove his good moral
character which is pertinent to the moral trait
Q: What steps should be taken before an expert involved in the offense charged.
witness is allowed to testify?
(2) Unless in rebuttal, the prosecution may not
A: -The expert witness should be qualified prove his bad moral character which is
before he is allowed to testify. pertinent to the moral trait involved in the
offense charged.
-Qualification is made by showing that the
witness possesses special knowledge, skill, (3) The good or bad moral character of the
experience or training. offended party may be proved if it tends to
establish in any reasonable degree the
probability or improbability of the offense
charged.
*An expert witness may testify based on
hypothetical or assumed facts. (b) In Civil Cases:

Q: When is the opinion of an ordinary witness Evidence of the moral character of a party in
admissible in evidence? civil case is admissible only when pertinent to
the issue of character involved in the case.
A: -The identity of a person about whom he has
adequate knowledge. In the case provided for in Rule 132, Section 14.

-A handwriting with which he has sufficient Q: Distinguish between character and


familiarity. reputation?

-The mental sanity of a person with whom he is A: Character is what you are, reputation what
sufficiently acquainted. the community knows you are. Reputation may
be used to prove character.

Rolan Jeff A. Lancion


Arellano University School of Law Page 214
Remedial Law 2- Brondial Notes AY 2017- 2018

GR: Character evidence is not admissible Burden of Proof- The duty of a party to present
evidence on the facts in issue necessary to
Exceptions: establish his claim or defense by the amount of
evidence required by law.
Criminal Cases
-The burden of proof rests upon a party
-The accused may prove his good moral asserting the affirmative of an issue.
character which is pertinent to the moral trait
involved in the offense charged. Burden of evidence- The duty of a party to go
forward with the evidence and which duty if
-Unless in rebuttal, the prosecution may not
fulfilled by the party shifts the burden of
prove his bad moral character which is
pertinent to the moral trait involved in the evidence to the adverse party.
offense charged.
Equipoise doctrine- It is where the evidence on
-The good or bad moral character of the a particular issue is in equipoise or level or
offended party may be proved if it tends to where the party not having the burden of proof
establish in any reasonable degree the has produced countervailing evidence, the
probability or improbability of the offense result is that the party with the burden of proof
charged. will lose out on that particular issue.

Civil Cases: Q: What is a Reputation?

-Evidence of the moral character of a party in A: It is an interference-It arrives at a logical


civil case is admissible only when pertinent to conclusion.
the issue of character involved in the case.
Q: What are the two kinds of inferences?
Q: How is character proved?
A: -Presumption of Fact- Simple Inference
A: It is proved by (1) reputation evidence, (2)
specific instances of conduct or (3) witness -Presumption of Law- It is an inference
opinion. determined by law

Reputation Evidence- it is a mode to prove -Under the ROC, there are only 2 kinds of
moral character. presumption: Disputable/Conclusive

-The modern view is that reputation among a Q: How many are the disputable presumptions
person’s associate is also admissible to prove provided by law?
his character.
A: 37 disputable presumptions Sec 3 (A to KK)
Rule 131- Burden of Proof and Presumptions
Q: How many are the conclusive presumptions
Burden of Proof vs Burden of Evidence provided by law?

Rolan Jeff A. Lancion


Arellano University School of Law Page 215
Remedial Law 2- Brondial Notes AY 2017- 2018

A: 2 conclusive presumptions (Sec 2) Example: A leases to B a parcel of land located


in Bulacan which the latter cultivates. Later on,
Section 2. Conclusive presumptions. — The B discovered that is not the real owner of the
following are instances of conclusive land and decided to stop paying rentals. Is the
presumptions: stoppage to pay rental valid?

(a) Whenever a party has, by his own Answer: NO, Since there is a conclusive
declaration, act, or omission, intentionally and presumption that when he entered into a
deliberately led to another to believe a contract of lease with A, he acknowledges the
particular thing true, and to act upon such fact that A has the right to lease such property
belief, he cannot, in any litigation arising out of unto him.
such declaration, act or omission, be permitted
to falsify it: (Estoppel in Pais) Disputable Presumption- The quantum of
evidence should be clear and convincing.
(b) The tenant is not permitted to deny the title
of his landlord at the time of commencement of Exception: Presumption of Innocence
the relation of landlord and tenant between
them. (Estoppel of a Tenant) (e) That evidence willfully suppressed would be
adverse if produced.
Q: What is a conclusive presumption?
Requirements:
A: It is a term that cannot be controverted and
the court has no other choice but to accept it. -The suppression must be willful

Q: What is estoppel in Pais? - The evidence must be material

-The evidence is not available to both parties


A: When a person represent somebody and a
third person acted on such representation
*Suppression is an exercise of privilege.
whether in a negative or positive way then the
former cannot deny the existence of such Example: In a case of rape, the victim was never
representation.
presented as a witness. When the accused was
-Such Representation shall be binding. convicted, he appealed to the appellate court
on the ground of suppression of evidence. SC
-It is also applicable in a contract of agency ruled that there is no suppression of evidence
since the evidence (testimonial) is available to
Estoppel By Tenant- A lessee who is the both parties. The accused should have
occupant of the premises on the basis of subpoena the rape victim to appear in court.
accepting the contract of lease, the lessee
(PP vs Padrigone)
accept the fact that lessor is owner of the land
and relation of landlord & tenant. Example 2: Benny filed a case against XYZ
Hospital, Doctor A & Nurse B for damages due

Rolan Jeff A. Lancion


Arellano University School of Law Page 216
Remedial Law 2- Brondial Notes AY 2017- 2018

to medical negligence. The son of Benny died The following shall be considered dead for all
due to medical complication on the ground of purposes including the division of the estate
medical negligence. During the trial of the case, among the heirs:
when the counsel for the plaintiff asked for the
(1) A person on board a vessel lost during a sea
hospital records and upon discovery of such
voyage, or an aircraft with is missing, who has
records, the documents pertaining to 42 to 44 th not been heard of for four years since the loss
day of records are not available. The court of the vessel or aircraft;
ordered the defendant to release such
document but they failed to do so. (2) A member of the armed forces who has
taken part in armed hostilities, and has been
Effect of Suppression of Evidence: Those that missing for four years;
the plaintiff wants to establish are deemed
established. (3) A person who has been in danger of death
under other circumstances and whose existence
has not been known for four years;
(m) That official duty has been regularly
performed.
(4) If a married person has been absent for four
consecutive years, the spouse present may
-When a police officer conducts a buy-busy
contract a subsequent marriage if he or she has
operation and the accused was killed. There is a
well-founded belief that the absent spouse is
presumption that the police officer performed
already death. In case of disappearance, where
his duty based on the rules & regulation.
there is a danger of death the circumstances
hereinabove provided, an absence of only two
Q: For this presumption to be applicable, what
years shall be sufficient for the purpose of
should be the rule?
contracting a subsequent marriage. However, in
any case, before marrying again, the spouse
A: If there are guidelines to be followed and one
present must institute a summary proceedings
of which is not followed, the disputable
as provided in the Family Code and in the rules
presumption of regularity in the performance of
for declaration of presumptive death of the
duty shall not be applicable.
absentee, without prejudice to the effect of
reappearance of the absent spouse.
(w) That after an absence of seven years, it
being unknown whether or not the absentee
4 years- A person on board a vessel lost during
still lives, he is considered dead for all purposes,
a sea voyage or an aircraft which is missing.
except for those of succession.
4 years- Member of the army who has taken
The absentee shall not be considered dead for
part in armed hostilities.
the purpose of opening his succession till after
an absence of ten years. If he disappeared after
4 years- A person who has been in danger of
the age of seventy-five years, an absence of five
death under other circumstances.
years shall be sufficient in order that his
succession may be opened.

Rolan Jeff A. Lancion


Arellano University School of Law Page 217
Remedial Law 2- Brondial Notes AY 2017- 2018

4 years- The spouse present may contract a (cc) That in cases of cohabitation by a man and
subsequent marriage if he/she has well founded a woman who are not capacitated to marry
belief of the absent spouse. each other and who have acquire properly
through their actual joint contribution of
2 years- In case of disappearance, when there is money, property or industry, such contributions
danger of death. and their corresponding shares including joint
deposits of money and evidences of credit are
GR: Any person that is absent for 7 consecutive equal.
years without news being alive is presumed
dead for all intents & purposes. (jj) That except for purposes of succession,
when two persons perish in the same calamity,
Exception: For purpose of succession- 10 years such as wreck, battle, or conflagration, and it is
absences not shown who died first, and there are no
particular circumstances from which it can be
Exception to Exception: Person disappeared inferred, the survivorship is determined from
was 75 years of age- 5 consecutive years to the probabilities resulting from the strength and
open succession. the age of the sexes, according to the following
rules:
Effects of Absence to Marriage
1. If both were under the age of fifteen years,
-2 years of presumptive death shall suffice the older is deemed to have survived;

-In the case of Santos vs Santos, for the purpose 2. If both were above the age sixty, the younger
of re-marriage there is a need for judicial is deemed to have survived;
declaration of absences.
3. If one is under fifteen and the other above
Factual Death- It is evidenced by the Corpus sixty, the former is deemed to have survived;
delicti (Fact of Death)
4. If both be over fifteen and under sixty, and
Presumptive Death- There is a necessity to the sex be different, the male is deemed to
apply for judicial declaration of absences. have survived, if the sex be the same, the older;

(aa) That a man and woman deporting 5. If one be under fifteen or over sixty, and the
themselves as husband and wife have entered other between those ages, the latter is deemed
into a lawful contract of marriage; to have survived.

(bb) That property acquired by a man and a Section 4. No presumption of legitimacy or


woman who are capacitated to marry each illegitimacy. — There is no presumption of
other and who live exclusively with each other legitimacy of a child born after three hundred
as husband and wife without the benefit of days following the dissolution of the marriage
marriage or under void marriage, has been or the separation of the spouses. Whoever
obtained by their joint efforts, work or industry. alleges the legitimacy or illegitimacy of such
child must prove his allegation.

Rolan Jeff A. Lancion


Arellano University School of Law Page 218
Remedial Law 2- Brondial Notes AY 2017- 2018

Rule 132- Presentation of Evidence Transcript of Stenographic Notes- The notes


prepared by the stenographer.
Three Parts of the presentation of evidence
-Under Sec 2, it shall be deemed prima facie a
-Examination of Witness (Sec 1-18) correct statement of such proceeding.
-Authentication and Proof of Documents (Sec
Q: Give exceptions to the rule that the witness’s
19-33) examination of a witness
-Offer and Objection (Sec 34-40)
A: -In Civil Cases covered by the Rule on
Summary Procedure, the testimony of a witness
Section 1. Examination to be done in open shall be by way of affidavit.
court. — The examination of witnesses
presented in a trial or hearing shall be done in -In Criminal cases, covered by the Rule on
open court, and under oath or affirmation. Summary Procedure, the affidavits submitted
Unless the witness is incapacitated to speak, or by the parties shall constitute the direct
the questions calls for a different mode of testimonies of the witnesses who executed the
answer, the answers of the witness shall be same, subject to cross-examination, redirect or
given orally. re-cross examination.

-Under the Constitution, a party has the right to -Cases covered by the Judicial Affidavit Rule
public, impartial and speedy trial.
-Matters regarding the admissibility and
Q: What do you mean of public? evidentiary weight of electronic documents may
be proved by affidavits to cross by adverse
A: It means it should be in open court the party.
presentation of evidence.
Section 3. Rights and obligations of a witness.
Q: What is open court? — A witness must answer questions, although
his answer may tend to establish a claim against
A: It should be accessible to the public. him. However, it is the right of a witness:

Q: What is the opposite of trial in open court? (1) To be protected from irrelevant, improper,
or insulting questions, and from harsh or
A: Trial in chambers insulting demeanor;

-All trials must be recorded and all of the courts (2) Not to be detained longer than the interests
right now are Courts of Record. of justice require;

Stenographer- The person assigned to record all (3) Not to be examined except only as to
the proceeding during the trial. matters pertinent to the issue;

Rolan Jeff A. Lancion


Arellano University School of Law Page 219
Remedial Law 2- Brondial Notes AY 2017- 2018

(4) Not to give an answer which will tend to ahead the direct, cross, re-direct & re-cross
subject him to a penalty for an offense unless questions.
otherwise provided by law; or
Section 5. Direct examination. — Direct
(5) Not to give an answer which will tend to examination is the examination-in-chief of a
degrade his reputation, unless it to be the very witness by the party presenting him on the facts
fact at issue or to a fact from which the fact in relevant to the issue.
issue would be presumed. But a witness must
answer to the fact of his previous final Q: What are the prohibited forms of question in
conviction for an offense. a direct examination?

Right of Self-Incrimination as Witness- Upon A: Leading Question and Misleading Question


proposal of questions
Q: What is a leading question?
Right of Self-Incrimination as accused-witness-
Total Refusal to take witness stand A: It is a question which suggests the answer.

-In Criminal Cases, the accused and the witness Q: What is a misleading question?
are protected by the Constitutional Right
against Self-Incrimination. A: One which assumes as true as fact not yet
testified to by the witness or contrary to the
Order of Trial vs Order of Presentation of previously stated.
Evidence
-Misleading questions are not allowed in the
Order of Trial proceeding/trial.

Rule 30- Civil Case Example: On March 5, 2010 at 7pm a crime of


rape happened.
Rule 119- Criminal Case
Question: You testified that on March 5, 2010
Order of Presentation of Evidence around 7:00 am, you were within the vicinity of
the crime?
-Direct Examination
-In Judicial Affidavit, there can be a lot of
-Cross Examination leading and misleading question.

-Re-Direct Examination Remedy of Adverse Party- A party should make


a manifestation that he object to the leading
-Re-Cross Examination question under judicial affidavit or to strike out
such leading questions.
*Under Rule 23- modes of discovery upon
written interrogatories- The party shall prepare Exceptions to Leading Question:

Rolan Jeff A. Lancion


Arellano University School of Law Page 220
Remedial Law 2- Brondial Notes AY 2017- 2018

-On Cross-Examination Section 8. Re-cross-examination. — Upon the


conclusion of the re-direct examination, the
-Preliminary Matters adverse party may re-cross-examine the witness
on matters stated in his re-direct examination,
-Children of Tender Years and also on such other matters as may be
allowed by the court in its discretion.
-Unwilling/Hostile Witness
Q: What is a misleading question? Is it allowed?
-Witness who is an adverse party
A: A misleading question is one which assumes
Q: Who is an adverse witness? as trues a fact not yet testified to by the witness
or contrary to that which he has previously
A: The opposing side and there is no need to stated. It is never allowed.
give the predicate.
Section 9. Recalling Witness
Q: Who is an adverse-party interest?
-After the examination of a witness by both
A: If the witness is an adverse party or a sides has been concluded, the witness cannot
director, officer or managing agent of a be recalled without leave of court.
corporation, partnership or association which is
an adverse party. -Only discretionary upon court.

Section 6. Cross-examination; its purpose and -Additional Direct & Re-Direct Examination
extent. — Upon the termination of the direct
examination, the witness may be cross- Section 11. Impeachment of adverse party's
examined by the adverse party as to many witness. — A witness may be impeached by the
matters stated in the direct examination, or party against whom he was called, by
connected therewith, with sufficient fullness contradictory evidence, by evidence that his
and freedom to test his accuracy and general reputation for truth, honestly, or
truthfulness and freedom from interest or bias, integrity is bad, or by evidence that he has
or the reverse, and to elicit all important facts made at other times statements inconsistent
bearing upon the issue. with his present, testimony, but not by
evidence of particular wrongful acts, except
Section 7. Re-direct examination; its purpose that it may be shown by the examination of the
and extent. — After the cross-examination of witness, or the record of the judgment, that he
the witness has been concluded, he may be re- has been convicted of an offense.
examined by the party calling him, to explain or
supplement his answers given during the cross- GR: A party cannot impeach his own witness.
examination. On re-direct-examination,
questions on matters not dealt with during the Exception: Hostile Witness
cross-examination, may be allowed by the court
in its discretion. (3) Three Ways to Impeach a Witness in
Cross-Examination

Rolan Jeff A. Lancion


Arellano University School of Law Page 221
Remedial Law 2- Brondial Notes AY 2017- 2018

- Contradictory Evidence Criminal Case


- General Reputation or integrity
- Prior inconsistent statement Penalty: 6 years and above- J.A is not
mandatory
-In order to impeach a witness, the basis shall
be the direct examination. -The option to avail J.A is with the accused.

Example of Prior Inconsistent Statement 6 years and below- J.A is mandatory

-A witness testified that he saw the accused Section 16. When witness may refer to
stabbed the deceased and he described the memorandum. — A witness may be allowed to
physical feature of the accused. refresh his memory respecting a fact, by
anything written or recorded by himself or
-In order to impeach such witness, a counsel under his direction at the time when the fact
occurred, or immediately thereafter, or at any
should rely on the sinumpaang salaysay and
other time when the fact was fresh in his
present it in the trial. memory and knew that the same was correctly
written or recorded; but in such case the
- Before a witness is impeached by evidence of
writing or record must be produced and may be
an inconsistent statement, the predicate must inspected by the adverse party, who may, if he
first be laid. chooses, cross examine the witness upon it, and
may read it in evidence. So, also, a witness may
Q: How is the predicate laid? testify from such writing or record, though he
retain no recollection of the particular facts, if
A: - It must be in writing, it must first be shown
he is able to swear that the writing or record
to the witness before any question is asked of correctly stated the transaction when made;
him concerning the statement. but such evidence must be received with
caution.
-The statement must be related to him, with the
circumstances of the time and place and the Principles: -Present Memory Revive
persons present.
-Past Recollection Recorded
=He must be asked whether he made such
statement. -When a witness is called to the witness stand,
he is not allowed to use as documentary paper
-If so, he must be asked to explain the as the basis.
inconsistency.
Present Memory Revive- With leave of court, a
Judicial-Affidavit Rule counsel can ask the trial court to allow his
witness to revive the memory.
GR: It shall be applicable in all courts
Example: On June 1, 2010 while on his way
Exception: Supreme Court
home. A saw a stabbing incident happened in

Rolan Jeff A. Lancion


Arellano University School of Law Page 222
Remedial Law 2- Brondial Notes AY 2017- 2018

the corner of Espana Avenue. When A arrived examination be continued on the next hearing
home, he writes what transpired in his daily schedule and it was granted by the court. On
diary. If A will be called to the witness stand, A the day set for cross-examination, the witness
can ask the court to allow him to use the daily died.
diary.
-The Incomplete testimony made by the witness
Limitation: The documentary evidence help will shall be admitted as evidence since the cause is
not form part as annexes. the defendant.

-It cannot be used as corroborative evidence. Example 2: After the counsel for the plaintiff
conducted its direct examination. The counsel
Past Recollection Recorded- The witness has no for the defendant will start the cross-
recollection about what transpired and he can examination but the former manifested before
only testify based on the memorandum or the court that he has an urgent hearing in
record. another place. On the day of the supposed
-The memorandum can be marked as evidence. cross-examination, the witness died.

Example: Dr. Arizala of NBI Forensics lab and his -The direct examination conducted by the
job is to conduct autopsy of dead bodies. If Dr. counsel of the plaintiff shall not form part as
Arizala will be called in the witness stand, he evidence since the cause of the incompleteness
can use a memorandum un order to identify the is the plaintiff.
dead body and his cause of death. B. Authentication and Proof of Documents
Q: How is testimonial evidence completed?
Q: What is authentication?
A: It is completed when all the parties are given A: Authentication is the process of establiing
opportunity as provided in the rules. that evidence is what is purports to be, in other
words, that it is genuine.

Q: Should a document be authenticated before


it may be received as evidence?
Q: Is incomplete testimony admissible?
Private documents- it must be authenticated
A: It will depend as to who cause the before they may be received.
incomplete testimony.
Public documents- It may be received in
Example 1: During the direct examination of the evidence without the need for authentication.
witness by the plaintiff, he manifested that he
completed the direct examination. The counsel Q: What is a private document?
for the defendant moved that the cross-

Rolan Jeff A. Lancion


Arellano University School of Law Page 223
Remedial Law 2- Brondial Notes AY 2017- 2018

A: A private document is a document other tribunals, and public officers whether


than a public document. Philippines or foreign country.

Q: How is a private document authenticated? Acknowledged documents except wills

A: Before a private document offered as Public records


authentic is received in evidence first be
authenticated, that is its due execution and Q: Are affidavits and sworn certifications public
documents?
authenticity should be proved by (a) anyone
who saw the document executed or written or A: Affidavits and sworn certifications are not
(b) evidence of the genuineness of the signature public documents because they do not contain
or handwriting of the maker. acknowledgment but a jurat.
Q: What is an ancient document? Does it need -A marriage contract is considered as private
to be authenticated? writing and authenticated.
A: An ancient document is a private document -If a public record is presented as evidence in
which is more than 30 years old, is produced the trial, there is no need to authenticate such
from a custody in which it would be found if document.
genuine, and is unblemished by any alteration
or circumstance of suspicion. -If a document carries an acknowledgment,
there is no need to authenticate such
-It does not need to be authenticated. document.
Q: How is a private electronic document Q: How is an official record proved?
authenticated?
A: -Through official publication thereof or –
A: - Evidence that it had been digitally signed. Copy attested by the officer having the legal
-Appropriate security procedure or devices as custody of the record and accompanied, if the
record is not kept in the Philippines, with a
may be authorized by law or rules have been
applied to the document. certificate that such officer has the custody.

-Other evidence showing its reliability and Q: How is a public record of a private document
integrity to the satisfaction of the judge. is proved?

Public Records: A: An authorized public record of a private


document may be proved by : (a) original
Official Records- Written official acts of the record copy attested by the legal custodian of
sovereign authority, official bodies and the record, with an appropriate certificates that
such officer has the custody.

Rolan Jeff A. Lancion


Arellano University School of Law Page 224
Remedial Law 2- Brondial Notes AY 2017- 2018

Q: May a judicial record be impeached? Testimonial Evidence- At the time the witness
is called to testify. The offer shall be made
A: Yes, a judicial record may be impeached by orally.
evidence of: (i) want of jurisdiction in the court
or judicial officer, (ii) collusion between the -Under Judicial Affidavit Rule, the party
parties or (iii) fraud in the party offering the presenting the judicial affidavit of his witness in
place of direct testimony shall state the purpose
record, in respect of the proceeding
of such testimony at the start of the
Q: When will an alteration affect the presentation of witness.
admissibility of a document?
Documentary Evidence- After the presentation
A: An alteration will affect the admissibility of a of a party’s testimonial evidence and before he
rests his case. The offer shall be done orally
document if :
unless allowed by the court to be done in
(1) The document is being offered as writing.
genuine
Object Evidence- Same as documentary
(2) The alteration was made after the evidence, unless it cannot be submitted to the
execution of the document. court in which case at the time it is presented to
(3) The alteration is in a part material to the court’s senses.
the question in dispute.
Q: May evidence offered for one purpose be
considered for another?

Offer and Objection A: NO

Section 34. Offer of evidence. — The court shall Formal Offer of Evidence
consider no evidence which has not been
formally offered. The purpose for which the GR: A formal offer of evidence must be done
evidence is offered must be specified. orally.

Section 35. When to make offer. — As regards Exception: In writing (Evidence consists of
the testimony of a witness, the offer must be voluminous documents)
made at the time the witness is called to testify.
Section 36. Objection. — Objection to evidence
Documentary and object evidence shall be offered orally must be made immediately after
offered after the presentation of a party's the offer is made.
testimonial evidence. Such offer shall be done
orally unless allowed by the court to be done in Objection to a question propounded in the
writing. course of the oral examination of a witness shall
be made as soon as the grounds therefor shall
become reasonably apparent.

Rolan Jeff A. Lancion


Arellano University School of Law Page 225
Remedial Law 2- Brondial Notes AY 2017- 2018

An offer of evidence in writing shall be objected


to within three (3) days after notice of the
unless a different period is allowed by the Q: When may a witness’s answer be stricken off
court. the record?

In any case, the grounds for the objections must A: Where a witness answered the question
be specified. before the adverse party had the opportunity to
voice fully its objection, and such objection is
Q: How can a party interpose an objection? found to be meritorious, the court shall sustain
the objection and order the answer given to be
A: A party can interpose an objection at the stricken off the record.
time of the offer of evidence.
-Where the answer is incompetent, irrelevant
Q: Why does a party object in the course of the or otherwise improper.
trial?
Q: When must the court give its ruling upon the
A: It is allowed by the rules as oral objection objection?
which must be ruled upon immediately by the
court. A: The ruling of the court must be given
immediately after the objection is made, unless
Q: What are the kinds of objection? the court desires to take a reasonable time to
inform itself on the question presented; but the
A: (a) formal objection, (b) substantial
ruling shall always be made during the trial and
objection, (c) formal & substantial objection.
at such time as will give the party against whom
Formal Objection- it only pertains to the form it is made an opportunity to meet the situation
presented by the ruling. (Rule 132, Sec 38)
Example: Leading Questions
Section 40. Tender of excluded evidence. — If
Substantial Objection- It violates the Best documents or things offered in evidence are
Evidence and Parole Evidence Rule excluded by the court, the offeror may have the
same attached to or made part of the record. If
Broad Side Objection- It is grounded on the evidence excluded is oral, the offeror may
irrelevancy and competency. state for the record the name and other
personal circumstances of the witness and the
Continuing Evidence-When a party has substance of the proposed testimony.
previously objected to a question and
succeeding questions are of the same class as Q: What is the remedy of the offeror if the
those previously objected. evidence is excluded?

General Objection- It is submitted that they are A: it will depend on what kind of evidence:
not prohibited where the evidence is orally
offered.

Rolan Jeff A. Lancion


Arellano University School of Law Page 226
Remedial Law 2- Brondial Notes AY 2017- 2018

Documentary/Object Evidence- The offeror may 3. Clear and Convincing Evidence (HK
have the same attached to or made part of the Government vs Olalia)
record.
-The quantum of evidence which is lower than
Oral Evidence- The offeror may state for the proof beyond reasonable doubt but higher than
record the name or other personal preponderance of evidence.
circumstance of the witness.
-In Labor Cases, it is clear and convincing
Q: What is the purpose of the tender of evidence
excluded evidence?
-Applicable to establish fraud and forgery cases.
A: To allow the appellate court to assess
whether the trial court’s exclusion of the -If a party want to overturn a disputable
evidence is proper. presumption.

Rule 133- Weight and Sufficiency of Evidence 4. Preponderance of Evidence

-Evidence may be admissible and it is a matter -Applicable in civil cases


of law.
-It provides for superior weight of evidence on
Weight and Sufficiency of Evidence – It is a the issues involved.
matter of perception or judicial evaluation.
Equipoise Doctrine- The evidence for both
Quantum of Evidence- The weight of evidence parties is on equilibrium
required to discharge the burden of proof.
-In civil cases, if the evidence of the plaintiff are
Kinds of Evidence on equilibrium and the determination of weight
shall be based on the evidence presented and
1. Overwhelming Evidence- Any evidence offered.
which is beyond controversy and it cannot be
controverted. 5. Substantial Evidence

2. Proof Beyond Reasonable Doubt -It is applicable in admin cases

-Applicable in Criminal Cases -It is a quantum of evidence required in


administrative cases.
-That quantum of evidence which produces in
unprejudiced mind and it produces conviction. -That amount of relevant evidence which a
reasonable mind might accept as adequate to
-Moral certainty is required. justify a conclusion.

-A person is sincerely convicted that the 6. Probable Cause


accused is guilty of charge.

Rolan Jeff A. Lancion


Arellano University School of Law Page 227
Remedial Law 2- Brondial Notes AY 2017- 2018

-The quantum of evidence consisting of several -It never shifts and always with the claimant.
facts and circumstance known to the person to
arrive at a probability that a person charged is Burden of Evidence- It is the duty to persuade
guilty of an offense. the court through evidence that he has a claim
over the case.
-It is used in Preliminary Examination and
Investigation. -It follows the burden of proof.

Preliminary Examination- Issuance of Warrant


of Arrest

Preliminary Investigation- Filing of an


information in court.

7. Prima Facie Evidence- The quantum of


evidence which becomes conclusive if not
controverted.

-An accused can be convicted of Prima Facie


Evidence.

8. Iota of Evidence- it is related to


circumstantial evidence

Requirements:

-More than one circumstance

-Facts from which the inferences are derived


and proven

-Combination of all circumstance as to produce


conviction beyond reasonable doubt.

-In a petition for Quo Warranto, it should be


proof beyond reasonable doubt that will lead to
the ouster of a judicial officer.

Burden of Proof vs Burden of Evidence

Burden of Proof- It is the obligation of the party


to establish his claim.

Rolan Jeff A. Lancion


Arellano University School of Law Page 228

Você também pode gostar